You are on page 1of 317

Geometry

1 INTRODUCTION TO EUCLID’S
GEOMETRY

INTRODUCTION

The credit for introducing geometrical concepts goes to the distinguished Greek mathematician ‘Euclid’ who is
known as the ‘Father of Geometry’ and the word ‘geometry’ comes from the Greek words ‘geo’ which means
‘Earth’ and ‘metreon’ which means ‘measure’.

A. EUCLID GEOMETRY

(a) Euclid’s definitions


(i) A point is that which has no part.
(ii) A line is breadthless length.
(iii) The ends of a line segment are points.
(iv) A straight line is that which has length only.
(v) A surface is that which has length and breadth only.
(vi) The edges of a surface are lines.
(vii) A plane surface is that which lies evenly with the straight lines on itself.

(b) Some Undefined terms


There are three basic concepts in geometry, namely, ‘point’, ‘line’ and ‘plane’. It is not possible to
define these three concepts precisely. We can, however have a good idea of these concept by
considering examples given below
Point: A point is represented by a fine dot make by sharp pencil on a sheet of paper. A Point is denoted
by capital letter A, B, P, Q, R etc.
Plane: The surface of a smooth wall or the surface of a sheet of paper or the surface of a smooth black
board are close example of a plane.
Line: If we fold a piece of paper, the crease in the paper represents a geometrical straight line. The edge
of ruler, the edge of the top of a table, the meeting place of two walls of a room are close examples of a
geometrical straight line.

(c) Some Geometrical concepts


Axioms: The basic facts which are taken for granted, without proof, are called axioms.
Ex. Halves of equal are equal.
Statements: A sentence which can be judged to be true or false is called a sentence.
Ex. The sum of the angles of a triangle is 180º, is a true statement.
Theorems: A statement that requires a proof, is called a theorem. Establishing the truth of a theorem is
known as proving the theorem.
Ex. The sum of all angles around a point is 360º
Corollary: A statement whose truth can easily be decided from a theorem, is called its corollary.
Conjecture: In mathematics, a conjecture is a conclusion or proposition based on incomplete
information, for which no proof has been found ‘Ex’ Goldbach Conjecture state that every even
integer greater than two is the sum of two prime numbers.

(d) Some terms related to geometry


Point : A point is an exact location. A fine dot represents a point. Point is denoted by capital letter A,
B, P, Q, R etc.
For Unacademy Subscription Use “PJLIVE” Code | Join t.me/pjsir42 for Updates
For More Info: “75970 – 84242, 94590 – 43333 / 2222”
Line segment: The straight path between two points A and B is called the line segment AB . The point
A and B are called the end points of the line segment AB .A line segment has a definite length.
Line : A line segment AB when extended indefinitely in both direction is called the line AB . A line
has no end points. A line has no definite length. Some time, we label lines by small letters l, m, n etc.

Interior point of a line segment : A point R is called an interior point of a line segment PQ if R lies
between P and Q but R is neither P nor Q.

Congruence of line segment : Two line segments AB and CD are congruent if trace copy of one can
be superposed on the other so as to cover it completely and exactly in this case we write AB  CD. In
other words, we can say two lines are congruent if their lengths is same.
Distance between two points : The distance between two points P and Q is the length of line segment PQ
Ray : Directed line segment is called a ray. If AB is a ray then it is denoted by AB . Point A is called
initial point of ray.

Opposite rays : Two rays AB and AC are said to be opposite rays if they are collinear and point A is
the only common point of the two rays.

Collinear points : Three or more points are said to be collinear if there is a line which contains all of them.

Concurrent Lines : Three or more lines are said to be concurrent if there is a point which lies on all of
them.
t
n

m
P


Intersecting lines : Two lines are intersecting if they have a common point. The common point is
called the ‘point of intersection’.

Parallel lines : Two lines l and min a plane are said to be parallel lines if they do not have a common point.

Plane : A plane is a surface such that every point of the line joining any two points on it lies on it.

(e) Euclid’s Axioms


(i) Things which are equal to the same thing are equal to one another.
(ii) If equal are added to equals, the wholes are equal.
(iii) If equal are subtracted from equals, the remainders are equal.
(iv) The things which coincide with one another are equal to one another.
(v) The whole is greater than the part.
(vi) The thing which are double of the same thing are equal to one another.
(vii) The thing which are halves of the same thing are equal to one another.

(f) Euclid’s Five Postulates


(i) A straight line may be drawn from any one point to any other point.
(ii) A terminated line or a line segment can be produced infinitely.

For Unacademy Subscription Use “PJLIVE” Code | Join t.me/pjsir42 for Updates
For More Info: “75970 – 84242, 94590 – 43333 / 2222”
(iii) A circle can be drawn with any centre and of any radius.
(iv) All right angles are equal to one another.
(v) If a straight line falling on two straight lines makes the interior angles on the same side of it taken
together less than two right angles, then the two straight lines if produced infinitely meet on that
side on which the sum of angles are less than two right angles.

(g) Incidence Axioms


(i) A line is the collection of infinite number of points.
(ii) Through a given point, an infinite lines can be drawn.

(iii) Given two distinct points, there is one and only one line that contains both the points.

(h) Parallel Axioms


If P is a point outside a line , then one and only one line can be drawn through P which is parallel to .

The above axioms may also be re stated as follows: Two intersecting lines cannot both be parallel to the
same line...

(i) Some more results


(i) Two distinct lines cannot have more than one point in common.

(ii) Two lines which are both parallel to the same line, are parallel to each other.
i.e.  || n, m || n   || m

Solved Examples

1
Example.1 If a point C lies between two points A and B such that AC = BC, then prove that AC = AB .
2
Explain by drawing the figure.
Solution. According to the given statement, the figure will be as shown
alongside in which the point C lies between two points A and B such
that AC = BC.
Clearly, AC + BC = AB
 AC + AC = AB [ AC = BC]
 2AC = AB

For Unacademy Subscription Use “PJLIVE” Code | Join t.me/pjsir42 for Updates
For More Info: “75970 – 84242, 94590 – 43333 / 2222”
1
And, AC = AC = AB
2
Example.2 Give a definition for each of the following terms. Are there other terms that need to be defined
first ? What are they, and how might you define them ?
(i) parallel lines (ii) perpendicular lines
(iii) line segment (iv) radius
Solution. (i) Parallel lines : Lines which don.t intersect any where are called parallel lines.
(ii) Perpendicular lines : Two lines which are at a right angle to each other are called
perpendicular lines.
(iii) Line segment : It is a terminated line.
(iv) Radius : The length of the line-segment joining the centre of a circle to any point on its
circumference is called its radius.

Example.3 How would you rewrite Euclid's fifth postulate so that it would be easier to understand ?
Solution. Two distinct intersecting lines cannot be parallel to the same line.

Example.4 Does Euclid's fifth postulate imply the existence of parallel lines? Explain.
Solution. If a straight line  falls on two straight lines m and n such that sum of the interior angles on one
side of  is two right angles, then by Euclid's fifth postulate the line will not meet on this side of
. Next, we know that the sum of the interior angles on the other side of line also be two right
angles. Therefore they will not meet on the other side. So, the lines m and n never meet and are,
therefore parallel.
Theorem : If , m, n are lines in the same plane such that  intersects m and n || m, then 
intersects n also.
Given : Three lines  , m, n in the same plane s. t.  intersects m and n || m.
To prove : Lines  and n are intersecting lines.

Proof : Let  and n be non-intersecting lines. Then,  || n.


But, n || m [Given]
  || n and n || m   || m
  and m are non-intersecting lines.
This is a contradiction to the hypothesis that  and m are intersecting lines.
So our supposition is wrong.
Hence,  intersects line n.

Example.5 If lines AB, AC, AD and AE are parallel to a line , then prove that points A, B, C, D and E are
collinear.
Solution. Lines AB, AC, AD and AE are parallel to a line .
To prove : A, B , C, D, E are collinear.
Proof : Since AB, AC, AD and AE are all parallel to a line . Therefore, point A is outside 
and lines AB, AC, AD, AE are drawn through A and each line is parallel to .

For Unacademy Subscription Use “PJLIVE” Code | Join t.me/pjsir42 for Updates
For More Info: “75970 – 84242, 94590 – 43333 / 2222”
But by parallel lines axiom, one and only one line can be drawn through the point A outside it
and parallel to .
This is possible only when A, B, C, D and E all lie on the same line. Hence, A, B, C, D and E
are collinear.

Check Point - A

1. Define :
(i) Axiom (ii) Theorem

2. Define:
(i) Collinear point (ii) Concurrent lines

3. How many lines can be drawn through two points?

4. In the figure below CA = BD. Show that CB = AD.

5. If a line segment can be drawn through 3 points A, B and C, what can you say about these points?

Answers
3. 1 5. points A, B, C are collinear points

For Unacademy Subscription Use “PJLIVE” Code | Join t.me/pjsir42 for Updates
For More Info: “75970 – 84242, 94590 – 43333 / 2222”
BOARD LEVEL EXERCISE

TYPE (I) : VERY SHORT ANSWER TYPE QUESTIONS : [01 MARK EACH]

1. Find the number of dimension, a point has.

2. A pyramid is a solid figure, which plane figure forms the base of it ?

3. It is known that if x + y = 10 then x + y + z = 10 + z. Write the Euclid's axiom that illustrates this statement.

4. Write whether the following statements are True or False? Justify your answer
(i) Euclidean geometry is valid only for curved surfaces.
(ii) The boundaries of the solids are curves.
(iii) The edges of a surface are curves.
(iv) The things which are double of the same thing are equal to one another.
(v) If a quantity B is a part of another quantity A, then A can be written as the sum of B and some third
quantity C.
(vi) The statements that are proved are called axioms.
(vii) "For every line l and for every point P not lying on a given line l, there exists a unique line m
passing through P and parallel to l " is known as Playfair 's axiom.

TYPE (II) : SHORT ANSWER TYPE QUESTIONS : [02 MARKS EACH]


5. Solve the equation a – 15 = 25 and state which axiom do you use here.

6. In figure, we have : AC = XD, C is the mid-point of AB and D is the mid-point of XY. Using an
Euclid's axiom, show that AB = XY.

7. In the figure, we have

1 1
BX = AB , BY = BC and AB = BC. Show that BX = BY.
2 2

8. In the figure, we have 1 =  3 and  2 =  4. Show that  A =  C.

For Unacademy Subscription Use “PJLIVE” Code | Join t.me/pjsir42 for Updates
For More Info: “75970 – 84242, 94590 – 43333 / 2222”
9. In the figure,

(i) AB = BC, M is the mid-point of AB and N is the mid- point of BC. Show that AM = NC.
(ii) BM = BN, M is the mid-point of AB and N is the mid-point of BC. Show that AB = BC.

TYPE (III) : LONG ANSWER TYPE QUESTIONS: [03 MARK EACH]


10. Read the following statement:
An equilateral triangle is a polygon made up of three line segments out of which two line segments are
equal to the third one and all its angles are 60° each.
Define the terms used in this definition which you feel necessary. Are there any undefined terms in
this? Can you justify that all sides and all angles are equal in a equilateral triangle.

11. Study the following statement:


"Two intersecting lines cannot be perpendicular to the same line".
Check whether it is an equivalent version to the Euclid's fifth postulate.

12. Read the following two statements which are taken as axioms:
(i) If two lines intersect each other, then the vertically opposite angles are not equal.
(ii) If a ray stands on a line, then the sum of two adjacent angles so formed is equal to 180°.
Is this system of axioms consistent? Justify your answer.

13. Read the following axioms:


(i) Things which are equal to the same thing are equal to one another.
(ii) If equals are added to equals, the wholes are equal.
(iii) Things which are double of the same thing are equal to one another.
Check whether the given system of axioms is consistent or inconsistent.

For Unacademy Subscription Use “PJLIVE” Code | Join t.me/pjsir42 for Updates
For More Info: “75970 – 84242, 94590 – 43333 / 2222”
EXERCISE – 01

SUBJECTIVE QUESTIONS
Section (A) : Euclid Geometry
A-1. How many lines can pass through :
(i) One point (ii) Two distinct points

A-2. Find the number of points in which two distinct straight lines may intersect.

A-3. A, B and C are three collinear points such that point A lies between B and C. Name all the line
segments determined by these points and write the relation between them.

A-4. If AB is a line and P is a fixed point, outside AB, how many lines can be drawn through P which are :
(i) Parallel to AB (ii) Not parallel to AB.

A-5. Out of the three lines AB, CD and EF, if AB is parallel to EF and CD is also parallel to EF, then what is
the relation between AB and CD.

A-6. If A, B and C three points on a line, and B lies between A and C, then prove that AB + BC = AC.

A-7. In the given figure, if AB = CD ; prove that AC = BD.

A-8. (i) How many lines can be drawn to pass through three given point if they are not collinear ?
(ii) How many line segments can be drawn to pass through two given points ?

A-9. Define
(a) line segment (b) radius of a circle

A-10. Define
(a) square (b) perpendicular lines

1
A-11. If a point C lies between two points A and B such that AC =BC, then prove that AC = AB . Explain
2
by drawing the figure.

A-12. Prove that every line segment has one and only one midpoint.

A-13. L, M, N are three lines in the same plane such that L intersect M and M || N. Show that L also intersects N

A-14. Using Euclid axiom find x, if 3x + 5=17

For Unacademy Subscription Use “PJLIVE” Code | Join t.me/pjsir42 for Updates
For More Info: “75970 – 84242, 94590 – 43333 / 2222”
OBJECTIVE QUESTIONS
Section (A) : Euclid Geometry
A-1. A proof is required for :
(A) Postulate (B) Axiom (C) Theorem (D) Definition

A-2. How many number of lines does pass through two distinct points.
(A) 1 (B) 2 (C) 3 (D) 4

A-3. Which of the following is an example of a geometrical line.


(A) Black Board (B) Sheet of paper
(C) Meeting place of two walls (D) Tip of the sharp pencil

A-4. Given four points such that no three of them are collinear, then the number of lines that can be drawn
through any of the two points is :
(A) 2 lines (B) 4 lines (C) 6 lines (D) 8 lines

A-5. Two planes intersect each other to form a :


(A) plane (B) point (C) straight line (D) angle

A-6. Lines are parallel if they do not intersect. is stated in the form of :
(A) an axiom (B) a definition (C) a postulate (D) a proof

A-7. Select the wrong statement :


(A) Only one line can pass through a single point
(B) Only one line can pass through two distinct points
(C) A line consists of infinite number of points.
(D) If two circles are equal, then their radii are equal

A-8. Number of dimension(s) a surface has :


(A) 0 (B) 1 (C) 2 (D) 3

A-9. The number of line segments determined by three collinear points is :


(A) Two (B) Three (C) Only one (D) Four

A-10. If the point P lies in between M and N and C is midpoint of MP, then :
(A) MC + PN = MN (B) MP +CP = MN (C) MC +CN = MN (D) None of these

VALUE BASED / PRACTICAL BASED QUESTIONS


MARKED QUESTIONS MAY HAVE FOR REVISION QUESTIONS.
1. Let A be a given point and B be some other point. If we draw several lines passing through the point A.
We see that only one of these lines also passes through the point B. Similarly, if we draw several lines
passing through the points, we see that only one of these lines also passes through the point A.
How many lines pass through the (i) points A and B ?
(ii) Which mathematical concept is used to solve the above problem ?
(iii) Which postulate is used in the given statement ?
(iv) Name the line which can be drawn from any point to any other point.

For Unacademy Subscription Use “PJLIVE” Code | Join t.me/pjsir42 for Updates
For More Info: “75970 – 84242, 94590 – 43333 / 2222”
Answer Key

BOARD LEVEL EXERCISE


TYPE (I) : VERY SHORT ANSWER TYPE QUESTIONS : [01 MARK EACH]
1. 0 2. Any polygon 3. Second axiom
4. (i) False (ii) False (iii) False (iv) True (v) True
(vi) False (vii) True

TYPE (II) : SHORT ANSWER TYPE QUESTIONS : [02 MARKS EACH]


5. (using Euclid's second axiom) a = 40

EXERCISE – 01

SUBJECTIVE QUESTIONS
Section (A) : Euclid Geometry
A-1. (i) Infinite (ii) Only one A-2. One
A-3. BA, AC & BC ; BA + AC = BC A-4. (i) Only one (ii) Infinite
A-5. AB || CD A-8. (i) Three lines (ii) one A-14. x = 4

OBJECTIVE QUESTIONS
Section (A) : Euclid Geometry
A-1. (C) A-2. (A) A-3. (C) A-4. (C) A-5. (C)
A-6. (B) A-7. (A) A-8. (C) A-9. (B) A-10. (C)

VALUE BASED / PRACTICAL BASED QUESTIONS


1. (i) Only 1 line
(ii) Euclid Geometry
(iii) Euclid 1 postulate. A straight line may be drawn from any one point to any other point.
(iv) Straight line

For Unacademy Subscription Use “PJLIVE” Code | Join t.me/pjsir42 for Updates
For More Info: “75970 – 84242, 94590 – 43333 / 2222”
2 LINES AND ANGLES

INTRODUCTION

Point
A point is represented by a dot. It has no dimensions like length, breadth or thickness. It has only position.
Points are denoted by capital letters A, B, C, D etc.
A dot made by sharp pencil is a point.

Line
A geometrical line is a set of points that extends endlessly in both the directions i.e., a line has no end points.
It has only length. The arrow heads show that the line goes on endlessly on either side.

Lines are denoted by small letters , m, n, .....

Line segment
A line segment is a line which has end points. In the figure, the part of the line between the points 'A' and 'B',
including 'A' and 'B' is a line-segment.

The line segment AB is represented as AB or segment AB.

Plane
A plane is a set of points. It is a flat surface with length and breadth. A geometrical plane extends endlessly in
all the directions. Small letters are used to denote a plane.

e.g., Surface of a sheet of paper, surface of a wall .

Coplanar points
The points that belong to the same plane are called coplanar points.

Coplanar lines
The lines that lie in the same plane are called coplanar lines.

For Unacademy Subscription Use “PJLIVE” Code | Join t.me/pjsir42 for Updates
For More Info: “75970 – 84242, 94590 – 43333 / 2222”
Space
The universal set of points, lines and planes is called a space. It has no end.

Distance between two points


The distance between two points A and B is the length of the line segment joining them. The distance between
A and B is denoted by d(A, B) or AB.

Midpoint
Given a line segment AB, a point M is said to be the midpoint of AB, if M is
an interior point of AB, such that AM= MB.

Perpendicular bisector
A line '' passing through the mid point 'M' of a line segment AB and perpendicular to AB is called the
perpendicular bisector of the line segment AB .

Collinear points
If three or more points lie on a straight line, then those points are called collinear points.
A, B, C, D, E are collinear.

Non-collinear points
The points which do not lie on the straight line are called non-collinear points.
G, H, I do not lie on the straight line ‘’
Hence, they are non-collinear points.

A. ANGLES AND THEIR TYPES

(a) Angles
An angle is the union of two non-collinear rays with a common initial point. The common initial point
is called the ‘vertex’ of the angle and two rays are called the ‘arms’ of the angles.

Remark : Every angle has a measure and unit of measurement is degree.


One right angle = 90º
1º = 60’ (minutes)
1’ = 60’’ (Seconds)

Angle addition axiom : If X is a point in the interior of BAC, then m BAC = m BAX + m XAC.

For Unacademy Subscription Use “PJLIVE” Code | Join t.me/pjsir42 for Updates
For More Info: “75970 – 84242, 94590 – 43333 / 2222”
(b) Types of Angles :
(i) Right angle : An angle whose measure is 90º is called a right angle.

(ii) Acute angle : An angle whose measure is less than 90º is called an acute angle.
0º < BOA < 900

(iii) Obtuse angle : An angle whose measure is more than 90º but less than 180º is called an obtuse
angle.

90º < AOB < 180º.


(iv) Straight angle : An angle whose measure is 180º is called a straight angle.

(v) Reflex angle : An angle whose measure is more than 180º is called a reflex angle.
180º < AOB < 360º.

(vi) Complementary angles : Two angles, the sum of whose measures is 90º are called
complementary angles.

AOC & BOC are complementary as their sum is 90º.


(vii) Supplementary angles : Two angles, the sum of whose measures is 180º, are called the
supplementary angles.

AOC & BOC are supplementary as their sum is 180º.


(viii) Angle Bisectors : A ray OX is said to be the bisector of AOB , if X is a point in the interior of
AOB, and AOX = BOX.

For Unacademy Subscription Use “PJLIVE” Code | Join t.me/pjsir42 for Updates
For More Info: “75970 – 84242, 94590 – 43333 / 2222”
(ix) Adjacent angles : Two angles are called adjacent angles, if
(I) they have the same vertex,
(II) they have a common arm,
(III) non common arms are on either side of the common arm.

AOX and BOX are adjacent angles, OX is common arm, OA and OB are non common arms
and lies on either side of OX.

(x) Linear pair of angles : Two adjacent angles are said to form a linear pair of angles, if their non
common arms are two opposite rays.
AOC + BOC = 180º.

(xi) Vertically opposite angles : Two angles are called a pair of vertically opposite angles, if their
arms form two pairs of opposite rays.

 AOC &  BOD form a pair of vertically opposite angles. Also  AOD &  BOC form a pair of
vertically opposite angles.
If two lines intersect, then the vertically opposite angles are equal i.e. AOC = BOD and
BOC = AOD.

Solved Examples

Example.1 Two supplementary angles are in ratio 4 : 5, find the angles.


Solution. Let angles are 4x & 5x.
 Angles are supplementary.
So, 4x + 5x = 180º
 9x = 180º
180
 x= = 20 
9
 Angles are 4  20º, 520º  80º & 100º.

Example.2 If an angle differs from its complement by 10º, find the angle.
Solution. Let angle is xº then its complement is 90 – xº.
Now given,
xº – (90 – xº) = 10º
 xº – 90º + xº = 10º
 2xº = 10º + 90º = 100º
100
 x = = 50 .
2
 Required angle is 50º.

For Unacademy Subscription Use “PJLIVE” Code | Join t.me/pjsir42 for Updates
For More Info: “75970 – 84242, 94590 – 43333 / 2222”
Example.3 In figure, OP and OQ bisects BOC and AOC respectively. Prove that POQ = 90º.

Solution.  OP bisects BOC


1
 POC = BOC ...(i)
2
Also, OQ bisects AOC
1
 COQ = AOC ...(ii)
2
 OC stands on AB
 AOC + BOC = 180º [Linear pair]
1 1 1
 AOC + BOC = 180
2 2 2
 COQ + POC = 90º [Using (i) & (ii)]
 POQ = 90º [By angle sum property] Hence Proved.

Example.4 In figure, lines AB, CD and EF intersect at O. Find the measures of AOC, DOE and BOF.

Solution. Given  AOE = 40º &  BOD = 35º


Clearly AOC =  BOD [Vertically opposite angles]
 AOC = 35º
BOF = AOE [Vertically opposite angles]
 BOF = 400
Now, AOB = 180º [Straight angles]
 AOC + COF + BOF = 180º [Angles sum property]
 35º + COF + 40º = 180º
 COF = 180º – 75º = 105º
Now, DOE = COF [Vertically opposite angles]
 DOE = 105º.

Check Point - A

1. Find the supplement of 100º 48.


2. Find the angle such that an angle is equal to its supplement.
3. Find the angle such that an angle is equal to its complement.
4. In the given figure below, find the value of y.

For Unacademy Subscription Use “PJLIVE” Code | Join t.me/pjsir42 for Updates
For More Info: “75970 – 84242, 94590 – 43333 / 2222”
5. In figure PQR is a straight line and PQS : SQR = 7 : 5 . Find SQR

Answers
1. 79º 12 2. 90º 3. 45º 4. 15º 5. 75º

B. ANGLES MADE BY TRANSVERSAL

(a) Parallel Lines


Parallel Lines : Two lines  and m in the same plane are said to be
parallel lines if they do not intersect when produced indefinitely in
either direction and we write  || m which is read as  is parallel to m.
Clearly, when  || m, we have m || .

(b) Parallel Rays


Two rays are parallel if the corresponding lines determined by them
are parallel. In other words, two rays in the same plane are parallel.
If they do not intersect each other even if extended indefinitely
beyond their initial points.
In fig. ray OA || ray PQ.
(c) Parallel segments :
Two segments are parallel if the corresponding lines determined by
them are parallel.
In other words, two segments which are in the same plane and do not
intersect each other even if extended indefinitely in both directions
are said to be parallel.
(d) Angles made by a transversal with two lines
A line which intersects two or more given lines at distinct points is called a transversal to the given lines.
(i) Exterior angles : The angles whose arms do not include the line
segment PQ are called exterior angles. In fig. angles 1, 2, 7 and 8
are exterior angles.
(ii) Interior angles : The angles whose arms include line segment PQ
are called interior angles. In fig. angles 3, 4, 5 and 6 are interior
angles.
(iii) Corresponding angles : A pair of angles in which one arm of
both the angles is on the same side of the transversal and their
other arms are directed in the same sense is called a pair of
corresponding angles.
In fig. 1, 5 ; 2, 6 ; 3, 7 and 4, 8 are four pairs of corresponding angles.
(iv) Alternate interior angles: A pair of angles in which one arm of each of the angles is on opposite
side of the transversal and whose other arm include the segment PQ is called a pair of alternate
interior angles. In fig 3 5 ; 4 and 6 are alternate interior angles.
(v) Alternate exterior angles : A pair of angles in which one arm of each of the angles is on opposite
sides of the transversal and whose other arms are directed in opposite direction and do not include
segment PQ is called a pair of alternate exterior angles. In fig. 2, 8 ; 1 and 7 are alternate
exterior angles.

Remark : Lines in a plane are parallel, if they do not intersect when produced indefinitely in either direction.
The distance between two intersecting lines is zero.
The distance between two parallel lines is the same everywhere and is equal to the perpendicular
distance between them.

For Unacademy Subscription Use “PJLIVE” Code | Join t.me/pjsir42 for Updates
For More Info: “75970 – 84242, 94590 – 43333 / 2222”
(e) Angles made by transversal to two parallel lines
If two parallel lines are intersected by a transversal, then
(i) Pairs of alternate (interior or exterior) angles are equal.
(ii) Pairs of corresponding angles are equal.
(iii) Interior angles on the same side of the transversal are supplementary.
If two non-parallel lines are intersected by transversal then none of (i), (ii) and (iii) hold true.
If two lines are intersected by a transversal, then they are parallel if anyone of the following is true:
(i) Pair of corresponding angles are equal.
(ii) Pair of alternate interior angles are equal.
(iii) Pair of interior angles on the same side of the transversal are supplementary.

Solved Examples

Example.5 In figure if  || m, n || p and  1 = 85º find  2.


Solution.  n || p and m is transversal.
 1 = 3 = 85º [Corresponding angles]
Also  || m & p is transversal.
 2 + 3 = 180º [Co - interior angles]
 2 + 85º = 180º
 2 = 180º – 85º
 2 = 95º.

Example.6 In the given Figure, AB || CD and AC || BD. If EIC = 40º, FDG = 55º, HAB = xº, then the
value of x is

Solution. (i) AIH = EIC (Vertically opp s)


 AIH = 40º
(ii) BDC = FDG (Vertically opp s)
 BDC = 55º
ICE = BDC (Corresponding s)
 ICE = 55º
AIJ = ICE (Corresponding s)
 AIJ = 55º
x = 180º – (AIJ + HIA) {Linear pair}
x = 180º – (55º + 40º)
x = 180º – 95º = 85º

Example.7 In the given figure AB || CD. Find FXE.

Solution. BFE = CEF = 110º [Alternate interior angles]


So, XFE = BFE – BFX
= (110º – 50º) = 60º
CEF + FEX + XED = 180º

For Unacademy Subscription Use “PJLIVE” Code | Join t.me/pjsir42 for Updates
For More Info: “75970 – 84242, 94590 – 43333 / 2222”
 110º + FEX + 30º = 180º
 FEX = 40º
Now, XFE + FEX + FXE = 180º
 60º + 40º + FXE = 180º
 FXE = 80º.

Check Point - B

1. In the following diagram, list out


(i) Pairs of corresponding angles.
(ii) Pairs of alternate angles (Do not produce the segments).

2. In the figure,4 = 4x, 3 = 2.5x + 24º, find the value of x, given that AD || CB.

3. In the given figure find x, if m || n.

4. In the figure, AB || CD || EF, find x and y.

Answers
1. (i) No pair of corresponding angles (ii)  1 &  6,  2 &  5
2. 16º 3. 37.5º 4. x = 105º, y = 75º

C. TRIANGLES

A plane figure bounded by three lines in a plane is called a triangle. Every triangle
have three sides and three angles. If ABC is any triangle then AB, BC & CA are
three sides and A, B and C are three angles.
(a) Types of triangles :
(i) On the basis of sides we have three types of triangle.
Scalene triangle – A triangle in which no two sides are equal is called a scalene triangle.
Isosceles triangle – A triangle having two sides equal is called an isosceles triangle.
Equilateral triangle – A triangle in which all sides are equal is called an equilateral triangle.
(ii) On the basis of angles we have three types :
For Unacademy Subscription Use “PJLIVE” Code | Join t.me/pjsir42 for Updates
For More Info: “75970 – 84242, 94590 – 43333 / 2222”
Right triangle – A triangle in which any one angle is right angle is called right triangle.
Acute triangle – A triangle in which all angles are acute is called an acute triangle.
Obtuse triangle – A triangle in which any one angle is obtuse is called an obtuse triangle.
SOME IMPORTANT THEOREMS :
Theorem : The sum of interior angles of a triangle is 180º.
To Prove : A + B + C = 180º or 1 + 2 + 3 = 180º.
Construction : Through A, draw a line  parallel to BC.
Proof : Since  || BC. Therefore,
2 = 4 [Alternate interior angles]
3 = 5 [Alternate interior angles]
 2 + 3 = 4 + 5
1 + 2 + 3 = 1 + 4 + 5
[Adding 1 both sides] 1 + 2 + 3 = 180º
[ Sum of angles at a point on a line is 180º]
A + B + C = 180º.
Theorem : if the bisectors of angles ABC and ACB of a triangle ABC meet at a point O, then
1
BOC = 90º +  A .
2
Given : A ABC such that the bisectors of ABC and ACB meet at a point O.
1
To Prove : BOC = 90º + A .
2
Proof : In BOC, we have
1 + 2 + BOC = 180º
1 + 2 = 180º – BOC .... (i)
In ABC, we have
 A + B + C = 180º
 A + 21 + 2 2 = 180º
A
 + 1 + 2 = 90º
2
1
 1 + 2 = 90º– A ...(ii)
2
From (i) and (ii)
1
180º – BOC = 90º – A
2
1
BOC = 90º + A . Hence Proved
2
Exterior Angle of a Triangle :
If the side of the triangle is produced, the exterior angle so formed is equal to the sum of two
interior opposite angles.
Given : A triangle ABC. D is a point on BC produced, forming exterior angle 4.
To Prove : 4 = 1 + 2 i.e. ACD = CAB + CBA.
Proof : In triangle ABC, we have
1 + 2 + 3 = 180º ...(i)
Also,
3 + 4 = 180º [ 3 and 4 form a linear pair] ..(ii)
From (i) and (ii), we have
1 + 2 + 3 = 3 + 4
 1 + 2 = 4
Hence, ACD = CAB + CBA.
Corollary : An exterior angle of a triangle is greater than either of the interior opposite angles.

For Unacademy Subscription Use “PJLIVE” Code | Join t.me/pjsir42 for Updates
For More Info: “75970 – 84242, 94590 – 43333 / 2222”
Proof : Let ABC be a triangle whose side BC is produced to form exterior angle A.
Then, 1 + 2 = 4  4 > 1 and 4 > 2
i.e., ACD > CAB and ACD > CBA
Theorem : The sides AB and AC of a ABC are produced to P and Q respectively. If the bisectors
1
of PBC and QCB intersect at O, then BOC = 90º A .
2
Given : A ABC in which sides AB and AC are produced to P and Q respectively.
The bisectors of PBC and QCB intersect at O.
1
To Prove : BOC = 90º – A
2
Proof : Since ABC and CBP form a linear pair.
 ABC + CBP = 180º
 B + 21 = 180º
[BO is the bisector of CBP  CBP = 21]
 21 = 180º – B
1
 1 = 90º – B ...(i)
2
Again, ACB and QCB form a linear pair.
 ACB + QCB = 180º
 C + 22 = 180º
[ OC is the bisector of QCB  QCB = 22]
1
 22 = 180º – C
2
1
 2 = 90º – C ...(ii)
2
In BOC, we have
1 + 2 + BOC = 180º
1 1
 90º – B + 90º – C + BOC = 180º [Using (i) and (ii)]
2 2
1
 180º – (B + C) + BOC = 180º
2
1
 BOC = (B + C)
2
1
 BOC = (180º – A)
2
[  A + B + C = 180º  B + C = 180º – A]
1
 BOC = 90º – A .
2

For Unacademy Subscription Use “PJLIVE” Code | Join t.me/pjsir42 for Updates
For More Info: “75970 – 84242, 94590 – 43333 / 2222”
Solved Examples

Example.8 In figure, If QT ⊥ PR, TQR = 40º and SPR = 30º, find x and y.
Solution. In TQR
TQR + QTR + TRQ = 180º
 40º + 90º + TRQ = 180º
 TRQ = 180º – 130º = 50º
 x = 50º
In PSR, using exterior angle property, we have
PSQ = PRS + RPS
 y = x + 30º
 y = 50º + 30º = 80º.

Example.9 The side BC of a ABC is produced, such that D is on ray BC. The bisector of A meets BC in
L as shown in figure. Prove that ABC + ACD = 2ALC.
Solution. In ABC, we have
ext. ACD = B + A
 ext. ACD = B + 21 ...(i)
[ AL is the bisector of A  A = 21]
 ACD = B + 21
In ABL, we have
ext. ALC = B + BAL
 ext. ALC = B +1
 2ALC = 2B + 21 ...(ii)
[Multiplying both sides by 2]
Subtracting (i) from (ii), we get
2ALC – ACD = B
 ACD + B = 2ALC
 ACD + ABC = 2 ALC.

Example.10 In figure, TQ and TR are the bisectors of Q and R respectively. If QPR = 80º and PRT
= 30º, determine TQR and QTR.
Solution. Since the bisectors of Q and R meet at T.
1
 QTR = 90º + QPR
2
1
 QTR = 90º + (80º)
2
 QTR = 90º + 40º = 130º
In QTR, we have
TQR + QTR + TRQ = 180º
 TQR + 130º + 30º = 180º [ TRQ = PRT = 30º]
 TQR = 20º
Thus, TQR = 20º and QTR = 130º.

For Unacademy Subscription Use “PJLIVE” Code | Join t.me/pjsir42 for Updates
For More Info: “75970 – 84242, 94590 – 43333 / 2222”
Check Point - C

1. The sum of the acute angles of an obtuse triangle is 70º and their difference is 10º. Find the bigger acute
angle.

2. If one angle of a triangle is equal to the sum of the other two, then determine the type of triangle

3. In ABC, 2A = 3B, 5B = 2C then determine the angles of the triangle

4. If one angle of a triangle is equal to half the sum of the other two equal angles, then detrmine type of
the triangle.

5. In the figure if AB = AC, CH = CB and HK || BC then find HCK.

Answers
1. 40º 2. Right angle triangle 3. A = 54º, B = 36º, C = 90º
4. Equilateral triangle 5. 30º

For Unacademy Subscription Use “PJLIVE” Code | Join t.me/pjsir42 for Updates
For More Info: “75970 – 84242, 94590 – 43333 / 2222”
BOARD LEVEL EXERCISE

TYPE (I) : VERY SHORT ANSWER TYPE QUESTIONS : [01 MARK EACH]
1. If two interior angles on the same side of a transversal intersecting two parallel lines are in the ratio 2 : 3,
then the greater of the two angles ?

2. In Figure, if AB || CD || EF, PQ || RS, RQD = 25º and CQP = 60º, then QRS = ?

3. An exterior angle of a triangle is 105º and its two interior opposite angles are equal. What is the
measure of each of these equal angle.

4. In Figure, POQ is a line. Find the value of x is

5. Angles of a triangle are in the ratio 2 : 4 : 3. Find the smallest angle of the triangle ?

TYPE (II) : SHORT ANSWER TYPE QUESTIONS : [02 MARKS EACH]


6. In Figure, AB, CD and EF are three lines concurrent at O. Find the value of y.

7. In Figure, x = y and a = b. Prove that l || n.

8. In Figure, OD is the bisector of AOC, OE is the bisector of BOC and OD ⊥ OE. Show that the
points A, O and B are collinear.
For Unacademy Subscription Use “PJLIVE” Code | Join t.me/pjsir42 for Updates
For More Info: “75970 – 84242, 94590 – 43333 / 2222”
9. AP and BQ are the bisectors of the two alternate interior angles formed by the intersection of a
transversal t with parallel lines l and m (see Figure). Show that AP || BQ.

10. In Figure, BA || ED and BC || EF. Show that ABC + DEF = 180º.

11. In Figure, DE || QR and AP and BP are bisectors of EAB and RBA, respectively. Prove that
APB = 90º.

12. The angles of a triangle are in the ratio 2 : 3 : 4. Find the angles of the triangle.

TYPE (III) : LONG ANSWER TYPE QUESTIONS: [04 MARK EACH]

13. In Figure, m and n are two plane mirrors perpendicular to each other. Show that incident ray CA is
parallel to reflected ray BD.

14. Bisectors of angles B and C of a triangle ABC intersect each other at the point O. Prove that

For Unacademy Subscription Use “PJLIVE” Code | Join t.me/pjsir42 for Updates
For More Info: “75970 – 84242, 94590 – 43333 / 2222”
1
BOC = 90º+ A.
2

15. Bisectors of interior B and exterior  ACD of a  ABC intersect at the point T. Prove that
1
BTC = BAC.
2

16. Prove that through a given point, we can draw only one perpendicular to a given line.

EXERCISE – 01

SUBJECTIVE QUESTIONS
Section (A) : Angles and their types
A-1. In figure, find COD when AOC + BOD = 100º .

A-2. In figure, x : y : z = 5 : 4 : 6. If XOY is a straight line find the values of x, y and z.

A-3. In the given figure, AB is a mirror, PO is the incident ray and OR, the reflected ray. If POR = 112º
find POA.

A-4. The supplement of an angle is one third of itself. Determine the angle and its supplement.
A-5. If two complementary angles are in the ratio 13:5, then find the angles.
A-6. In figure, if x + y = w + z then prove that AOB is a straight line.

A-7. Two complementary angles are such that two times the measure of one is equal to three times measure
of the other. Find the measure of the larger angle.
A-8. Find the complement of each of the following angles :
(i) 36º 40 (ii) 42º 25 36
For Unacademy Subscription Use “PJLIVE” Code | Join t.me/pjsir42 for Updates
For More Info: “75970 – 84242, 94590 – 43333 / 2222”
A-9. Write the supplementary angles of the following angles.
(i) 54º 28 (ii) 98º 35 20

A-10. In figure, POQ is a line. Ray OR is perpendicular to line PQ. OS is another ray lying between rays OP
1
and OR. Prove that ROS = (QOS – POS).
2

Section (B) : Angles made by transversal


B-1. In figure, AB || ED and ABC = 30º, EDC = 70º then find xº

B-2. In figure, AB || CD and CD || EF. Also EA ⊥ AB, if BEF = 55º, find the values of x, y and z.

B-3. If two parallel lines are intersected by a transversal, prove that the bisectors of the interior angles on the
same side of transversal intersect each other at right angles.

B-4. If two parallel lines are intersect by a transversal, prove that the bisectors of the two pairs of interior
angle enclose a rectangle.

B-5. In the diagram, If AB = AD = BD = DC, then find xº.

B-6. In figure, if AB || CD || EF and y : z = 3 : 7, find x.

For Unacademy Subscription Use “PJLIVE” Code | Join t.me/pjsir42 for Updates
For More Info: “75970 – 84242, 94590 – 43333 / 2222”
B-7. In figure, if AB ||CD, EF ⊥ CD and GED = 126º, find AGE, GEF and FGE.

Section (C) : Triangles


C-1.  ABC is an isosceles triangle in which B = C and L & M are points on AB & AC respectively such
that LM || BC. If A = 50º find LMC.
C-2. In figure if AB || DF, AD| | FG, BAC = 65º, ACB = 55º Find FGH.

1
C-3. In figure, PS is the bisector of QPR and PT ⊥ QR. Show that TPS = (Q – R).
2

C-4. In figure, if PQ ⊥ PS, PQ || SR, SQR = 28º and QRT = 65º, then find the value of x and y.

C-5. Prove that sum of all the angles of a triangle is 180º. Also, find the angle of a triangle, if they are in the
ratio 5:6:7.

OBJECTIVE QUESTIONS
Section (A) : Angles and their types
A-1. Two parallel lines have :
(A) a common point (B) two common point
(C) no common point (D) infinite common points

For Unacademy Subscription Use “PJLIVE” Code | Join t.me/pjsir42 for Updates
For More Info: “75970 – 84242, 94590 – 43333 / 2222”
A-2. An angle is 14º more than its complementary angle then angle is :
(A) 38º (B) 52º (C) 50º (D) none of these
A-3. The angle between the bisectors of two adjacent supplementary angles is :
(A) acute angle (B) right angle (C) obtuse angle (D) none of these

A-4. X lies in the interior of BAC. If BAC = 70º and BAX = 42º then XAC =
(A) 28º (B) 29º (C) 27º (D) 30º
A-5. If the supplement of an angle is three times its complement, then angle is :
(A) 40º (B) 35º (C) 50º (D) 45º

4a
A-6. Two angles forms a linear pair whose measures are a & b are such that 2a – 3b = 60º, then =?
5b
8 1 2
(A) 0 (B) (C) (D)
5 2 3
A-7. The complement of (90º – a) is :
(A) – aº (B) 90º + a (C) 90º – a (D) aº

Section (B) : Angles made by transversal


B-1. If two lines intersected by a transversal, then each pair of corresponding angles so formed is :
(A) Equal (B) Complementary (C) Supplementary (D) None of these

B-2. In the given figure, AB || CD, ABF = 45º and CFD = 110º. Then, FDC is :

(A) 25º (B) 45º (C) 35º (D) 30º

B-3. In the given figure PQ || RS, QPR = 70º, ROT = 20º. Then, find the value of x.

(A) 20º (B) 70º (C) 110º (D) 50º

Section (C) : Triangles


C-1. If one angle of triangle is equal to the sum of the other two then triangle is :
(A) acute a triangle (B) obtuse triangle (C) right triangle (D) none

C-2. In the adjoining figure, AD = BD = AC ; CAE = 75º and ACD = xº. Then the value of x is :


(A) 45º (B) 50º (C) 60º (D) 37
2
For Unacademy Subscription Use “PJLIVE” Code | Join t.me/pjsir42 for Updates
For More Info: “75970 – 84242, 94590 – 43333 / 2222”
C-3. In the given figure, PQR is :

(A) 40º (B) 50º (C) 30º (D) 105º

C-4. A triangle can have :


(A) Two right angles (B) Two obtuse angles
(C) All angles more than 60º (D) Two acute angles

C-5. In the given figure, the ratio ABD : ACD is :

(A) 1 : 1 (B) 2 : 1 (C) 1 : 2 (D) 3 : 1

EXERCISE – 02

OBJECTIVE QUESTIONS
1. If one angle of a triangle is 72º and the difference of the other two angles is 12º, Find the ratio of other
two angles.
(A) 4 : 5 (B) 2 : 1 (C) 3 : 4 (D) 5 : 3

2. An angle is 18º less than its complementary angle. The measure of this angle is
(A) 36º (B) 48º (C) 83º (D) 81º

3. In  ABC,  A : B :  C = 2 : 3 : 5, then angle at B is


(A) 54º (B) 126º (C) 136º (D) 64º

4. Which of the following statement is not false ?


(A) If two angles forming a linear pair, then each of these angle is 90º.
(B) Angles forming a linear pair can both be acute angles.
(C) Both of the angles forming a linear pair can be obtuse.
(D) Bisectors of the adjacent angles forming linear pair form a right angle.

5. If two interior angles on the same side of a transversal intersecting two parallel lines are in the ratio 5: 4,
then the greatest angle is
(A) 54º (B) 100º (C) 120º (D) 136º

6. If angle with measure x and y form a complementary pair, then angles with which of the following
measures will form a supplementary pair ?
(A) (x + 47º), (y + 43º) (B) (x – 23º), (y + 23º)
(C) (x – 43º), (y – 47º) (D) No such pair is possible

7. If one angle of a triangle is equal to the sum of the other two angles, then triangle is a/an
(A) Acute angled triangle (B) Obtuse angled triangle
(C) Right angled triangle (D) None of these

For Unacademy Subscription Use “PJLIVE” Code | Join t.me/pjsir42 for Updates
For More Info: “75970 – 84242, 94590 – 43333 / 2222”
8. In figure, lines AB and CD intersect at O. If  AOC +  BOE = 100º and  BOD = 60º, Find  BOE
and reflex  COE respectively.

(A) 40º, 280º (B) 40º, 260º (C) 30, 260º (D) 30º, 250º

9. In the diagram if ABC and PQR are equilateral. The CXY equals

(A) 35º (B) 40º (C) 45º (D) 50º

10. In the figure shown, BEA = 100º. Point F is chosen inside DBEA so that line FA bisects EAB and
line FB bisects EBA. The measure of BFA, is :

(A) 140º (B) 145º (C) 150º (D) 155º

11. The altitudes of triangle are 12, 15 and 20 units. The largest angle in the triangle is :
(A) 75º (B) 90º (C) 120º (D) 135º

12. Lines PS, QT and RU intersect at a common point O, as shown P is joined to Q, R to S and T to U, to
form triangles. The value of P + Q + R + S + T + U is :

(A) 270º (B) 360º (C) 450º (D) 540º

13. Triangle ABC is isosceles with AB = AC. The measure of angle BAD is 30º and AD = AE. The
measure of angle EDC, is :

(A) 5º (B) 10º (C) 15º (D) 20º

For Unacademy Subscription Use “PJLIVE” Code | Join t.me/pjsir42 for Updates
For More Info: “75970 – 84242, 94590 – 43333 / 2222”
14. Given triangle PQR with RS bisecting R, PQ extended to D and n a right angle, then :

1 1 1 1
(A) m = (p – q) (B) m = (p + q) (C) d = (q + p) (D) d = m
2 2 2 2

15. In a rectangle ABCD, as shown in figure, a point P is taken on the side CD such that PC = 9, BP = 15
and AB = 14 then the correct relation between angles of APB is :

(A)  >  >  (B)  >  >  (C)  >  >  (D)  >  > 

16. In the figure, AB = BC = CD = DE = EF = FG = GA, then DAE is equal to :

180
(A) 24º (B) 25º (C) 27º (D)
7
17.

In the above figure (not to scale),  and m are parallel lines and p is a transversal. If c = 45º, then f is
(A) 45º (B) 135º (C) 145º (D) 170º

18.

In the above figure (not to scale), PQ || RS and PR || QS. Each pair of parallel rays is intersecting the
other pair and a, b, c and d are the angles formed. If c = 120º, then a is
(A) 80º (B) 70º (C) 60º (D) 90º

19. If xº is the measure of an angle which is equal to its complement and yº is the measure of the angle
which is equal to its supplement, then xº/yº is
(A) 1 (B) 3 (C) 0.5 (D) 2

For Unacademy Subscription Use “PJLIVE” Code | Join t.me/pjsir42 for Updates
For More Info: “75970 – 84242, 94590 – 43333 / 2222”
20. In the figure (not to scale), ABC is a straight line. If FBE = 60º, CBG = 120º, ABG = xº, ABF
= yº and CBE = zº = 2yº, then (xº + zº) : yº is

(A) 2 : 7 (B) 1 : 1 (C) 1 : 2 (D) 7 : 2

21. In the figure given below (not to scale), MN and OP intersect at right angles.
If (MAB : BAP) = 2 : 1 = (PAC : CAN), then (MAB + PAC) is

(A) 120º (B) 90º (C) 30º (D) 60º

22. In the figure below (not to scale), MN is a straight line. AOB = 120º, DOC = 100º, the relation
between xº and yº is

(A) xº = 2yº (B) yº = 2xº (C) yº = xº (D) xº = 3yº

23. In the figure below, EF and GH are parallel to each other. If GI is the transversal, IGH = yº and
FIG = 3yº then the ratio of the supplement and the complement of yº is

(A) 2 : 1 (B) 3 : 1 (C) 4 : 3 (D) 3 : 2


24. If the figure below, CD is parallel to RS . EMG = 90º, GMD = yº, CME = xº and yº = . Find
2
FNS : FNR.

(A) 1 : 2 (B) 2 : 1 (C) 1 : 1 (D) 3 : 2

25.

In the above figure (not to scale), AB|| CD. If BAE = 25º and DCE = 30º, then find AEC.
(A) 30º (B) 45º (C) 50º (D) 55º

For Unacademy Subscription Use “PJLIVE” Code | Join t.me/pjsir42 for Updates
For More Info: “75970 – 84242, 94590 – 43333 / 2222”
26.

In the above figure (not to scale), x = 125º, y = 135º and AB || CD . Find AOB.
(A) 40º (B) 60º (C) 80º (D) 100º

27. In the figure above (not to scale),  || m and also x : y = 5 : 3. Find the measure of angles (x – 40º) and
(y + 40º) respectively.

(A) 60º, 100º (B) 60º, 60º (C) 100º, 60º (D) 60º, 80º

28. In the figure below (not to scale),  || m || n, p and q are transversals. AB : BC = 2 : 1 and DE = 5 cm.
Find EF

(A) 3 cm (B) 2 cm (C) 5 cm (D) 2.5 cm

29. In the figure below (not to scale), a || b, and x = 130º AB || CD . Find the difference between x and y.

(A) 80º (B) 70º (C) 60º (D) Data inadequate

30. In the figure below (not to scale), AB || DE and BC || EF. If HEF = 60º and DEB = 120º, then find
the ratio of ABG and EBC.

(A) 1 : 1 (B) 2 : 1 (C) 3 : 2 (D) 5 : 4

For Unacademy Subscription Use “PJLIVE” Code | Join t.me/pjsir42 for Updates
For More Info: “75970 – 84242, 94590 – 43333 / 2222”
31.

In the above figure, || m and 1||m1. If ACB = 40º and DBC = 110º, then find QPR.
(A) 60º (B) 70º (C) 80º (D) 110º

32.

In the above ABC, AC || DE and DF || EG. If FDE = 65º then find EGA.
(A) 65º (B) 70º (C) 115º (D) 150º

33.

In the above figure  and m are parallel lines, and p is the transversal intersecting them. If C = 70º,
then G and H respectively are
(A) 70º, 160º (B) 80º, 140º (C) 70º, 110º (D) 60º, l40º

34. In the below figure PQ || RS and PR || QS . Each pair of parallel rays is intersecting the other pair. A,
B, C and D, are the angles formed. If C = 110º, then B =

(A) 90º (B) 110º (C) 100º (D) 120º

35. The ratio between the complement and the supplement of an angle is 1 : 2. The angle is
(A) 90º (B) 180º (C) 135º (D) 0º

36. AFB is a straight line and EF is perpendicular to AB. If x = 2y = z then (x + y) : (y + 2z) is

(A) 1 : 1 (B) 1 : 2 (C) 2 : 1 (D) 3 : 5

For Unacademy Subscription Use “PJLIVE” Code | Join t.me/pjsir42 for Updates
For More Info: “75970 – 84242, 94590 – 43333 / 2222”
37. MN and OP are two straight lines intersecting at R. If ORN = 5/2 (ORM), find NRP : MRP.

(A) 2 : 5 (B) 5 : 2 (C) 1 : 1 (D) 3 : 7

38. AB, EF, and GH are parallel lines, also EB and GF are parallel to each other. If the supplement of the
ABE is 151.5º then the complement of the FGH is

(A) 61.5º (B) 79.8º (C) 90º (D) 28.5º

39. MN and OP are parallel to each other and EF is the transversal. FBO = (3x + 5)º and MAE = (2x)º.
The supplement of PBA : complement of NAB

(A) 2 : 7 (B) 7 : 2 (C) 1 : 1 (D) 1 : 6

40. Both AB and CD are parallel to EF and are perpendicular to GH. GH and IJ are parallel to each other. If KL
: LM = 2 : 3 then ON : OP =

(A) 2 : 3 (B) 3 : 2 (C) 2: 5 (D) 3 : 5

41.

In the above figure, AB|| CD || EF , DPA = 130º, and AQE = 80º. Find PAQ.
(A) 30º (B) 25º (C) 60º (D) 80º

42.

In the above figure, AF || BE and AB || FE. AFE = 40º and BAC = 80º. Find DCE.
(A) 120º (B) 100º (C) 80º (D) 60º

For Unacademy Subscription Use “PJLIVE” Code | Join t.me/pjsir42 for Updates
For More Info: “75970 – 84242, 94590 – 43333 / 2222”
43.

In the above given figure,  || m and x and y are complementary angles. Find x and y.
(A) 50º, 40º (B) 20º, 70º (C) 10º, 80º (D) 30º, 60º

44.

In the above given figure, || m || n. PQ: QR = 4: 3 and LN = 21 cm. Find the lengths of LM and MN
(in cm) respectively.
(A) 9 cm, 15 cm (B) 9cm, 12cm (C) 12 cm, 9 cm (D) 15 cm, 9 cm

45.

In the figure  and m are


(A) Parallel lines (B) Intersecting lines
(C) Perpendicular lines (D) Cannot be determined

46. In the given figure, AB || EC and BC || DF . If EDF = 120º, then find the measure of ABC.

(A) 50º (B) 45º (C) 30º (D) 60º

47. In the given figure (not to scale), APB is a straight line, BPC : APE = 1 : 2 and EPD: BPD = 2 : 3.
If APC = 130º then find the measure of BPD – EPD.

(A) 32º (B) 48º (C) 16º (D) None of the above
48. CD and GH are two straight lines intersecting at O. If the supplement and the complement of the
GOC are in the ratio 31 : 13, then HOC =

(A) 155º (B) 125º (C) 25º (D) 75º

For Unacademy Subscription Use “PJLIVE” Code | Join t.me/pjsir42 for Updates
For More Info: “75970 – 84242, 94590 – 43333 / 2222”
49.

In the above figure || m and 1 || m1. If x = y and PBQ = 60º, then find z.
(A) 60º (B) 80º (C) 100º (D) 120º

50.

In the above figure if  || m and 1 || m1. If BAC = 55º then find QPR.
(A) 60º (B) 75º (C) 55º (D) 125º

51.

In the figure above (not to scale), DE || BC , EF || DC , EFC = 30º and FED = 40º. Find BCF.
(A) 45º (B) 50º (C) 60º (D) 70º

52.

In the figure above (not to scale), EF || GD , AF || EG , AD || BC and DCG = 100º. If CDG = 402, then
find AEF.
(A) 30º (B) 40º (C) 150º (D) 60º

53.

In the figure above (not to scale), AB || PQ and AC || PR . If BAC = 70º and PQR = 50º find PRQ.
(A) 30º (B) 50º (C) 60º (D) 80º

For Unacademy Subscription Use “PJLIVE” Code | Join t.me/pjsir42 for Updates
For More Info: “75970 – 84242, 94590 – 43333 / 2222”
54.

In the figure above (not to scale), AF || BG || PQ , AB|| EF and AD || FG . ABC = 30º and FGE =
70º. Find CDE.
(A) 20º (B) 40º (C) 60º (D) 80º

55.

In the figure above (not to scale), x and y are complementary angles. Then  and m are
(A) parallel lines (B) non parallel lines (C) of equal length (D) None of the above

56.

In the figure above (not to scale), AC, and CD are bisectors of BAD and ADE. If AB and DE are
parallel, then find CAD + CDA.
(A) 80º (B) 100º (C) 90º (D) 70º

57.

In the figure above (not to scale), EFG is a straight segment. ABD = 60º, HDC = 60º, BGE = 80º
and IFE = 30º. If DH bisects BDC and FI bisects DFE then find DBG.
(A) 30º (B) 40º (C) 50º (D) 60º

58.

In the figure above (not to scale), the values of angles a, b, c, d, e, f, g and h are consecutive integers in
increasing order. Find the value of a.
(A) 38 (B) 39 (C) 40 (D) 41

59. There are two angles a and b such that a = b + 15º. If the supplement of 3b is twice the complement of b
+ 15º find the value of a.
(A) 30 (B) 45 (C) 50 (D) 55

For Unacademy Subscription Use “PJLIVE” Code | Join t.me/pjsir42 for Updates
For More Info: “75970 – 84242, 94590 – 43333 / 2222”
60. In the figure below (not to scale), BF is parallel to CG , ECH = 80º and CEJ = 130º. Find A

(A) 80º (B) 30º (C) 50º (D) None of the above

61. In the figure below (not to scale), AC and DE are bisectors of BAD and ADF respectively. If
AC || DE , then find the measure BAD + ADF.

(A) 100º (B) 80º (C) 50º (D) None of the above

62. In the given figure, AB || EF,DE || BC and AC || DF . If EDF· = 70º, then find KLC + LKC.

(A) 70º (B) 110º (C) 120º (D) 80º

63. In the figure below (not to scale), AB || CDand BD || EF . If ABD = DBC = x, BDC = ADB = y,
BCF = s and DCE = t, then which of the following is correct?

(A) x = t and y = s (B) x = s and y = t (C) x = y = t = S (D) None of the above

64. In the figure below (not to scale), AB || CD , AHF = 50º and DEG = 140º. If BHJ = HGE then
find BHJ.

(A) 10º (B) 20º (C) 30º (D) 40º

For Unacademy Subscription Use “PJLIVE” Code | Join t.me/pjsir42 for Updates
For More Info: “75970 – 84242, 94590 – 43333 / 2222”
65. In the figure below (not to scale), AB and CD intersect at the point O. AOC = 100º and OP is the
bisector of AOD. Find the measure of reflex POB.

(A) 200º (B) 190º (C) 210º (D) 220º


66. If the difference between an angle and its supplement Is 100º, then find the ratio of the larger and the
smaller angles.
(A) 3 : 2 (B) 7 : 1 (C) Both (1) and (2) (D) None of the above

67.

In the above figure,  || m, 1 || m1, AB || CD and BC || DE . If PAB = 28º. Then find DEQ.
(A) 28º (B) 56º (C) 84º (D) 52º

68.

In the above figure, AF || ED, CG || AB and AE || HD. If FPD = 40º. Then find AED.
(A) 40º (B) 80º (C) 120º (D) 140º

69.

In the above figure, EF || AG, AB||CD||FG and AG||BC. If EFG = 70º then find BAG – BCD.
(A) 70º (B) 40º (C) 80º (D) 110º
70.

In the above figure (not to scale), E and Fare the mid points of AB and CD respectively. AB || CD, BC||AD
and ADE = 70º, and BCE = 40º. DEC is
(A) 70º (B) 40º (C) 110º (D) 120º

For Unacademy Subscription Use “PJLIVE” Code | Join t.me/pjsir42 for Updates
For More Info: “75970 – 84242, 94590 – 43333 / 2222”
71.

In the figure above (not to scale), AB produced to E, ABC = 3BAC, ACB – 6BAC and AC is
parallel to BD then find DBE and CBD.
(A) 54º, 108º (B) 18º, 108º (C) 54º, 100º (D) 18º, 54º
72.

In the above figure (not to scale), GF||BC,AB||PQ and AC||PR. If x = 40º and y = 110º, then find
QPR.
(A) 70º (B) 80º (C) 60º (D) None of the above

73.

In the above figure (not to scale), AB|| QP || SD and also QR || DP. Find x.
(A) 40º (B) 140º (C) 100º (D) None of the above

74. In the given figure, BDE = xº, FBG = (x + 2)º and BED = (x + 7)º. Find the value of x.

(A) 58 (B) 57 (C) '60 (D) 61

75.

In the above figure, GH || IJ, AC || BD, AB and CD are bisectors of EAH and FCJ. Find ABD +
BDC if BAC = BDC.
(A) 80º (B) 90º (C) 100º (D) 110º

For Unacademy Subscription Use “PJLIVE” Code | Join t.me/pjsir42 for Updates
For More Info: “75970 – 84242, 94590 – 43333 / 2222”
76. The supplement of an angle and the complement of another have the sum equal to half of a complete
angle. If the greater angle is 10º more than the smaller, find the smaller angle.
(A) 40º (B) 35º (C) 45º (D) 30º

77. In the figure below FJ || AD, BE || CD , BGH = 130º and CHG = 120º. If AB || CE , then find
A + D.

(A) 130º (B) 120º (C) 110º (D) 100º

78. In the figure below, m ||  || n and PT || QR. If TUV = x, QRS = y and QVW = z, then which of the
following is necessarily true ?

(A) x > y = z (B) x < y = z


(C) x = y = z (D) Cannot be determined

79. In the figure below, BC || AE and AF || BD. If CBD = x and FAE = y, then find | x – y |.

(A) 10º (B) 20º


(C) Cannot be determined (D) None of the above

80. In the figure below AB|| FC, AE || BC and AF || BD. If F = x, C = y, EAB = k and ABD = p,
then which of the following is correct?

(A) x = k and y = p (B) x = p and y = k


(C) Both (1) and (2) (D) None of the above

81. In the figure below, AD || GE, GB|| FC || ED and BF || CE (not to scale). If ABG = x, GBF = y,
CED = p and CDE = q, then x – y is

(A) p – q (B) q – p
(C) Cannot be determined (D) None of these

For Unacademy Subscription Use “PJLIVE” Code | Join t.me/pjsir42 for Updates
For More Info: “75970 – 84242, 94590 – 43333 / 2222”
82. In the figure below, RT || SQ (not to scale), Q = 80º and T = 30º, find P.

(A) 30º (B) 110º (C) 50º (D) None of the above

83. In the figure below, PQ || TS , Q = 130º and S = 150º. Find the measure of QRM.

(A) 80º (B) 90º (C) 100º (D) 120º

84. In the figure below (not to scale), PQ || TS , reflex QRS = 300º and x – y = 30º. Find the measure of y.

(A) 25º (B) 15º (C) 20º (D) 30º

85. In the figure below (not to scale), MR ⊥ MP, MQ,MN , and MS is bisector of RMQ. If PMN = 50º,
then find the measure of RMS.

(A) 25º (B) 20º (C) 30º (D) 35º

86. If a, b are complementary angles and b, c are supplementary angles. If the average of a and c is 85º,
then find a + b + c.
(A) 200º (B) 220º (C) 210º (D) 240º

EXERCISE – 03

NTSE PROBLEMS (PREVIOUS YEARS)


1. In the given figure mA + mB + mC + mD + mE + mF + mG = _________.
[U.P. NTSE Stage-1 2012]

(A) 360º (B) 500º (C) 520º (D) 540º


2. In a given figure PQ || ST, PQR = 110º, RST = 130º then value of QRS is
For Unacademy Subscription Use “PJLIVE” Code | Join t.me/pjsir42 for Updates
For More Info: “75970 – 84242, 94590 – 43333 / 2222”
[Raj. NTSE Stage-1 2013]

(A) 20º (B) 50º (C) 60º (D) 70º

3. In figure, if QT ⊥ PR, TQR = 40º and SPR = 30º, then y is [Raj. NTSE Stage-1 2014]

(A) 70º (B) 110º (C) 90º (D) 80º

4. In given figure AB || CD, ABE = 120º DCE = 110º and BEC = xº then xº will be.
[U.P. NTSE Stage-1 2014]

(A) 60º (B) 50º (C) 40º (D) 70º

5. In a  PRS, PRS = 120º.A point Q is taken on PR such that PQ = QS and QR = RS then QPS = ........
[Bihar NTSE Stage-1 2014]
(A) 15º (B) 30º (C) 45º (D) 12º

6. In the figure given below, equilateral triangle EDC surmounts square ABCD. Find the angle DEB
represented by x [Delhi NTSE Stage-1 2015]

(A) 60º (B) 15º (C) 30º (D) 45º

7. [Maharashtra NTSE Stage-1 2016]

In the above figure APQ, P −B −C −Q and AB = AC = PB = CQ. Find the angle congruent to PAQ.
(A) ACP (B) ABP (C) APC (D) BAQ

8. In the given figure, AB | | ED and BC | | EF, then the value of ABC + DEF is
[Rajasthan NTSE Stage-I/18]
(A) 90º (B) 180º (C) 120º (D) 360º

9. In ABC, m B = 90º, AB = BC. Then AB : AC = _____ . [Gujarat NTSE Stage-I/18]


(A) 1 : 3 (B) 1 : 2 (C) 1 : 2 (D) 2 : 1
For Unacademy Subscription Use “PJLIVE” Code | Join t.me/pjsir42 for Updates
For More Info: “75970 – 84242, 94590 – 43333 / 2222”
VALUE BASED / PRACTICAL BASED QUESTIONS
MARKED QUESTIONS MAY HAVE FOR REVISION QUESTIONS.
1. Sanjeev was asked to find the sum of the four angles of a quadrilateral. He found the sum of the four
angles as 270º by giving the reason as follow.
Sum of the three angles of a triangle [made up of the three sides = 2 right angles i.e. (3 – 1) right
angles] So, sum of the four angles of quadrilateral [made up of the four sides = (4 – 1) right angles, i.e.
3 right angles = 3  90º = 270º] is 270º His mistake Rajeev pointed out that the sum obtained is
incorrect and found the correct sum
(i) Write the correct solution.
(ii) What value is depicted from this action ?

2. In a mathematical activity, a teacher asks students to divide a circular sheet of radius 13 cm into 5 equal
parts. A student states that each sheet will subtend central angle of 72º, is it correct and which moral
value is depicted ?

For Unacademy Subscription Use “PJLIVE” Code | Join t.me/pjsir42 for Updates
For More Info: “75970 – 84242, 94590 – 43333 / 2222”
Answer Key

BOARD LEVEL EXERCISE


TYPE (I) : VERY SHORT ANSWER TYPE QUESTIONS : [01 MARK EACH]
 1 º
1. 108º 2. 145º 3.  52  4. 20º 5. 40º
 2

TYPE (II) : SHORT ANSWER TYPE QUESTIONS : [02 MARKS EACH]


6. 20º 12. 40º, 60º, 80º

EXERCISE – 01

SUBJECTIVE QUESTIONS
Section (A) : Angles and their types
A-1.  COD = 80º. A-2. x = 60º, y = 48º, z = 72º. A-3.  POA = 34º.

A-4. Angle = 135º and supplement = 45º. A-5. 65, 25 A-7. 54º

A-8. (i) 53º 20 (ii) 47º 34 24

A-9. (i) 125º 32´ (ii) 81º 24º 40´´

Section (B) : Angles made by transversal

B-1. 260º B-2. x = y = 125º, z = 35º. B-5. x = 30º.

B-6. x = 126º. B-7. AGE = 126º, GEF = 36º, FGE = 54º

Section (C) : Triangles

C-1.  LMC = 115º. C-2. FGH = 125º C-4. y = 53º, x = 37º

OBJECTIVE QUESTIONS
Section (A) : Angles and their types
A-1. (C) A-2. (B) A-3. (B) A-4. (A) A-5. (D)
A-6. (B) A-7. (D)

Section (B) : Angles made by transversal


B-1. (D) B-2. (A) B-3. (D)

Section (C) : Triangles


C-1. (C) C-2. (B) C-3. (C) C-4. (D) C-5. (A)

For Unacademy Subscription Use “PJLIVE” Code | Join t.me/pjsir42 for Updates
For More Info: “75970 – 84242, 94590 – 43333 / 2222”
EXERCISE – 02

OBJECTIVE QUESTIONS
Ques. 1 2 3 4 5 6 7 8 9 10 11 12 13 14 15 16 17 18 19 20
Ans. A A A D B A C A B A B B C B A D B C C D
Ques. 21 22 23 24 25 26 27 28 29 30 31 32 33 34 35 36 37 38 39 40
Ans. A B B A D C A D A B B A C B D D A A B A
Ques. 41 42 43 44 45 46 47 48 49 50 51 52 53 54 55 56 57 58 59 60
Ans. A D C C B D C A D C D B C D B C B D B B
Ques. 61 62 63 64 65 66 67 68 69 70 71 72 73 74 75 76 77 78 79 80
Ans. A B C B D D A D B C B A B B B A C C D B
Ques. 81 82 83 84 85 86
Ans. B C C B A B

EXERCISE – 03

Ques. 1 2 3 4 5 6 7 8 9
Ans. D C D B A D B B C

For Unacademy Subscription Use “PJLIVE” Code | Join t.me/pjsir42 for Updates
For More Info: “75970 – 84242, 94590 – 43333 / 2222”
3 TRIANGLES
INTRODUCTION

Scalene triangle
(i) A triangle in which none of the three sides are equal is called a scalene triangle
(ii) All the three angles are also different

abc
Isosceles triangle
(i) A triangle in which at least two sides are equal is called an isosceles
triangle.
(ii) In this triangle, the angles opposite to the congruent sides are also equal
(iii) 2 medians, 2 altitudes equal.
(iv) Internal bisectors of 2 angles are equal.
(v) Bisector of vertical angle bisects the base and perpendicular to the base.
(vi) May be acute, obtuse or right-angled triangle.
AB = AC
B = C
Equilateral triangle
(i) A triangle in which all the three sides are equal is called an equilateral triangle.
(ii) In this triangle each angle is congruent and equal to 60º
(iii) Always acute angled.
(iv) Incentre, circumcentre, orthocentre and centroid coincide.

AB = BC = AC
A = B = C = 60º
Isosceles right-angled triangle
(i) 2 sides are equal
(ii) Angle included by the equal sides is 90º.
(iii) Side opposite to 90º is hypotenuse and is the greatest side.
(iv) Median to the hypotenuse is half of the hypotenuse.

For Unacademy Subscription Use “PJLIVE” Code | Join t.me/pjsir42 for Updates
For More Info: “75970 – 84242, 94590 – 43333 / 2222”
Fundamental properties of triangles
1. Sum of any two sides is always greater than the third side.
2. The difference of any two sides is always less than the third side.
3. Greater angle has a greater side opposite to it and smaller angle has a smaller
side opposite to it i.e., if two sides of triangle are not congruent then the angle
opposite to the greater side is greater.
4. Let a, band c be the three sides of a AABC and cis the largest side, then
(i) if c2 < a2 + b2, the triangle is acute angle triangle
(ii) if c2 = a2 + b2, the triangle is right angled triangle
(iii) If c2 > a2 + b2, the triangle is obtuse angle triangle
5. The sum of all the three interior angles is always 180º i.e., CAB + ABC + BCA = 180º

6. The sum of three (ordered) exterior angles of a triangle is 360º.

Fig.(i) Fig.(ii)
In fig (i) : FAC + ECB + DBA = 360º
In fig (ii) : FAB + DBC + ECA = 360º
7. A triangle must have at least two acute angles.
8. In a triangle, the measure of an exterior angle equals the sum of the measures of the interior opposite angles.
9. The measure of an exterior angle of a triangle is greater than the measure of each of the opposite
interior angles.

A. CONGRUENT TRIANGLES

(a) Congruent figures


The figures are called congruent if they have same shape and same size. In other words, two figures are
called congruent if they are having equal length, width and height.

Fig.(i) Fig.(ii)
In the above figures {fig.(i) and fig.(ii)} both are equal in length, width and height, so these are
congruent figures.

(b) Congruent Triangles


Two triangles are congruent if and only if one of
them can be made to superimposed on the other,
so as to cover it exactly.
If two triangles ABC and DEF are congruent
then A = D, B = E, C = F and AB =
DE, BC = EF, AC = DF.
If two ABC & DEF are congruent then we write  ABC   DEF, we cannot write as  ABC
  DFE or  ABC   EDF.
Hence, we can say that two triangles are congruent if and only if there exists a one-one correspondence
between their vertices such that the corresponding sides and the corresponding angles of the two
triangles are equal.
For Unacademy Subscription Use “PJLIVE” Code | Join t.me/pjsir42 for Updates
For More Info: “75970 – 84242, 94590 – 43333 / 2222”
(c) Sufficient Conditions for Congruence of two Triangles :
(i) SAS Congruence Criterion :

Two triangles are congruent if two sides and the included angle of one are equal to the
corresponding sides and the included angle of the other triangle.
(ii) ASA Congruence Criterion :

Two triangles are congruent if two angles and the included side of one triangle are equal to the
corresponding two angles and the included side of the other triangle.
(iii) AAS Congruence Criterion :

If any two angles and a non-included side of one triangle are equal to the corresponding angles and
side of another triangle, then the two triangles are congruent.
(iv) SSS Congruence Criterion :

Two triangles are congruent if the three sides of one triangle are equal to the corresponding three
sides of the other triangle.
(v) RHS Congruence Criterion :

Two right triangles are congruent if the hypotenuse and one side of one triangle are respectively
equal to the hypotenuse and one side of the other triangle.

Remark : If two triangles are congruent then their corresponding sides and angles are also congruent by
CPCT (corresponding parts of congruent triangles are also congruent).
Theorem : Angles opposite to equal sides of an isosceles triangle are equal.
Given : ABC in which AB = AC.
To Prove : B = C.
Construction : We draw the bisector AD of A which meets BC in D.
Proof : In ABD and ACD we have
AB = AC [Given]
BAD = CAD [ AD is bisector of A]
And, AD = AD [Common side]

For Unacademy Subscription Use “PJLIVE” Code | Join t.me/pjsir42 for Updates
For More Info: “75970 – 84242, 94590 – 43333 / 2222”
 By SAS criterion of congruence, we have
 ABD   ACD
 B = C [By CPCT] Hence Proved.
Converse : If two angles of a triangle are equal, then sides opposite to them are also equal.
Theorem : If the bisector of the vertical angle bisects the base of the triangle, then the triangle is isosceles.
Given : A ABC in which AD is the bisector of A meeting BC in D such that BD = CD.
To Prove : ABC is an isosceles triangle.
Construction : We produce AD to E such that AD = DE and join EC.
Proof : In ADB and EDC we have
AD = DE [By construction]
ADB = CDE [Vertically opposite angles]
BD = DC [Given]
 By SAS criterion of congruence, we get
 ADB   EDC
 AB = EC and, BAD = CED [By CPCT]
But, BAD = CAD
 CAD = CED [ AD is the bisector of A]
 AC = EC
[Sides opposite to equal angles are equal]
 AC = AB. [By equation (i)] Hence Proved.

Solved Examples

Example.1 Prove that measure of each angle of an equilateral triangle is 60º.


Solution. Let ABC be an equilateral triangle, then we have
AB = BC = CA ... (i)
 AB = BC
 C = A ... (ii)
[Angles opposite to equal sides are equal]
Also, BC = CA
 A = B ... (iii)
[Angles opposite to equal sides]
By (ii) & (iii) we get A = B = C
Now in ABC
A + B + C = 1800
 3A = 1800 [A = B = C]
 A = 600 = B = C. Hence Proved.
Example.2 If D is the mid-point of the hypotenuse AC of a right triangle ABC, prove that BD = AC.
Solution. Given : ABC is a right triangle such that B = 900 and D is midpoint of AC.
1
To prove : BD = AC.
2
Construction : Produce BD to E such that BD = DE and join EC.
Proof : In ADB and CDE
AD = DC [Given]
BD = DE [By construction]
And, ADB = CDE [Vertically opposite angles]
 By SAS criterion of congruence, we have
 ADB   CDE
 EC = AB and CED = ABD ... (i) [By CPCT]]
But CED & ABD are alternate interior angles
 CE || AB
 ABC + ECB = 180º [Consecutive interior angles]
 90 + ECB = 180º
 ECB = 90º.
Now, In ABC & ECB we have
For Unacademy Subscription Use “PJLIVE” Code | Join t.me/pjsir42 for Updates
For More Info: “75970 – 84242, 94590 – 43333 / 2222”
AB = EC [By (i)]
BC = BC [Common]
And, ABC = ECB = 90º
 By SAS criterion of congruence
 ABC   ECB
 AC = EB [By CPCT]
1 1
 AC = EB
2 2
1
 BD = AC. Hence Proved.
2
Example.3 In a right angled triangle, one acute angle is double the other. Prove that the hypotenuse is
double the smallest side.
Solution. Given : ABC is a right triangle such that B = 900 and ACB = 2CAB.
To Prove : AC = 2BC.
Construction : Produce CB to D such that BD = CB and join AD.
Proof : In ABD and ABC we have
BD = BC [By construction]
AB = AB [Common]
ABD = ABC = 900
 By SAS criterion of congruence, we get
 ABD   ABC
 AD = AC and DAB = CAB [By cpct]
 AD = AC and DAB = x [CAB = x]
Now, DAC = DAB + CAB = x + x = 2x
 DAC = ACD
 DC = AD [Side Opposite to equal angles]
 2BC = AD [DC = 2BC]
 2BC = AC [AD = AC] Hence Proved.
Example.4 In figure, two sides AB and BC and the median AM of a ABC are respectively equal to sides
DE and EF and the median DN of DEF. Prove that  ABC   DEF.

Solution.  AM and DN are medians of ABC & DEF respectively


 BM = MC & EN = NF
1 1
 BM = BC & EN = EF
2 2
But, BC = EF BM = EN ... (i)
In ABM & DEN we have
AB = DE [Given]
AM = DN [Given]
BM = EN [By (i)]
 By SSS criterion of congruence we have
 ABM   DEN
 B = E ... (ii) [By CPCT]
Now, In ABC & DEF
AB = DE [Given]
B = E [By (ii)]
BC = EF [Given]
 By SAS criterion of congruence, we get
 ABC   DEF Hence Proved.
For Unacademy Subscription Use “PJLIVE” Code | Join t.me/pjsir42 for Updates
For More Info: “75970 – 84242, 94590 – 43333 / 2222”
Example.5 Prove that the medians of an equilateral triangle are equal.
Solution. Given : A ABC in which AB = BC = AC, and AD, BE and CF are its medians.
To prove : AD = BE = CF.
Prove : In ADC and BEA, we have :
1 1
DC = EA [BC = AC  BC = AC]
2 2
ACD = BAE [Each equal to 60º]
AC = AB [Given]
 ADC  BEA [SAS criteria]
So, AD = BE [By CPCT]
Similarly, BE = CF
Hence, AD = BE = CF.

Example.6 In the given figure, AY ⊥ ZY and BY ⊥ XY such that AY = ZY and BY = XY. Prove that AB = ZX.
Solution. BYX = 90º and AYZ = 90º.
BYX = AYZ
 BYX + AYX = AYZ + AYX
 AYB = ZYX
Now, in AYB and  ZYX
AY = ZY [Given]
BY = XY [given]
AYB = ZYX [proved above]
 AYB  ZYX [By SAS congruency]
Hence, AB = ZX [By CPCT]
Example.7 In the given figure, AB = AC and OB = OC. Prove that ABO = ACO.
Solution. AB = AC
So B = C ...(i)
and OB = OC
 OBC = OCB ....(ii)
Subtract equation (ii) from (i)
 B – OBC = C – OCB
ABO = ACO.
Example.8 In the adjoining figure, X and Y are respectively two points on equal sides AB and AC of
ABC such that AX = AY. Prove that CX = BY.
Solution. In AXC and AYB
AX = AY [Given]
AC = AB [Given]
A = A [Common]
So, by SAS congruency
AXC  AYB
So, by CPCT
CX = BY.
Example.9 In the given figure, if x = y and AB = CB, then prove that AE = CD.
Solution. In ABE and CDB
AB = CB [Given]
ABE = CBD [Common]
and AEB = CDB
[ xº = yº  180º – xº = 180º – yº  CDB = AEB]
So, by AAS Congruency
ABE  CBD
 By CPCT, AE = CD.

For Unacademy Subscription Use “PJLIVE” Code | Join t.me/pjsir42 for Updates
For More Info: “75970 – 84242, 94590 – 43333 / 2222”
Example.10 In ABC, D is the midpoint of BC. If DL ⊥ AB and DM ⊥ AC such that DL = DM, prove that
AB = AC.
Solution. In BDL and DMC
DL = DM [Given]
DLB = DMC [Each 90º]
BD = DC [D is the midpoint of BC]
 DLB  DMC [By RHS congruency]
So, by CPCT
B = C
Hence, AB = AC.

Example.11 In the given figure, ABC is an equilateral triangle; PQ || AC and AC is produced to R such that
CR = BP. Prove that QR bisects PC.
Solution. Let QR intersect PC at M.
 BPQ = BCA [Alternate interior angles PQ ||AR]
So, B = BPQ = BQP = 60º
 BQP is an equilateral triangle.
So, PQ = BP = CR.
In PMQ and CMR
QMP = RMC [Vertically opposite angles]
MQP = MRC [Alternate angles]
PQ = CR [CR = BP and BP = PQ]
So, PMQ  CMR [By AAS congruency]
 By CPCT
PM = MC.
Hence, QR bisects PC.

Check Point - A

1. In the following example the data is marked on the diagrams. Write criterion of congruence that
supports your conclusion. If more than one criterion is involved state all.

(i) (ii) (iii)

2. Given AB = 4 cm, DC = 4 cm, AD = BC. Prove  ABC   ADC.

3. Given AD bisects BC and AB = AC. Prove  ADB   ADC.

4. Given AB = CF, EF = BD and AFE = DBC Prove AFE  CBD.

For Unacademy Subscription Use “PJLIVE” Code | Join t.me/pjsir42 for Updates
For More Info: “75970 – 84242, 94590 – 43333 / 2222”
5. Given P = R, PQ = RQ. Prove  PQT   RQS

6. Given AB = AC, AD = AE, BD = CE. Prove  BAE   CAD

7. Through any point on the bisector of an angle, a straight line is drawn parallel to either arm of an angle.
Prove that the triangle so formed is isosceles

8. In a quadrilateral ABCD, AB || DC. The bisectors of angles D and C meet at E. Prove that AB = AD + BC.

Answers
1. (i) SAS (ii) ASA (iii) RHS

B. SOME INEQUALITY RELATIONS IN A TRIANGLE

Theorem : If two sides of a triangle are unequal, the longer side has greater angle opposite to it.
Given : A ABC in which AC > AB.
To Prove : ABC > ACB.
Construction : Mark a point D on AC such that AB = AD. Join BD.
Proof : In ABD, we have
AB = AD [By construction]
 1 = 2 ...(i)
[Angle opposite to equal sides are equal] Now, consider BCD. We find that 2, is the exterior angle
of BCD and an exterior angle is always greater than interior opposite angle. Therefore,
2 > DCB
 2 > ACB ....(ii) [ ACB = DCB]
From (i) and (ii), we have
1 = 2 and 2 > ACB
 1 > ACB ....(iii)
But, 1 is a part of ABC.
 ABC > 1 ....(iv)
From (iii) and (iv), we get
ABC > ACB.

For Unacademy Subscription Use “PJLIVE” Code | Join t.me/pjsir42 for Updates
For More Info: “75970 – 84242, 94590 – 43333 / 2222”
Theorem : The sum of any two sides of a triangle is greater than the third side.
Given : A ABC.
To Prove : AB + AC > BC, AB + BC > AC and BC + AC > AB.
Construction : Produce side BA to D such that AD = AC. Join CD.
Proof : In ACD, we have
AC = AD [By construction]
 ADC = ACD
[Angles opposite to equal sides are equal]
 ACD = ADC
 BCA + ACD > ADC [ BCA + ACD > ACD]
 BCD > ADC
 BCD > BDC [ ADC = BDC]
 BD > BC [ Side opposite to greater angle is larger]
 BA + AD > BC
 BA + AC > BC [ AC = AD (By construction)]
 AB + AC > BC
Thus, AB + AC > BC
Similarly, AB + BC > AC and BC + AC > AB.
Theorem : Of all the line segments that can be drawn to a given line, from a point, not lying on it,
the perpendicular line segment is the shortest.
Given : A straight line and a point P not lying on . PM ⊥  and N is any point on other than M.
To Prove : PM < PN.
Proof : In PMN, we have
M = 90º
 N < 90º [ M = 90º  MPN + PNM = 90º
 P + N = 90º  N < 90º]
 N < M
 PM < PN [Side opposite to greater angle is larger]
Thus, PM < PN.
Hence, PM is the shortest of all line segments from P to .
Theorem : Prove that the difference between any two sides of a triangle is less than its third side.
Given : A ABC.
To Prove :
(i) AC – AB < BC
(ii) BC – AC < AB
(iii) BC – AB < AC
Construction : Let AC > AB. Then, along AC, set off AD = AB. Join BD.
Proof : AB = AD  1 = 2.
Side CD of BCD has been produced to A.
 2 > 4 [ exterior angle > each interior opposite angle]
Again, side AD of ABD has been produced to C.
 3 > 1 [ exterior angle > each interior opposite angle]
Consequently, 3 > 2 [ 1 = 2]
Now, 3 > 2 and 2 > 4  3 > 4.
 BC > CD [side opposite to greater angle is longer]
 CD < BC
 AC – AD < BC
 AC – AB < BC [ AD = AB]
Hence, AC – AB < BC.
Similarly, BC – AC < AB and BC – AB < AC.

For Unacademy Subscription Use “PJLIVE” Code | Join t.me/pjsir42 for Updates
For More Info: “75970 – 84242, 94590 – 43333 / 2222”
Solved Examples

Example.12 Prove that any two sides of the triangle are together greater than twice the median drawn to the
third side.
Solution. Given : ABC and AD is the median.
To prove : AB + AC > 2AD
Construction : Produce AD to E such that AD = DE. Join EC.
Proof : In ADB and CDE
AD = DE [By construction]
BD = DC [AD is the median]
 ADB =  CDE [Vertically opposite angles]
ADB  CDE [By SAS congruency]
So, by CPCT
AB = EC
In AEC
AC + EC > 2AD. [Sum of two sides of a triangle is always greater than the third side]
So, AC + AB > 2AD. [As EC = AB].

Example.13 In figure, PQ = PR, show that PS > PQ.


Solution. In PQR
 PQ = PR
 PRQ = PQR ... (i)
In PSQ, SQ is produced to R
 Ext. PQR > PSQ ... (ii)
 PRQ > PSQ [From equation (i) and (ii)]
 PRS > PSR
 PS > PR [Side opposite to greater angle is larger]
But, PR = PQ
 PS > PQ Hence Proved.

Example.14 In figure, T is a point on side QR of PQR and S is a point such that RT = ST. Prove that PQ +
PR > QS
Solution. In PQR we have
PQ + PR > QR
 PQ + PR > QT + TR
 PQ + PR > QT + ST ... (i) [ RT = ST] S T
In QST
QT + ST > SQ ... (ii)
From (i) & (ii)
 PQ + PR > SQ Hence Proved.

Check Point - B

1. In the figure, compare BCD with A

2. If  ABC   CDE, compare FED with B

For Unacademy Subscription Use “PJLIVE” Code | Join t.me/pjsir42 for Updates
For More Info: “75970 – 84242, 94590 – 43333 / 2222”
1
3. If AB = BC and B = A , prove that AB > AC?
2

4. List all sides of  ABC in the increasing order of their length.

5. List the angles of the triangle ABC in the increasing order of their magnitude.

Answers
1. BCD > A (Exterior angle of a triangle is greater than either of the two interior opposite angles).
2. FED > B 4. CA < BC < AB 5. B < A < C

For Unacademy Subscription Use “PJLIVE” Code | Join t.me/pjsir42 for Updates
For More Info: “75970 – 84242, 94590 – 43333 / 2222”
BOARD LEVEL EXERCISE

TYPE (I) : VERY SHORT ANSWER TYPE QUESTIONS : [01 MARK EACH]
1. In  ABC, AB = AC and  B = 50º. Then find  C

2. In PQR,  R =  P and QR = 4cm and PR = 5 cm. Find the length of PQ.

3. It is given that ABC  FDE and AB = 5 cm, B = 40º and  A = 80º. Then find DF ?

4. In  PQR, if  R >  Q, then which is longer between PQ and PR ?

5. In triangles ABC and DEF, AB = FD and  A =  D. The two triangles will be congruent by SAS
axiom then find the side corresponding to AC.

TYPE (II) : SHORT ANSWER TYPE QUESTIONS : [02 MARKS EACH]


6. In triangles ABC and PQR, A = Q and B = R. Which side of PQR should be equal to side AB
of ABC so that the two triangles are congruent? Give reason for your answer.

7. In Figure, PQ = PR and Q = R. Prove that PQS  PRT.

8. In Figure, two lines AB and CD intersect each other at the point O such that BC || DA and BC = DA.
Show that O is the mid- point of both the line-segments AB and CD.

9. In Figure, PQ > PR and QS and RS are the bisectors of  Q and  R, respectively. Show that SQ > SR.

10. ABC is an isosceles triangle with AB = AC and BD and CE are its two medians. Show that BD = CE.

11. CDE is an equilateral triangle formed on a side CD of a square ABCD (according to figure). Show that
 ADE   BCE.

For Unacademy Subscription Use “PJLIVE” Code | Join t.me/pjsir42 for Updates
For More Info: “75970 – 84242, 94590 – 43333 / 2222”
12. D is any point on side AC of a  ABC with AB = AC. Show that CD < BD.

13. In Figure, l || m and M is the mid-point of a line segment AB. Show that M is also the mid-point of any
line segment CD, having its end points on l and m, respectively.

14. In Figure, AD is the bisector of BAC. Prove that AB > BD.

TYPE (III) : LONG ANSWER TYPE QUESTIONS: [03 MARK EACH]


15. In Figure, ABC is a right triangle and right angled at B such that  BCA = 2  BAC. Show that
hypotenuse AC = 2 BC.

16. Prove that if in two triangles two angles and the included side of one triangle are equal to two angles
and the included side of the other triangle, then the two triangles are congruent.

17. If the bisector of an angle of a triangle also bisects the opposite side, prove that the triangle is isosceles.

18. S is any point in the interior of  PQR. Show that SQ + SR < PQ + PR.

19. ABCD is a quadrilateral in which AB = BC and AD = CD. Show that BD bisects both the angles ABC
and ADC.

20. O is a point in the interior of a square ABCD such that OAB is an equilateral triangle. Show that OCD
is an isosceles triangle.

21. ABC is an isosceles triangle in which AC = BC. AD and BE are respectively two altitudes to sides BC
and AC. Prove that AE = BD.
For Unacademy Subscription Use “PJLIVE” Code | Join t.me/pjsir42 for Updates
For More Info: “75970 – 84242, 94590 – 43333 / 2222”
22. In a right triangle, prove that the line-segment joining the mid-point of the hypotenuse to the opposite
vertex is half the hypotenuse

23. ABCD is quadrilateral such that AB = AD and CB = CD. Prove that AC is the perpendicular bisector of BD.

TYPE (IV): VERY LONG ANSWER TYPE QUESTIONS [04 MARK EACH]
24. The image of an object placed at a point A before a plane mirror LM is seen at the point B by an
observer at D as shown in Figure. Prove that the image is as far behind the mirror as the object is in
front of the mirror.

25. Show that in a quadrilateral ABCD, AB + BC + CD + DA < 2 (BD + AC)

26. Line segment joining the midpoints M and N of parallel sides AB and DC, respectively of a trapezium
ABCD is perpendicular to both the sides AB and DC. Prove that AD = BC.

27. ABC is a right triangle such that AB = AC and bisector of angle C intersects the side AB at D. Prove
that AC + AD = BC.

28. Prove that in a triangle, other than an equilateral triangle, angle opposite the longest side is greater than
2
of a right angle.
3

EXERCISE – 01

SUBJECTIVE QUESTIONS
Section (A) : Congruent triangles
A-1. In figure, it is given that AB = CF, EF = BD and AFE = CBD. Prove that AFE  CBD.

A-2. In figure, it is given that A = C and AB = BC. Prove that ABD  CBE.

A-3. In the figure, AB = AC = AD, prove that BCD is a right angle.

For Unacademy Subscription Use “PJLIVE” Code | Join t.me/pjsir42 for Updates
For More Info: “75970 – 84242, 94590 – 43333 / 2222”
A-4. ABC is a triangle and D is the mid point of BC. The perpendiculars from D to AB and AC are equal.
Prove that the triangle is isosceles.

A-5. In figure AC = BC, DCA = ECB and DBC = EAC. Prove that triangles DBC and EAC are
congruent, and hence DC = EC and BD = AE.

A-6. In figure, BD and CE are altitudes. If BE = CD, prove that BD = CE.

A-7. In figure,  ABC is a right angled triangle at B. ADEC and BCFG are squares. Prove that AF = BE.

A-8. ABC and DBC are two isosceles triangle on the same base BC, and vertices A and D are on the same
side of BC. If AD is extended to intersect BC at P, show that
(i) ABD  ACD (ii) ABP  ACP
(iii) AP bisects A as well as D (iv) AP is the perpendicular bisector of BC.

Section (B) : Some Inequalities in triangles


B-1. If D is any point on the base BC produced, of an isosceles triangle ABC, prove that AD > AB.

B-2. In PQR, S is any point on the side QR. Show that PQ + QR + RP > 2 PS.

B-3. Prove that the perimeter of a triangle is greater than the sum of its three medians.

B-4. O is any point in the interior of a triangle ABC. Prove that :


(i) AB + AC > OB + OC (ii) AB + BC + CA > OA + OB + OC
1
(iii) OA + OB + OC > (AB + BC + AC).
2
B-5. PQRS is a quadrilateral in which diagonal PR and QS intersect in O. Show that :
(i) PQ + QR + RS + SP > PR + QS (ii) PQ + QR + RS + SP < 2( PR + QS)

For Unacademy Subscription Use “PJLIVE” Code | Join t.me/pjsir42 for Updates
For More Info: “75970 – 84242, 94590 – 43333 / 2222”
B-6. Prove that the sum of three altitudes of a triangle is less than the sum of three sides of the triangle.

OBJECTIVE QUESTIONS
Section (A) : Congruent Triangles
A-1. If the three altitudes of a  are equal then triangle is :
(A) isosceles (B) equilateral (C) right angled (D) none

A-2. ABCD is a square and P, Q, R are points on AB, BC and CD respectively such that AP = BQ = CR and
PQR = 90º, then QPR :
(A) 45º (B) 50º (C) 60º (D) 70º

A-3. In a XYZ, LM || YZ and bisectors YN and ZN of Y & Z respectively meet at N on LM. Then
YL + ZM =
(A) YZ (B) XY (C) XZ (D) LM

A-4. In AOC and XYZ, A = X, AO = XZ, AC = XY, then by which congruence rule is AOC  XZY
(A) SAS (B) ASA (C) SSS (D) RHS

A-5. Two equilateral triangles are congruent when :


(A) their angles are equal (B) their sides are equal
(C) their sides are proportional (D) their areas are proportional

A-6. In the given figure, if AB = AC and BD = DC.  ABD and  ACD are congruent by which criterion.

(A) SSS (B) ASA (C) SAS (D) RHS

A-7. In the given figure, AD is the median, then BAD is :

(A) 55º (B) 50º (C) 100º (D) 40º

A-8.  ABC  PQR. If AB = 5cm, B = 40º and A = 80º, then which of the following is true ?
(A) QP = 5 cm,  P = 60º (B) QP = 5cm , R = 60º
(C) QR = 5 cm, R= 80º (D) QR = 5cm, Q = 40º

For Unacademy Subscription Use “PJLIVE” Code | Join t.me/pjsir42 for Updates
For More Info: “75970 – 84242, 94590 – 43333 / 2222”
Section (B) : Some inequalities in triangles
B-1. In a PQR, PS is bisector of P and Q = 70º R = 30º, then :
(A) QS > PQ > PR (B) QS < PQ < PR (C) PQ > QS > SR (D) PQ < QS < SR

B-2. If D is any point on the side BC of a ABC, then :


(A) AB + BC + CA > 2AD (B) AB + BC + CA < 2AD
(C) AB + BC + CA > 3 AD (D) None

B-3. If length of the largest side of a triangle is 12 cm, then other two sides of triangle can be :
(A) 4.8 cm, 8.2 cm (B) 3.2 cm, 7.8 cm (C) 6.4 cm, 2.8 cm (D) 7.6 cm, 3.4 cm

B-4. It is not possible to construct a triangle when its sides are :


(A) 8.3 cm, 3.4 cm, 6.1 cm (B) 5.4 cm, 2.3 cm, 3.1 cm
(C) 6 cm, 7 cm, 10 cm (D) 3cm, 5 cm, 5cm

B-5. P is point on side BC of ABC such that AP bisects BAC. Then :


(A) BP = CP (B) BA = BP (C) BP > BA (D) CP < CA

B-6. In the given figure, which of the following statement is true ?

(A) B = C (B) B is the greatest angle in triangle


(C) B is the smallest angle in triangle (D) A is the smallest angle in triangle

EXERCISE – 02

OBJECTIVE QUESTIONS
1. A square board side 10 centimeters, standing vertically, is tilted to the left so that the bottom-right
corner is raised 6 centimeters from the ground.

By what distance is the top-left corner lowered from its original position ?
(A) 1 cm (B) 2 cm (C) 3 cm (D) 0.5 cm

2. In a right-angled triangle ABC, P is midpoint of AC. Which one is true ?


AC AC
(A) PA = (B) PB = (C) PA = PB (D) All of these
2 2
3. In the given figure AC = CB, PCA = QCB and PBC = QAC, then the true statement is :

For Unacademy Subscription Use “PJLIVE” Code | Join t.me/pjsir42 for Updates
For More Info: “75970 – 84242, 94590 – 43333 / 2222”
(A) PB > QA (B) CPB  CQA (C) PC  QC (D) PCB  QCA
4. For given figure, which one is correct :

(A)  ABC   DEF (B)  ABC  FED (C)  ABC   DFE (D)  ABC   EDF
5. The sides of a triangle with positive area have lengths 4, 6 and x. The sides of a second triangle with
positive area have length 4, 6 and y. The smallest positive number that is not the possible value of |x . y|
is (x and y are integers) :
(A) 2 (B) 4 (C) 6 (D) 8
6. The sides of a triangle are in the ratio 4 : 6 : 11. Which of the following words best described the triangle?
(A) obtuse (B) isosceles (C) acute (D) impossible
7. The number of triangles with any three of the lengths 1, 4, 6 and 8 cm, are :
(A) 4 (B) 2 (C) 1 (D) 0
8. The perimeter of a triangle is :
(A) greater than the sum of its altitudes (B) less than the sum of its altitudes
(C) equal to the sum of its altitudes (D) none of these
9. In the given diagram B = C = 65º and D = 30º, then the true statement is :

(A) BC = CA (B) CA > CD (C) BD > AD (D) AC = AD


10. In a PQR, PS is bisector of P, Q = 70º and R = 30º, then :
(A) QR < PR > PQ (B) QR > PR > PQ (C) QR = PR = PQ (D) QR < PR = PQ
11. In an isosceles triangle MNO, MN = OM. If P is a point on the side ON such that MP is the bisector of
the M, then NP : PO is
(A) 1 : 1 (B) 1 : 2 (C) 2 : 1 (D) 1 : 3

12.

In the above figure, DEF is a triangle. The side DF is produced to G. If GFH = xº and HFE = yº, then
(D + E) is equal to
(A) xº + yº (B) 2xº + yº (C) xº + 2yº (D) 2xº – yº

13. In ABC, AB = AC and AD is the median. If AC produced' to E and DAC = 20º, then find DCE.
(A) 30º (B) 60º (C) 70º (D) 110º

14.

For Unacademy Subscription Use “PJLIVE” Code | Join t.me/pjsir42 for Updates
For More Info: “75970 – 84242, 94590 – 43333 / 2222”
In the figure above, AC and BF intersect at E, CAB = 80º, ABD = 20º, BDC = 120º and
FCA = 30º. Find DCE and EFC.
(A) 20º, 70º (B) 20º, 80º (C) 30º, 70º (D) 30º, 60º

15. In PQR, PQ = 6 cm, PR = 8 cm and P is the orthocentre. What is the length of QR ?


(A) 9 cm (B) 8 cm (C) 10 cm (D) Cannot be determined

16.

In the above figure (not to scale), AC || PQ. What is the measure of y?


(A) 90º (B) 120º (C) 60º (D) 30º

17.

In the triangle above, AB = AC and I is the incentre. Then BAC = __________.


(A) 30º (B) 60º (C) 90º (D) 120º

18.

A In the figure above, ABC is a triangle and AB || DE . Find BAC and EDC when DCE = 80º.
(A) 30º, 30º (B) 60º, 60º (C) 60º, 50º (D) 40º, 40º

19. In ABC, A= 70º, B = 60º and I is incentre then BIC = ______.
(A) 110º (B) 120º (C) 125º (D) 135º

20.

In the above triangle ABC, AB = AC, AC is produced to E and BC produced to D. What is the
measure of BAC?
(A) 90º (B) 100º (C) ll0º (D) 120º

For Unacademy Subscription Use “PJLIVE” Code | Join t.me/pjsir42 for Updates
For More Info: “75970 – 84242, 94590 – 43333 / 2222”
21.

In ABC, AD is the altitude from A to the side BC. BC is produced to E, and CA is produced to F. If
5 5
ACE = DAC and BAF = DAC, then find ABC.
2 3
(A) 35º (B) 45º (C) 70º (D) 90º

22.

In the figure above (not to scale) AB|| CE, DB|| AC , DBA = 40º and ECF = 50º, find ABC + ACB
(A) 140º (B) 90º (C) 100º (D) 135º

23. In ABC and PQR given below, AB = PQ, BC = QR and B = Q = 90º. Which of the following
congruency axioms is/are suitable to prove that BC  PQR?

(A) R.H.S (B) S.A.S (C) both (1) and (2) (D) None of these

24. In the figure given below, AB = AC and AD bisects A. Which of the following is true?
(a) ADB = 90º (b) BD = CD (c) AB = BC

(A) Only (a) (B) Only (b) (C) Both (a) and (b) (D) All (a), (b) and (c)

25. In the figure given below, AB and CD bisect each other at E. Which of the following is/are true?
(a) AD = BC (b) A = C (c) B = D

(A) Only (a) (B) Only (b) (C) Only (c) (D) All (a), (b) and (c)

For Unacademy Subscription Use “PJLIVE” Code | Join t.me/pjsir42 for Updates
For More Info: “75970 – 84242, 94590 – 43333 / 2222”
26.

In the above figure, ABC is _____.


(A) equilateral (B) isosceles (C) right angled (D) Both (1) and (2)

27.

In the above diagram, CAB = 30º, BCA = 50º. If xº and yº are exterior angles, then find x + y.
(A) 180º (B) 280º (C) 300º (D) 360º

28.

In the figure above, ABE Is a triangle and AC = AD. What is the measure of CBA?
(A) 30º (B) 40º (C) 60º (D) 80º

29.

In the figure above, DEG is a triangle and F is lying on EG. DE = EF and DF = FG. If DGF = 25º then
EDG = ____.
(A) 75º (B) 100º (C) 50º (D) 25º

30. In a triangle ABC, if B = 90º, AB = 5 cm and BC = 12 cm, then the length of the altitude drawn from
B to the side AC (in cm) is
120 120 60 60
(A) (B) (C) (D)
5 13 13 5

31.

In the figure above, BC || PQ,BC is produced to D and ACD = 120º. What type of triangle is ABC?
(A) Scalene triangle (B) Right angled triangle (C) Isosceles triangle (D) Equilateral triangle

32.

In the figure above, DAB = 120º. Then ABC is a/an ___ triangle.
For Unacademy Subscription Use “PJLIVE” Code | Join t.me/pjsir42 for Updates
For More Info: “75970 – 84242, 94590 – 43333 / 2222”
(A) equilateral (B) isosceles (C) acute (D) right angled

33.

1
In the above triangle ABC, EF || DC and AC || FD. EFD = BAC and ABC = 40º. Find FEA.
3
(A) 35º (B) 40º (C) 90º (D) 105º

34.

In the figure above, ABC and DEC are two triangles. BAC = 40º, CDE = 30º and AFD = 100º. Find
ACB.
(A) 30º (B) 40º (C) 50º (D) 100º

35.

In the figure above (not to scale), ABC is a triangle. AE is bisector of BAC, ACD = 150º.
1
BAE = (BEA). If BC is produced to D, then find ABE.
2
(A) 30º (B) 45º (C) 60º (D) 90º

3
36. In ABC, AD is the median. If AD produced to F, AC produced to E, DCE = CDF and
2
DAC = 20º, then find BDA.
(A) 30º (B) 60º (C) 80º (D) 100º

37. In the figure given below, DE = GH, FE = FG, FD = FH and EFH = 90º. Which of the following
congruency/axiom is suitable to prove that DEF  HGF?

(A) S.S.S (B) S.A.S (C) R.H.S (D) All of the above

38. In the figure given below, AB || CD and AC and BD intersect at E. Which of the following is
necessarily true?

(a) EBC  EAD (b) ABC  BAD (c) ACD  BDC


(A) Only (a) (B) Both (b) and (c) (C) Both (a) and (c) (D) None of the above
For Unacademy Subscription Use “PJLIVE” Code | Join t.me/pjsir42 for Updates
For More Info: “75970 – 84242, 94590 – 43333 / 2222”
39. In parallelogram SRQP, T and U are the mid points of PS and QR respectively. Which of the
following is necessarily true?
(A) TR = PU (B) TRU = UPT (C) Both (1) and (2) (D) None of the above

40. In the figure given below, PT ⊥ QR , where PT is the median drawn to QR . Which of the following
is/are sufficient to prove that PQT  SRT?

(A) P, T and S are collinear points (B) PT = TS


(C) PR = RS (D) [(1) and (3)] or [(1) and (3)] or [(2) and (3)

41. In a triangle PQR, D is a point on QR such that PRD = QPD. If QPR = 122½º, then PDQ = ____?
(A) 122½º (B) 61¼º (C) 90º (D) 45º

42. In a triangle ABC, if A > B > C and A, B and C are integers, then the least possible value
for A is
(A) 70º (B) 65º (C) 60º (D) 61º

43.

In the above triangle DEF, the bisectors of the angles E and F intersect each other at a point G. If angle
EDF = 63½º, then angle EGF = ____.

(A) 63½º (B) 90º (C) 121 (D) 127º
4

44.

In the above EFG, side FG is produced on either side. EGM = 120º and EFN = 110º. OF and OG are
the angle bisectors of EFG and EGF respectively, which intersect at the point O. The FOG is _____.
(A) 115º (B) 70º (C) 60º (D) 130º

45.

In the above triangle DEF, side DE is produced to point the M. Ray EO is the bisector of MEF.
FEO is equal to

For Unacademy Subscription Use “PJLIVE” Code | Join t.me/pjsir42 for Updates
For More Info: “75970 – 84242, 94590 – 43333 / 2222”
1 1
(A) 90 − ( DEF) (B) 90º + (DEF) (C) 90º – (DEF) (D) 90º + (DEF)
2 2

46.

In the above figure, DEF is a triangle whose side DF is produced to G. HF and HD are the bisectors of
1
the EFG and EDG respectively. If DEF = 23 , then DHF in degrees is equal to
2
1 2 3 1
(A) 11 (B) 11 (C) 11 (D) 11
2 5 4 3

47. In PQR, PD is perpendicular to QR and PO is the bisector of the angle QPR. If PQR = 65º and
1
PRQ = 23 , then DPO in degrees is equal to
2
3 1 1 1
(A) 20 (B) 20 (C) 20 (D) 20
4 2 5 4

48.

In the above figure, AB is parallel to DE, AC is parallel to FD , EDC = 30º and BDF = 50º. Find
AFD ·
(A) 100º, 80º (B) 80º, 80º (C) 100º, 100º (D) 50º, 30º

49.

In the figure above, ABD = 20º, BDC = 110º and DCA = 30º. What is the value of BAC?
(A) 30º (B) 60º (C) 90º (D) 120º

50.

In the figure above (not to scale), ABE = ECD and EBD = ACE. If BAC = 80º and BEC = 100º,
then BDC = _____.
(A) 80º (B) 100º (C) 110º (D) 120º

For Unacademy Subscription Use “PJLIVE” Code | Join t.me/pjsir42 for Updates
For More Info: “75970 – 84242, 94590 – 43333 / 2222”
51.

In the figure above (not to scale), AD is the angle bisector of EAF, AFC = 110º and DCF = 20º. If
DAF = 30º and EBD = 10º, then find AEB.
(A) 110º (B) 120º (C) 150º (D) 160º

52.

1 1 1
In the triangle above, if DCE = BCD = DBC = BDC, then find BAC.
2 3 4
(A) 90º (B) 80º (C) 120º (D) 60º

53. In the figure below, BC || EF , BC = EF and DF = AC. Which of the following congruency axiom(s)
is/are suitable to prove that BCA  EFD?

(A) S.S.S. (B) S.A.S (C) R.H.S (D) All of the above

54. In a rhombus PQRS, diagonals intersect at M. Which of the following is true?


(a) RMQ  PMS (b) RMS  PMS (c) PMQ  RMQ
(A) Only (a) (B) Only (b) (C) Both (a) and (b) (D) All (a), (b) and (c)

55. In the figure, AB = AD, BC = DC and AC ⊥ BD. Which of the following is necessarily true ?

(A) ABC = ADC (B) BAD = BCD (C) Both (A) and (B) (D) None of these

56. In the figure below, BC is produced to F and CB is produced to the point E. AB,ED intersect at G.
Find BAC.

(A) 30º (B) 45º (C) 60º (D) 75º

For Unacademy Subscription Use “PJLIVE” Code | Join t.me/pjsir42 for Updates
For More Info: “75970 – 84242, 94590 – 43333 / 2222”
57.

In the above figure AD and BE intersect at C, AC = CD, ABC = 60º and CED = 30º.
Find CDE – BAC.
(A) 30º (B) 45º (C) 60º (D) 90º

58.

In the figure above, EFG is a triangle whose sides EF and EG are produced to the points M and N
respectively. OF and OG are the bisectors of the MFG and FGN respectively. Then (GFO + FGO)
is
1 1
(A) 180º – (EFG + EGF) (B) 180º + (EFG + EGF)
2 2
(C) 180º + (EFG + EGF) (D) 180º – (EFG + EGF)

59.

In the above figure (not to scale), AD is the bisector of BAC, AB = AD and AD || BE and AB || DE .
Find ACB.
(A) 20º (B) 40º (C) 30º (D) 60º
60.

In the above MNO, MC is the bisector of the NMO and MD is perpendicular to ON . If M = 40º
and N = 80º, then DMC is ___.
(A) 5º (B) 15º (C) 10º (D) None of the above
61.

In the figure above, A, D and C are collinear DBE = 40º and DCE = 30º. If x + y = 70º, then find BEC.
For Unacademy Subscription Use “PJLIVE” Code | Join t.me/pjsir42 for Updates
For More Info: “75970 – 84242, 94590 – 43333 / 2222”
(A) 70º (B) 140º (C) 35º (D) None of the above

62.

In the above figure (not to scale), ABE = 30º, ACF = 20º and EDF = 110º. If BE || DF and
ED || FC then find BAC
(A) 30º (B) 60º (C) 50º (D) 55º

63.

5
In the above ABC, AB|| DE, ABC = QPD. If DCP = 20º and QC is the bisector of ACB,
3
then find BAC.
(A) 20º (B) 40º (C) 60º (D) 80º
64.

In the figure above (not to scale), DCE = 50º and DEC = 60º. If AB || DE , then find ABC.
(A) 70º (B) 40º (C) 50º (D) 60º

63.

In the above triangle ABC, AD is the bisector of BAC and DAC = 30º then find BPC.
(A) 60º (B) 80º (C) 120º (D) Cannot be determined

66. In the following figure, ABC is a right triangle, where BAC = 90º. EF || AB and ACB = 30º. If
BC = 20 cm and EF = 8 cm, then what is the length of BF?

(A) 4 cm (B) 6 cm (C) 4 3 cm (D) 6 3 cm

For Unacademy Subscription Use “PJLIVE” Code | Join t.me/pjsir42 for Updates
For More Info: “75970 – 84242, 94590 – 43333 / 2222”
67. Triangle ABC is such that AB = 3 cm, BC = 2 cm and CA = 2.5 cm. Triangle DEF is similar to ABC.
If EF = 4 cm, then the perimeter of DEF is
(A) 7.5 cm (B) 15 cm (C) 22.5 cm (D) 30 cm

68. The line segments joining the mid points of the sides of a triangle form four triangles each of which is
(A) similar to the original triangle (B) congruent to the original triangle
(C) an equilateral triangle (D) an isosceles triangle

69. In ABC and DEF, A = 50º, B = 70º, C = 60º, D = 60º, E = 70º, F = 50º, then ABC is
similar to
(A) DEF (B) EDF (C) DFE (D) FED

70. D, E, F are the mid points of the sides BC, CA and AB respectively of ABC. Then DEF is congruent
to triangle
(A) ABC (B) AEF (C) BFD, CDE (D) AFE, FBD, EDC

71. The ratio of the corresponding sides of two similar triangles is 1 : 3. The ratio of their corresponding
heights is
(A) 1 : 3 (B) 3: 1 (C) 1 : 9 (D) 9: 1

72. The areas of two similar triangles are 49 cm2 and 64 cm2 respectively. The ratio of their corresponding
sides is
(A) 49: 64 (B) 7:8 (C) 64: 49 (D) None of these

73. The areas of two similar triangles are 12 cm2 and 48 cm2. If the height of the smaller one is 2.1 cm, then
the corresponding height of the bigger one is
(A) 4.41 cm (B) 8.4 cm (C) 4.2 cm (D) 0.525 cm

74. In the adjoining figure, ABC and DBC are two triangles on the same base BC, AL ⊥ BC and
area( ABC)
DM ⊥ BC. Then, is equal to
area( DBC)

AO AO2 AO OD 2
(A) (B) (C) (D)
OD OD 2 AD AO2

75. In a right angled triangle, if the square of the hypotenuse is twice the product of the other two sides,
then one of the angles of the triangle is
(A) 15º (B) 30º (C) 45º (D) 60º

76. Consider the following statements :


1. If three sides of a triangle are equal to three sides of another triangle, then the triangles are
congruent.
2. If three angles of a triangle are respectively equal to three angles of another triangle, then the two
triangles are congruent.
Of these statements,
(A) 1 is correct and 2 is false (B) Both 1 and 2 are false
(C) Both 1 and 2 are correct (D) 1 is false and 2 is correct

77. If in two triangles DEF and PQR, D = Q and R = E, then which of the following is not true?
EF DF DE EF DE DF EF DF
(A) = (B) = (C) = (D) =
PR PQ PQ RP QR PQ RP QR
For Unacademy Subscription Use “PJLIVE” Code | Join t.me/pjsir42 for Updates
For More Info: “75970 – 84242, 94590 – 43333 / 2222”
78. In triangles ABC and DEF, B = E, F = C and AB = 3 DE. Then, the two triangles are
(A) Congruent but not similar (B) Similar but not congruent
(C) Neither congruent nor similar (D) Congruent as well as similar

AB BC
79. If in triangles ABC and DEF, = , then they will be similar, when
DE FD
(A) B = E (B) A = D (C) B = D (D) A= F

ar(ABC) 9
80. If ABC ~ QRP, = , AB = 18 cm and BC = 15 cm, then PR is equal to
ar(PQR) 4
20
(A) 10 cm (B) 12 cm (C) cm (D) 8 cm
3

EXERCISE – 03

NTSE PROBLEMS (PREVIOUS YEARS)


1. In ABC and DEF, AC =BC=DF=EF, length AB=2FH, where FH ⊥ DE. Which of the following
statements is (are) true ? [Haryana NTSE Stage-1 2013]
I. ACB and DFE are complementary II. ACB and DFE are supplementary
III. Area of ABC = Area of DEF IV. Area of ABC = 1.5x (Area of DEF)
(A) II only (B) III only (C) I and III only (D) II and III only

2. The length of sides of triangle are integers and its perimeter is 14. How many such distinct triangles are
possible? [Bihar NTSE Stage-1 2015]
(A) 6 (B) 5 (C) 4 (D) 3

4. Which of the following statements is false for the quadrilateral ABCD ? [Rajasthan NTSE Stage-I/18]
(A) AB + BC + CD + DA > AC (B) AB + BC+ CD + DA > AB + AC
(C) AB + BC + CD + DA > AC + BD (D) AB+ BC+ CD + DA < 2AC.

5. AD and BE are the altitudes of ABC. If AD = 6 cm, BC = 16 cm, BE = 8 cm, then CA = ______ cm.
[Gujarat NTSE Stage-I/18]
(A) 12 (B) 18 (C) 24 (D) 10

VALUE BASED / PRACTICAL BASED QUESTIONS


1. Two friends Rahim and Meera constructed their houses in the same colony. Rahim wanted to make a
bamboo stair to go on the roof of his house. He called a carpenter for taking measurement of the stair to
be constructed. Now, Rahim’s friend Meera desires to have a stair for her roof. She measures the
heights of the two buildings and finds that they are the same. What criterion of congruency can use to
make her bamboo stair without taking measurement of the bamboo stair? What would be the length of
the stair ? What value is depicted by this question?

For Unacademy Subscription Use “PJLIVE” Code | Join t.me/pjsir42 for Updates
For More Info: “75970 – 84242, 94590 – 43333 / 2222”
Answer Key

BOARD LEVEL EXERCISE


TYPE (I) : VERY SHORT ANSWER TYPE QUESTIONS : [01 MARK EACH]
1. 50º 2. 4 cm 3. 5 cm 4. PQ > PR 5. DE
6. PQ, ASA criterion

EXERCISE – 01

OBJECTIVE QUESTIONS
Section (A) : Congruent Triangles
A-1. (B) A-2. (A) A-3. (D) A-4. (A) A-5. (B)
A-6. (A) A-7. (B) A-8. (B)

Section (B) : Some inequalities in triangles


B-1. (B) B-2. (A) B-3. (A) B-4. (B) B-5. (D)
B-6. (C)

EXERCISE – 02

OBJECTIVE QUESTIONS
Ques. 1 2 3 4 5 6 7 8 9 10 11 12 13 14 15 16 17 18 19 20
Ans. B D D C D D C A C B A A D A C B B B C D
Ques. 21 22 23 24 25 26 27 28 29 30 31 32 33 34 35 36 37 38 39 40
Ans. C A C C A A B C A C D C A A D C D D C D
Ques. 41 42 43 44 45 46 47 48 49 50 51 52 53 54 55 56 57 58 59 60
Ans. A D C A A C A B B D D D B D A B A A C C
Ques. 61 62 63 64 65 66 67 68 69 70 71 72 73 74 75 76 77 78 79 80
Ans. B B B A C A B A D D A B C A C A B B C A

EXERCISE – 03

Ques. 1 2 3 4 5
Ans. D C C D A

VALUE BASED / PRACTICAL BASED QUESTIONS

1. By SAS criteria
same length
Value depicted are friendly and cooperative nature between two friends ignoring the religion.

For Unacademy Subscription Use “PJLIVE” Code | Join t.me/pjsir42 for Updates
For More Info: “75970 – 84242, 94590 – 43333 / 2222”
4 QUADRILATERALS
INTRODUCTION

Quadrilaterals
A quadrilateral is a four sided closed figure.

Let A, B, C and D be four points in a plane such that :


(i) No three of them are collinear.
(ii) The line segments AB, BC, CD and DA do not intersect except at their end points,
then figure obtained by joining A, B, C & D is called a quadrilateral.
Properties: –
(i) Sum of four interior angles is 360º.
(ii) The figure formed by joining the mid-points of a quadrilateral is a parallelogram.
(iii) The sum of opposite sides of a quadrilateral circumscribed about a circle, is
always equal.
1
(iv) Area of quadrilateral = × one of the diagonals × sum of the perpendiculars
2
drawn to the diagonals from the opposite vertices.
1
i.e, Ar ( ABCD) = × AC × (DE + BF)
2

A. TYPES OF QUADRILATERALS

(a) Parallelogram : A parallelogram is a quadrilateral in which both pairs of opposite sides are parallel.
In figure, AB || DC, AD || BC therefore, ABCD is a parallelogram.
Properties :–
(i) The opposite sides are parallel and equal.
(ii) Opposite angles are equal.
(iii) Sum of any two adjacent angles is 180º.
(iv) Diagonals bisect each other.
(v) Diagonals need not be equal in length.
(vi) Diagonals need not bisect at right angle.
(vii)Diagonals need not bisect angles at the vertices.
(viii)Each diagonal divides a parallelogram into two congruent triangles.
(ix) Lines joining the mid-points of the adjacent sides of a quadrilateral form a
parallelogram.
(x) Lines joining the mid-points of the adjacent sides of a parallelogram is a parallelogram.
(xi) The parallelogram that is inscribed in a circle is a rectangle.
(xii) The parallelogram that is circumscribed about a circle is a rhombus.
(xiii) (AC)2 + (BD)2 = (AB)2 + (BC)2 + (CD)2 + (AD)2 = 2(AB2 + BC2)
(xiv) Parallelogram that lie on the same base and between the same parallel lines are equal in area.
(xv) A parallelogram is a rectangle if its diagonals are equal.

For Unacademy Subscription Use “PJLIVE” Code | Join t.me/pjsir42 for Updates
For More Info: “75970 – 84242, 94590 – 43333 / 2222”
(b) Rectangle : A rectangle is a parallelogram, in which each of its angle is a right angle. If ABCD is a
rectangle then A = B = C = D = 90º, AB = CD, BC = AD and diagonals AC = BD.
Properties : –
(i) Opposite sides are parallel and equal.
(ii) Opposite angles are equal and of 90º.
(iii) Diagonals are equal and bisect each other, but not necessarily at right
angles,
(iv) When a rectangle is inscribed in a circle, the diameter of the circle is equal to
the diagonal of the rectangle.
(v) For the given perimeter of rectangles, a square has maximum area.
(vi) The figure formed by joining the mid-points of the adjacent sides of a rectangle is
a rhombus.
(vii)The quadrilateral formed by joining the mid-points of intersection of the angle bisectors of a
parallelogram is a rectangle.
(viii) Every rectangle is a parallelogram.

(c) Rhombus : A rhombus is a parallelogram in which all its sides are equal in length. If ABCD is a
rhombus then, AB = BC = CD = DA.
The diagonals of a rhombus are perpendicular to each other.
Properties : –
(i) Opposite sides are parallel and equal.
(ii) Opposite angles are equal.
(iii) Diagonals bisect each other at right angle, but they are not necessarily equal.
(iv) Diagonals bisect the vertex angles.
(v) Sum of any two adjacent angles is 180º.
(vi) Figure formed by joining the mid-points of the adjacent sides of a rhombus is
a rectangle.

(vii)A parallelogram is a rhombus if its diagonals are perpendicular to each other.

(d) Square : A square is a parallelogram having all sides equal and each angle equal to right angle. If ABCD is
a square then AB = BC = CD = DA, diagonal AC = BD and  A =  B =  C =  D = 90º.
The diagonals of a square are perpendicular to each other.
Properties : –
(i) All sides are equal and parallel.
(ii) All angles are right angles.
(iii) Diagonals are equal and bisect each other at right angle.
(iv) The figure formed by joining the mid-points of the adjacent sides of a square
is a square.

(e) Trapezium : A trapezium is a quadrilateral with only one pair of opposite sides parallel. In figure,
ABCD is a trapezium with AB || DC.
Properties : –
(i) The line joining the mid-points of the oblique (non-parallel) sides is half
the sum of the parallel sides and is called the median.
1 1
[ i.e., Median = × sum of parallel sides = × (AB + DC) = EF]
2 2
(ii) If the non-parallel sides are equal, then the diagonals will also be equal to each
other.
(iii) Diagonals intersect each other proportionally in the ratio of lengths of parallel
sides.

(iv) By joining the mid-points of adjacent sides of a trapezium, four similar triangles are obtained.
(v) If a trapezium is inscribed in a circle, then it is an isosceles trapezium with equal oblique sides.
For Unacademy Subscription Use “PJLIVE” Code | Join t.me/pjsir42 for Updates
For More Info: “75970 – 84242, 94590 – 43333 / 2222”
(vi) AC2 + BD2 = BC2 + AD2 + 2AB. CD

(f) Kite : A kite is a quadrilateral in which two pairs of adjacent sides are equal. If
ABCD is a kite then AB = AD and BC = CD.
Properties : –
(i) AB = BC and AD = CD.
(ii) Diagonals intersect at right angles.
(iii) Shorter diagonal is bisected by the longer diagonal.
S. No. Property Parallelogram Rectangle Rhombus Square
1. Opposite side are equal ✓ ✓ ✓ ✓
2. All side are equal   ✓ ✓
3. Opposite are equal and right angle ✓ ✓ ✓ ✓
4. All angles are equal and right angle  ✓  ✓
5. Diagonals bisect each other ✓ ✓ ✓ ✓
6. Diagonals bisect each other at right angles   ✓ ✓
7. Diagonals are equal   ✓ ✓
8. Diagonals form four triangles of equal area  ✓  ✓
9. Diagonals from four congruent triangles ✓ ✓ ✓ ✓
10. Diagonals from four tringles   ✓ ✓

Isosceles trapezium : A trapezium is said to be an isosceles trapezium, if its


non-parallel sides are equal. Thus a quadrilateral ABCD is an isosceles trapezium,
if AB || DC and AD = BC.
In isosceles trapezium A = B and C = D.

Remark : (i) Square, rectangle and rhombus are all parallelograms.


(ii) Kite and trapezium are not parallelograms.
(iii) A square is a rectangle.
(iv) A square is a rhombus.
(v) A parallelogram is a trapezium.

(g) Important theorems related to Quadrilaterals :


(i) Theorem : A diagonal of a parallelogram divides the parallelogram into two congruent triangles.
Given : A parallelogram ABCD.
To Prove : A diagonal divides the parallelogram into two congruent triangles.
i.e., if diagonal AC is drawn then  ABC  CDA.
and if diagonal BD is drawn then  ABD  CDB.
Construction : Join A and C.
Proof : Since, ABCD is a parallelogram.
 AB || DC and AD || BC.
In  ABC and CDA
BAC = DCA [Alternate angles]
BCA = DAC [Alternate angles]
And, AC = AC [Common side]
 ABC  CDA [By ASA congruency]
Similarly, we can prove that
ABD  CDB Hence Proved.
(ii) Theorem : The diagonals of a parallelogram bisect each other.
Given : A parallelogram ABCD. Its diagonals AC and BD intersect each
other at point O.
To Prove : Diagonals AC and BD bisect each other
i.e., OA = OC and OB = OD.
For Unacademy Subscription Use “PJLIVE” Code | Join t.me/pjsir42 for Updates
For More Info: “75970 – 84242, 94590 – 43333 / 2222”
Proof : In  AOB and COD
 AB || DC and BD is a transversal line.
 ABO = CDO [Alternate angles]
 AB || DC and AC is a transversal line.
 BAO = DCO [Alternate angles]
And, AB = DC
  AOB  COD [By ASA congruency]
 OA = OC and OB = OD [By CPCT] Hence Proved.
(iii) Theorem : The diagonals of a rhombus are perpendicular to each other.
Given : A rhombus ABCD whose diagonals AC and BD intersect at O.
To prove :  BOC =  DOC =  AOD =  AOB = 90º.
Proof : Parallelogram is a rhombus, if all of its sides are equal.
AB = BC = CD = DA … (i)
Since the diagonals of a parallelogram bisect each other.
 OB = OD and OA = OC ... (ii)
In BOC and DOC
BO = OD [From (ii)]
BC = DC [From (i)]
OC = OC
So, by SSS criterion of congruence
 BOC  DOC
 BOC =  DOC [By CPCT]
But,  BOC +  DOC = 180º [Linear pair]
 BOC =  DOC = 90º
Similarly,  AOD =  AOB = 90º
Hence,  BOC =  DOC =  AOD =  AOB = 90º.

Solved Examples

Example.1 The angles of a quadrilateral are in the ratio 3 : 5 : 9 : 13. Find all the angles of the quadrilateral.
Solution. Let the angles be 3x, 5x, 9x and 13x.
 3x + 5x + 9x + 13x = 360º
360
 30x = 360º and x = = 12º
30
 1st angle = 3x = 3 × 12º = 36º,
2nd angle = 5x = 5 × 12º = 60º,
3rd angle = 9x = 9 × 12º = 108º,
And, 4th angle = 13 × 12º = 156º.
Example.2 Use the information’s given in adjoining figure to calculate the value of x.
Solution. Since, EAB is a straight line.
 DAE + DAB = 180º
 73º + DAB = 180º
 DAB = 180º – 73º = 107º.
Since, the sum of the angles of quadrilateral ABCD is 360º.
 107º + 105º + x + 80º = 360º
 292º + x = 360º
 x = 360º – 292º
 x = 68º.
Example.3 Prove that the angle bisectors of a parallelogram form a rectangle.
Solution. A parallelogram ABCD in which bisectors of angles A, B, C, D
intersect at P, Q, R, S to form a quadrilateral PQRS.
Since, ABCD is a parallelogram, so AD || BC and transversal AB
intersects them at A and B respectively.
A + B = 180º

For Unacademy Subscription Use “PJLIVE” Code | Join t.me/pjsir42 for Updates
For More Info: “75970 – 84242, 94590 – 43333 / 2222”
1 1
 A + B = 90
2 2
 BAS + ABS = 90º ....(i) [AS and BS are bisectors of A and B respectively]
BAS + ABS +ASB = 180º
 90º + ASB = 180º
 ASB = 90º
 RSP = 90º [ASB and RSP are vertically opposite angles]
Similarly, SRQ = 90º, RQP = 90º and SPQ = 90º.
Hence, PQRS is rectangle.
Example.4 The diagonals of a parallelogram ABCD intersect at O. A line through O intersects AB at X
and DC at Y. Prove that OX = OY.
Solution. Since AB || CD.
In OAX and OCY
1 = 2 [Alternate angles]
OA = OC
and 3 = 4 [Vertically opposite angles]
So, by ASA criterion of congruence,
OAX  OCY  OX = OY. [By CPCT]
Example.5 In the adjoining figure, ABCD is a parallelogram and X, Y are the points on diagonal BD such
that DX = BY. Prove that CXAY is a parallelogram.

Solution. Join AC, meeting BD at O.


Since the diagonals of a parallelogram bisect each other.
OA = OC and OD = OB
Now, OD = OB and DX = BY
 OD – DX = OB – BY  OX = OY
Now, OA = OC and OX = OY.
 CXAY is a quadrilateral whose diagonal bisect each other.
 CXAY is a parallelogram.
Example.6 In the adjoining figure, ABCD is a parallelogram and the bisector of A bisect BC at X. Prove
that AD = 2AB.
Solution. ABCD is a parallelogram.
 AD || BC and AX cuts them.
1
 BXA = DAX = A [Alternate interior angles]
2
1
 2 = A.
2
1
Also, 1 = A
2
 2 = 1  AB = BX
 AB = BC  AB = AD  AD = 2 AB.

Example.7 ABCD is a trapezium in which AB || CD and AD = BC. Show that :


(i) A = B (ii) C = D
(iii) ABC   BAD (iv) diagonal AC = diagonal BD.
Solution. (i) Extend AB and draw a line through C parallel to DA intersecting AB produced at E.
Since, AD || CE and transversal AE cuts them at A and E respectively.
 A + E = 180º
 180º – E = A
Since, AB || CD and AD || CE
 AECD is a parallelogram.
For Unacademy Subscription Use “PJLIVE” Code | Join t.me/pjsir42 for Updates
For More Info: “75970 – 84242, 94590 – 43333 / 2222”
 AD = CE
 BC = CE
Thus, in BCE
BC = CE
 CBE = CEB
 180º – B = E
 180º – E = B
 A = B.
(ii) Consecutive interior angles on the same side of a transversal are supplementary.
 A + D = 180º and E + C = 180º
 A + D = E + C
 B + D = E + C
 D = C [B = E]
(iii) In ABC and BAD
AB = BA
B = A
and BC = AD
So, by SAS congruence criterion
ABC  BAD
(iv) Since, ABC  BAD
AC = BD
Hence, diagonal AC = diagonal BD.

Example.8 In ABC, lines are drawn through A, B and C parallel respectively to the sides BC, CA and
1
AB, forming PQR. Show that BC = QR.
2
Solution. AQ || CB and AC || QB
 AQCB is parallelogram.
 BC = AQ [Opposite sides of a ||gm is equal] Q A R
AR || CB and AB || RC
 ARCB is parallelogram.
 BC = AR [Opposite sides of a ||gm are equal]
So, AQ = AR
1
 AQ = AR = QR
2
1
 BC = QR.
2

Check Point - A

1. Show that the diagonals of a rhombus are perpendicular to each other.

2. If ABCD is a rhombus, find AOD, where O is the point of intersection of the diagonals.
3. If AD = (x + 2y) cm, BC = (2x + 3) cm, DC = (x + 7) cm and AB = (3y + 2) cm find AB and BC in
parallelogram ABCD.

4. Given a parallelogram ABCD. DE perpendicular to AC and BF perpendicular to AC.


Prove that: DE = BF

For Unacademy Subscription Use “PJLIVE” Code | Join t.me/pjsir42 for Updates
For More Info: “75970 – 84242, 94590 – 43333 / 2222”
5. Given a parallelogram ABCD, EF || AD, GH || CB. Prove that EFHG is a parallelogram.

6. Given a parallelogram PQRS in which QX || SY. Prove that QX = SY

7. The diagonals of a square intersect at O. From AB a part AQ = AO is cut off. Prove that AOQ = 3BOQ

Answers
2. 90º 3. AB = 8 cm, BC = 5 cm.

B. MID POINT THEOREM AND ITS CONVERSE

(a) Mid point theorem


In a triangle, the line segment joining the mid-points of any two sides is parallel to the third side
and is half of it.
Given : A triangle ABC in which P is the mid-point of side AB and Q is the mid-point of side AC.
1
To Prove : PQ is parallel to BC and is half of it i.e., PQ || BC and PQ = BC.
2
Construction : Produce PQ upto point R such that PQ = QR. Join R and C.
Proof : In APQ and CRQ
PQ = QR [By construction]
AQ = QC [Given]
And, AQP = CQR [Vertically opposite angles]
So, APQ  CRQ [By SAS]
 AP = CR [By CPCT]
And, APQ = CRQ [By CPCT]
But, APQ and CRQ are alternate angles and whenever the alternate angles are equal, the lines are parallel.
 AP || CR
 AB || CR
 BP || CR
AP = BP [Given, P is mid-point of AB]
 CR = BP [As, AP = CR]
Now, BP = CR and BP || CR
 BCRP is a parallelogram.
[When any pair of opposite sides are equal and parallel, the quadrilateral is a parallelogram]
BCRP is a parallelogram and opposite sides of a parallelogram are equal and parallel.
 PR = BC and PR || BC
Since, PQ = QR

For Unacademy Subscription Use “PJLIVE” Code | Join t.me/pjsir42 for Updates
For More Info: “75970 – 84242, 94590 – 43333 / 2222”
1 1
 PQ = PR = BC [As, PR = BC]
2 2
Also, PQ || BC [As, PR || BC]
1
 PQ || BC and PQ = BC Hence Proved.
2

(b) Converse of mid point theorem


The line drawn through the mid-point of one side of a triangle parallel to the another side; bisects
the third side.
Given : A triangle ABC in which P is the mid-point of side AB and PQ is parallel to BC.
To prove : PQ bisects the third side AC i.e., AQ = QC.
Construction : Through C, draw CR parallel to BA, which meets PQ produced at point R.
Proof : Since, PQ || BC i.e., PR || BC [Given]
and CR || BA i.e., CR || BP. [By construction]
 Opposite sides of quadrilateral PBCR are parallel.
 PBCR is a parallelogram
 BP = CR
Also, BP = AP [As, P is mid-point of AB]
 CR = AP
 AB || CR and AC is transversal, PAQ = RCQ [Alternate angles]
 AB || CR and PR is transversal, APQ = CRQ [Alternate angles]
In  APQ and CRQ
CR = AP, PAQ = RCQ and APQ = CRQ
  APQ  CRQ [By ASA]
 AQ = QC [By CPCT] Hence Proved.

Remark : In quadrilateral ABCD, if side AD is parallel to side BC; ABCD is a trapezium.

1
Now, P and Q are the mid-points of the non-parallel sides of the trapezium; then PQ = (AD + BC).
2
i.e. The length of the line segment joining the mid-points of the two non-parallel sides of a
trapezium is always equal to half of the sum of the length of its two parallel sides.

(c) Intercept theorem


Theorem : If there are three or more parallel lines and the intercepts made by them on a
transversal are equal, then the corresponding intercepts on any other transversal are also equal.
Given : Three parallel lines l, m and n i.e., l || m || n. A transversal p meets these parallel lines at points
A, B and C respectively such that AB = BC. Another transversal q also meets parallel lines l, m and n
at points D, E and F respectively.

For Unacademy Subscription Use “PJLIVE” Code | Join t.me/pjsir42 for Updates
For More Info: “75970 – 84242, 94590 – 43333 / 2222”
To Prove : DE = EF
Construction : Through point A, draw a line parallel to DEF; which meets BE at point P and CF at point Q.
Proof : In  ACQ, B is mid-point of AC and BP is parallel to CQ and we know that the line through the
midpoint of one side of the triangle and parallel to another side bisects the third side.
 AP = PQ ... (i)
When the opposite sides of a quadrilateral are parallel, it is a parallelogram and so its opposite sides are
equal.
 AP || DE and AD || PE
 APED is a parallelogram.
 AP = DE ...(ii)
And PQ || EF and PE || QF
 PQFE is a parallelogram
 PQ = EF ...(iii)
From above equations, we get
DE = EF Hence Proved.

Solved Examples

Example.9 ABCD is a rhombus and P, Q, R and S are the mid-points of the sides AB, BC, CD and DA
respectively. Prove that the quadrilateral PQRS is a rectangle.
Solution.

1
In  ABC, PQ || AC and PQ = AC ... (i) [By mid-point theorem]
2
1
In  ADC, SR || AC and SR = AC ... (ii) [By mid-point theorem]
2
 PQ = SR and PQ || SR [From (i) and (ii)]
 PQRS is a parallelogram.
Now, PQRS will be a rectangle if any angle of the parallelogram PQRS is 90º.
PQ || AC [By mid-point theorem]
QR || BD [By mid-point theorem]
But, AC ⊥ BD [Diagonals of a rhombus are perpendicular to each other]
 PQ⊥ QR [Angle between two lines = angle between their parallels]
 PQRS is a rectangle. Hence Proved.

Example.10 ABCD is a trapezium in which AB || DC, BD is a diagonal and E is the mid-point of AD. A line is
drawn through E parallel to AB intersecting BC at F (Figure). Prove that F is the mid-point of BC.
Solution. Given line EF is parallel to AB and AB || DC.
 EF || AB || DC.
According to the converse of the mid-point theorem, in  ABD, E is
the mid-point of AD and EP is parallel to AB. [As EF || AB]
 P is the mid-point of side BD.
[The line through the mid-point of a side of a triangle and parallel to
the other side, bisects the third side]
Now, in  BCD, P is mid-point of BD and, PF is parallel to DC. [As EF || DC]
 F is the mid-point of BC
[The line through the mid-point of a side of a triangle and parallel to the other side, bisects the
third side]
Hence Proved.

For Unacademy Subscription Use “PJLIVE” Code | Join t.me/pjsir42 for Updates
For More Info: “75970 – 84242, 94590 – 43333 / 2222”
Example.11 In the given figure, E and F are respectively, the mid-points of non-parallel sides of a trapezium
ABCD.
Prove that :
1
(i) EF || AB (ii) EF = (AB + DC).
2

Solution. Join BE and produce it to intersect CD produced at point P. In  AEB and DEP, AB || PC and
BP is transversal.
ABE = DPE [Alternate interior angles]
AEB = DEP [Vertically opposite angles]
And AE = DE [E is mid-point of AD]
So, AEB  DEP [By AAS congruency]
BE = PE [By CPCT]
And AB = DP [By CPCT]
Since, the line joining the mid-points of any two sides of a triangle is parallel and half of the
third side, Therefore, in BPC,
E is mid-point of BP [As, BE = PE] and F is mid-point of BC [Given]
1
 EF || PC and EF = PC
2
1
 EF || DC and EF = (PD + DC)
2
1
 EF || AB and EF = (AB + DC) [As, DC || AB and PD = AB] Hence Proved.
2

1
Example.12 AD and BE are medians of ABC and BE || DF. Prove that CF = AC.
4
Solution. In BEC, DF is a line through the mid - point D of BC and parallel to
BE intersecting CE at F. Therefore, F is the midpoint of CE. Because
the line drawn through the mid point of one side of a triangle and
parallel to another sides bisects the third side.
Now, F is the mid point of CE
1 1 1 1
 CF = CE  CF = ( AC)  CF = AC.
2 2 2 2

Example.13 Prove that the figure formed by joining the mid - points of the pairs of consecutive sides of a
quadrilateral is a parallelogram.
Solution. ABCD is a quadrilateral in which P, Q, R and S are the midpoints of the sides AB, BC, CD and
DA respectively.
Join A and C.
In ABC, P and Q are the midpoints of sides AB and AC respectively.
1
 PQ || AC and PQ = AC [By midpoint theorem]
2
In ABC, P and Q are the midpoints of sides AB and AC respectively.
1
 RS || AC and RS = AC [By midpoint theorem]
2
 PQ = RS and PQ || RS.
Thus in quadrilateral PQRS one pair of opposite sides are equal and parallel.
Hence, PQRS is a parallelogram.

For Unacademy Subscription Use “PJLIVE” Code | Join t.me/pjsir42 for Updates
For More Info: “75970 – 84242, 94590 – 43333 / 2222”
Check Point - B

1. Prove that the median to the hypotenuse of a right angled triangle is half the length of the hypotenuse.

2. The diagonals of a quadrilateral ABCD are perpendicular to each other. Show that the quadrilateral,
formed by joining the middle points of its sides is a rectangle.

3. Perpendiculars dropped from the mid points of two sides of a triangle to the third side are equal.

4. In ABC, the medians CD and BE are produced to X and Y such that CD = DX and BE = EY. Prove
that the points X, A, Y are collinear.

5. Show that the three line segments which join the middle points of the sides of a triangle, divide it into
four triangles which are congruent to each other.

C. POLYGON

A closed plane figure bounded by line segments is called a polygon.


The line segments are called its sides and the points of intersection of consecutive sides are called its
vertices. An angle formed by two consecutive sides of a polygon is called an interior angle or simply an
angle of the polygon.
Number of side Name
3 Triangles
4 Quadrilateral
5 Pentagon
6 Hexagon
7 Heptagon
8 Octagon
9 Nonagon
10 Decagon
A polygon is named according to the number of sides, it has.
In general, a polygon of "n" sides is called n-gon. Thus, a polygon having 18 sides is called 18-gon.
Diagonal of a polygon
Line segment joining any two non-consecutive vertices of a polygon is called its diagonal.
(a) Convex polygon
If all (interior) angles of a polygon are less than 180º, it is called a convex polygon.
In the figure given below, ABCDEF is a convex polygon.
In other words, a polygon is a convex polygon if the line segment joining any two
points inside it lies completely inside the polygon.

For Unacademy Subscription Use “PJLIVE” Code | Join t.me/pjsir42 for Updates
For More Info: “75970 – 84242, 94590 – 43333 / 2222”
(b) Concave polygon
If one or more of the (interior) angles of polygon is greater than 180º i.e., reflex, it is called concave (or
re-entrant) polygon. In the figure given below, ABCDEFG is a concave polygon.

(c) Exterior angle of convex polygon


If we produce a side of polygon, the angle it makes with the next side is called an exterior angle. In the
diagram given below, ABCDE is a pentagon. Its side AB has been produced to P, then CBP is an
exterior angle.

Corresponding to each interior angle, there is an exterior angle. Also, an exterior angle and its adjacent
interior angle make a straight line, i.e. an exterior angle + adjacent interior angle = 180º

Regular polygon
A polygon is called a regular polygon if all its sides have equal length and all its angles have equal size.
Thus, in a regular polygon
(i) All sides are equal in length.
(ii) All interior angles are equal in size.
(iii) All exterior angles are equal in size.
All regular polygons are convex.

Check Point - C

1. The difference between an exterior angle of (n – 1) sided regular polygon and an exterior angle of (n + 2)
sided regular polygon is 6º, then the value of “n” is
(A) 14 (B) 15 (C) 13 (D) 12

2. How many diagonals are there in a decagon?


(A) 19 (B) 29 (C) 35 (D) 45

Answers.
1. (C) 2. (C)

Add to Your Knowledge

1. (i) If there is a polygon of “n” sides (n  3), we can cut it into (n – 2) triangles with a common vertex
and so the sum of the interior angles of a polygon of “n” sides would be
(n – 2) × 180º = (n – 2) × 2 right-angles = (2n – 4) right angles
(ii) If there is a regular polygon of “n” sides (n  3), then its each interior angle is equal to
 2n − 4 º
  90  .
 n 

For Unacademy Subscription Use “PJLIVE” Code | Join t.me/pjsir42 for Updates
For More Info: “75970 – 84242, 94590 – 43333 / 2222”
 360  º
(iii) Each exterior angle of a regular polygon of “n” sides is equal to   .
 n 
2. The sum of all the exterior angles formed by producing the sides of a convex polygon in the same order
is equal to four right angles.
Explanation
If in a convex polygon P1P2P3P4P5, all the sides are produced in order, forming
exterior angles 1,2, 3, 4 and 5, then 1 + 2 + 3 + 4 + 5 = 4 right
angles = 360º
3. If each exterior angle of a regular polygon is xº, then the number of sides in the
360º
polygon = . Greater the number of sides in a regular polygon, greater is the
x
value of its each interior angle and smaller is the value of each exterior angle.
n(n − 1)
4. If polygon has "n" sides, then the number of diagonals of the polygon = −n
2
Sum of all Each interior angle Each exterior angle No. of
No. of sides Polygon
the angles (regular polygon) (regular polygon) diagonals
3 Triangle 180º 60º 120º 0
4 Quadrilateral 360º 90º 90º 2
5 Pentagon 540º 108º 72º 5
6 Hexagon 720º 120º 60º 9
 4 º  3 º
7 Heptagon 900º 128   51  14
 7  7
8 Octagon 1080º 135º 45º 20
9 Nonagon 1260º 140º 40º 27
10 Decagon 1440º 144º 36º 35

For Unacademy Subscription Use “PJLIVE” Code | Join t.me/pjsir42 for Updates
For More Info: “75970 – 84242, 94590 – 43333 / 2222”
BOARD LEVEL EXERCISE

TYPE (I) : VERY SHORT ANSWER TYPE QUESTIONS : [01 MARK EACH]
1. Diagonals of a parallelogram ABCD intersect at O. If  BOC = 90º and  BDC = 50º, then find OAB.

2. Three angles of a quadrilateral are 75º, 90º and 75º. Find the fourth angle

3. A diagonal of a rectangle is inclined to one side of the rectangle at 25º. Find the acute angle between
the diagonals.

4. ABCD is a rhombus such that  ACB = 40º. Then find  ADB.

5. If angles A, B, C and D of the quadrilateral ABCD, taken in order, are in the ratio 3:7:6:4, then identify
the type of quadrilateral ABCD ?

6. If APB and CQD are two parallel lines, then find the type of quadrilateral formed by the bisectors of the
angles APQ, BPQ, CQP and PQD.

7. Diagonals AC and BD of a parallelogram ABCD intersect each other at O. If OA = 3 cm and OD = 2


cm, determine the lengths of AC and BD.

TYPE (II) : SHORT ANSWER TYPE QUESTIONS : [02 MARKS EACH]


8. One angle of a quadrilateral is of 108º and the remaining three angles are equal. Find each of the three
equal angles.

9. The angle between two altitudes of a parallelogram through the vertex of an obtuse angle of the
parallelogram is 60º. Find the angles of the parallelogram.

10. ABCD is a rhombus in which altitude from D to side AB bisects AB. Find the angles of the rhombus.

11. E and F are points on diagonal AC of a parallelogram ABCD such that AE = CF. Show that BFDE is a
parallelogram.

12. D, E and F are the mid-points of the sides BC, CA and AB, respectively of an equilateral triangle ABC.
Show that  DEF is also an equilateral triangle.

13. Points P and Q have been taken on opposite sides AB and CD, respectively of a parallelogram ABCD
such that AP = CQ see figure. Show that AC and PQ bisect each other.

14. A diagonal of a parallelogram bisects one of its angle. Prove that it will bisect its opposite angle also.

TYPE (III) : LONG ANSWER TYPE QUESTIONS: [03 MARK EACH]


15. Show that the quadrilateral formed by joining the mid-points the sides of a rhombus, taken in order,
form a rectangle.

16. In a parallelogram ABCD, AB = 10 cm and AD = 6 cm. The bisector of A meets DC in E. AE and


BC produced meet at F. Find the length of CF.

17. P, Q, R and S are respectively the mid-points of the sides AB, BC, CD and DA of a quadrilateral
ABCD in which AC = BD. Prove that PQRS is a rhombus.
For Unacademy Subscription Use “PJLIVE” Code | Join t.me/pjsir42 for Updates
For More Info: “75970 – 84242, 94590 – 43333 / 2222”
18. ABCD is a quadrilateral in which AB || DC and AD = BC. Prove that A = B and C = D.

1
19. E is the mid-point of a median AD of  ABC and BE is produced to meet AC at F. Show that AF = AC.
3

TYPE (IV): VERY LONG ANSWER TYPE QUESTIONS [04 MARK EACH]
20. PQ and RS are two equal and parallel line-segments. Any point M not lying on PQ or RS is joined to Q
and S and lines through P parallel to QM and through R parallel to SM meet at N. Prove that line
segments MN and PQ are equal and parallel to each other.

21. P, Q, R and S are respectively the mid-points of sides AB, BC, CD and DA of quadrilateral ABCD in
which AC = BD and AC ⊥ BD. Prove that PQRS is a square.

22. Prove that the line joining the mid-points of the diagonals of a trapezium is parallel to the parallel sides
of the trapezium.

23. P is the mid-point of the side CD of a parallelogram ABCD. A line through C parallel to PA intersects
AB at Q and DA produced at R. Prove that DA =AR and CQ = QR.

EXERCISE – 01

SUBJECTIVE QUESTIONS
Section (A) : Types of Quadrilaterals
A-1. In the following figure, ABCD is a parallelogram DAO = 40º, BAO = 35º and COD = 65º. Find :

(i) ABO (ii) ODC (iii) ACB (iv) CBD

1
A-2. ABCD is a parallelogram. P is a point on AD such that AP = AD. Q is a point on BC such that
3
1
CQ = BC. Prove that AQCP is a parallelogram.
3

A-3. In the following figure, ABCD is a parallelogram in which A = 60º. If the bisectors of A and B
meet at P, prove that APB = 90º. Also, prove that AD = DP, PC = BC and DC = 2AD.

A-4. In a parallelogram ABCD, the bisector of A also bisects BC at P. Prove that AD = 2AB.

A-5. ABCD is a parallelogram and X and Y are points on the diagonal BD such that DX = BY. Prove that :
(i) AXCY is a parallelogram (ii) AX = CY, AY = CX

For Unacademy Subscription Use “PJLIVE” Code | Join t.me/pjsir42 for Updates
For More Info: “75970 – 84242, 94590 – 43333 / 2222”
(iii) AYB  CXD (iv) AXD  CYB

Section (B) : Mid-point theorem and its converse


B-1. In the following figure, AD is a median and DE || AB. Prove that BE is a median.

B-2. ABCD is a trapezium in which side AB is parallel to side DC and E is the mid-point of side AD. If F is
1
a point on side BC such that segment EF is parallel to side DC. Prove that EF = (AB + DC).
2

B-3. Prove that the line joining the mid-points of the diagonals of a trapezium is parallel to each of the
parallel sides and is equal to half of the difference of these sides.

B-4. In figure, ABCD is a parallelogram. E and F are the mid-points of the sides AB and CD respectively.
Prove that the line segments AF and CE trisect (divide into three equal parts) the diagonal BD.

B-5. P is the mid-point of side AB of a parallelogram ABCD. A line through B parallel to PD meet DC at Q
and AD produced at R. Prove that :
(i) AR = 2BC (ii) BR = 2BQ

B-6. In the adjoining figure, D, E, F are the midpoints of the sides BC, CA and AB of ABC. If BE and DF
1
intersect at X while CF and DE intersect at Y, prove that XY = BC.
4

OBJECTIVE QUESTIONS
Section (A) : Types of Quadrilaterals
A-1. In a parallelogram ABCD, D = 105º, then the A and B will be :
(A) 105º, 75º (B) 75º, 105º (C) 105º, 105º (D) 75º, 75º

A-2. When the diagonals of a parallelogram are perpendicular to each other then it is called :
(A) Square (B) rectangle (C) rhombus (D) parallelogram

A-3. ABCD is a rhombus with ABC = 56º, then the ACD will be :
(A) 56º (B) 62º (C) 124º (D) 34º

A-4. In an Isosceles trapezium ABCD if A = 45º then C will be :


(A) 90º (B) 135º (C) 120º (D) none of these
For Unacademy Subscription Use “PJLIVE” Code | Join t.me/pjsir42 for Updates
For More Info: “75970 – 84242, 94590 – 43333 / 2222”
A-5. In the adjoining figure, AP and BP are angle bisectors of A and B which meets at P on the
parallelogram ABCD. Then 2APB =

(A) C + D (B) A + C (C) B + D (D) 2C

A-6. In a quadrilateral ABCD, AO & DO are angle bisectors of A and D and given that C = 105º, B
= 70º, then the AOD is :
(A) 67.5º (B) 77.5º (C) 87.5º (D) 99.75º

A-7. From the figure, find the value of SQP and QSP of parallelogram PQRS.

(A) 60º, 50º (B) 60º, 45º (C) 70º, 35º (D) 35º, 70º

A-8. Two opposite angles of a parallelogram are (3x – 2)º and (50 – x)º then the value of x will be:-
(A) 17º (B) 16º (C) 15º (D) 13º

A-9. Which of the following properties are not TRUE for parallelogram ?
(A) Its diagonals are perpendicular to each other
(B) The diagonals divide the figure into four congruent triangles
(C) Its diagonals are equal
(D) All of the above

A-10. If the diagonals of a parallelogram are equal then it is a :


(A) rectangle (B) trapezium (C) rhombus (D) square

A-11. The diagonals of a rectangle ABCD meet at O. If BOC = 44º, then OAD is :
(A) 68º (B) 44º (C) 54º (D) None of these

A-12. If ABCD is a trapezium in which AB || CD and AD = BC, then A is equal to :


(A) 180º (B) 90º (C) B (D) None of these

A-13. In the adjoining figure ABCD is a parallelogram, then the measure of x is :

(A) 45º (B) 60º (C) 90º (D) 135º


For Unacademy Subscription Use “PJLIVE” Code | Join t.me/pjsir42 for Updates
For More Info: “75970 – 84242, 94590 – 43333 / 2222”
A-14. ABCD is a parallelogram and AP and CQ are the perpendiculars from A and C on its diagonal BD,
respectively. Then AP is equal to :
(A) DP (B) CQ (C) PQ (D) AB

A-15. In fig. ABCD is a parallelogram. P and Q are mid points of the sides AB and CD, respectively. Then
PRQS is

(A) parallelogram (B) trapezium (C) rectangle (D) none of these

Section (B) : Mid-point theorem and its converse.


B-1. In a triangle, P, Q and R are the mid-points of the sides BC, CA and AB respectively. If AC = 16 cm,
BC = 20 cm and AB = 24 cm, then the perimeter of the quadrilateral ARPQ will be :
(A) 60 cm (B) 30 cm (C) 40 cm (D) none of these

B-2. LMNO is a trapezium with LM || NO. If P and Q are the mid-points of LO and MN respectively and
LM = 5 cm and ON = 10 cm, then PQ =
(A) 2.5 cm (B) 5 cm (C) 7.5 cm (D) 15 cm

B-3. In a right angle triangle ABC is right angled at B. Given that AB = 9 cm, AC = 15 cm and D, E are the
midpoints of the sides AB and AC respectively, then the area of  ADE =
(A) 67.5 cm2 (B) 13.5 cm2 (C) 27 cm2 (D) data insufficient

B-4. In  ABC, AD is the median through A and E is the mid-point of AD. BE produced meets AC in F.
Then
1 1 1
(A) AF = AC. (B) AF = AC. (C) AF = AC. (D) None of these
4 3 2

Section (C) : Polygon


C-1. One angle of a pentagon is 140º. If the remaining angles are in the ratio 1 : 2 : 3 : 4. The size of the
greatest angle is
(A) 150º (B) 180º (C) 160º (D) 170º

C-2. Two regular polygons are such that the ratio between their number of sides is 1 : 2 and the ratio of
measures of their interior angles is 3 : 4. The number of sides of each polygon is
(A) 5, 10 (B) 6, 12 (C) 4, 8 (D) 2, 3

C-3. Each interior angle of a regular polygon is 144º. Find the interior angle of a regular polygon which has
double the number of sides as the first polygon.
(A) 100º (B) 160º (C) 36º (D) 162º

For Unacademy Subscription Use “PJLIVE” Code | Join t.me/pjsir42 for Updates
For More Info: “75970 – 84242, 94590 – 43333 / 2222”
EXERCISE – 02

OBJECTIVE QUESTIONS
1. In given figure, ABCD is a parallelogram. Compute the values of x and y.

(A) x = 6º and y = 7º (B) x = 3º and y = 2º (C) x = 5º and y = 6º (D) None of these


2. If an angle of a parallelogram is two-third the adjacent angles, Then both angles of a parallelogram.
(A) 60º, 120º (B) 36º, 144º (C) 72º, 108º (D) None of these
3. A quadrilateral having only one pair of opposite sides parallel is called a
(A) Square (B) Rhombus (C) Trapezium (D) Parallelogram
4. The angles of a quadrilateral are xº, (x – 10)º, (x + 30º) and (2x)º, the smallest angle is equal to
(A) 68º (B) 52º (C) 58º (D) 47º

5. In the given figure, ABCD is a rhombus. If A = 70º, then CDB is equal to

(A) 65º (B) 55º (C) 75º (D) 80º


6. Which is not correct about rectangle EFGH ?
(A) E = F = G = H = 90º (B) EG = FH
(C) EF = GH and HE = FG (D) EG and FH are angle bisectors.

7. PQRS is a square, PR and SQ intersect at O. The 19 easure of POQ is


(A) 45º (B) 90º (C) 180º (D) None of these

8. The measure all the angles of a parallelogram, if an angle is 24º less than twice the smallest angle, is
(A) 37º, 143º, 37º, 143º (B) 108º, 72º, 108º, 72º
(C) 68º, 112º, 68º, 112º (D) None of these

9. To construct rhombus uniquely it is necessary to know at least _______of its parts.


(A) 1 (B) 2 (C) 3 (D) 5

10. ABCD is a trapezium in which AB || CD. If ADC = 2ABC, AD = a cm and CD = b cm, then the
length (in cm) of AB is :
a 2 2
(A) + 2b (B) a + b (C) a+b (D) a + b
2 3 3

11. ABCD is a quadrilateral whose diagonals intersect each other at the point O such that OA = OB = OD.
If OAB = 30º, then the measure of ODA is :
(A) 30º (B) 45º (C) 60º (D) 90º

5xº 7xº
12. A quadrilateral ABCD has four angles xº, 2xº, and respectively. What is the difference
2 2
between the value of biggest and the smallest angles.
(A) 40º (B) 100º (C) 80º (D) 20º

For Unacademy Subscription Use “PJLIVE” Code | Join t.me/pjsir42 for Updates
For More Info: “75970 – 84242, 94590 – 43333 / 2222”
13. In the figure, z =

2xy x+y
(A) (B) xy (C) x 2 + y2 (D)
x+y 2
14. In the trapezium shown, AB || DC, and E and F are the midpoints of the two diagonals. If DC = 60 and
EF = 5 then the length of AB is equal to :

(A) 40 (B) 45 (C) 50 (D) 55


15. Suppose the triangle ABC has an obtuse angle at C and let D be the midpoint of side AC. Suppose E is
on BC such that the segment DE is parallel to AB. Consider the following three statements.
(i) E is the midpoint of BC
(ii) The length of DE is half the length of AB
(iii) DE bisects the altitude from C to AB
(A) only (i) is true (B) only (i) and (ii) are true
(C) only (i) and (iii) are true (D) all three are true.
16. The line joining the mid points of the diagonals of a trapezium has length 3. If the longer base is 97,
then the shorter base is :
(A) 94 (B) 92 (C) 91 (D) 90

17. P is the mid-point of side AB to a parallelogram ABCD. A line through B parallel to PD meets DC at Q
and AD produced at R. Then BR is equal to

1
(A) BQ (B) (C) 2BQ (D) none of these
2

18. PQRS is a parallelogram. PX and QY are, respectively, the perpendicular from P and Q to SR and SR
produced. Then PX is equal to

1
(A) QY (B) 2QY (C) QY (D) XR
2

19. X, Yare the mid-points of opposite sides AB and DC of a parallelogram ABCD. AY and DX are joined
intersecting in P; CX and BY are joined intersecting in Q. Then PXQY is a

(A) rectangle (B) rhombus (C) parallelogram (D) square


20. PQRS is a square. The SRP is equal to

For Unacademy Subscription Use “PJLIVE” Code | Join t.me/pjsir42 for Updates
For More Info: “75970 – 84242, 94590 – 43333 / 2222”
(A) 45º (B) 90º (C) 100º (D) 60º

21. ABCD is a trapezium in which AB || DC and AD = BC. If P, Q, R, S be respectively the mid-points of


BA, BD, CD and CA. Then PQRS is a

(A) Rhombus (B) Rectangle (C) Parallelogram (D) Square

22. In a ABC, P, Q and R are the mid-points of sides BC, CA and AB respectively. If AC = 21 cm,
BC = 29 cm and AB = 30 cm. The perimeter of the quad. ARPQ is

(A) 91 cm (B) 60 cm (C) 51 cm (D) 70 cm

23. Find the measure of each angle of a parallelogram, if one of its angle is 30º less than twice the smallest
angle.
(A) 60º, 100º, 90º, 20º (B) 80º, 40º, 120º, 90º (C) 100º, 90º, 90º, 80º (D) 70º, 110º, 70º, 110º

24. ABCD is a parallelogram and X, Y are the mid-points of sides AB and CD respectively. Then
quadrilateral AXCY is a

(A) parallelogram (B) rhombus (C) square (D) rectangle

25. ABCD is a parallelogram, E is the mid-point of AB and CE bisects BCD. The DEC is

(A) 60º (B) 90º (C) 100º (D) 120º

26. The measures of the angles of a quadrilateral ABCD are respectively in the ratio 1 : 2 : 3 : 4. Then
which of the following is true?

(A) AC = BD (B) AD || BC (C) AB || DC (D) None of these

27. ABCD is a trapezium in which AB || CD. Then AC2 + BD2 is equal to

For Unacademy Subscription Use “PJLIVE” Code | Join t.me/pjsir42 for Updates
For More Info: “75970 – 84242, 94590 – 43333 / 2222”
(A) AD2 + BC2 – 2AB. CD. (B) AD2 + BC2 + 2AB. CD.
(C) AD2 – BC2 + 2AB. CD. (D) AD2 – BC2 – 2AB. CD.

28. In the given figure, PQRS is a parallelogram. PO and QO are respectively, the bisectors of P and Q.
Line LOM is drawn parallel to PQ. Then PL is equal to

(A) PQ (B) QR (C) QM : (D) OQ

29. In a quadrilateral ABCD, the line segments bisecting C and D meet at E. Then A + B is equal to

1
(A) CED (B) CED (C) 2CED (D) None of these
2

30. ABCD is a trapezium and P, Q are the mid-points of the diagonal’s AC and BD. Then PQ is equal to

1 1 1 1
(A) (AB) (B) (CD) (C) (AB – CD) (D) (AB + CD)
2 2 2 2

31. The ratio of the measure of an angle of a regular octagon to the measure of its exterior angle is
(A) 3 : 1 (B) 2 : 1 (C) 1 : 3 (D) 1 : 2

32. The exterior angle of a regular polygon is one-third of its interior angle. The number of the sides of the
polygon is
(A) 9 (B) 8 (C) 10 (D) 12

33. The bisectors of the angles of a parallelogram encloses a

(A) Rhombus (B) Rectangle (C) Square (D) None of these

ADE
34. ABCDE is a regular pentagon, Diagonal AD divides CDE in two parts, then the ratio of is
ADC
equal to
(A) 3 : 1 (B) 1 : 4 (C) r : 3 (D) 1 : 2

35. The ratio of an interior angle to the exterior angle of a regular polygon is 7 : 2. The number of sides of
the polygon is
(A) 9 (B) 19 (C) 7 (D) 8
For Unacademy Subscription Use “PJLIVE” Code | Join t.me/pjsir42 for Updates
For More Info: “75970 – 84242, 94590 – 43333 / 2222”
36. Find P + Q + R + S + T.

(A) 90º (B) 60º (C) 45º (D) 180º

37. ABCDE is a regular pentagon. The bisector of A of the pentagon meets the side CD in M. Then the
measure of AMC is
(A) 54º (B) 45º (C) 90º (D) 100º

38. In the given figure, AE = BC and AE || BC and the three sides AB, CD and ED are equal in length. Find
the measure of ECD.

(A) 138º (B) 60º (C) 88º (D) None of these

39. ABCD is a square, A is joined to a point P on BC and D is joined to a point Q on AB. If AP = DQ and
AP intersects DQ at R, then DRP is
(A) 60º (B) 120º (C) 90º (D) can't be determined

40. A point X inside a rectangle PQRS is joined to the vertices then, which of the following is true?
(A) Area ( PSX) = Area ( RXQ)
(B) Area ( PSX) + Area ( PXQ) = Area ( RSX) + Area ( RQX)
(C) Area ( PXS) + Area ( RXQ) = Area ( SRX) + Area ( PXQ)
(D) None of the above

41. If ABCD is a parallelogram in which P and Q are the centroids of ABD and BCD, then, PQ equals
(A) AQ (B) AP (C) BP (D) DQ

42. If area of a parallelogram with sides "' and "b" is "A" and that of a rectangle with sides "" and "b" is
"B", then
(A) A < B (B) A = B (C) A > B (D) None of these

43. Diagonals of a parallelogram are 8m and 6m respectively. If one of sides is 5 m, then the area of
parallelogram is
(A) 18m2 (B) 30m2 (C) 24m2 (D) 48m2

44. ( ABCD) is a parallelogram, AB = 14 cm, BC = 18 cm and AC = 16 cm. Find the length of the other
diagonal.
(A) 24 cm (B) 28 cm (C) 36 cm (D) 32 cm

45. ABCD is parallelogram. The diagonals AC and BD intersect at a point "O". If E, F, G and H are the
mid-points of AO, DO, CO and BO respectively, then the ratio of (EF + FG + GH + HE) to (AD + DC
+ CB + BA) is
(A) 1 : 1 (B) 1 : 2 (C) 1: 3 (D) 1 : 4

For Unacademy Subscription Use “PJLIVE” Code | Join t.me/pjsir42 for Updates
For More Info: “75970 – 84242, 94590 – 43333 / 2222”
46. In a rectangle ABCD, P and Q are the mid-points of BC and AD respectively and R is any point on PQ,
then ar(ARB) equals
1 1 1
(A) ( ABCD) (B) ( ABCD) (C) ( ABCD) (D) None of these
2 3 4

EXERCISE – 03

NTSE PROBLEMS (PREVIOUS YEARS)


1. In the given diagram, ABCD is a parallelogram. Bisectors of A and B meet at the point X. Then the
value of A X B is - [Raj. NTSE Stage-1 2005]

(A) 150º (B) 120º (C) 90º (D) 60º

2. The short diagonal of a rhombus is equal to its side, then the value of angle opposite to its longer
diagonal is [Raj. NTSE Stage-1 2005]
(A) 30º (B) 60º (C) 90º (D) 120º

3. The bisectors of angles of a parallelogram makes a figure which is [Raj. NTSE Stage-1 2013]
(A) Rectangle (B) Circle (C) Pentagon (D) Octagon

4. PQRS is a parallelogram and M, N are the mid-points of PQ and RS respectively. Which of the
following is not true ? [M.P. NTSE Stage-1 2013]

(A) RM trisects QS (B) PN trisects QS (C) PSN  RQM (D) MS is not parallel to QN

5. An obtuse angle of a rhombus is greater than twice the acute angle by 30º. Find the measure of each
angle. [Maharashtra NTSE Stage- 1_2014]
(A) 50º (B) 130º (C) 80º (D) 60º

6. ABCD is a rhombus and P, Q, R, S are respectively mid points of sides AB, BC, CD, DA. Then RSP
is : [MP NTSE Stage-I_2014]
(A) 120º (B) 90º (C) 60º (D) 30º

7. If the angle bisectors of DAB and CBA of any quadrilateral ABCD intersects at the Point P, then
find 2mAPB : [Maharashtra NTSE Stage- 1_2014]

For Unacademy Subscription Use “PJLIVE” Code | Join t.me/pjsir42 for Updates
For More Info: “75970 – 84242, 94590 – 43333 / 2222”
(A) C + D (B) C + B (C) A + B (D) A + D

1
8. The side of a rhombus is 10 cm. The smaller diagonal is of the greater diagonal. Find the length of
3
the greater diagonal : [MP NTSE Stage-I_2015]
(A) 6 10 cm (B) 10 6cm (C) 6 5 cm (D) 5 6 cm

9. Sides AB and CD of a quadrilateral. ABCD are extended as in figure. Then a + b is equal to


[Raj. NTSE Stage-1 2016]

(A) x + 2y (B) x – y (C) x + y (D) 2x + y.

10. Angle at A in trapezium ABCD if AB = 18 cm, BC = 10 cm, CD = 12 cm, DA = 8cm, AB || CD, will be
: [West Bengal NTSE Stage-1 2016]
(A) 80º (B) 45º (C) 90º (D) None of these

11. The line segment joining the mid-points of the adjacent sides of a quadrilateral form :
[MP NTSE Stage-I_2016]
(A) parallelogram (B) Square (C) Rhombus (D) Rectangle

12. In a rhombus of side 10 cm, one of the diagonal is 12 cm long, the length of second diagonal will be :
[MP NTSE Stage-I_2016]
(A) 4 cm (B) 8 cm (C) 12 cm (D) 16 cm

13. The length of the side of a rhombus is 4 cm. If one of the diagonals is equal to the side of rhombus, then
the length of other diagonal in cm will be [Rajasthan NTSE Stage-I/18]
3
(A) (B) 3 (C) 2 3 (D) 4 3
2
14. The difference between the two adjacent angles of a parallelogram is 20º. What would be the ratio
between the smaller and the longer angles of the parallelogram respectively.
[Jharkhand NTSE Stage–I/18]
(A) 4:5 (B) 4 : 7 (C) 3:5 (D) 5: 6

15. Shape made by the bisectors of angles of a parallelogram is. [Madhya Pradesh NTSE Stage-I/18]
(A) Rectangle (B) Square (C) Circle (D) Straight line

16. Which of the following statements is false for the quadrilateral ABCD? [Rajasthan NTSE Stage-I/18]
(A) AB + BC + CD + DA > AC (B) AB + BC + CD + DA > AB + AC
(C) AB + BC + CD + DA > AC + BD (D) AB + BC + CD + DA < 2AC.

For Unacademy Subscription Use “PJLIVE” Code | Join t.me/pjsir42 for Updates
For More Info: “75970 – 84242, 94590 – 43333 / 2222”
17. The sum of all interior angles of a Heptagon is [Rajasthan NTSE Stage-I/19]
(A) 360º (B) 540º (C) 720º (D) 900º

VALUE BASED / PRACTICAL BASED QUESTIONS

1. There is a visit in a school by Directorate of Education. Girls are asked to prepare a rangoli in triangular
shape. Dimensions of ABC are 26 cm, 28 cm, 25 cm. Garland is to be placed along the side of PQR,
which is formed by joining mid-points of sides of ABC.

(i) Find the length of garland.


(ii) Which values are depicted here?

2. A class teacher gave students colored papers made by recycling of waste products in shape of
quadrilateral. She asked them to make a parallelogram from it using paper folding.

(i) How can a parallelogram be formed by using paper folding ?


(ii) Prove that it is a parallelogram.
(iii) Which values are depicted here?

For Unacademy Subscription Use “PJLIVE” Code | Join t.me/pjsir42 for Updates
For More Info: “75970 – 84242, 94590 – 43333 / 2222”
Answer Key

BOARD LEVEL EXERCISE


TYPE (I) : VERY SHORT ANSWER TYPE QUESTIONS : [01 MARK EACH]
1. 40º 2. 120º 3. 50º 4. 50º 5. Trapezium
6. Rectangle 7. AC = 6 cm, BD = 4 cm

TYPE (II) : SHORT ANSWER TYPE QUESTIONS : [02 MARKS EACH]


8. 84º 9. 60º, 120º, 60º, 120º 10. A = C = 60º, B = D = 120º

TYPE (III) : LONG ANSWER TYPE QUESTIONS: [03 MARK EACH]


16. 4 cm

EXERCISE – 01

SUBJECTIVE QUESTIONS
Section (A) : Types of Quadrilaterals
A-1. (i) 80º (ii) 80º (iii) 40º (iv) 25º

OBJECTIVE QUESTIONS
Section (A) : Types of Quadrilaterals
A-1. (B) A-2. (C) A-3. (B) A-4. (B) A-5. (A)
A-6. (C) A-7. (A) A-8. (D) A-9. (D) A-10. (A)
A-11. (A) A-12. (C) A-13. (C) A-14. (B) A-15. (A)

Section (B) : Mid-point theorem and its converse.


B-1. (C) B-2. (C) B-3. (B) B-4. (B)

Section (C) : Polynomial


C-1. (C) C-2. (A) C-3. (D)

EXERCISE – 02

OBJECTIVE QUESTIONS
Ques. 1 2 3 4 5 6 7 8 9 10 11 12 13 14 15 16 17 18 19 20
Ans. A C C C B D B C B B C B D C D C C A C A
Ques. 21 22 23 24 25 26 27 28 29 30 31 32 33 34 35 36 37 38 39 40
Ans. A C D A B C B C C C A B B D A D C B C C
Ques. 41 42 43 44 45 46
Ans. B A C B B C

EXERCISE – 03

For Unacademy Subscription Use “PJLIVE” Code | Join t.me/pjsir42 for Updates
For More Info: “75970 – 84242, 94590 – 43333 / 2222”
NTSE PROBLEMS (PREVIOUS YEARS)
Ques. 1 2 3 4 5 6 7 8 9 10 11 12 13 14 15 16 17
Ans. C D A D B B A A C C A D D A A D D

VALUE BASED / PRACTICAL BASED QUESTIONS

1. (i) 39.5 cm
(ii) Encouraging creativity, working in groups.

2. (i) By joining the mid- points of sides of a quadrilateral.


(ii) PQRS is a parallelogram proved.
(iii) Environment protection

For Unacademy Subscription Use “PJLIVE” Code | Join t.me/pjsir42 for Updates
For More Info: “75970 – 84242, 94590 – 43333 / 2222”
AREA OF PARALLELOGRAMS AND
5 TRIANGLES
INTRODUCTION

Polygon region
Polygon region can be expressed as the union of a finite number of triangular regions in a plane such that if two
of these intersect, their intersection is either a point or a line segment. It is the shaded portion including its sides
as shown in the figure.

Area Axioms :
Every polygonal region R has an area, measured in square units and denoted by ar(R).
(i) Congruent area axiom : If R1 and R2 be two regions such that R1  R2 then ar(R1) = ar (R2).
(ii) Area addition axiom : If R1 and R2 are two polygonal regions, whose intersection is a finite number of
points & line segments such that R = R1  R2, then ar (R) = ar (R1) + ar (R2).
(iii) Rectangular area axiom : If AB = a metre and AD = b metre then, ar (Rectangular region ABCD) = ab
sq.m.

A. AREA OF PARALLELOGRAM

(a) Base and Altitude of a Parallelogram :


(i) Base : Any side of a parallelogram can be called its base.
(ii) Altitude : The length of the line segment which is perpendicular to the base from the opposite side is
called the altitude or height of the parallelogram corresponding to the given base.

(i) DL is the altitude of ||gm ABCD, corresponding to the base AB.


(ii) DM is the altitude of ||gm ABCD, corresponding to the base BC.
Theorem : A diagonal of a parallelogram divides it into two triangles of equal area.
Given : A parallelogram ABCD whose one of the diagonals is BD.
To prove : ar (ABD) = ar (CDB).
Proof : In ABD and CDB.
AB = DC [Opposite sides of a ||gm]
AD = BC [Opposite sides of a ||gm]
BD = BD [Common side]
 ABD  CDB [By SSS congruency]
 ar (ABD) = ar (CDB) Hence Proved.
Theorem : Parallelograms on the same base and between the same parallels are equal in area.
Given : Two ||gms ABCD and ABEF on the same base AB and between the same parallels AB and FC.
To prove : ar(||gm ABCD) = ar(||gm ABEF)

For Unacademy Subscription Use “PJLIVE” Code | Join t.me/pjsir42 for Updates
For More Info: “75970 – 84242, 94590 – 43333 / 2222”
Proof : In ADF and BCE, we have
AD = BC [Opposite sides of a ||gm]
AF = BE [Opposite sides of a ||gm]
DAF = CBE [ AD || BC and AF || BE]
[Angle between AD and AF = Angle between BC and BE]
 ADF  BCE [By SAS congruency]
 ar(ADF) = ar(BCE) ...(i)
 ar(||gm ABCD) = ar(ABED) + ar(BCE)
= ar(ABED) + ar(ADF) [Using (i)]
gm
= ar(|| ABEF).
Hence, ar(||gm ABCD) = ar(||gm ABEF). Hence Proved.
Theorem : The area of parallelogram is the product of its base and the corresponding altitude.
Given : A ||gm ABCD in which AB is the base and AL is the corresponding height.
To prove : Area (||gm ABCD) = AB × AL.
Construction : Draw BM ⊥ DC, so that rectangle ABML is formed.
Proof : ||gm ABCD and rectangle ABML are on the same base AB and between
the same parallel lines and LC.
 ar(||gm ABCD) = ar(rectangle ABML) = AB × AL.
 area of a ||gm = base × height. Hence Proved.

(b) Area of Triangle


Theorem : Two triangles on the same base (or equal bases) and between the same parallels are
equal in area.
Given : Two triangles ABC and PBC on the same base BC and between the same parallel lines BC and AP.
To prove : ar(ABC) = ar(PBC).
Construction : Through B, draw BD || CA intersecting AP produced in D and through C, draw CQ || BP,
intersecting PA produced in Q.
Proof : BD || CA [By construction]
And, BC || DA [Given]
 Quadrilateral BCAD is a parallelogram.
Similarly, Quadrilateral BCQP is a parallelogram.
Now, parallelogram BCQP and BCAD are on the same base BC, and
between the same parallels.
 ar (||gm BCQP) = ar (||gm BCAD) ....(i)
Diagonals of a parallelogram divides it into two triangles of equal area.
1
 ar (PBC) = ar (||gm BCQP) ...(ii)
2
1
And ar (ABC) = ar (||gm BCAD) ...(iii)
2
Now, ar (||gm BCQP) = ar (||gm BCAD) [From (i)]
1 1
 ar (||gm BCQD) = ar(||gm BCQP)
2 2
Hence, ar (ABC) = ar (PBC) [Using (ii) and (iii)] Hence Proved.
Theorem : If a triangle and a parallelogram are on the same base and between the same parallels,
the area of the triangle is equal to half of the parallelogram.
Given : A ABC and a paralellogram BCDE on the same as BC and between the same parallels BC
and AD.
1
To prove : ar (ABC) = ar (parallelogram BCDE)
2
Construction : Draw AL ⊥ BC and DM ⊥ BC, meeting BC producted in M.
Proof : Since, E and D are colinear and BC || AD
 AL = DM ..............(i)
[ distance between parallel lines is always same]

For Unacademy Subscription Use “PJLIVE” Code | Join t.me/pjsir42 for Updates
For More Info: “75970 – 84242, 94590 – 43333 / 2222”
1
Now, ar (ABC) = (BC × AL)
2
1
 ar (ABC) = (BC × DM) [ AL = DM (from (i)]
2
1
 ar (ABC) = ar (parallelogam BCDE).
2
Theorem : The area of a trapezium is half the product of its height and the sum of the parallel sides.
Given : Trapezium ABCD in which AB || DC, AL ⊥ DC, CN ⊥ AB and
AL = CN = h (say), AB = a, DC = b.
1
To prove : ar(trapezium ABCD) = h × (a + b).
2
Construction : Join AC.
Proof :  AC is a diagonal of quad. ABCD.
 ar(trapezium ABCD) = ar(ABC) + ar(ACD)
1 1 1
= h × a + h × b = h(a + b). Hence Proved.
2 2 2
Theorem : Median of a triangle divides it into two triangles of equal area.
Given : A ABC in which AD is the median.
To Prove : ar (ABD) = ar(ADC).
Construction : Draw AL ⊥ BC.
Proof : Since, AD is the median of ABC. Therefore, D is the midpoint of BC.
1
 ar (ABD) = (BD × AL) .. (i)
2
1
 ar (ADC) = (CD × AL)
2
1
 ar (ADC) = (BD × AL) .. (ii) [BD = CD, AD is the median of ABC]
2
From (i) & (ii)
ar (ABD) = ar(ADC). Hence Proved.

Solved Examples

Example.1 In a parallelogram ABCD, AB = 8 cm. The altitudes corresponding to sides AB and AD are
respectively 4 cm and 5 cm. Find AD.
Solution. Area of a ||gm = Base × corresponding altitude
 Area of parallelogram ABCD = AD × BN = AB × DM

 AD × 5 = 8 × 4
8 4
 AD = = 6.4 cm.
5
Example.2 The diagonals of a parallelogram ABCD intersect in O. A line through O meets AB in X and
1
the opposite side CD in Y. Show that ar (quadrilateral AXYD) = ar(parallelogram ABCD).
2
Solution. AC is a diagonal of the parallelogram ABCD.
1
ar (ACD) = ar(|| gm ABCD) ...(i)
2
Now, in s AOX and COY,

AO = CO [Diagonals of a parallelogram bisect each other]


AOX = COY [Vertically opposite s]
For Unacademy Subscription Use “PJLIVE” Code | Join t.me/pjsir42 for Updates
For More Info: “75970 – 84242, 94590 – 43333 / 2222”
OAX = OCY [Alternate interior s]
[ AB || DC and transversal AC intersects them]
 AOX  COY [By ASA congruency]
 ar(AOX) = ar(COY) ...(ii)
Adding ar(quad. AOYD) to both sides of (ii), we get
ar(quad. AOYD) + ar(AOX) = ar (quad. AOYD) + ar (COY)
1
 ar(quad. AXYD) = ar(ACD) = ar(|| gm ABCD) Hence Proved.
2

Example.3 ABCD is a trapezium with AB || DC. A line parallel to AC intersects AB at X and BC at Y.


Prove that ar(ADX) = ar(ACY).
Solution. Join CX, DX and AY.
Clearly, triangles ADX and ACX are on the same base AX
and between the parallels AB and DC.
 ar (ADX) = ar (ACX) ... (i)
Also, ACX and ACY are on the same base AC and
between the parallels AC and XY.
 ar (ACX) = ar (ACY) ...(ii)
From (i) and (ii), we get
ar (ADX) = ar (ACY).

Example.4 ABCD is a quadrilateral. A line through D, parallel to AC,


meets BC produced in P as shown in figure. Prove that ar
(ABP) = ar(quad. ABCD).
Solution. Since s ACP and ACD are on the base AC and between
the same parallels AC and DP.
 ar(ACP) = ar(ACD)
 ar(ACP) + ar(ABC) = ar(ACD) + ar(ABC)
 ar(ABP) = ar(quad. ABCD).

Example.5 In figure, E is any point on median AD of a ABC. Show that ar(ABE) = ar(ACE).

Solution.

Construction : From A, draw AG ⊥ BC and from E draw EF ⊥ BC.


BD  AG DC  AG
Proof : ar(ABD) = and ar(ADC) =
2 2
But, BD = DC [D is the mid-point of BC, AD being the median]
ar(ABD) = ar(ADC) ... (i)
BD  EF DC  EF
Again, ar(EBD) = and ar(EDC) =
2 2
But, BD = DC
 ar(EBD) = ar(EDC) ... (ii)
Subtracting (ii) from (i), we get
ar(ABD) – ar(EBD) = ar(ADC) – ar(EDC)
 ar(ABE) = ar(ACE). Hence Proved.

Example.6 Triangles ABC and DBC are on the same base BC; with A, D on opposite sides of the line BC,
such that ar(ABC) = ar(DBC). Show that BC bisects AD.
Solution. Construction : Draw AL ⊥ BC and DM⊥ BC.

For Unacademy Subscription Use “PJLIVE” Code | Join t.me/pjsir42 for Updates
For More Info: “75970 – 84242, 94590 – 43333 / 2222”
Proof : ar(ABC) = ar(DBC) [Given]
BC  AL BC  DM
 =
2 2
 AL = DM ...(i)
Now in s OAL and OMD
AL = DM [From (i)]
 ALO = DMO [Each = 90º]
 AOL = MOD [Vertically opposite s]
 OLA  OMD [By AAS congruency]
 OA = OD [By CPCT]
i.e., BC bisects AD. Hence Proved.

Example.7 ABC is a triangle in which D is the mid-point of BC and E is the mid-point of AD. Prove that
1
the area of BED = area of ABC.
4
Solution. Given : A ABC in which D is the mid-point of BC and E is the mid-point of AD.
1
To prove : ar(BED) = ar(ABC).
4
Proof :  AD is a median of ABC.
1
 ar (ABD) = ar(ADC) = ar (ABC) ... (i)
2
[Median of a triangle divides it into two triangles of equal area]
Again,
 BE is a median of ABD.
1
 ar (BEA) = ar(BED) = ar (ABD)
2
[Median of a triangle divides it into two triangles of equal area]
1 1 1
And ar(BED) = ar(ABD) = × ar (ABC) [From (i)]
2 2 2
1
 ar (BED) = ar(ABC). Hence Proved.
4
1
Example.8 If the medians of a ABC intersect at G, show that ar(AGB) = ar(AGC) = ar(BGC) =
3
ar(ABC).
Solution. Given : A ABC and its medians AD, BE and CF intersect at G.
1
To prove : ar(AGB) = ar(AGC) = ar(BGC) = ar(ABC).
3
Proof : A median of a triangle divides it into two triangles of equal area.
In ABC, AD is the median.
 ar(ABD) = ar(ACD) ...(i)
In GBC, GD is the median.
 ar(GBD) = ar(GCD) ...(ii)
Subtract equation (ii) from (i), we get
ar(ABD) – ar(GBD) = ar(ACD) – ar(GCD)
 ar(AGB) = ar(AGC) ...(iii)
Similarly, ar(AGB) = ar(BGC) ...(iv)
From (iii) & (iv)
ar(AGB) = ar(AGC) = ar(BGC)
But, ar(ABC) = ar(AGB) + ar(AGC) + ar(BGC) = 3 ar (AGB)
1
 ar(AGB) = ar(ABC).
3

For Unacademy Subscription Use “PJLIVE” Code | Join t.me/pjsir42 for Updates
For More Info: “75970 – 84242, 94590 – 43333 / 2222”
1
Hence, ar(AGB) = ar(AGC) = ar(BGC) = ar(ABC). Hence proved
3

Example.9 D, E and F are respectively the mid points of the sides BC, CA and AB of a ABC. Show that :
1
(i) BDEF is a parallelogram (ii) ar(||gmBDEF) = ar(ABC)
3
1
(iii) ar(DEF) = ar(ABC)
4
Solution. (i) In ABC,
 F is the mid-point of side AB and E is the mid point of side AC.
 EF || BD [Line joining the mid-points of any two sides of a is
parallel to the third side.]
Similarly, ED || FB.
Hence, BDEF is a parallelogram. Hence Proved.

(ii) Similarly, we can prove that AFDE and FDCE are parallelograms.
 FD is a diagonal of parallelogram BDEF.
 ar(FBD) = ar(DEF) ...(i)
Similarly, ar(FAE) = ar(DEF) ...(ii)
And, ar(DCE) = ar(DEF) ...(iii)
From above equations, we have
ar(FBD) = ar(FAE) = ar(DCE) = ar(DEF)
and ar (FBD) + ar (DCE) + ar (DEF) + ar (FAE) = ar (ABC)
 2 [ar(FBD) + ar(DEF)] = ar(ABC) [By using (ii) and (iii)]
1
 2 [ar. (IIgm BDEF)] = ar (ABC)  ar (||gm BDEF) = ar (ABC).
2
(iii) Since, ABC is divided into four non-overlapping triangles FBD, FAE, DCE and DEF.
 ar(ABC) = ar(FBD) + ar(FAE) + ar(DCE) + ar(DEF)
 ar (ABC) = 4 ar (ABC) [Using (i), (ii) and (iii)]
1
 ar(DEF) = ar(ABC). Hence Proved.
4

Example.10 In figure, P is a point in the interior of a rectangle ABCD. Show that

1
(i) ar(APD) + ar(PBC) = ar(rectangle ABCD)
2
(ii) ar(APD) + ar(PBC) = ar(APB) + ar(PCD)
Solution. Construction : Draw EPF || AB || CD and LPM || AD || BC.
Proof :
(i) EPF || AB and DA cuts them.
 DEP = EAB = 90º [Corresponding angles]
 PE ⊥ AD.
Similarly, PF ⊥ BC; PL ⊥ AB and PM ⊥ DC.
1  1  1
 ar(APD) + ar(BPC) =   AD  PE  +   BC  PE  = AD × (PE + PF)
2  2  2
[ BC = AD]

For Unacademy Subscription Use “PJLIVE” Code | Join t.me/pjsir42 for Updates
For More Info: “75970 – 84242, 94590 – 43333 / 2222”
1 1 1
=  AD  EF =  AD  AB = × ar(rectangle ABCD).
2 2 2
1  1  1
(ii) ar (APB) + ar(PCD) =   AB  PL  +   DC  PM  =  AB  (PL+ PM)
2  2  2
[ DC = AB]
1 1
= × AB × LM = × AB × AD [ LM = AD]
2 2
1
= × ar(rect. ABCD).
2
 ar(APD) + ar(PBC) = ar(APB) + ar(PCD) Hence Proved.

Check Point - A

1. Find the area of the parallelogram whose base is 8.5 cm and height 4 cm.
2. Find the base of a parallelogram whose area is 85 sq. cm and the altitude are 17 cm.
3. Prove that the area of a rhombus is equal to half the product of its diagonals.

4. In ABC, P is any point on the base BC. Q is the midpoint of AP. Show that area of the
1
QBC = area of ABC.
2

5. ABC is a triangle and a straight line DE, drawn parallel to BC cuts the sides AB and AC at D and E
respectively. Prove that area of ABE = area of ACD.

6. In the quadrilateral ABCD, diagonal BD bisects AC at right angles. If P and Q are the middle points of
3
AB and AD respectively, prove that PQC = quadrilateral ABCD.
8

For Unacademy Subscription Use “PJLIVE” Code | Join t.me/pjsir42 for Updates
For More Info: “75970 – 84242, 94590 – 43333 / 2222”
7. ABCD is a quadrilateral. A line drawn through D parallel to AC meets BC produced at P. Prove that
(i) Area of BAP = Area of quadrilateral ABCD
(ii) Area of AOD = Area of COP

8. ABCD is a parallelogram whose diagonals AC and BD meet at O. M and N are the mid points of OB
and OD respectively. Prove that AMCN is a parallelogram whose area is half that of ABCD.

9. In the figure ABCD is a parallelogram. P, Q, R and S are the mid points of AB, BC, CD and DA
respectively. Prove that the area of the parallelogram PQRS in equal to the half the area of the
parallelogram ABCD.

10. In the figure ABCD is a trapezium and YX || AC. Show that area of triangle BCX is equal to area of
triangle ACY.

Answers
1. 34 cm2 2. 5 cm

For Unacademy Subscription Use “PJLIVE” Code | Join t.me/pjsir42 for Updates
For More Info: “75970 – 84242, 94590 – 43333 / 2222”
BOARD LEVEL EXERCISE

TYPE (I) : VERY SHORT ANSWER TYPE QUESTIONS : [01 MARK EACH]
1. Find the area of the figure formed by joining the mid-points of the adjacent sides of a rhombus with
diagonals 12 cm and 16 cm.

2. In which of the following figures (Figure), you find two polygons on the same base and between the
same parallels?

(i) (ii)

(iii) (iv)

3. Name the figure obtained by joining the mid-points of the adjacent sides of a rectangle of sides 8 cm
and 6 cm and also find its area.

4. The mid-point of the sides of a triangle along with any of the vertices as the fourth point make a
parallelogram of area equal to x (ar ABC). Find x.

5. If a triangle and a parallelogram are on the same base and between same parallels, then find the ratio of
the area of the triangle to the area of parallelogram.

TYPE (II) : SHORT ANSWER TYPE QUESTIONS : [02 MARKS EACH]


1
6. If in Figure, PQRS and EFRS are two parallelograms, then prove that ar (MFR) = ar (PQRS).
2

7. PQRS is a rectangle inscribed in a quadrant of a circle of radius 13 cm. A is any point on QR. If PS = 05 cm,
then prove that ar (PAS) = 30 cm2.

8. ABC and BDE are two equilateral triangles such that D is the mid-point of BC. Then prove that ar
1
(BDE) = ar(ABC).
4

For Unacademy Subscription Use “PJLIVE” Code | Join t.me/pjsir42 for Updates
For More Info: “75970 – 84242, 94590 – 43333 / 2222”
9. PQRS is a square. T and U are respectively, the mid-points of PS and QR (Figure). Find the area of
OTS, if PQ = 8 cm, where O is the point of intersection of TU and QS.

10. ABCD is a parallelogram and BC is produced to a point Q such that AD = CQ (Figure). If AQ


intersects DC at P, show that ar (BPC) = ar (DPQ)

11. In Figure, PSDA is a parallelogram. Points Q and R are taken on PS such that PQ = QR = RS and PA ||
QB || RC. Prove that ar (PQE) = ar (CFD).

12. X and Y are points on the side LN of the triangle LMN such that LX = XY = YN. Through X, a line is
drawn parallel to LM to meet MN at Z (See Figure). Prove that ar (LZY) = ar (MZYX)

For Unacademy Subscription Use “PJLIVE” Code | Join t.me/pjsir42 for Updates
For More Info: “75970 – 84242, 94590 – 43333 / 2222”
13. The area of the parallelogram ABCD is 90 cm2 (see Figure). Find

(i) ar (ABEF) (ii) ar (ABD) (iii) ar (BEF)

14. ABCD is a square. E and F are respectively the mid- points of BC and CD. If R is the mid-point of EF
(Figure), prove that ar (AER) = ar (AFR)

TYPE (III) : LONG ANSWER TYPE QUESTIONS: [03 MARK EACH]


15. ABCD is a trapezium with parallel sides AB = a cm and DC = b cm (Figure). E and F are the mid-
points of the non-parallel sides. Then find ratio of ar (ABFE) and ar (EFCD).

16. ABCD is a parallelogram in which BC is produced to E such that CE = BC (Figure). AE intersects CD


at F. If ar (DFB) = 3 cm2, find the area of the parallelogram ABCD.

For Unacademy Subscription Use “PJLIVE” Code | Join t.me/pjsir42 for Updates
For More Info: “75970 – 84242, 94590 – 43333 / 2222”
17. If the mid-points of the sides of a quadrilateral are joined in order, prove that the area of the
parallelogram so formed will be half of the area of the given quadrilateral (Figure).

18. In Figure l, m, n, are straight lines such that l || m and n intersect l at P and m at Q. ABCD is a
quadrilateral such that its vertex A is on l. The vertices C and D are on m and AD || n. Show that ar
(ABCQ) = ar (ABCDP)

1
19. In Figure, BD || CA, E is mid-point of CA and BD = CA. Prove that ar (ABC) = 2ar(DBC).
2

20. A point E is taken on the side BC of a parallelogram ABCD. AE and DC are produced to meet at F.
Prove that ar (ADF) = ar (ABFC)

21. The diagonals of a parallelogram ABCD intersect at a point O. Through O, a line is drawn to intersect
AD at P and BC at Q. Show that PQ divides the parallelogram into two parts of equal area.

22. The medians BE and CF of a triangle ABC intersect at G. Prove that the area of GBC = area of the
quadrilateral AFGE.

For Unacademy Subscription Use “PJLIVE” Code | Join t.me/pjsir42 for Updates
For More Info: “75970 – 84242, 94590 – 43333 / 2222”
23. In Figure, CD || AE and CY || BA. Prove that ar (CBX) = ar (AXY)

TYPE (IV): VERY LONG ANSWER TYPE QUESTIONS [04 MARK EACH]
24. In Figure, ABCD is a parallelogram. Points P and Q on BC trisects BC in three equal parts. Prove that
1
ar (APQ) = ar (DPQ) = ar (ABCD)
6

25. ABCD is a trapezium in which AB || DC, DC = 30 cm and AB = 50 cm. If X and Y are, respectively the
7
mid-points of AD and BC, prove that ar (DCYX) = ar (XYBA).
9
26. In Figure, ABCDE is any pentagon. BP drawn parallel to AC meets DC produced at P and EQ drawn
parallel to AD meets CD produced at Q. Prove that ar (ABCDE) = ar (APQ)

1
27. If the medians of a ABC intersect at G, show that ar (AGB) = ar (AGC) = ar (BGC) = ar (ABC)
3

28. In Figure, X and Y are the mid-points of AC and AB respectively, QP || BC and CYQ and BXP are
straight lines. Prove that ar (ABP) = ar (ACQ).

For Unacademy Subscription Use “PJLIVE” Code | Join t.me/pjsir42 for Updates
For More Info: “75970 – 84242, 94590 – 43333 / 2222”
29. In Figure, ABCD and AEFD are two parallelograms. Prove that ar (PEA) = ar (QFD)

EXERCISE – 01

SUBJECTIVE QUESTIONS
Section (A) : Area of parallelograms and triangles
A-1. ABCDE is a pentagon. A line through B parallel to AC meets DC produced at F. Show that :
(i) ar (ACB) = ar (ACF) (ii) ar (AEDF) = ar (ABCDE)

A-2. P and Q are any two points lying on the sides DC and AD respectively of a parallelogram ABCD. Prove
that : ar (APB) = ar (BQC).

A-3. In the figure, PQRS and ABRS are parallelograms and X is any point on side BR. Prove that :

1
(i) ar(||gm PQRS) = ar(||gm ABRS) (ii) ar(AXS) = ar(||gm PQRS)
2

A-4. BD is one of the diagonals of a quadrilateral ABCD. If AL ⊥ BD and CM⊥ BD, show that :
1
ar(quadrilateral ABCD) = × BD × (AL + CM).
2

A-5. In the adjoining figure, PQRS and PABC are two parallelograms of equal area. Prove that QC || BR.

For Unacademy Subscription Use “PJLIVE” Code | Join t.me/pjsir42 for Updates
For More Info: “75970 – 84242, 94590 – 43333 / 2222”
A-6. O is any point on the diagonal BD of the parallelogram ABCD. Prove that ar (OAB) = ar(OBC).

A-7. In figure, AP || BQ || CR. Prove that ar(AQC) = ar(PBR).

1 1
A-8. The base BC of ABC is divided at D such that BD = DC. Prove that ar(ABD) = ar(ABC).
2 2

A-9. In the given figure, XY is a line parallel to side BC of a ABC. BE || AC and CF || AB meet XY in E
and F respectively. Show that ar(ABE) = ar(ACF).

A-10. Diagonals AC and BD of a quadrilateral ABCD intersect at O, such that OB = OD. If AB = CD, then
show that :

(i) ar (DOC) = ar (AOB) (ii) ar (DCB) = ar (ACB)


(iii) DA || CB or ABCD is a parallelogram.

A-11. In figure, ABCD is a parallelogram and BC is produced to point Q such that AD = CQ. If AQ intersect
at P, show that ar(BPC) = ar(DPQ).

For Unacademy Subscription Use “PJLIVE” Code | Join t.me/pjsir42 for Updates
For More Info: “75970 – 84242, 94590 – 43333 / 2222”
A-12. In figure, ABCD is a trapezium in which AB || DC and DC = 40 cm and AB = 60 cm. If X and Y are,
respectively, the mid-points of AD and BC, prove that :
(i) XY = 50 cm (ii) DCYX is a trapezium
9
(iii) Area (trapezium DCYX) = Area (trapezium XYBA)
11

A-13. In ABC, D is the midpoint of AB. P is any point on BC. CQ || PD meets AB in Q. Show that
1
ar(BPQ) = ar(ABC).
2

A-14. D is the midpoint of side BC of ABC and E is the midpoint of BD. If O is the midpoint of AE, prove
that ar(BOE) = ar(ABC).

For Unacademy Subscription Use “PJLIVE” Code | Join t.me/pjsir42 for Updates
For More Info: “75970 – 84242, 94590 – 43333 / 2222”
OBJECTIVE QUESTIONS
Section (A) : Area of parallelograms and triangles
A-1. In parallelogram ABCD, AB = 12 cm. The altitudes corresponding to the sides AB and AD are
respectively9 cm and 11 cm. Find AD.

108 108 99 108


(A) cm (B) cm (C) cm (D) cm
11 10 10 17

A-2. ABCD is a parallelogram. Points P and Q, on BC trisects it in three equal parts. PR and QS are also
drawn parallel to AB, then ar(APQ) = ........... ar(ABCD).

1 1 1 1
(A) (B) (C) (D)
3 2 4 6

A-3. In the figure, D and E are the mid-point of the sides AC and BC respectively of ABC.
If ar(BED) = 12 cm2, then ar (AEC) =

(A) 48 cm2 (B) 24 cm2 (C) 36 cm2 (D) none of these

A-4. In ABC, AD is a median and P is a point on AD such that AP : PD = 1 : 2, then the area of ABP =
1 2 1 1
(A) × Area of ABC (B) × Area of ABC (C) × Area of ABC (D) × Area of ABC
2 3 3 6

A-5. In ABC, if AD divides BC in the ratio m : n then area of ABD : area of ABC is :

(A) m : n (B) (m + 1) : n (C) m : (n + m) (D) n : m

A-6. The area of figure formed by joining the mid points of the adjacent sides of a rhombus with diagonals
(A) 48 cm2 (B) 64 cm2 (C) 96 cm2 (D) 192 cm2

A-7. In figure, if ar(ABC) = 28 cm2 then ar (AEDF) =

For Unacademy Subscription Use “PJLIVE” Code | Join t.me/pjsir42 for Updates
For More Info: “75970 – 84242, 94590 – 43333 / 2222”
(A) 21 cm2 (B) 18 cm2 (C) 16 cm2 (D) 14 cm2

ar( ABP)
A-8. If the area of ABC is 120 cm2 and the median AD is bisected at point P. Then find
ar( ACD)
1 1 1 1
(A) (B) (C) (D)
4 3 6 2

EXERCISE – 02

OBJECTIVE QUESTIONS
1. In quadrilateral ABCD, diagonals AC and BD intersect at point E. Then
(A) ar (AED) + ar (BCD) = ar (ABE) + ar (CDE) (B) ar (AED) – ar (BCD) = ar (ABE) – ar (CDE)
(C) ar (AED) ÷ ar (BCD) = ar (ABE) ÷ ar (CDE) (D) ar (AED) × ar (BCD) = ar (ABE) × ar (CDE)

2. AD is a median of ABC. If X is any point on AD, then find ratio of ar (ABX) to the ar(ACX).
1
(A) 1 (B) (C) 2 (D) None of these
2

ar(BPD)
3. In ABC, P is mid-point of median AD. Then =
ar(ABC)
1 1 1 1
(A) (B) (C) (D)
2 3 4 6

4. In ABC, D is a point on BC such that it divides BC in the ratio 3 : 5 i.e., BD : DC = 3 : 5. Find ar (ADC) : ar
(A) 3 : 5 (B) 5 : 8 (C) 3 : 8 (D) None of these

5. In the given figure, ABC and BDE are two equilateral triangles and D is the mid-point of BC. Then
ar(BDE)
=
ar(ABC)
1 1 1 1
(A) (B) (C) (D)
2 3 4 6

6. In the figure, ABCD is a parallelogram and PBQR is a rectangle.

If AP : PB = 1 : 2 = PD : DR, what is the ratio of the area of ABCD to the area of PBQR ?
(A) 1 : 2 (B) 2 : 1 (C) 1 : 1 (D) 2 : 3

1
7. ABCD is a parallelogram. DEC is drawn such that BE = AE. Sum of the areas of ADE and BEC is:
3

For Unacademy Subscription Use “PJLIVE” Code | Join t.me/pjsir42 for Updates
For More Info: “75970 – 84242, 94590 – 43333 / 2222”
1 1
(A) area of parallelogram ABCD (B) area of parallelogram ABCD
3 2
2 1
(C) area of DEC (D) area of DEC
3 2

8. E is the midpoint of diagonal BD of a parallelogram ABCD. If the point E is joined to a point F on DA


1
such that DA, then the ratio of the area of DEF to the area of quadrilateral ABEF is :
3
(A) 1 : 3 (B) 1: 4 (C) 1 : 5 (D) 2 : 5

12
9. ABCD (in order) is a rectangle with AB = CD = and BC = DA = 5. Point P is taken on AD such that
5
BPC = 90º. The value of (BP + PC) is equal to :
(A) 5 (B) 6 (C) 7 (D) 8

10. In the diagram, ABCD is a rectangle and point E lies on AB. Triangle DEC has DEC = 90º, DE = 3
and EC = 4. The length of AD is :

(A) 2.4 (B) 2.8 (C) 1.8 (D) 3.2

11. In the figure PQRS is a rectangle, which one is true?

(A) area of APS = area of QRB (B) PA = RB


(C) area of PQS = area of QRS (D) all of these
12. ABCD is parallelogram, AE ⊥ DC and CF ⊥ AD. If AB = 16 cm, AE = 8 cm, CF = 10 cm, find AD.
(A) 16 cm (B) 12 cm (C) 12.8 cm (D) 10.2 cm

13. The perimeter of an isosceles triangle is 32 cm and its base is 12 cm. One of its equal sides forms the
diagonal of a parallelogram. Find the area of parallelogram.
(A) 48 cm2 (B) 38 cm2 (C) 96 cm2 (D) None of these

EXERCISE – 03

NTSE PROBLEMS (PREVIOUS YEARS)


1. The area of a rhombus is 36 cm2. If one diagonal is double of second, then the length of bigger diagonal
is - [Rajasthan NTSE Stage-1 2005]
(A) 6 cm (B) 12 cm (C) 16 cm (D) 36 cm

For Unacademy Subscription Use “PJLIVE” Code | Join t.me/pjsir42 for Updates
For More Info: “75970 – 84242, 94590 – 43333 / 2222”
2. In the following figure, the area of the shaded portion is [Rajasthan NTSE Stage-1 2007]

(A) 85 cm2 (B) 420 cm2 (C) 750 cm2 (D) 1500 cm2

1 1
3. In the figure AD = DB, BE = EC and CF = AF. If the area of ABC = 120 cm2, the area (in cm2) of
2 3
DEF is : [Haryana NTSE Stage-1 2013]

(A) 21 (B) 35 (C) 40 (D) 45

4.

ABCD is a trapezium, AB || DC. Diagonals of trapezium intersect to each other at point O :


A (AOB) = 3 sq.cm
A (COD) = 12 sq. cm
A (ABCD) = ...................... . [Maharashtra NTSE Stage-1 2013]
(A) 27 sq. cm (B) 45 sq. cm (C) 36 sq. cm (D) 18 sq. cm

5. In the figure given below, points P and Q are mid points on the sides AC and BP respectively. Area of
each part is shown in the figure, then find the value of x + y. [Maharashtra NTSE Stage-1 2013]

(A) 11 (B) 4 (C) 7 (D) 18


6. PQRS is a parallelogram and M, N are the mid-points of PQ and RS respectively. Which of the
following is not true ? [M.P. NTSE Stage-1 2013]

(A) RM trisects QS (B) PN trisects QS (C) PSN  RQM (D) MS is not parallel to QN

7. In ABC, E divides AB in the ratio 3 : 1 and F divides BC in the ratio 3 : 2, then the ratio of areas of
BEF and ABC is : [Jharkhand NTSE Stage-1 2014]
(A) 3 : 5 (B) 3 : 10 (C) 1 : 5 (D) 3 : 20

8. In the figure, the area of square ABCD is 4 cm2 and E any point on AB. F, G, H and K are the mid point
of DE, CF, DG, and CH respectively. The area of KDC is - [Delhi NTSE Stage-1 2016]

For Unacademy Subscription Use “PJLIVE” Code | Join t.me/pjsir42 for Updates
For More Info: “75970 – 84242, 94590 – 43333 / 2222”
1 2 1 2 1 1
(A) cm (B) cm (C) cm 2 (D) cm 2
4 8 16 32
9. ABCD is a square of area of 4 square units which is divided into 4 non overlapping triangles as shown
in figure, then sum of perimeters of the triangles so formed is [Delhi NTSE Stage-1 2016]

(A) 8 (2 + 2 ) (B) 8 (1 + 2 ) (C) 4 (1 + 2 ) (D) 4 (2 + 2 )

10. In the diagram ABCD is a rectangle with AE = EF = FB, the ratio of the areas of triangle CEF and that
of rectangle ABCD is [Delhi NTSE Stage-1 2016]

(A) 1 : 6 (B) 1 : 8 (C) 1 : 9 (D) 1 : 10

VALUE BASED / PRACTICAL BASED QUESTIONS

1. A villager X has a plot of land of the shape of a quadrilateral. The Gram Panchayat of the village
decided to take over some portion of plot from one of the corners to construct a Health Centre. X agrees
to the above proposal with the condition that he should be given equal amount of land in lieu of his land
adjoining his plot so as to form a triangular plot. Explain how his proposal will be implemented.

2. For ‘Sarva Shiksha Abhiyan’ a rally was organised by a school. Students were given triangular cardboard
pieces to write slogans. They divided the triangular shape into three equal parts by drawing medians.
1
(i) Prove that ar (AGC) = ar (AGB) = ar (BGC) = ar (ABC).
3
(ii) Which values are included through this activity?

3. A plot is in the form of a parallelogram ABCD. Owner of this plot wants to build old age home, dispensary,
park and health centre for elderly people as shown in the figure. P is a point on the diagonal BD.
(i) Prove that area allotted to old age home and dispensary are same.
(ii) Which values are depicted here ?

For Unacademy Subscription Use “PJLIVE” Code | Join t.me/pjsir42 for Updates
For More Info: “75970 – 84242, 94590 – 43333 / 2222”
Answer Key

BOARD LEVEL EXERCISE


TYPE (I) : VERY SHORT ANSWER TYPE QUESTIONS : [01 MARK EACH]
1. 48 cm2 2. (iv) 3. a rhombus of area 24 cm2
1
4. ar (ABC) 5. 1 : 2 9. 8 cm2
2

TYPE (II) : SHORT ANSWER TYPE QUESTIONS : [02 MARKS EACH]


13. (i) 90 cm2 (ii) 45 cm2 (iii) 45 cm2

TYPE (III) : LONG ANSWER TYPE QUESTIONS: [04 MARK EACH]


15. (3a + b) : (a + 3b) 16. 12 cm2

EXERCISE – 01

OBJECTIVE QUESTIONS
Section (A) : Area of parallelograms and triangles
A-1. (A) A-2. (D) A-3. (B) A-4. (D) A-5. (C)
A-6. (C) A-7. (D) A-8. (D)

EXERCISE – 02

OBJECTIVE QUESTIONS
Ques. 1 2 3 4 5 6 7 8 9 10 11 12 13
Ans. D A C B C A B C C A D C C

EXERCISE – 03

Ques. 1 2 3 4 5 6 7 8 9 10
Ans. B C B A D D D B D A

For Unacademy Subscription Use “PJLIVE” Code | Join t.me/pjsir42 for Updates
For More Info: “75970 – 84242, 94590 – 43333 / 2222”
6 CIRCLES
INTRODUCTION

Circle : The collection of all the points in a plane, which are at a fixed distance from a fixed point in the plane,
is called a circle.
The fixed point is called the centre of the circle and the fixed distance is called the radius of the circle.

In figure, O is the centre and the length OP is the radius of the circle. So the line segment joining the centre
and any point on the circle is called a radius of the circle.
Chord : If we take two points P and Q on a circle, then the line segment PQ is called a chord of the circle.

Diameter : The chord which passes through the centre of the circle, is called the diameter of the circle.

A diameter is the longest chord and all diameters of same circle have the same length, which is equal to two
times the radius. In figure, AOB is a diameter of circle.
Arc : A piece of a circle between two points is called an arc. The longer one is called the major arc PQ and
the shorter one is called the minor arc PQ. The minor arc PQ is also de-remarked by PQ and the major arc PQ
by QP . When P and Q are ends of a diameter, then both arcs are equal and each is called a semi circle.

Circumference : The length of the complete circle is called its circumference.


Segment : The region between a chord and either of its arcs is called a segment of the circular region or simply
a segment of the circle. There are two types of segments which are the major segment and the minor segment
(as in figure).

For Unacademy Subscription Use “PJLIVE” Code | Join t.me/pjsir42 for Updates
For More Info: “75970 – 84242, 94590 – 43333 / 2222”
Sector : The region between an arc and the two radii, joining the centre to the end points of an arc is called a
sector. Minor arc corresponds to the minor sector and the major arc corresponds to the major sector.

When two arcs are equal, then both segments and both sectors become the same and each is known as a semi-
circular region.

A. CHORD PROPERTIES OF CIRCLES

(a) Important theorems related to chords


Theorem : Equal chords of a circle subtend equal angles at the centre.
Given : AB and CD are the two equal chords of a circle with centre O.
To Prove : AOB = COD.
Proof : In  AOB and COD,
OA = OC [Radii of a circle]
OB = OD [Radii of a circle]
AB = CD [Given]
 AOB  COD [By SSS congruency]
 AOB = COD. [By CPCT] Hence Proved.
Converse :
If the angles subtended by the chords of a circle at the centre are equal, then the chords are
equal.
Theorem : The perpendicular from the centre of a circle to a chord bisects the chord.
Given : A circle with centre O. AB is a chord of this circle. OM ⊥ AB.
To Prove : MA = MB.
Construction : Join OA and OB.
Proof : In right triangles OMA and OMB,
OA = OB [Radii of a circle]
OM = OM [Common]
OMA = OMB [90º each]
 OMA  OMB [By RHS]
 MA = MB [By CPCT] Hence Proved.
Converse :
The line drawn through the centre of a circle to bisect a chord is perpendicular to the chord.
Theorem : There is one and only one circle passing through three given non-collinear points.
Proof : Take three points A, B and C, which are not in the same line, or in other words, they are not
collinear [as in figure ]. Draw perpendicular bisectors of AB and BC say, PQ and RS respectively. Let
these perpendicular bisectors intersect at one point O.(REMARK that PQ and RS will intersect because
they are not parallel) [as in figure].

For Unacademy Subscription Use “PJLIVE” Code | Join t.me/pjsir42 for Updates
For More Info: “75970 – 84242, 94590 – 43333 / 2222”
 O lies on the perpendicular bisector PQ of AB.
 OA = OB
[ Every point on the perpendicular bisector of a line segment is equidistant from its end points]
Similarly,
 O lies on the perpendicular bisector RS of BC.
 OB = OC
[ Every point on the perpendicular bisector of a line segment is equidistant from its end points]
So, OA = OB = OC
i.e., the points A, B and C are at equal distances from the point O.
So, if we draw a circle with centre O and radius OA it will also pass through B and C. This shows that
there is a circle passing through the three points A, B and C. We know that two lines (perpendicular
bisectors) can intersect at only one point, so we can draw only one circle with radius OA. In other
words, there is a unique circle passing through A, B and C. Hence Proved.

Solved Examples

Example.1 In figure, AB = CB and O is the centre of the circle. Prove that BO bisects ABC.
Solution. Given : In figure, AB = CB and O is the centre of the circle.
To Prove : BO bisects ABC.
Construction : Join OA and OC.

Proof : In OAB and OCB,


OA = OC [Radii of the same circle]
AB = CB [Given]
OB = OB [Common]
 OAB  OCB [By SSS congruency]
 ABO = CBO [By CPCT]
 BO bisects ABC. Hence Proved.

Example.2 Two circles with centres A and B intersect at C and D. Prove that ACB = ADB.
Solution. Given : Two circles with centres A and B intersect at C and D.
To Prove : ACB = ADB.
Construction : Join AC, AD, BC, BD and AB.
Proof : In  ACB and  ADB,
AC = AD [Radii of the same circle]
BC = BD [Radii of the same circle]
AB = AB [Common]
  ACB   ADB [By SSS congruency]
 ACB = ADB. [By CPCT] Hence Proved.

For Unacademy Subscription Use “PJLIVE” Code | Join t.me/pjsir42 for Updates
For More Info: “75970 – 84242, 94590 – 43333 / 2222”
Example.3 In figure, AB  AC and O is the centre of the circle. Prove that OA is the perpendicular
bisector of BC.
Solution. Given : In figure, AB  AC and O is the centre of the circle.
To Prove : OA is the perpendicular bisector of BC.
Construction : Join OB and OC.
Proof :
 AB  AC [Given]
 chord AB = chord AC.
[If two arcs of a circle are congruent, then their corresponding
chords are equal]
 AOB = AOC ...(i)
[ Equal chords of a circle subtend equal angles at the centre]
In OBD and OCD,
DOB = DOC [From (i)]
OB = OC [Radii of the same circle]
OD = OD [Common]
 OBD  OCD [By SAS congruency]
 ODB = ODC ...(ii) [By CPCT]
And, BD = CD ...(iii) [By CPCT]
But BDC = 180º
 ODB + ODC = 180º
 ODB + ODB = 180º [From equation(ii)]
 2ODB = 180º
 ODB = 90º
 ODB = ODC = 90º ...(iv) [From (ii)]
So, by (iii) and (iv), OA is the perpendicular bisector of BC. Hence Proved.

Example.4 Prove that the line joining the mid-points of the two parallel chords of a circle passes through
the centre of the circle.
Solution. Let AB and CD be two parallel chords of a circle whose centre is O.
Let L and M be the mid-points of the chords AB and CD respectively. Join OL and OM.
Draw OX || AB or CD.

 L is the mid-point of the chord AB and O is the centre of the circle


 OLB = 90º
[ The perpendicular drawn from the centre of a circle to a chord bisects the chord]
But, OX || AB
 LOX = 90º ...(i)
[ Sum of the consecutive interior angles on the same side of a transversal is 180º]
 M is the mid-point of the chord CD and O is the centre of the circle
 OMD = 90º
[ The perpendicular drawn from the centre of a circle to a chord bisects the chord]
But OX || CD ...(ii)
[ Sum of the consecutive interior angles on the same side of a transversal is 180º]
 MOX = 90º
For Unacademy Subscription Use “PJLIVE” Code | Join t.me/pjsir42 for Updates
For More Info: “75970 – 84242, 94590 – 43333 / 2222”
From above equations, we get
LOX + MOX = 90º + 90º = 180º
 LOM = 180º
 LM is a straight line passing through the centre of the circle. Hence Proved.

Example.5  is a line which intersects two concentric circles (i.e., circles with the same centre) with
common centre O at A, B, C and D (as in figure). Prove that AB = CD.
Solution. Given :  is a line which intersects two concentric circles (i.e., circles with the same centre)
with common centre O at A, B, C and D.
To Prove : AB = CD.
Construction : Draw OE ⊥ .
Proof :  The perpendicular drawn from the centre of a circle to a
chord bisects the chord.
 AE = ED ...(i)
And BE = EC ...(ii)
Subtracting (ii) from (i), we get
AE – BE = ED – EC  AB = CD. Hence Proved.

Example.6 PQ and RS are two parallel chords of a circle whose centre is O and radius is 10 cm. If PQ = 16
cm and RS = 12 cm, find the distance between PQ and RS, if they lie
(i) on the same side of the centre O.
(ii) on the opposite sides of the centre O.
Solution. (i) Draw the perpendicular bisectors OL and OM of PQ and RS respectively.
 PQ || RS
 OL and OM are in the same line.
 O, L and M are collinear.
Join OP and OR.
In right triangle OLP,
OP2 = OL2 + PL2 [By Pythagoras Theorem]
2
1 
 (10) = OL +   PQ 
2 2

2 
[ The perpendicular drawn from the centre of a circle to a chord bisects the chord]
2
1 
 100 = OL +   16 
2

2 
 100 = OL2 + (8)2  100 = OL2 + 64
 OL = 100 – 64
2
 OL2 = 36 = (6)2
 OL = 6 cm
In right triangle OMR,
OR2 = OM2 + RM2 [By Pythagoras Theorem]
2
1 
 OR = OM +   RS 
2 2

2 
[ The perpendicular drawn from the centre of a circle to a chord bisects the chord]
2
1 
 (10) = OM +   12 
2 2
 (10)2 = OM2 + (6)2
2 
 OM2 = (10)2 – (6)2 = (10 – 6)(10 + 6) = (4)(16) = 64 = (8)2.
 OM = 8 cm
 LM = OM – OL = 8 – 6 = 2 cm.
Hence, the distance between PQ and RS, if they lie on the same side of the centre O, is 2 cm.
(ii) Draw the perpendicular bisectors OL and OM of PQ and RS respectively.

For Unacademy Subscription Use “PJLIVE” Code | Join t.me/pjsir42 for Updates
For More Info: “75970 – 84242, 94590 – 43333 / 2222”
 PQ || RS
 OL and OM are in the same line.
 L, O and M are collinear.
Join OP and OR.
In right triangle OLP,
OP2 = OL2 + PL2 [By Pythagoras Theorem]
2
1 
 OP = OL +   PQ 
2 2

2 
[ The perpendicular drawn from the centre of a circle to a chord bisects the chord]
2
1 
 (10) = OL +   16 
2 2

2 
 100 = OL2 + (8)2  100 = OL2 + 64
 OL2 = 100 – 64  OL2 = 36 = (6)2
 OL = 6 cm.
In right triangle OMR,
OR2 = OM2 + RM2 [By Pythagoras Theorem]
2
1 
 OR2 = OM2 +   RS 
2 
[ The perpendicular drawn from the centre of a circle to a chord bisects the chord]
2
1 
 (10) = OM +   12 
2 2

2 
 (10)2 = OM2 + (6)2
 OM2 = (10)2 – (6)2
= (10 – 6)(10 + 6) = (4)(16) = 64 = (8)2.
 OM = 8 cm
 LM = OL + OM = 6 + 8 = 14 cm
Hence, the distance between PQ and RS, if they lie on the opposite side of the centre O, is 14 cm.
Theorem : Equal chords of a circle (or of congruent circles) are equidistant from the
centre (or centres).
Given : A circle have two equal chords AB & CD. i.e. AB = CD and OM
⊥ AB, ON ⊥ CD.
To Prove : OM = ON
Construction : Join OB & OD
Proof : AB = CD (Given)
[ The perpendicular drawn from the centre of a circle to a chord bisects the chord]
1 1
 AB = CD  BM = DN
2 2
In OMB & OND
OMB = OND = 90º [Given]
OB = OD [Radii of same circle]
BM = DN [Proved above]
 OMB  OND [By R.H.S. congruency]
 OM = ON [By CPCT]
Hence Proved.

Remark : Chords equidistant from the centre of a circle are equal in length.

Example.7 AB and CD are equal chords of a circle whose centre is O. When produced, these chords meet
at E. Prove that EB = ED.

For Unacademy Subscription Use “PJLIVE” Code | Join t.me/pjsir42 for Updates
For More Info: “75970 – 84242, 94590 – 43333 / 2222”
Solution. Given : AB and CD are equal chords of a circle whose centre is O. When produced, these
chords meet at E.
To Prove : EB = ED.
Construction : From O draw OP ⊥ AB and OQ ⊥ CD. Join OE.
Proof : AB = CD [ Given]
 OP = OQ
[  Equal chords of a circle are equidistant from the centre]
Now in OPE and OQE,
OPE = OQE [Each 90º]
OE = OE [Common]
OP = OQ [Proved above]
 OPE  OQE [By RHS congruency]
 PE = QE [By CPCT]
1 1
 PE – AB = QE – CD [ AB = CD (Given)]
2 2
 PE – PB = QE – QD  EB = ED. Hence Proved.

Example.8 AB and CD are the chords of a circle whose centre is O. They intersect each other at P. If PO
be the bisector of APD, prove that AB = CD.
OR
In the given figure, O is the centre of the circle and PO bisects the angle APD. Prove that AB = CD.
Solution. Given : AB and CD are the chords of a circle whose centre is O. They intersect each other at P.
PO is the bisector of APD.
To Prove : AB = CD.
Construction : Draw OR ⊥ AB and OQ ⊥ CD.
Proof : In OPR and OPQ,
OPR = OPQ [Given]
OP = OP [Common]
And ORP = OQP [Each = 90º]
 OPR  OPQ [By AAS congruency]
 OR = OQ [By CPCT]
 AB = CD [ Chords of a circle which are equidistant from the centre are equal].

Check Point - A

1. Find the number of least non-collinear points required to draw a circle passing through them.

2. Given an arc of a circle, show how to complete the circles ?

3. If two circles intersect each other, prove that the line joining their centres bisects the common chord at
right angle.

4. Find the length of a chord which is at a distance of 16 cm from the centre of a circle whose radius is 20 cm.

5. Two circles of radii 10 cm and 17 cm intersect at two points and the distance between their centres is 21
cm. Find the length of the common chord.
Answers
1. 3 4. 24cm 5. 16cm

For Unacademy Subscription Use “PJLIVE” Code | Join t.me/pjsir42 for Updates
For More Info: “75970 – 84242, 94590 – 43333 / 2222”
B. RESULTS ON ANGLES SUBTENDED BY
ARCS AND CYCLIC QUADRILATEERAL

(a) Result on angles subtended by arcs


In figure, the angle subtended by the minor arc PQ at O is POQ and the angle subtended by the major
arc PQ at O is reflex angle POQ.

Theorem : The angle subtended by an arc at the centre is double the angle subtended by it at any
point on the remaining part of the circle.
Given : An arc PQ of a circle subtending angles POQ at the centre O and PAQ at a point A on the
remaining part of the circle.
To Prove : POQ = 2PAQ.
Construction : Join AO and extend it to a point B.

Proof : There arises three cases:-


(A) arc PQ is minor (B) arc PQ is a semi-circle (C) arc PQ is major.
In all the cases, BOQ = OAQ + AQO ...(i)
[ An exterior angle of a triangle is equal to the sum of the two interior opposite angles]
In OAQ,
OA = OQ [Radii of a circle]
 OQA = OAQ ...(ii)
[Angles opposite equal sides of a triangle are equal]
From (i) and (ii)
BOQ = 2OAQ ...(iii)
Similarly,
BOP = 2OAP ...(iv)
Adding (iii) and (iv), we get
BOP + BOQ = 2(OAP + OAQ)
 POQ = 2PAQ. ...(v)

Remark : For the case (C), where PQ is the major arc, (v) is replaced by reflex angles.
Thus, reflex POQ = 2PAQ.

Theorem : Angles in the same segment of a circle are equal.


Proof : Let P and Q be any two points on a circle to form a chord PQ, A and C any other points on the
remaining part of the circle and O be the centre of the circle. Then,

For Unacademy Subscription Use “PJLIVE” Code | Join t.me/pjsir42 for Updates
For More Info: “75970 – 84242, 94590 – 43333 / 2222”
POQ = 2PAQ ...(i)
And POQ = 2PCQ ...(ii)
[Angle subtended at the centre is double than the angle subtended by it on the remaining part of the circle]
From equation (i) & (ii)
2PAQ = 2PCQ
 PAQ = PCQ. Hence Proved.
Theorem : Angle in the semicircle is a right angle.
Proof :  PAQ is an angle in the segment, which is a semicircle.
 PAQ =
1 1
POQ = POQ = × 180º = 90º
2 2
[POQ is straight line angle or POQ = 180º]
If we take any other point C on the semicircle, then again, we get
1 1
PCQ = POQ = × 180º = 90º. Hence Proved.
2 2

(b) Cyclic quadrilaterals


A quadrilateral ABCD is called cyclic if all the four vertices of it lie on a circle.

Theorem : The sum of either pair of opposite angles of a cyclic quadrilateral is 180º.
Given : A cyclic quadrilateral ABCD.

To Prove : A + C = B + D = 180º.
Construction : Join AC and BD.
Proof : ACB = ADB ...(i)
And BAC = BDC ...(ii) [Angles of same segment of a circle are equal]
Adding equation (i) & (ii)
 ACB + BAC = ADB + BDC
ACB + BAC = ADC.
Adding ABC to both sides, we get
ACB + BAC + ABC = ADC + ABC.
 ADC + ABC = 180º
i.e., D + B = 180º
 A + C = 360º – (B + D) = 180º [ A + B + C + D = 360º ] Hence Proved.

For Unacademy Subscription Use “PJLIVE” Code | Join t.me/pjsir42 for Updates
For More Info: “75970 – 84242, 94590 – 43333 / 2222”
Solved Examples

Example.9 If a side of a cyclic quadrilateral is produced, then the exterior angle is equal to the interior
opposite angle.
Solution. Let ABCD be a cyclic quadrilateral inscribed in a circle with centre O. The side AB of
quadrilateral ABCD is produced to E. Then, we have to prove that CBE = ADC.
Since the sum of opposite pairs of angles of a cyclic
quadrilateral is 180º
 ABC + ADC = 180º
But ABC + CBE1 = 180º
[ABC and CBE form a linear pair]
 ABC + ADC = ABC + CBE
 ADC = CBE or CBE = ADC. Hence proved

Example.10 In figure, ABC = 69º, ACB = 31º, find BDC.


Solution. In  ABC,
BAC + ABC + ACB = 180º
 BAC + 69º + 31º = 180º
 BAC + 100º = 180º
 BAC = 180º – 100º = 80º
Now, BDC = BAC = 80º. [Angles in the same segment of a circle are equal]

Example.11 ABCD is a cyclic quadrilateral whose diagonals intersect at a point E. If DBC = 70º, BAC
= 30º, find BCD. Further, if AB = BC, find ECD.
Solution. CDB = BAC = 30º ...(i)
[Angles in the same segment of a circle are equal]
DBC = 70º ...(ii)
In BCD,
BCD + DBC + CDB = 180º
 BCD + 70º + 30º = 180º [Using (i) and (ii)]
 BCD + 100º = 180º
 BCD = 180º – 100º
 BCD = 80º ...(iii)
In ABC, AB = BC
 BCA = BAC = 30º ...(iv) [ Angles opposite to equal sides of a triangle are equal]
 BCA + ECD = 80º  30º + ECD = 80º
 ECD = 80º – 30º  ECD = 50º.

Example.12 In figure, find the measures of ABD, CDP, PDA, CAB and CBD.

Solution. Clearly, ABD and ACD are in the same segment determined by chord AD.
 ABD = ACD
 ABD = 60º [Given ACD = 60º]
Now, CPD = BPA [Vertically opp. angles]
 CPD = 90º [Given BPA = 90º]
In CPD, we have
For Unacademy Subscription Use “PJLIVE” Code | Join t.me/pjsir42 for Updates
For More Info: “75970 – 84242, 94590 – 43333 / 2222”
CDP +CPD +PCD = 180º
 CDP + 60º + 90º = 180º
 CDP + 150º = 180º
 CDP = 180º – 150º = 30º
Since EDF and ADC are vertically opposite angles.
 ADC = EDF
 ADC = 85º
 PDA + PDC = 85º
 PDA + 30º = 85º
 PDA = 85º – 30º = 55º
Since, CDB = CDP and CAB are the angles in the same segment determined by chord CB.
 CAB = CDP
 CAB = 30º
In ACD, we have
CAD + ACD + ADC = 180º
CAD + 60º + 85º = 180º
 CAD = 180º – (60º + 85º) = 35º
Clearly, CAD and CBD are angles in the same segment determined by chord CD.
 CBD = CAD = 35º.

Example.13 If the non parallel side of a trapezium are equal, prove that it is cyclic.
Solution. Given : ABCD is a trapezium whose two non-parallel sides AD and BC are equal.
To Prove : Trapezium ABCD is a cyclic.
Construction : Draw BE || AD.
Proof :  AB || DE [Given]
and AD || BE [By construction]
 Quadrilateral ABED is a parallelogram.
 BAD = BED ...(i) [Opp. angles of a ||gm]
And , AD = BE ...(ii) [Opp. sides of a ||gm]
But AD = BC ...(iii) [Given]
From (ii) and (iii),
BE = BC
 BCE = BEC ...(iv) [Angles opposite to equal sides]
BEC + BED = 180º [Linear Pair Axiom]
 BCE + BAD = 180º [From (iv) and (i)]
 Trapezium ABCD is cyclic.
[If a pair of opposite angles of a quadrilateral is 180º, then the quadrilateral is cyclic]
Hence Proved.

Example.14 Prove that a cyclic parallelogram is a rectangle.


Solution. Given : ABCD is a cyclic parallelogram.
To Prove : ABCD is a rectangle.
Proof : ABCD is a cyclic quadrilateral
 1 + 2 = 180º ...(i)
[ Opposite angles of a cyclic quadrilateral are supplementary]
ABCD is a parallelogram
 1 = 2 ...(ii) [Opp. angles of a ||gm]
From (i) and (ii),
1 = 2 = 90º
 Parallelogram ABCD is a rectangle. Hence Proved.

Example.15 In figure, PQ is a diameter of a circle with centre O. If PQR = 65º, SPR = 40º, PQM = 50º,
find QPR,PRS and QPM.
Solution. (i) QPR

For Unacademy Subscription Use “PJLIVE” Code | Join t.me/pjsir42 for Updates
For More Info: “75970 – 84242, 94590 – 43333 / 2222”
 PQ is a diameter
 PRQ = 90º [Angle in a semi-circle is 90º] In  PQR,
QPR + PRQ + PQR = 180º
 QPR + 90º + 65º = 180º
 QPR + 155º = 180º
 QPR = 180º – 155º
 QPR = 25º.
(ii) PRS
 PQRS is a cyclic quadrilateral.
 PSR + PQR = 180º [ Opposite angles of a cyclic quadrilateral are supplementary]
 PSR + 65º = 180º
 PSR = 180º – 65º
 PSR = 115º
In  PSR,
PSR + SPR + PRS = 180º
 115º + 40º + PRS = 180º
 155º + PRS = 180º
 PRS = 180º – 155º
 PRS = 25º.
(iii) QPM
 PQ is a diameter
 PMQ = 90º [ Angle in a semi-circle is 90º]
In  PMQ,
PMQ + PQM + QPM = 180º
 90º + 50º + QPM = 180º  140º + QPM = 180º
 QPM = 180º – 140º  QPM = 40º.

Example.16 In figure, O is the centre of the circle. Prove that x + y = z.


1
Solution. EBF = EOFz
2
[Angle subtended by an arc of a circle at the centre is twice the angle subtended by it at any
point of the remaining part of the circle]
1
 ABF = 180º – z ...(i) [Linear Pair Axiom]
2
1 1
EDF = EOF = z
2 2
[Angle subtended by any arc of a circle at the centre is twice the
angle subtended by it at any point of the remaining part of the circle]
1
 ADE = 180º – z ...(ii) [Linear Pair Axiom]
2
BCD = ECF = y [Vertically Opp. Angles]
BAD = x
In quadrilateral ABCD
ABC + BCD + CDA + BAD = 2 × 180º
1 1
 180º – z + y + 180º – z + x = 2 × 180º  x + y = z. Hence Proved.
2 2

Example.17 AB is a diameter of the circle with centre O and chord CD is equal to radius OC. AC and BD
produced meet at P. Prove that CPD = 60º.
Solution. Given : AB is a diameter of the circle with centre O and chord CD is equal to radius OC. AC
and BD produced meet at P.

For Unacademy Subscription Use “PJLIVE” Code | Join t.me/pjsir42 for Updates
For More Info: “75970 – 84242, 94590 – 43333 / 2222”
To Prove : CPD = 60º.
Construction : Join AD.
Proof : In OCD,
OC = OD ...(i) [Radii of the same circle]
OC = CD ...(ii) [Given]
From (i) and (ii),
OC = OD = CD
 OCD is an equilateral triangle.
 COD = 60º
1 1
 CAD = COD = (60) = 30
2 2
COD =(60º) = 30º
[Angle subtended by any arc of a circle at the centre is twice the angle subtended by it at any
point of the remaining part of the circle]
 PAD = 30º ...(iii)
And, ADB = 90º ...(iv) [Angle in a semi-circle]
 ADB + ADP = 180º [Linear Pair Axiom]
 90º + ADP = 180º [From (iv)]
 ADP = 90º ...(v)
In ADP,
APD + PAD + ADP = 180º
 APD + 30º + 90º = 180º [From (iii) and (v)]
 APD + 120º = 180º
 APD = 180º – 120º = 60º
 CPD = 60º. Hence Proved.

Example.18 Prove that the quadrilateral formed by angle bisectors of a cyclic quadrilateral is also cyclic.
Solution. Given : ABCD is a cyclic quadrilateral. Its angle bisectors form a quadrilateral PQRS.
To Prove : PQRS is a cyclic quadrilateral.
Proof : In APB
1 + 2 + 3 = 180º ...(i)
In DRC
4 + 5 + 6 = 180º ...(ii)
 1 + 2 + 3 + 4 + 5 + 6 = 360º ...(iii)
[ Adding (i) and (ii)]
1
But 2 + 3 + 6 + 5 = [A + B + C + D]
2
1
= (360º) = 180º  1 + 4 = 360º – (2 + 3 + 6 + 5)
2
= 360º – 180º = 180º.
 PQRS is a cyclic quadrilateral.
[ If the sum of any pair of opposite angles of a quadrilateral is 180º, then the quadrilateral is a
cyclic] Hence Proved.

Example.19 Two concentric circles with centre O have A, B, C, D as the points of intersection with the line  as
shown in the figure. If AD = 12 cm and BC = 8 cm, find the length of AB, CD, AC and BD.
Solution. Since OM ⊥ BC, a chord of the circle,
 It bisects BC.
1 1
 BM = CM = (BC) = (8) = 4 cm
2 2
Since, OM ⊥ AD, a chord of the circle,
 It bisects AD.
1 1
 AM = MD = AD = (12) = 6 cm
2 2
For Unacademy Subscription Use “PJLIVE” Code | Join t.me/pjsir42 for Updates
For More Info: “75970 – 84242, 94590 – 43333 / 2222”
Now, AB = AM – BM = 6 – 4 = 2 cm
CD = MD – MC = 6 – 4 = 2 cm
AC = AM + MC = 6 + 4 = 10 cm
BD = BM + MD = 4 + 6 = 10 cm

Example.20 OABC is a rhombus whose three vertices A, B and C lie on a circle with centre O. If the radius
of the circle is 10 cm. Find the area of the rhombus.
Solution. Since, OABC is a rhombus
 OA = AB = BC = OC = 10 cm
1 1
Now, OD ⊥ BC  CD = BC = (10) = 5 cm
2 2
 By Pythagoras theorem,
OC2 = OD2 + DC2
 OD2 = OC2 – DC2 = (10)2 – (5)2 = 100 – 25 = 75
 OD = 75 = 5 3
1 1
 Area (OBC) = BC × OD = (10) × 5 3 = 25 3 sq.cm.
2 2
Area of Rhombus = 2 (Area of OBC) = 2( 25 3 ) = 50 3 sq. cm.

Example.21 In the given figure, AB is the chord of a circle with centre O. AB is produced to C such that BC
= OB. CO is joined and produced to meet the circle in D. If ACD = yº and AOD = xº, prove
that xº = 3yº.
Solution. Since BC = OB [Given]
 OCB = BOC = yº
[ Angles opposite to equal sides are equal]

OBA = BOC + OCB = yº + yº = 2yº.


[Exterior angle of a  is equal to the sum of the opposite interior angles]
Also OA = OB [Radii of the same circle]
OAB = OBA = 2yº [Angles opposite to equal sides of a triangle are equal]
AOD = OAC + OCA = 2yº + yº = 3yº
[Exterior angle of a  is equal to the sum of the opposite interior angles]
Hence, xº = 3yº. Hence Proved.

Check Point - B

1. Two angles in the same segment of a circle are (2x + 10)º and (x + 45º). Then x is equal to

2. One angle of a cyclic quadrilateral is twice its opposite angle. Then find the smaller of the two angles
3. In the figure, O is the centre of the circle, find PQR

For Unacademy Subscription Use “PJLIVE” Code | Join t.me/pjsir42 for Updates
For More Info: “75970 – 84242, 94590 – 43333 / 2222”
4. In the figure, O is the centre and ACB = 20º, find AOD -

5. In this figure, find sum of Q and S.

Answers
1. 35º 2. 60º 3. 123º 4. 140º 5. 90º

Add to Your Knowledge

1. The tangent at any point of a circle and the radius through the point are perpendicular to each other.
Given : A circle with centre O. AB is a tangent to the circle at a point P and OP is the radius through P.
Then : OP ⊥ AB.

2. If O be the centre of a circle and tangents drawn to the circle at the points A and B of the circle intersect
each other at P.
Then : AOB + APB = 180º.

3. If two tangents are drawn to a circle from an exterior point, then


(i) the tangents are equal in length
(ii) the tangents subtend equal angles at the centre
(iii) the tangents are equally inclined to the line joining the point and the centre of the circle.

Given : PA and PB are two tangents drawn to a circle with centre O, from an exterior point P.
Then: (i) PA = PB (ii) AOP = BOP (iii) APO = BPO.

For Unacademy Subscription Use “PJLIVE” Code | Join t.me/pjsir42 for Updates
For More Info: “75970 – 84242, 94590 – 43333 / 2222”
4. Its I If two chords AB and CD of a circle, intersect inside a circle (outside the circle when produced at a
point E).
Then: AE × BE = CE × DE.

5. If PB be a secant which intersects the circle at A and B and PT be a tangent at T.


Then : PA.PB = (PT)2.

6. For the two circles with centre X and Y and radii r1 and r2. AB and CD are two Direct Common
Tangents (OCT), then the length of OCT = (distance between centres) 2 − (r2 − r1 ) 2

7. For the two circles with centre X and Y and radii r1 and r2. PQ and RS are two transverse common
tangents, then length of TCT = (distance between centres) 2 − (r2 + r1 ) 2

8. If a circle touches all the four sides of a quadrilateral then the sum of the two opposite sides is equal to
the sum of other two. AB + DC = AD + BC.

For Unacademy Subscription Use “PJLIVE” Code | Join t.me/pjsir42 for Updates
For More Info: “75970 – 84242, 94590 – 43333 / 2222”
BOARD LEVEL EXERCISE

TYPE (I) : VERY SHORT ANSWER TYPE QUESTIONS : [01 MARK EACH]

1. In Figure, two congruent circles have centres O and O'. Arc AXB subtends an angle of 75º at the centre
O and arc A'YB' subtends an angle of 25º at the centre O'. Then the ratio of arcs AXB and A'YB' is :

2. In Figure, AB and CD are two equal chords of a circle with centre O. OP and OQ are perpendiculars on
chords AB and CD, respectively. If POQ = 150º, then find APQ ?

3. In Figure, if OA = 5 cm, AB = 8 cm and OD is perpendicular to AB, then find CD.

4. If AB = 12 cm, BC = 16 cm and AB is perpendicular to BC, then find the radius of the circle passing
through the points A, B and C.

5. In Figure, if ABC = 20º, then find AOC.

6. In Figure, if AOB is a diameter of the circle and AC = BC, then find CAB.

For Unacademy Subscription Use “PJLIVE” Code | Join t.me/pjsir42 for Updates
For More Info: “75970 – 84242, 94590 – 43333 / 2222”
7. In Figure, if OAB = 40º, then find ACB.

8. In Figure, if DAB = 60º, ABD = 50º, then find ACB.


D C

60º 50º
A B

9. In Figure, BC is a diameter of the circle and BAO = 60º. Then find ADC.

TYPE (II) : SHORT ANSWER TYPE QUESTIONS : [02 MARKS EACH]


1
10. In Figure, AOC is a diameter of the circle and arc AXB = arc BYC. Find BOC.
2

11. In Figure, ABC = 45º, prove that OA ⊥ OC.

12. If a line segment joining mid-points of two chords of a circle passes through the centre of the circle,
prove that the two chords are parallel.
For Unacademy Subscription Use “PJLIVE” Code | Join t.me/pjsir42 for Updates
For More Info: “75970 – 84242, 94590 – 43333 / 2222”
13. ABCD is such a quadrilateral that A is the centre of the circle passing through B, C and D. Prove that
1
CBD + CDB = BAD
2

14. If a line is drawn parallel to the base of an isosceles triangle to intersect its equal sides, prove that the
quadrilateral so formed is cyclic.

15. In Figure, AB is a diameter, ADC = 130º and chord BC = chord BE. Find CBE.
C
D
130

A B
O

E
16. Two circles with centres O and O' intersect at two points A and B. A line PQ is drawn parallel to OO'
through A(or B) intersecting the circles at P and Q. Prove that PQ = 2 OO'.

17. In Figure, AOB is a diameter of the circle and C, D, E are any three points on the semi-circle. Find the
value of ACD + BED

18. In Figure, OAB = 30º and OCB = 57º. Find BOC and AOC.

19. Prove that among all the chords of a circle passing through a given point inside the circle that one is
smallest which is perpendicular to the diameter passing through the point.

TYPE (III) : LONG ANSWER TYPE QUESTIONS: [03 MARK EACH]


20. If P, Q and R are the mid-points of the sides BC, CA and AB of a triangle and AD is the perpendicular
from A on BC, prove that P, Q, R and D are concyclic.

21. Prove that angle bisector of any angle of a triangle and perpendicular bisector of the opposite side if
intersect, they will intersect on the circumcircle of the triangle.

22. If two chords AB and CD of a circle AYDZBWCX intersect at right angles (see Figure), prove that arc
CXA + arc DZB = arc AYD + arc BWC = semi- circle.

For Unacademy Subscription Use “PJLIVE” Code | Join t.me/pjsir42 for Updates
For More Info: “75970 – 84242, 94590 – 43333 / 2222”
23. In Figure, AB and CD are two chords of a circle intersecting each other at point E. Prove that AEC = 2
(Angle subtended by arc CXA at centre + angle subtended by arc DYB at the centre).

24. A circle has radius 2 cm. It is divided into two segments by a chord of length 2 cm. Prove that the
angle subtended by the chord at a point in major segment is 45º.

25. AB and AC are two chords of a circle of radius r such that AB = 2AC. If p and q are the distances of
AB and AC from the centre, prove that 4q2 = p2 + 3r2.

For Unacademy Subscription Use “PJLIVE” Code | Join t.me/pjsir42 for Updates
For More Info: “75970 – 84242, 94590 – 43333 / 2222”
EXERCISE – 01

SUBJECTIVE QUESTIONS
Section (A) : Chord properties of Circles
A-1. The radius of a circle is 13 cm and the length of one of its chords is 10 cm. Find the distance of the
chord from the centre.

A-2. In figure, O is the centre of the circle of radius 5 cm. OP ⊥ AB, OQ ⊥ CD, AB || CD, AB = 6 cm and
CD = 8 cm. Determine PQ.

A-3. AB and CD are two parallel chords of a circle such that AB = 10 cm and CD = 24 cm. If the chords are
on the opposite side of the centre and the distance between them is 17 cm, find the radius of the circle.

A-4. In a circle of radius 5 cm, AB and AC are two chords such that AB = AC = 6 cm. Find the length of the
chord BC.

A-5. A circular park of radius 20 m is situated in a colony. Three boys Ankur, Syed and David are sitting at
equal distances on its boundary each having a toy telephone in his hands to talk to each other. Find the
length of the string of each phone.

Section (B) : Results on angles subtended by arcs and Cyclic quadrilateral


B-1. In the given figure, BC is diameter bisecting ACD, find the values of a, b (O is centre of circle).

B-2. Find the value of a & b.

B-3. A chord of a circle is equal to the radius of the circle, find the angle subtended by the chord at a point
on the minor arc and also at a point on the major arc.

For Unacademy Subscription Use “PJLIVE” Code | Join t.me/pjsir42 for Updates
For More Info: “75970 – 84242, 94590 – 43333 / 2222”
B-4. Find the value of a & b.

B-5. Find the value of a and b, if b = 2a.

B-6. In the given figure, the chord ED is parallel to the diameter AC. Find CED.

B-7. In the given figure, P is the centre of the circle. Prove that : XPZ = 2 (XZY + YXZ).

B-8. In a circle with centre O, chords AB and CD intersects inside the circumference at E. Prove that
AOC + BOD = 2AEC.

For Unacademy Subscription Use “PJLIVE” Code | Join t.me/pjsir42 for Updates
For More Info: “75970 – 84242, 94590 – 43333 / 2222”
B-9. In figure, P is any point on the chord BC of a circle such that AB = AP. Prove that CP = CQ.

B-10. If two sides of a cyclic quadrilateral are parallel, prove that the remaining two sides are equal and the
diagonals are also equal.

B-11. D is a point on the circumcircle of ABC in which AB = AC such that B and D are on the opposite side
of line AC. If CD is produced to a point E such that CE = BD, prove that AD = AE.

B-12. Two circles intersect at two points B and C. Through B, two line segments ABD and PBQ are drawn to
intersect the circles at A, D and P, Q respectively as shown in figure. Prove that ACP = QCD.

B-13. In the figure given below, two circles intersect at A and D, and AC, AB are respectively the diameters
of the circles. Prove that the points C, D, B are collinear.

B-14. ABCD is a parallelogram. The circle through A, B, C intersects CD (produced if necessary) at E.


Prove that AD = AE.

B-15. Bisectors of angles A, B and C of a triangle ABC intersect its circumcircle at D, E and F respectively.
1 1 C
Prove that the angles of the triangle DEF are 90º – A, 90º – B and 90º – .
2 2 2

For Unacademy Subscription Use “PJLIVE” Code | Join t.me/pjsir42 for Updates
For More Info: “75970 – 84242, 94590 – 43333 / 2222”
OBJECTIVE QUESTIONS
Section (A) : Chord properties of Circles
A-1. In a circle of radius 10 cm, the length of chord whose distance is 6 cm from the centre is :
(A) 4 cm (B) 5 cm (C) 8 cm (D) 16 cm

A-2. If BAD = ADC , then

(A) AB = CD (B) AB  CD (C) AC  BD (D) AD = BC

A-3. O is the centre of a circle, AB and CD are two chords of the circle. OM ⊥ AB and ON ⊥ CD. If OM =
ON = 3 cm and AM = BM = 4.5 cm, then CD =

(A) 9 cm (B) 10 cm (C) 8 cm (D) 6 cm

A-4. If AB  CD , then :

(A) 1 = 2 (B) 3 = 4 (C) 2 = 3 (D) None of these.

Section (B) : Results on angles subtended by arcs and Cyclic quadrilateral


B-1. In the given circle ABCD, O is the centre and BDC = 42º. The ACB is equal to :

(A) 48º (B) 45º (C) 42º (D) 60º

B-2. In the diagram, O is the centre of the circle. The angles CBD is equal to :

(A) 25º (B) 50º (C) 40º (D) 130º

For Unacademy Subscription Use “PJLIVE” Code | Join t.me/pjsir42 for Updates
For More Info: “75970 – 84242, 94590 – 43333 / 2222”
B-3. In the given figure, CAB = 80º, ABC = 40º. The sum of DAB + ABD is equal to :

(A) 80º (B) 100º (C) 120º (D) 140º

B-4. In the given figure, if C is the centre of the circle and PQC = 25º and PRC = 15º, then QCR is
equal to :

(A) 40º (B) 60º (C) 80º (D) 120º

B-5. The sides BA and CD of a cyclic quadrilateral ABCD are produced to meet at P, the sides DA and CB
are produced to meet at Q. If ADC = 85º and BPC = 40º, then CQD equals :
(A) 50º (B) 45º (C) 30º (D) 75º

B-6. In the given figure, if ACB = 40º, DPB = 120º, then y will be :

(A) 40º (B) 20º (C) 0º (D) 60º

B-7. In a cyclic quadrilateral if A – C = 70º, then the greater of the angles A and C is equal to :
(A) 95º (B) 105º (C) 125º (D) 115º

B-8. The length of a chord of a circle is equal to the radius of the circle. The angle which this chord subtends
on the longer segment of the circle is equal to :
(A) 30º (B) 45º (C) 60º (D) 90º

B-9. In the given figure, AB = BC = CD, If BAC = 25º, then value of AED is :

(A) 50º (B) 60º (C) 65º (D) 75º

For Unacademy Subscription Use “PJLIVE” Code | Join t.me/pjsir42 for Updates
For More Info: “75970 – 84242, 94590 – 43333 / 2222”
B-10. A, B and C are three points on the circle whose centre is O. If BAC = x, CBO = BCO = y,
BOC = t, reflex BOC = z, then :

(A) x + y = 90º (B) x – y = 90º (C) t + 2yº = 90º (D) None of these

B-11. O is the centre of the circle. BC is a chord of the circle and point A lies on the circle. If BAC = x,
OBC = y, then x + y :

(A) > 90º (B) = 90º (C) < 90º (D) > 180º

EXERCISE – 02

OBJECTIVE QUESTIONS
1. Let P be a point on the circumference of a circle. Perpendiculars PA and PB are drawn to points A and
B on two mutually perpendicular diameters. If AB = 36 cm, the diameter of the circle is :
(A) 16 cm (B) 24 cm (C) 36 cm (D) 72 cm

2. A semicircle is drawn with AB as its diameter. From C, a point on AB, a line perpendicular to AB is
drawn meeting the circumference of the semicircle at D. Given that AC = 2 cm and CD = 6 cm, the area
of the semicircle is:
(A) 32 (B) 50 (C) 40 (D) 36

3. In the figure given above, A and B are the centers of the two congruent circles with radius 17 units.
If AB = 30 units, the length of the common chord DC is :

(A) 25 units (B) 18 units (C) 10 units (D) 16 units

4. In the diagram the circle contains the vertices A, B, C of triangle ABC. Now ABC is 30º and the
length of AC is 5. The diameter of the circle is :

(A) 5 3 (B) 8 (C) 10 (D) 5 5

For Unacademy Subscription Use “PJLIVE” Code | Join t.me/pjsir42 for Updates
For More Info: “75970 – 84242, 94590 – 43333 / 2222”
5. Find the measure of angle y in the figure if P is the centre of the circle :

(A) 99º (B) 105º (C) 90º (D) 109º

6. The centre of a circle is at O. AB and CD are two chords of length d and  respectively. If P is the mid
point of CD, then the length OP is :

1 2 1 2
(A) d2 + 2
(B) d2 − 2
(C) d + 2
(D) d − 2

2 2

7. BC is the diameter of a circle. Points A and D are situated on the circumference of the semi-circle
ABD = 35º and BCD = 60º, ADB equals to :

(A) 20º (B) 25º (C) 30º (D) 115º

8. Find the value of a + b, if b = 2a.

(A) 40º (B) 80º (C) 120º (D) 160º

9. In the given figure value of 'a' is :

(A) 30º (B) 40º (C) 60º (D) 90º

For Unacademy Subscription Use “PJLIVE” Code | Join t.me/pjsir42 for Updates
For More Info: “75970 – 84242, 94590 – 43333 / 2222”
10. ABCD is a cyclic quadrilateral inscribed in a circle with the centre O. Then OAD is equal to :

(A) 30º (B) 40º (C) 50º (D) 60º

11. In figure, O is centre, then BXD =

(A) 65º (B) 60º (C) 70º (D) 55º

12. Which of the following shapes of equal perimeter, the one having the largest area is :
(A) circle (B) equilateral triangle (C) square (D) regular pentagon

13. Let XYZ be right angle triangle, with right angle at Z. Let Ax de-remarks the area of the circle with
diameter YZ. Let Ay de-remark the area of the circle with diameter XZ and let Az de-remarks the area
of the circle diameter XY. Which of the following relations is true ?
(A) Az = Ax + Ay (B) A Z = A 2x + A 2y (C) A 2z = A 2x + A 2y (D) A 2z = A 2x − A 2y

14. A triangle with side lengths in the ratio 3 : 4 : 5 is inscribed in a circle of radius 3. The area of the
triangle is equal to :
(A) 8.64 (B) 12 (C) 6 (D) 10.28

15. Two legs of a right triangle are 8  and 9  as shown. A circle is drawn so that the area inside the
circle but outside the triangle equals the area inside the triangle but outside the circle. The radius of the
circle is (Use  = 22/7)

(A) 6  (B) 6 (C) 5 (D) 8

16. In the diagram O is the centre of a circle. AE + EB = CE + ED. OP ⊥ AB and OQ ⊥ CD, then true
relation between OP and OQ is :

1
(A) OP > OQ (B) OP < OQ (C) OP = OQ (D) OP = OQ
2

For Unacademy Subscription Use “PJLIVE” Code | Join t.me/pjsir42 for Updates
For More Info: “75970 – 84242, 94590 – 43333 / 2222”
17. PQ and RS are the chords of a circle each of which is at a distance of 5 cm from the centre. If the radius
of the circle is 10 cm then the length of the chords PQ and RS are __ and __ respectively.
(A) 5 3 cm, 10 3 cm (B) 5 3 cm, 5 3 cm (C) 10 3 cm, 10 3 cm (D) 10 3 cm, 5 3 cm

18. EF and FG are two equal chords of a circle intersecting each other at F. If EG is the diameter of the
circle, EF = FG = 7 cm, find the length of EG.
(A) 7 2 cm (B) 49 cm (C) 49 2 cm (D) 7 cm

19. If the distance between two parallel chords of a circle is 6 cm and each chord is 8 cm long, then the
radius of the circle is
(A) 6 cm (B) 5 cm (C) 4 cm (D) 3 cm

20. If the angle between two equal chords which are intersecting on the circle is 60º, then what are the
angles subtended by the chords at the centre of the circle?
(A) 30º, 30º (B) 60º, 60º (C) 90º, 90º (D) 120º, 120º

21.

In the figure above (not to scale), AB is the diameter of the circle with centre O. If ACO = 30º, then
find BOC.
(A) 45º (B) 50º (C) 55º (D) 60º

22.

In the figure above (not to scale), if AB, BC, CD and DA are equal chords. Find the angle between any
two adjacent chords.
(A) 60º (B) 90º (C) 70º (D) 80º

23.

In the figure above (not to scale), AB || CD , AB = CD and if BCD = 55º and O is the centre of the
circle, then AOB = _____.
(A) 55º (B) 65º (C) 70º (D) 75º

24.

In the figure above (not to scale), chord AB = chord AC and if BAO = 25º, then find BOC where
'O' is the centre.
(A) 50º (B) 60º (C) 100º (D) Cannot be determined

For Unacademy Subscription Use “PJLIVE” Code | Join t.me/pjsir42 for Updates
For More Info: “75970 – 84242, 94590 – 43333 / 2222”
25.

In the figure above (not to scale), O is the centre of the circle and OBA = 30º. Find ACS.
(A) 60º (B) 90º (C) 30º (D) 120º

26. MN and KL are two chords of a circle of length 12 cm each, when produced they meet at the point C. If
CM = 20 cm, then CL = ___.
(A) 12 cm (B) 10 cm (C) 8 cm (D) 6 cm

27.

In the figure above (not to scale), AB = CD and AB and CD are produced to meet at the point P. If
BAC = 70º, then find P.
(A) 30º (B) 40º (C) 60º (D) 50º

28.

In the figure above (not to scale), PAO = 40º, O is the centre of the circle and PB is the diameter, find
OBA.
(A) 30º (B) 40º (C) 50º (D) 60º

29.

In the figure above (not to scale), CAD = 40º, ADC = 100º and ACB = 20º then BAC is ___.
(A) 60º (B) 80º (C) 100º (D) 120º

30.

In the figure above (not to scale), AB = AD, BC = CD and AE is the perpendicular from A to BD . If
DAE = xº, then find BCD.
(A) xº (B) 2xº (C) (90 – x)º (D) (180 – 2x)º

31. MN and ST are two equal chords of a circle and the distance between them is 12 cm. If the length of
each chord is 15 cm, find the radius of the circle approximately.
(A) 9.6 cm (B) 8.6 cm (C) 7 cm (D) 10 cm

32. In the figure given below, O is the centre of the circle, CD and DE are chords measuring 8 cm each,
intersecting at D. And the radius (OD) of the circle.
For Unacademy Subscription Use “PJLIVE” Code | Join t.me/pjsir42 for Updates
For More Info: “75970 – 84242, 94590 – 43333 / 2222”
(A) 8 2 cm (B) 8 cm (C) 4 2 cm (D) 4 cm

33.

In the above figure, BC is the diameter of the circle with centre O and AB || OD . If DOC = 40º, then
find ACB.
(A) 50º (B) 60º (C) 70º (D) 90º

34. Two equal chords of a circle intersect on the circle, make angle xº and form a triangle with a diameter.
What is the angle subtended by each chord at the centre?
(A) 30º (B) 60º (C) 90º (D) Cannot be determined

35.

In the above figure, AB and CD are equal chords. O is the centre of the circle and AD and BC are
intersecting at O. If OAC = 40º then find BCD and CBA.
(A) 40º, 40º (B) 50º, 50º (C) 40º, 50º (D) 50º, 40º

36.

In the above figure (not to scale), AC is the diameter and ABCD is the cyclic quadrilateral. If EAB = 80º
and DCF = 110º, then find the angle made by AB at the centre of the circle.
(A) 80º (B) 60º (C) 40º (D) 20º

37. If the adjacent angles of a cyclic quadrilateral are 40º and 100º, then the angle made by the diagonals at
the centre of the circle is
(A) 40º, 50º (B) 20º, 25º (C) 80º, 100º (D) 80º, 160º

38.

In the above figure (not to scale), ABCDEF is a regular hexagon inscribed in a circle, what is AOE?
(A) 60º (B) 100º (C) 120º (D) 140º

For Unacademy Subscription Use “PJLIVE” Code | Join t.me/pjsir42 for Updates
For More Info: “75970 – 84242, 94590 – 43333 / 2222”
39.

In the above figure, AB = AC and O is the centre of the circle. If ACO = 40º, then find ABO.
(A) 40º (B) 80º (C) 35º (D) 55º

40.

In the above figure (not to scale), ABC = 2ACB. Find the angle made by AC at the centre of the
circle, if BAC = 60º.
(A) 80º (B) 120º (C) 160º (D) 180º
41.

In the above figure, AB and AC are equal chords and O is the centre. If BOC = 100º, then find ACO.
(A) 10º (B) 20º (C) 30º (D) 25º

42.

In the above figure, AB and AC are equal chords of the given circle, BP = PC and ABP = PBC. If
BAC = 40º, then find BPC.
(A) 90º (B) 100º (C) 110º (D) 120º

43.

In the above figure (not to scale), ABCD is a cyclic quadrilateral, AB = BC, BAC = 40º and DCA = 70º
then find CAD.
(A) 30º (B) 50º (C) 80º (D) 100º

44.

In the above figure (not to scale), PA and PB are equal chords and ABCD is a cyclic quadrilateral. If
DCE = 80º, DAP = 30º then find APB.
(A) 40º (B) 80º (C) 90º (D) 160º

For Unacademy Subscription Use “PJLIVE” Code | Join t.me/pjsir42 for Updates
For More Info: “75970 – 84242, 94590 – 43333 / 2222”
45. In the figure below, PQ and RS are chords of length 10 cm each intersecting at B. If PBS = 90º and
the area of the PBS is 32 sq. cm., then find the length of BR.

(A) 2 cm (B) 4 cm (C) 6 cm (D) 8 cm

46. In the figure below, EF is a chord produced to G. O is the centre of the circle. EF = 12 cm and FG = 8 cm.
The radius of the circle is 10 cm. Find the approximate length of OG.

(A) 8 cm (B) 12 cm (C) 16 cm (D) 20 cm

47.

In the above diagram (not to scale), AB = AC. O is the centre of the circle. If ABC = 80º, then find BOC.
(A) 20º (B) 40º (C) 60º (D) 80º

48. In the figure below, CD is a chord of the semi-circle with centre O.

OA is the radius of the circle. If CD = 10 cm, AB = 2 cm and OA ⊥ CD , then find the length of OB.
(A) 3.25 cm (B) 5.25 cm (C) 4.25 cm (D) 6.25 cm

49. A, B, C, D and E are concyclic such that AB = BC = CD = DE = EA. Find the angle between any two
adjacent chords.
(A) 32º (B) 42º (C) 54º (D) 108º

50.

In the figure above (not to scale), AB = AC, BD = DC and 'O' is centre of the circle. If ACO = 35º
then find ABD and BDC.
(A) 90º, 90º (B) 110º, 110º (C) 90º, 110º (D) 80º, 90º

51. There are four chords AB, BC, CD and DA in a circle making angles , 2, 3 and 4 respectively at
the centre of a circle. Find the angle between BC and CD.
(A) 30º (B) 45º (C) 60º (D) 90º

52. In the given figure (not to scale), O is the centre of the circle and AB = BC. If AQC = 80º, then find
the angle subtended by the chord AB at the centre.
For Unacademy Subscription Use “PJLIVE” Code | Join t.me/pjsir42 for Updates
For More Info: “75970 – 84242, 94590 – 43333 / 2222”
(A) 70º (B) 80º (C) 100º (D) Cannot be determined

53. In the figure below (not to scale), AB and CD are equal chords. O is the centre of the circle. If AOP = 120º,
then find OAC.

(A) 30º (B) 60º (C) 80º (D) None of these

54.

In the above figure, O is the centre of the circle and AB = CD. If APB = 110º, then find the angle
made by the chord CD at the centre.
(A) 220º (B) 110º (C) 120º (D) 140º

55. In the above figure, ED || AB and EF || BC . If FED = 40º and DEC = 20º, then find the angle made
by BC at the centre.

(A) 20º (B) 40º (C) 60º (D) 80º

56.

In the above figure, O is the centre of the circle ABO = OCO and ACO = OBO. If BAC = 40º
then find BDC.
(A) 100º (B) 110º (C) 120º (D) 150º

For Unacademy Subscription Use “PJLIVE” Code | Join t.me/pjsir42 for Updates
For More Info: “75970 – 84242, 94590 – 43333 / 2222”
57.

In the figure above, DBA = 2DAB = 4CAD. If ADC = 120º then find the angle made by AB at
the centre of the circle. ·
(A) 20º (B) 40º (C) 60º (D) 80º

58.

In the above figure, AB and AC are equal chords and AD is perpendicular to AC . If COD = 60º,
then angle between the chords is
(A) 30º (B) 60º (C) 90º (D) Cannot be determined

59.

In the above figure (not to scale) AB , BC , CF , DE and FE are chords of the circle. If ABC = 100º
and FED = 110º, then find FPA.
(A) 20º (B) 30º (C) 40º (D) 70º

60.

In the above figure (not to scale), O is the centre of the circle. BC and CD are equal chords. If OBC = 70º,
then what is BAD?
(A) 40º (B) 60º (C) 80º (D) 100º

61. In the following figure (not to scale), O is the centre of the circle. A, B, C and Dare concyclic and
AB = CD. If MON = 120º, then find OPN.

(A) 20º (B) 30º (C) 40º (D) 60º

For Unacademy Subscription Use “PJLIVE” Code | Join t.me/pjsir42 for Updates
For More Info: “75970 – 84242, 94590 – 43333 / 2222”
62. In the following figure, MN and OP are equal chords of length 10 cm each intersecting at the point A
If OA = 3 cm and PAM = 90º, then find the ratio of the areas of the triangles PAM and OAN.

(A) 9 : 49 (B) 7 : 3 (C) 49 : 9 (D) 7 2 : 3 2


63.

In the figure (not to scale) AB = CD, DAP = DPA and A, B, C, Dare concyclic. If ADC = 80º
then find ACD.
(A) 50º (B) 60º (C) 70º (D) 80º
64.

In the above figure, O is the centre of the circle and CD || AB . If DAO = 20º, then find AOB.
(A) 40º (B) 60º (C) 100º (D) 120º

65.

In the above figure, O is the centre of the circle and chords AB and CD are equal. If ABP = 40º and
PQ ⊥ AB , then find the angle made by the chord CD at the centre.
(A) 160º (B) 110º (C) 120º (D) 140º

66.

In the above figure, O is the centre of the circle and chords AB and AC are equal. If ABO = 20º, then
find BOC.
(A) 40º (B) 60º (C) 80º (D) 100º
67.

In the above figure, (not to scale), O is the centre of the circle. AP and BP are two chords. C is point
of intersection of AP and OB . OAC = 30º and PBC = 80º then find AOB.
(A) 50º (B) 100º (C) 80º (D) 120º

For Unacademy Subscription Use “PJLIVE” Code | Join t.me/pjsir42 for Updates
For More Info: “75970 – 84242, 94590 – 43333 / 2222”
68.

In the above figure AP = PB and O is the centre of the circle. If AOB = 80º and OAP = 20º, then
find APB.
(A) 60º (B) 90º (C) 120º (D) 160º

69. A point P is 10 cm from the centre of a circle. The length of the tangent drawn from P to the circle is 8
cm. The radius of the circle is equal to
(A) 4 cm (B) 5 cm (C) 6 cm (D) None of these.

70.

In the above figure, ABCD is a cyclic quadrilateral and BCD = 2BAD, find the angle made by the
diagonal BD at the centre of the circle.
(A) 60º (B) 80º (C) 100º (D) 120º

71.

In the above figure (not to scale), ABCD is a cyclic quadrilateral, DE ⊥ AB , BAO = 40º, OAD = 20º
and OCO= 50º. find ABC.
(A) 70º (B) 100º (C) 110º (D) 120º

72. In the following figure (not to scale), ADC = 60º, BAD = 80º and EBC = 2PDE. And APE.

(A) 60º (B) 80º (C) 120º (D) 140º

73. In figure below, PA and PB are the two tangents drawn to the circle. O is the centre of the circle. A and
Bare the points of contact of the tangents PA and PB with the circle. If OPA = 35º, then POB =

(A) 55º (B) 65º (C) 75º (D) 85º

74. A tangent PQ at a point P of a circle of radius 5 cm meets a line through the centre O at a point Q such
that OQ = 12 cm. Length PQ is
(A) 12 cm (B) 13 cm (C) 8.5 cm (D) 119 cm

For Unacademy Subscription Use “PJLIVE” Code | Join t.me/pjsir42 for Updates
For More Info: “75970 – 84242, 94590 – 43333 / 2222”
75. If tangents PA and PB from a point P to a circle with centre O are inclined to each other at an angle of
80º then POA is equal to
(A) 50º (B) 60º (C) 70º (D) 80º

76. If TP and TQ are two tangents to a circle with centre O so that POQ = 110º, then, PTQ is equal to
(A) 60º (B) 70º (C) 80º (D) 90º

77. Two circle touch each other externally at C and AB is a common tangent to the circles. Then, ACB =
(A) 60º (B) (C) 30º (D) 90º

78. ABC is a right angled triangle, right angled at B such that BC = 6 cm and AB = 8 cm. A circle with
centre O is inscribed in ABC. The radius of the circle is
(A) 1 cm (B) 2 cm (C) 3 cm (D) 4 cm

79. The length of the tangent drawn from a point 8 cm away from the centre of a circle of radius 6 cm is
(A) 7 cm (B) 2 7 cm (C) 10 cm (D) 5 cm

80. In the adjoining figure, if AD, AE and BC are tangents to the circle at D, E and F respectively. Then,

(A) AD = AB + BC + CA (B) 2AD = AB + BC + CA


(C) 3AD = AB + BC + CA (D) 4AD = AB + BC + CA

81. In the given figure, tangent PT = 5 cm, PA = 4 cm, find AB

7 11 9
(A) cm (B) cm (C) cm (D) can't be determined
4 4 4

EXERCISE – 03

NTSE PROBLEMS (PREVIOUS YEARS)


1. In the following figure, O is the centre of the circle. The value of x is :
[Rajasthan NTSE Stage-1 2006]

(A) 60º (B) 45º (C) 40º (D) 80º

For Unacademy Subscription Use “PJLIVE” Code | Join t.me/pjsir42 for Updates
For More Info: “75970 – 84242, 94590 – 43333 / 2222”
2. In the figure, O is the centre of the circle and OABC is rectangle : [Kerala NTSE Stage-1 2007]

What is the length of AC ?


(A) 4 cm (B) 4.5 cm (C) 5 cm (D) 5.5 cm
3. In a circle, a 16 unit long chord is at a distance 6 units away from the center. Find the distance of a 12
unit long chord from the centre. [Gujarat NTSE Stage-1 2007]
(A) 5 (B) 6 (C) 7 (D) 8
4. In the following figure. O is the centre of the circle. The value of x is [Rajasthan NTSE Stage-1 2007]

(A) 45º (B) 65º (C) 85º (D) 95º


5. The chord of maximum length in a circle is called : [Rajasthan NTSE Stage-1 2013]
(A) Radius (B) Arc (C) Diameter (D) Point
6. In figure, A, B, C and D are four point on a cirlcle. AC and BD intersect at a point E such that BEC = 125º
and ECD = 30º. Then BAC = [Rajasthan NTSE Stage-1 2013]

(A) 95º (B) 110º (C) 85º (D) 105º


7. AB and CD are two parallel chords of a circle such that AB = 10 cm and CD = 24 cm. If the chords are
on the opposite sides of the centre and the distance between them is 17 cm, the radius of the circle is :
[Delhi NTSE Stage-1 2013]
(A) 14 cm (B) 10 cm (C) 13 cm (D) 15 cm

8. If two equal circles of radius r passes through centre of the other then the length of their common chord is.
[U.P. NTSE Stage-1 2014]
r 3
(A) (B) r 3 (C) r (D) r 2
3 4

9. In the following figure O is the centre of circle and BAC = nº, OCB = mº then
[U.P. NTSE Stage-1 2014]

(A) mº + nº = 90º (B) mº + nº = 180º (C) mº + nº = 120º (D) mº + nº = 150º

For Unacademy Subscription Use “PJLIVE” Code | Join t.me/pjsir42 for Updates
For More Info: “75970 – 84242, 94590 – 43333 / 2222”
10. AB and AC are equal chord of a circle with centre O. Then by which angle OA bisects BC.
[M.P. NTSE Stage-1 2014]
(A) 30º (B) 60º (C) 90º (D) 120º

11. In the given figure find PQR (where O is centre of the circle) [U.P. NTSE Stage-1 2014]

(A) 60º (B) 80º (C) 100º (D) 120º

12. If two chords of a circle are equidistance from the centre of the circle, then they are............
[M.P. NTSE Stage-1 2015]
(A) Equal to each other (B) Not equal to each other.
(C) Intersect each other. (D) None of these

13. In the given figure, DBC = 22º and DCB = 78º then BAC is equal to
[Rajasthan NTSE Stage-1 2015]

(A) 90º (B) 80º (C) 78º (D) 22º

14. The lengths of two parallel chords of a circle are 6 cm and 8 cm. If the smaller chord is at distance 4 cm
from the centre, then the distance of the other chord from the cnetre is [Rajasthan NTSE Stage-1 2015]
(A) 5 cm (B) 4 cm (C) 3 cm (D) 2 cm

15. In the figure, the radius of the larger circle is 2 cm and the radius of the smaller circle is 1 cm and the
larger circle passes through the centre of the smaller circle. The length (in cm) of the chord AB is :
[Haryana NTSE Stage-1 2016]

15 3 5 34
(A) (B) 2 (C) (D)
2 2 17

For Unacademy Subscription Use “PJLIVE” Code | Join t.me/pjsir42 for Updates
For More Info: “75970 – 84242, 94590 – 43333 / 2222”
16. In the figure O is the centre of the circle and POR = 80º. Then RQS is
[Rajasthan NTSE Stage-1 2016]
S R

80º O
Q

P
(A) 30º (B) 40º (C) 1400 (D) 50º

17. In the figure, the semicircle centered at O has a diameter 6 cm. The chord BC is parallel to AD and
1
BC = AD. The area of the trapezium ABCD in cm 2, is : [Haryana NTSE Stage-1 2016]
3

B C

A O D
(A) 4 (B) 4 2 (C) 8 (D) 8 2

18. A chord of length 24 cm is situated 5 cm from the centre of a circle. The diameter of the circle will be
[Chhattisgarh NTSE Stage-I/18]
(A) 24 cm (B) 29 cm (C) 26 cm (D) 13 cm

VALUE BASED / PRACTICAL BASED QUESTIONS

1. An amusement fair was organised in a circular park for the children of slum clusters. Free food was
supplied to them at 4 stalls situated at A,B, C and D as shown in the figure.
(i) Find the angle between the lines joining stalls C, D and stalls C, B.
(ii) Are such activities helpful for society ?

2. 3 STD booths situated at A, B and C in the figure are operated by handicapped persons. These three
booths are equidistant from each other as shown in the figure.
(i) Find BAC.
(ii) Find BOC
(iii) Do you think employment provided to handicapped persons is important for the development of a
society?

For Unacademy Subscription Use “PJLIVE” Code | Join t.me/pjsir42 for Updates
For More Info: “75970 – 84242, 94590 – 43333 / 2222”
Answer Key

BOARD LEVEL EXERCISE


TYPE (I) : VERY SHORT ANSWER TYPE QUESTIONS : [01 MARK EACH]
1. 3 : 1 2. 75º 3. 2 cm 4. 10 cm 5. 40º
6. 45º 7. 50º 8. 70º 9. 60º

TYPE (II) : SHORT ANSWER TYPE QUESTIONS : [02 MARKS EACH]


10. 120º 17. 270º 18. BOC = 66º and AOC = 54º

EXERCISE – 01

SUBJECTIVE QUESTIONS
Section (A) : Chord properties of Circles
A-1. 12 cm. A-2. 1 cm. A-3. 13 cm. A-4. 9.6 cm.
A-5. 20 3m.

Section (B) : Results on angles subtended by arcs and Cyclic quadrilateral


B-1. a = 45º & b = 90º. B-2. a = 140º and b = 70º B-3. 300º B-4. a = 40º and b = 90º.
B-5. 80º. B-6. 150º.

OBJECTIVE QUESTIONS
Section (A) : Chord properties of Circles
A-1. (D) A-2. (A) A-3. (A) A-4. (C)

Section (B) : Results on angles subtended by arcs and Cyclic quadrilateral


B-1. (A) B-2. (A) B-3. (C) B-4. (C) B-5. (A)
B-6. (B) B-7. (C) B-8. (A) B-9. (D) B-10. (B)
B-11. (B)

EXERCISE – 02

OBJECTIVE QUESTIONS
Ques. 1 2 3 4 5 6 7 8 9 10 11 12 13 14 15 16 17 18 19 20
Ans. D B D C D D B B B C C A A A B D C A B D
Ques. 21 22 23 24 25 26 27 28 29 30 31 32 33 34 35 36 37 38 39 40
Ans. D B C C D C B C B D A C A C B D D C A C
Ques. 41 42 43 44 45 46 47 48 49 50 51 52 53 54 55 56 57 58 59 60
Ans. D C A B A C B B D C D B A D B C D B B A
Ques. 61 62 63 64 65 66 67 68 69 70 71 72 73 74 75 76 77 78 79 80
Ans. B C C C A C B C C D A C A D A B D B B B
Ques. 81
Ans. C

For Unacademy Subscription Use “PJLIVE” Code | Join t.me/pjsir42 for Updates
For More Info: “75970 – 84242, 94590 – 43333 / 2222”
EXERCISE – 03

Ques. 1 2 3 4 5 6 7 8 9 10 11 12 13 14 15 16 17 18 19
Ans. A C D C C A C B A C B A B C A B D C A

VALUE BASED / PRACTICAL BASED QUESTIONS

1. (i) C = 80 (ii) Yes


2. (i) 60º (ii) 120º (iii) Yes

For Unacademy Subscription Use “PJLIVE” Code | Join t.me/pjsir42 for Updates
For More Info: “75970 – 84242, 94590 – 43333 / 2222”
7 HERON’S FORMULA

INTRODUCTION

Heron (or Hero) of Alexandria (c.10 – 70AD) was an ancient Greek mathematician and engineer who was
active in his native city of Alexandria, Roman Egypt. He is considered the greatest experimenter of antiquity
and his work is representative of the Hellenistic scientific tradition.
Heron’s works as an inventor truly reveal his genius but he is also accredited as a mathematician who delivered
a lot to the field with his practical approach. From approximations of square roots and formulating the rea of a
triangle to his treatise in geometry, Heron.s contributions are wide ranging. The ‘Metrica’ is a series of three
books, found by R.Schone in Istanbul in 1896, in which Heron focuses on calculating areas and volumes of
bodies such as pyramids, cones, cylinders, prisms etc. ‘Hero’s formula. was found in this book which stated the
area of a triangle with given sides.

A. HERON’S FORMULA

If a, b, c denote the lengths of the sides of a triangle ABC. Then,


Area of ABC = s(s − a )(s − b)(s − c)
a+b+c
s= is the semi - perimeter of ABC.
2

(a) Perimeter and Area of a Triangle :


(i) Right-angled triangle
For an right-angled triangle, let b be the base, h be the perpendicular and d
be the hypotenuse. Then
Perimeter = b + h + d
1 1
Area = (Base × Height) = bh
2 2
Hypotenuse, d = b2 + h 2 [Pythagoras theorem]

(ii) Isosceles right-angled triangle


For an isosceles right-angled triangle, let a be the equal sides, then
Hypotenuse = a + a = 2a
2 2

Perimeter = 2a + 2a

1 1 1
Area = (Base × Height) = (a × a) = a2.
2 2 2

For Unacademy Subscription Use “PJLIVE” Code | Join t.me/pjsir42 for Updates
For More Info: “75970 – 84242, 94590 – 43333 / 2222”
(iii) Equilateral triangle
For an equilateral triangle, let each side be a, and the height of the triangle is h, then
A = B = C = 60º
BAD = CAD = 30º
AB = BC = AC = a (say)
a
BD = DC =
2
2
a 3a 2
  +h =a 
2 2
h2 =
 
2 4
3
 Height (h) = a
2
1 1
Area = (Base × Height) = × a × 3 a = 3 a 2
2 2 2 4
Perimeter = a + a + a = 3a.

(b) Application of Heron’s formula


With the help of heron's formula we can find the area of Quadrilateral whose all side and one diagonal
is given to us, Quadrilateral whose all sides and angle between any two adjacent side is right angle,
Trapezium whose all sides are given and we can find the area of Parallelogram and its types.

Solved Examples

Example.1 The perimeter of a triangular field is 450 m and its sides are in the ratio 13 : 12 : 5. Find the
area of the triangle.
Solution. Let a = 13x, b = 12x and c = 5x
 Perimeter = 450
13x + 12x + 5x = 450
30x = 450
x = 15
So, the sides of the triangle are :
a = 13 × 15 = 195 m, b = 12 × 15 = 180 m and c = 5 × 15 = 75 m
2s = 195 + 180 + 75
2s = 450
s = 225.
Hence, Area = s(s − a )(s − b)(s − c) = 225(225 − 195)(225 − 180)(225 − 75)
= 225 ( 30 )( 45 )(150 ) = 6750 m2.

3
Example.2 The perimeter of an isosceles triangle is 42 cm and its base is times each of the equal sides.
2
Find the length of each side of the triangle, area of the triangle and height of the triangle.
Solution. Let the equal sides be a and unequal side be b.
 2a + b = 42
3
Given : b = a
2
3
2a + a = 42
2
7
a = 42
2
a = 12
3
so b = (12) = 18 cm.
2
For Unacademy Subscription Use “PJLIVE” Code | Join t.me/pjsir42 for Updates
For More Info: “75970 – 84242, 94590 – 43333 / 2222”
Also, perimeter = 2s = 42 cm
s = 21 cm.
Area of triangle = 21(21 − 12)(21 − 12)(21 − 18) = 21( 9 )( 9 )( 3) = 27 7 cm
2

1
Area of triangle = × base × height
2
1
27 7 =  18 × height
2
Height = 54 7 = 3 7 cm
18

Example.3 A triangle and parallelogram have the same base and the same area. If the sides of the triangle
are 26 cm, 28 cm and 30 cm, and the parallelogram stands on the base 28 cm, find the height of
the parallelogram.
26 + 28 + 30
Solution. s= = 42cm
2
 Area of the triangle = 42(42 − 26)(42 − 28)(42 − 30) = 42 (16 )(14 )(12 ) = 336 cm2.
Let h be the height of the parallelogram.
It is given that the triangle and the parallelogram have the same base and same area.
 Area of the parallelogram = 336 cm2
336
 Base × height = 336  28 × h = 336  h = = 12cm
28

Example.4 Find the area of a trapezium whose parallel sides are 25 cm, 13 cm and other sides are 15 cm
and 15 cm.
Solution. Let ABCD be the given trapezium in which AB = 25 cm, CD = 13 cm, BC = 15 cm and
AD = 15 cm.
Draw CE || AD.
Now, ADCE is a parallelogram is which AD || CE and AE || CD.
 AE = DC = 13 cm and BE = AB – AE = 25 – 13 = 12 cm.
In BCE, we have
15 + 15 + 12
s= = 21
2
 Area of BCE = s(s − a )(s − b)(s − c)
 Area of BCE = 21(21 − 15)(21 − 15)(21 − 12)
 Area of BCE = 21 6  6  9
= 18 21 cm2 ...(i)
Let h be the height of BCE, then
1
Area of BCE = (Base × Height)
2
1
=  12  h = 6h ..(ii)
2
From (i) and (ii), we have,
6h = 18 21
 h = 3 21 cm
Clearly, the height of trapezium ABCD is same as that of BCE.
1
 Area of trapezium = (AB + CD) × h
2
1
 Area of trapezium = (25 + 13) × 3 21 cm2 = 57 21 cm2.
2

For Unacademy Subscription Use “PJLIVE” Code | Join t.me/pjsir42 for Updates
For More Info: “75970 – 84242, 94590 – 43333 / 2222”
Example.5 Find the percentage increase in the area of a triangle if its each side is doubled.
Solution. Let a, b, c be the sides of the given triangle and s be its semi-perimeter.
1
 s = (a + b + c) ...(i)
2
The sides of the new triangle are 2a, 2b and 2c.
Let s’ be its semi-perimeter.
1
 s’= 2 (2a + 2b + 2c) = a + b + c = 2s [Using (i)]
2
Let  = Area of given triangle
= s(s − a )(s − b)(s − c) ... (ii)
And, ’ = Area of new triangle
’ = s'(s'− 2a )(s'− 2b)(s '− 2c)
= 2s(2s − 2a )(2s − 2b)(2s − 2c) [Using (i)]

= 16s(s − a )(s − b)(s − c)


’ = 4  [Using (ii)]
 Increase in the area of the triangle
= ’ –  = 4 –  = 3
 % increase in area =  3  100  % = 300 .
  

Example.6 An umbrella is made by stitching 10 triangular pieces of cloth of two different colours (see
figure), each piece measuring 20 cm, 50 cm and 50 cm. How much cloth of each colour is
required for the umbrella ?
Solution. The sides of a triangular piece are 20 cm, 50 cm and 50 cm.
a + b + c 20 + 50 + 50
s=− = = 60cm.
2 2
Area of one triangular piece = s(s − a )(s − b)(s − c)
= 60(60 − 20)(60 − 50)(60 − 50) = 60  40 10 10
= 240000 = 200 6 cm2
Area of cloth of each colour for five triangular pieces = 5 × 200 6
= 1000 6 cm2.

Example.7 A rhombus shaped field has green grass for 18 cows to graze. If each side of the rhombus is 30
m and its longer diagonal is 48 m, how much area of grass field will each cow be grazing ?
Solution. Clearly, triangles ABC and ADC are congruent.
 Area of ABC = Area of ADC
Let s be the semi - perimeter of ABC and ADC. Then,
30 + 30 + 48
s= = 54m.
2
 Area of ABC = Area of ADC
= 54(54 − 30)(54 − 30)(54 − 48) = 54(24)(24)(6) = 432 m2
So, area of rhombus ABCD = 2 (Area of ABC) = 2 (432) = 864 m2
864
 Area of grass field each cow will graze = = 48 m2.
18

Example.8 The sides of a quadriangular field, taken in order are 26 m, 27 m, 7 m and 24 m respectively.
The angle contained by the last two sides is a right angle. Find its area. [Take 14 = 3.751]
1 1
Solution. Area of ADC = (AD × DC) = × 24 × 7m2 = 84 m2
2 2
In ADC, we have 90º
AC2 = AD2 + CD2
For Unacademy Subscription Use “PJLIVE” Code | Join t.me/pjsir42 for Updates
For More Info: “75970 – 84242, 94590 – 43333 / 2222”
 AC2 = 242 + 72
 AC2 = 252
 AC = 25 m.
Thus, in ABC,we have
a = BC = 27 m, b = CA = 25 m
and c = AB = 26 m
Let 2s be the perimeter of ABC. Then,
2s = a + b + c
2s = 27 + 25 + 26 = 78
 s = 39 m
 Area of ABC = s(s − a )(s − b)(s − c) = 39 12 14 13
 Area of ABC = 13  3  3  4  2  7 13 = 78 14 cm2 = 292.57 m2
Area of quadrilateral ABCD = 84 + 291.57 = 376.57 m2.

Check Point - A

1. Find the area of the triangles whose sides are 25 m, 60 m and 65 m

2. Two sides of a triangular field are 85 m and 154 m and its perimeter is 324 m. Find its area.

3. Find the area of a triangle whose sides are 50 m, 78 m and 112 m and also find the length of the
perpendicular from the opposite vertex to the side of length 112 m.

4. Find the area of a trapezium whose parallel sides are 11 m and 25 m and the two non parallel sides are
15 m and 13 m.

5. Find the area of a quadrilateral piece of land one of whose diagonals is 32 m long and the length of the
perpendiculars from the other two vertices are 20 m and 12 m.

Answers
1. 750 m2 2. 2772 m2 3. 1680 m2, 30m 4. 216 m2 5. 512 m2

For Unacademy Subscription Use “PJLIVE” Code | Join t.me/pjsir42 for Updates
For More Info: “75970 – 84242, 94590 – 43333 / 2222”
BOARD LEVEL EXERCISE

TYPE (I) : VERY SHORT ANSWER TYPE QUESTIONS : [01 MARK EACH]

1. The base of a right triangle is 8 cm and hypotenuse is 10 cm. Its area will be

2. An isosceles right triangle has area 8 cm2. Find the length of its hypotenuse.

3. The perimeter of an equilateral triangle is 60 m. Find its area ?

4. The sides of a triangle are 56 cm, 60 cm and 52 cm long. Find the area of the triangle is

5. Find the area of an equilateral triangle with side 2 3 cm

6. Find the length of each side of an equilateral triangle having an area of 9 3 cm2 ?

TYPE (II) : SHORT ANSWER TYPE QUESTIONS : [02 MARKS EACH]

7. Find the area of an isosceles triangle having base 2 cm and the length of one of the equal sides 4 cm ?

8. The sides of a triangle are 35 cm, 54 cm and 61 cm, respectively. The length of its longest altitude ?

9. The sides of a triangular field are 41 m, 40 m and 9 m. Find the number of rose beds that can be
prepared in the field, if each rose bed, on an average need 900 cm2 space.

10. The perimeter of an isosceles triangle is 32 cm. The ratio of the equal side to its base is 3 : 2. Find the
area of the triangle

11. A field in the form of a parallelogram has sides 60 m and 40 m and one of its diagonals is 80 m long.
Find the area of the parallelogram.

TYPE (III) : LONG ANSWER TYPE QUESTIONS: [03 MARK EACH]

12. Calculate the area of the shaded region in figure.

13. The perimeter of a triangular field is 420 m and its sides are in the ratio 6 : 7 : 8. Find the area of the
triangular field

14. A rhombus shaped sheet with perimeter 40 cm and one diagonal 12 cm, is painted on both sides at the
rate of Rs 5 per cm2. Find the cost of painting.

For Unacademy Subscription Use “PJLIVE” Code | Join t.me/pjsir42 for Updates
For More Info: “75970 – 84242, 94590 – 43333 / 2222”
15. Find the area of the trapezium PQRS with height PQ given in Figure.

16. The perimeter of a triangle is 50 cm. One side of a triangle is 4 cm longer than the smaller side and the
third side is 6 cm less than twice the smaller side. Find the area of the triangle.

17. The area of a trapezium is 475 cm2 and the height is 19 cm. Find the lengths of its two parallel sides if
one side is 4 cm greater than the other.

18. In Figure, ABC has sides AB = 7.5 cm, AC = 6.5 cm and BC = 7 cm. On base BC a parallelogram
DBCE of same area as that of ABC is constructed. Find the height DF of the parallelogram.

19. The dimensions of a rectangle ABCD are 51 cm × 25 cm. A trapezium PQCD with its parallel sides QC
and PD in the ratio 9 : 8, is cut off from the rectangle as shown in the Figure. If the area of the
5
trapezium PQCD is th part of the area of the rectangle, find the lengths QC and PD.
6

For Unacademy Subscription Use “PJLIVE” Code | Join t.me/pjsir42 for Updates
For More Info: “75970 – 84242, 94590 – 43333 / 2222”
20. A design is made on a rectangular tile of dimensions 50 cm × 70 cm as shown in Figure. The design
shows 8 triangles, each of sides 26 cm, 17 cm and 25 cm. Find the total area of the design and the
remaining area of the tiles.

TYPE (IV): VERY LONG ANSWER TYPE QUESTIONS [04 MARK EACH]

21. From a point in the interior of an equilateral triangle, perpendiculars are drawn on the three sides. The
lengths of the perpendiculars are 14 cm, 10 cm and 6 cm. Find the area of the triangle

22. If each side of a triangle is doubled, then find the ratio of area of the new triangle thus formed and the
given triangle.

23. A field is in the shape of a trapezium having parallel sides 90 m and 30 m. These sides meet the third
side at right angles. The length of the fourth side is 100 m. If it costs Rs 4 to plough 1m2 of the field,
find the total cost of ploughing the field.

For Unacademy Subscription Use “PJLIVE” Code | Join t.me/pjsir42 for Updates
For More Info: “75970 – 84242, 94590 – 43333 / 2222”
EXERCISE – 01

SUBJECTIVE QUESTIONS
Section (A) : Heron’s formula
A-1. Calculate the area of the triangle whose sides are 18 cm, 24 cm and 30 cm in length. Also, find the
length of the altitude corresponding to the smallest side of the triangle.

A-2. The sides of a triangle are 10 cm, 24 cm and 26 cm. Find its area and the longest altitude.

A-3. Two sides of a triangular field are 85 cm and 154 cm in length, and its perimeter is 324 cm. Find (i) the area
of the field, and (ii) the length of the perpendicular from the opposite vertex on the side measuring 154 cm.

A-4. The sides of a triangular field are 165 cm, 143 cm and 154 cm. Find the cost of ploughing it at 12 paise
per sq.cm.

A-5. If the height of an equilateral triangle is 3 cm. Then find the area of the triangle.

A-6. Students of a school staged a rally for cleanliness campaign. They walked through the lanes in two groups.
One group walked through the lanes AB, BC and CA; while other through AC, CD and DA (figure). Then
they cleaned the area enclosed within their lanes. If AB = 9 m, BC = 40 m, CD = 15 m, DA = 28 m and B
= 90º. Which group cleaned more area and how much ? Find the total area cleaned by the students.

A-7. The perimeter of a right angle triangle is 40 cm. Its hypotenuse is 17 cm. Find the sides containing the
right angle. Also find the area of the triangle using heron’s formula.

A-8. Find the perimeter and area of the quadrilateral ABCD in which AB = 17 cm, AD = 9 cm, CD = 12 cm,
ACB = 90º and AC = 15 cm.

A-9. The perimeter of an isosceles triangle is 32 cm. The ratio of one of the equal sides to the base is 3:2.
Find the area of the triangle.

A-10. Rajesh has a triangular field with sides 240 m, 200 m, 360 m, where he grew wheat. In another triangular
field with sides 240 m, 320 m, 400 m adjacent to the previous field, he wanted to grow potatoes and onions
as shown in figure. He divided the field in two parts by joining the mid-point of the longest side to the
opposite vertex and grew potatoes in one part and onions in the other part. How much area (in hectares) has
been used for wheat, potatoes and onions ? [ 2 = 1.41 and 1 hectare = 10000 m2]

For Unacademy Subscription Use “PJLIVE” Code | Join t.me/pjsir42 for Updates
For More Info: “75970 – 84242, 94590 – 43333 / 2222”
A-11. A triangular park ABC has sides 120 m, 80 m and 50 m. A gardener Ramu has to put a fence all around
it and also plant grass inside. How much area does he need to plant? Find the cost of fencing it with
barbed wire at the rate of Rs. 20 per meter leaving a space 3 m wide for a gate on one side.

A-12. A field is in the shape of a trapezium whose parallel sides are 60 m and 77 m. The non-parallel sides are
25 m and 26 m. Find the area of the field.

OBJECTIVE QUESTIONS
Section (A) : Heron’s formula
A-1. The triangular side walls of a flyover have been used for advertisements from both sides. The sides of
each wall are 120 m, 110 m and 20 m. The advertisement yield and earning of Rs. 100 per m 2 per year.
Find the amount of revenue earned in one year. [Take 7 = 2.65]

(A) Rs. 3, 37, 500 (B) Rs. 3, 97, 500 (C) Rs. 5, 73, 300 (D) Rs. 4, 73, 500

A-2. If the adjacent sides of a parallelogram ABCD measure 34 cm and 20 cm, and the diagonal AC
measures 42 cm, then its area in cm2 is :
(A) 796 (B) 672 (C) 692 (D) none of these

A-3. The lengths of four sides and a diagonal of the given quadrilateral are indicated in the diagram. If A
denotes the area of quadrilateral, then A in cm2 is :

(A) 12 6 (B) 6 (C) 6 6 (D) 6

A-4. If the sides of a triangle are doubled, then its area :


(A) Remains the same (B) Becomes doubled
(C) Becomes three times (D) Becomes four times

A-5. Find the cost of painting of a rhombus sheet, whose perimeter is 32 m and whose one diagonal is 10 m
long is painted on both sides at the rate of Rs. 5 per m2. [Use 39 = 6.25]
(A) Rs. 500 (B) Rs. 600 (C) Rs. 625 (D) Rs. 650

A-6. Find the height of the trapezium in which parallel sides are 25 cm and 10 cm and non parallel sides are
14 cm and 13 cm.
For Unacademy Subscription Use “PJLIVE” Code | Join t.me/pjsir42 for Updates
For More Info: “75970 – 84242, 94590 – 43333 / 2222”
(A) 13 cm (B) 14 cm (C) 11.2 cm (D) 13.2 cm

A-7. Calculate the area of the shaded portion.

(A) 30 cm2 (B) 84 cm2 (C) 144 cm2 (D) 54 cm2

A-8. The unequal side of an isoseles triangle is 6 cm and its perimetre is 24 cm. Find its area.
(A) 6 2 cm2 (B) 12 2 cm2 (C) 18 2 cm2 (D) 24 2 cm2

A-9. A triangle and a parallelogram have the same base and same area. If the sides of the triangle are 15 cm,
14 cm and 13 cm and the parallelogram stand on the base 15 cm, find the height of the parallelogram.
(A) 5 cm (B) 5.6 cm (C) 6 cm (D) 6.2 cm

A-10. In triangle if each side of triangle is halfed then what is the % change in its area.
(A) 75% increase (B) 75% decrease (C) 25% increase (D) 25% decrease

EXERCISE – 02

OBJECTIVE QUESTIONS
1 All the 3 sides of a right triangle are integers and one side has a length 11 units. Area of the triangle in
square units lies between
(A) 1 and 100 (B) 100 and 200 (C) 200 and 300 (D) More than 300
2. In the figure given PM = 10 cm, MN = 15 cm and PN = 17 cm. Also QM = QX and XR = RN.
Perimeter of the PQR, is :

(A) 32 (B) 27 (C) 25 (D) 21


3. A triangle EFG is inscribed in a unit square ABCD with E on AB, F on DA, G on CD such that
AE = DF = CG = 1/3. The area of the triangle EFG is :
5 1 5 4
(A) (B) (C) (D)
18 3 9 9

4. A triangle of area 9 y cm2 has been drawn such that its area is equal to the area of an equilateral triangle
of side 6 cm. Then, the value of y is :
(A) 2 cm (B) 3 cm (C) 2 cm (D) 3 cm
5. A plot of land is in the shape of a right angled isosceles triangle. The length of the hypotenuse is 50 2 .
The cost of fencing it at Rs. 3 per metre will be :
(A) less than Rs. 300 (B) less than Rs. 400 (C) more than Rs. 500 (D) more than Rs. 600

For Unacademy Subscription Use “PJLIVE” Code | Join t.me/pjsir42 for Updates
For More Info: “75970 – 84242, 94590 – 43333 / 2222”
6. The perimeter of an isosceles triangle is equal to 14 cm, the lateral side is to the base in the ratio 5 : 4.
The area of the triangle is :
1 3
(A) 21cm 2 (B) 21cm 2 (C) 21cm 2 (D) 2 21cm 2
2 2

7. In a trapezium ABCD with bases AB and CD, where AB = 52, BC = 12, CD = 39 and DA = 5. The area
of the trapezium ABCD, is :
(A) 182 (B) 195 (C) 210 (D) 260

8. ABCD is a rectangle with AB = 12 cm and BC = 7 cm. Point E is on AD with DE = 2 cm. Point P is on


AB. How far to the right of point. A should point P be placed so that the shaded area comprises exactly
40% of the area of the rectangle ?

(A) 8 (B) 8.4 (C) 8.2 (D) 8.6

9. Given an isosceles trapeziumABCD in order with AB = 6, CD = 12 and area 36 sq. units. Length of the
side BC is :
(A) 6 (B) 5 (C) 4.5 (D) 5.5

10. The length of the side of a rhombus is 10 units and its diagonals differ by 4. The area of the rhombus is:
(A) 108 (B) 96 (C) 84 (D) 48

11. The side lengths of trapezium are 4 3 , 4 3 , 4 3 and 2 × 4 3 . Its area in the ratio of two relatively prime
positive integers, m and n. The value of (m + n) is equal to :
(A) 5 (B) 7 (C) 9 (D) 13

12. A rectangle is inscribed in a square creating four isosceles right triangle. If the total area of these four
triangles is 200. The length of the diagonal of the rectangle is :

(A) 10 (B) 15 (C) 20 (D) 25

13. The cost of levelling a rectangular ground at Rs.1.25 per sq. metre is Rs 900. If the length of the ground
is 30 metres, then the width is :
(A) 330 metres (B) 34 metres (C) 24 metres (D) 18 metres

14. A rectangular lawn 60 metres by 40 metres has two roads each 5 metres wide running in the middle of
it, one parallel to length and the other parallel to breadth. The cost of gravelling the roads at 60 paise
per sq. metre is :
(A) Rs. 300 (B) Rs. 280 (C) Rs. 285 (D) Rs. 250

15. A child draws the figure of an aeroplane as given. Here thewings EDCF and AGHB are parallelograms, the
tail ADK is an isosceles triangle, the cockpit BLC is a semi-circle and the portion ABCD is a square. Let FP
⊥ CD & HQ ⊥ AB, AB = 6 cms. KD = 5 cms FP = HQ = 2 cms. The area of the figure is: [ = 3.14]

For Unacademy Subscription Use “PJLIVE” Code | Join t.me/pjsir42 for Updates
For More Info: “75970 – 84242, 94590 – 43333 / 2222”
(A) 86.14 cm2 (B) 87.25 cm2 (C) 84.63 cm2 (D) 91.56 cm2

16. The area of a rhombus is 28 cm2 and one of its diagonals in 4cm. Its perimeter is :
(A) 4 53 (B) 36 cm (C) 2 53 (D) 53 cm

17. The area of a trapezium shaped field is 960 m2 the distance between two parallel sides is 30 m. and one
of the parallel sides is 20 m. Find the length of other parallel side.
(A) 44 m. (B) 22 m. (C) 88 m. (D) 11 m.

EXERCISE – 03

NTSE PROBLEMS (PREVIOUS YEARS)


1. A triangle with integral sides has perimeter 8 units. The area of the triangle is __________ sq units.
[UP NTSE Stage - 1 2012]
(A) 2 (B) 2 2 (C) 3 2 (D) 4

2. Area of triangle ABC whose sides are 24 m. 40 m. and 32 m. is : [Raj. NTSE Stage-1 2013]
(A) 96 m2 (B) 384 m2 (C) 43 m2 (D) 192 m2

3. The perimeters of a reqular hexagon and a square are equal. The ratio of the area of the square to the
area of the hexagon is : [Harayana NTSE Stage-1 2014]
(A) 3 : 2 (B) 2 : 3 3 (C) 1 : 3 (D) 3 : 2 3

4. In the figure given below, ABC is an equilateral triangle. D, E, F, G, H and I are the trisector points of
the sides as shown. If the side of the triangle ABC is 6 cm, then the area of the regular hexagon
DEFGHI is [Raj. NTSE Stage-1 2014]

(A) 3 3 cm2 (B) 4 3 cm2 (C) 5 3 cm2 (D) 6 3 cm2

5. If every side of a triangle is doubled then a new triangle is formed. The ratio of areas of these two
triangles is [Raj. NTSE Stage-1 2016]
(A) 1 : 2 (B) 1 : 3 (C) 1 : 4 (D) 2 : 3

6. Equilateral triangles I, II, III and IV are such that the altitude of triangle I is the side of triangle II, the
altitude of triangle II is the side of triangle III and the altitue of the triangle is the side of triangle IV. If
the area of triangle I is 2 cm2, then the area (in cm2) of triangle IV is : [Haryana NTSE Stage-1 2016]
9 27 45
(A) 3 (B) (C) (D)
2 16 32 64

For Unacademy Subscription Use “PJLIVE” Code | Join t.me/pjsir42 for Updates
For More Info: “75970 – 84242, 94590 – 43333 / 2222”
7. ABC is an equilateral triangle, we have BD = EG = DF = DE = EC, then the ratio of the area of the
shaded portion to area of ABC is [Delhi NTSE Stage-1 2016]

4 7 5 6
(A) (B) (C) (D)
11 9 12 7

8. The height of an equilateral triangle is 6 cm. Its area is : [MP NTSE Stage - 1 2016]
(A) 2 2 cm2 (B) 6 2 cm2 (C) 2 3 cm2 (D) 3 3 cm2

9. Heron's formula for the Area of triangle is : [Madhya Pradesh NTSE Stage-I/18]
(A) ½ (Base × Height) (B) s(s− a)(s− b)(s− c)
a+b+c
(C) (D) s.a.b.c
2

10. A Parallelogram has sides 6cm and 4cm and one of its diagonals is 8cm, then its area is–
[Uttar Pradesh NTSE Stage–I/18]
(A) 36 cm2 (B) 3 15 cm2 (C) 6 15 cm2 (D) 12 210 cm2

11. If the perimeter of an equilateral triangle is 24 cm, then its area will be [Raj. NTSE Stage-1 2019]
(A) 16 3 sq.cm (B) 32 3 sq.cm (C) 48 3 sq.cm (D) 64 3 sq.cm

VALUE BASED / PRACTICAL BASED QUESTIONS

1. Anita has a piece of land which is in the shape of a rhombus. She has two children, one daughter Preeti
and one son Narendra to work on the land and produce different types of crops. She divided the land in
two equal parts. If the perimeter of the land is 200 m and one of the diagonals is 70 m, then solve the
following questions.
(i) How much area each of them will get for their crops ?
(ii) Anita covered the land by wires. How much wire is required to cover the land ?
(iii) As Anita divided the land into two equal parts. Justify her decision ?

2. Students of a school staged a rally for plantation campaign. They walked through the lanes in two
groups. First group walked through the lanes AB, BC and CA, while the second group through AC, CD
and DA. They planted sapling in the area enclosed within their lanes. Suppose the length of field are
AB = 80 m, BC = 60m, CD = 65m and DA = 55 m and B = 90°
(i) Which group planted saplings in larger area ?
(ii) Which mathematical concept is used to solve the above problem ?
(iii) What message is spread by the students through the campaign ?

For Unacademy Subscription Use “PJLIVE” Code | Join t.me/pjsir42 for Updates
For More Info: “75970 – 84242, 94590 – 43333 / 2222”
Answer Key

BOARD LEVEL EXERCISE

TYPE (I) : VERY SHORT ANSWER TYPE QUESTIONS : [01 MARK EACH]
1. 24 2. − 32 3. 100 3 m2 4. 1344 cm2
5. 5.196 cm2 6. 6 cm

TYPE (II) : SHORT ANSWER TYPE QUESTIONS : [02 MARKS EACH]

7. − 15 cm2 8. 24 5 9. 2000 10. 32 2 cm2


11. 600 15 m2.

TYPE (III) : LONG ANSWER TYPE QUESTIONS: [03 MARK EACH]

12. 1074 m2 13. 2100 15 m2 14. Rs. 960 15. 114 m2


16. 20 30m 2
17. 23 cm and 27 cm 18. 3 cm 19. QC = 45 cm, PD = 40 cm
20. Total area of design = 1632 cm2
Remaining area of tile = 1869 cm2

TYPE (IV): VERY LONG ANSWER TYPE QUESTIONS [04 MARK EACH]

21. 300 3 cm2 22. 4 : 1 23. Rs. 19200

EXERCISE – 01

SUBJECTIVE QUESTIONS
Section (A) : Heron's formula
A-1. 24 cm A-2. 24 cm. A-3. 2772 cm2, 36 cm A-4. Rs. 1219.68
A-5. 3 3 cm2 A-6. 306 m2. A-7. 8 cm ,15 cm, 60 cm2.
A-8. 46 cm, 114 cm2. A-9. 32 2 cm2 A-10. 2.26 hectares, 1.92 hectares and 1.92 hectares.
A-11. Rs. 4940. A-12. 1644 m2.

OBJECTIVE QUESTIONS
Section (A) : Heron's formula
A-1. (B) A-2. (B) A-3. (A) A-4. (D) A-5. (C)
A-6. (C) A-7. (D) A-8. (C) A-9. (B) A-10. (B)

EXERCISE – 02

OBJECTIVE QUESTIONS
Ques. 1 2 3 4 5 6 7 8 9 10 11 12 13 14 15 16 17
Ans. D B A B C D C B B B D C C C A A A

For Unacademy Subscription Use “PJLIVE” Code | Join t.me/pjsir42 for Updates
For More Info: “75970 – 84242, 94590 – 43333 / 2222”
EXERCISE – 03

Ques. 1 2 3 4 5 6 7 8 9 10 11
Ans. B B D D C C B C B C A

VALUE BASED / PRACTICAL BASED QUESTIONS

1. (i) 175 51 m2 2. (i) AB, BC, CA

For Unacademy Subscription Use “PJLIVE” Code | Join t.me/pjsir42 for Updates
For More Info: “75970 – 84242, 94590 – 43333 / 2222”
8 SIMILAR TRIANGLES

INTRODUCTION

The study of Geometry is concerned with knowing properties of various shapes and structures. Arithmetic and
Geometry were considered to be the two oldest branches of mathematics. Greeks held Geometry in high esteem
and used its properties to discuss various scientific principles which otherwise would have been impossible.
Eratosthenes used the similarity of circle to determine the circumference of the Earth, distances of the moon and
the sun from the Earth, to a remarkable accuracy. Apart from these achievements, similarity is used to find
width of rivers, height of trees and much more.
In this chapter, we will be discussing the concepts mainly as continuation of previous classes and discuss most
important concepts like Similar Triangles, Basic Proportionality Theorem, Angle Bisector Theorem, the most
prominent and widely acclaimed Pythagoras Theorem and much more.
Geometry plays vital role in the field of Science, Engineering and Architecture. We see many Geometrical
patterns in nature. We are familiar with triangles and many of their properties from earlier classes.

A. SIMILAR TRIANGLES

(a) Congruent figures


Two geometric figures which have the same shape and size are known as congruent figures.
Congruent figures are alike in every respect.

(b) Similar figures


Geometric figures which have the same shape but different sizes are known as similar figures.
Two congruent figures are always sim ilar but two similar figures need not be congruent.
Examples
(i) Any two line segments are similar.
(ii) Any two equiangular triangles are similar.
(iii) Any two squares are similar.
(iv) Any two circles are similar.

(c) Similar polygons


Two polygons are said to be similar if
(i) their corresponding angles are equal and
(ii) the lengths of their corresponding sides are proportional.
If two polygons ABCDE and PQRST are similar we write, ABCDE ~ PQRST, where the symbol ‘~’
stands for ‘is similar to’.

(d) Equiangular triangles


Two triangles are said to be equiangular if their corresponding angles are equal.

(e) Similar Triangles


Two triangles ABC and DEF are said to be similar if their
(i) Corresponding angles are equal.
i.e. A = D, B = E, C = F
For Unacademy Subscription Use “PJLIVE” Code | Join t.me/pjsir42 for Updates
For More Info: “75970 – 84242, 94590 – 43333 / 2222”
And

(ii) Corresponding sides are proportional.


AB BC AC
i.e. = = .
DE EF DF

(f) Congruency and similarity of triangles


Congruency is a particular case of similarity. In both the cases, three angles of one triangle are equal to
the three corresponding angles of the other triangle. But in congruent triangles, the corresponding sides
are equal. While in similar triangles, the corresponding sides are proportional.
(i) Congruent triangles
ABC  PQR
A = P, B = Q, C = R.
AB = PQ, BC = QR, CA = RP
AB BC CA
= = =1
PQ QR RP
Same shape and same size.

(ii) Similar triangles


ABC ~ PQR
A = P, B = Q, C = R
AB  PQ, BC  QR, CA  RP
AB BC CA
but = = > 1 or < 1
PQ QR RP
Same shape but not same size.

(g) Types of triangles


(i) Scalene triangle
(i) A triangle in which none of the two sides are equal is called a
scalene triangle
(ii) All the three angles are also different

abc
(ii) Isosceles triangle
(i) A triangle in which at least two sides are equal is called an
isosceles triangle.
(ii) In this triangle, the angles opposite to the congruent sides are also
equal
(iii) 2 medians, 2 altitudes equal.
(iv) Internal bisectors of 2 angles are equal.
(v) Bisector of vertical angle bisects the base and perpendicular to AB = AC
the base. B = C
(vi) May be acute, obtuse or right angled triangle.

(iii) Equilateral triangle


(i) A triangle in which all the three sides are equal is called an
equilateral triangle.
(ii) In this triangle each angle is congruent and equal to 60º
(iii) Always acute angled.
(iv) Incentre, circumcentre, orthocentre and centroid coincide.

AB = BC = AC
A = B = C = 60º

For Unacademy Subscription Use “PJLIVE” Code | Join t.me/pjsir42 for Updates
For More Info: “75970 – 84242, 94590 – 43333 / 2222”
(iv) Isosceles right angled triangle
(i) 2 sides are equal
(ii) Angle included by the equal sides is 90º.
(iii) Side opposite to 90º is hypotenuse and is the greatest side.
(iv) Median to the hypotenuse is half of the hypotenuse.

(h) Fundamental properties of triangles


Sum of any two sides is always greater than the third side.
The difference of any two sides is always less than the third side.
Greater angle has a greater side opposite to it and smaller angle has a smaller
side opposite to it i.e., if two sides of triangle are not congruent then the
angle opposite to the greater side is greater.
Let a, b and c be the three sides of a ABC and c is the largest side, then
If c2 < a2 + b2, the triangle is acute angle triangle
If c2 = a2 + b2, the triangle is right angled triangle
If c2 > a2 + b2, the triangle is obtuse angle triangle
The sum of all the three interior angles is always 180º i.e.,
CAB + ABC + BCA = 180º
The sum of three (ordered) exterior angles of a triangle is 360º.
In fig (i) : FAC + ECB + DBA = 360º
In fig (ii) : FAB + DBC + ECA = 360º
A triangle must have at least two acute angles.
In a triangle, the measure of an exterior angle equals the
sum of the measures of the interior opposite angles.
The measure of an exterior angle of a triangle is
greater than the measure of each of the opposite
interior angles. Fig.(i) Fig.(ii)

(i) Congruence of triangles


(i) S – S – S (Side-Side-Side)
If the three sides of one triangle are equal to the
corresponding three sides of the other triangle, then the two
triangles are congruent.
AB  PQ, AC  PR, BC  QR
 ABC  PQR

(ii) S–A–S (Side-Angle-Side)


If two sides and the included angle between them of one
triangle be congruent to the corresponding sides and the angle
included between them, of the other triangle then the two
triangles are congruent.
AB  PQ, ABC  PQR, BC  QR
 MBC  PQR

(iii) A–S–A (Angle-Side-Angle)


If two angles and the included side of a triangle are congruent
to the corresponding two angles and the included side of the
other triangle, then the two triangles are congruent.
ABC  PQR, BC  QR, ACB  PRQ
 ABC  PQR

(iv) A–A–S (Angle-Angle-Side)


If two angles and a side other than the included side of a one triangle are congruent to the
corresponding angles and a corresponding side other than the included side of the other triangle in
order, then the two triangles are congruent.
ABC  PQR, ACB  PRQ, AC  PR

For Unacademy Subscription Use “PJLIVE” Code | Join t.me/pjsir42 for Updates
For More Info: “75970 – 84242, 94590 – 43333 / 2222”
 ABC  PQR

(v) R–H–S (Right angle-Hypotenuse-Side)


If the hypotenuse and one side of the right angled triangle are
congruent to the hypotenuse and a corresponding side of the other
right angled triangle, then the two given triangles are congruent.
AC  PR, B = Q and BC  QR
 ABC  PQR

(j) Important Definitions


(i) Altitude (or height)
The perpendicular drawn from the opposite vertex of a side in a triangle called an altitude of the
triangle. There are three altitudes in a triangle.

AE, CD and BF are the altitudes


(ii) Median
The line segment joining the mid-point of a side to the A
vertex opposite to the side is called a median. There are
three medians in a triangle.
A median bisects the area of the triangle i.e,
1
Ar(ABE) = Ar(AEC) = Ar(ABC)
2
Point of intersection is called Centroid.
AE, CD and BF are the medians
Centroid divides median in the ratio 2 : 1.
(BE = CE, AD = BD, AF = CF)

(iii) Angle bisector


A line segment which originates from a vertex and bisects the same
angle is called an angle bisector.
1
(BAE = CAE = BAC) etc.
2
Point of Intersection of angle bisectors is called Incentre.
AE, CD and BF are the
angles bisectors
(iv) Perpendicular bisector
A line which bisects a side perpendicularly A (i.e. at right angle) is
called a perpendicular bisector of a side of triangle.
All points on the perpendicular bisector of a line segment are
equidistant from the ends of the line segment.
Point of Intersection of perpendicular bisectors is called
Circumcentre.
DO, EO and FO are the
perpendicular bisectors

For Unacademy Subscription Use “PJLIVE” Code | Join t.me/pjsir42 for Updates
For More Info: “75970 – 84242, 94590 – 43333 / 2222”
(v) Orthocentre
The point of Intersection of the three altitudes of the triangle is
called as the orthocentre.
BOC = 180º – A
COA = 180º – B
AOB = 180º – C
'O' is the orthocenter
(k) Basic Proportionality Theorem (BPT) or Thales Theorem
Theorem : If a line is drawn parallel to one side of a triangle to intersect the other two sides in distinct
points, then the other two sides are divided in the same ratio.
Given : A ABC in which a line parallel to side BC intersects other two sides AB and AC at D and E
respectively.
AD AE
To Prove : =
DB EC
Construction : Join BE and CD and draw DM ⊥ AC and EN ⊥ AB.
1 1
Proof : Area of  ADE = (base × height) = AD × EN.
2 2
Area of  ADE is denoted as ar(ADE).
1 1
So, ar(ADE) = AD × EN and ar(BDE) = DB × EN.
2 2
1
ar ( ADE ) 2 AD  EN AD
Therefore, = = ... (i)
ar ( BDE ) 1 DB  EN DB
2
1 1
Similarly, ar(ADE) = AE × DM and ar(DEC) = EC × DM.
2 2
1
ar ( ADE ) 2 AE  DM AE
And = = ... (ii)
ar ( DEC ) 1 EC  DM EC
2
Note that  BDE and  DEC are on the same base DE and between the two parallel lines BC and DE.
So, ar(BDE) = ar(DEC) ... (iii)
Therefore, from (i), (ii) and (iii), we have :
AD AE
= Hence Proved.
DB EC
Corollary :
If in a ABC, a line DE || BC, intersects AB in D and AC in E, then

DB EC AB AC AD AE AB AC
(i) = (ii) = (iii) = (iv) =
AD AE AD AE AB AC DB EC
DB EC
(v) =
AB AC

(l) Converse of Basic Proportionality Theorem (Thales Theorem)


Theorem : If a line divides any two sides of a triangle in the same ratio, then the line must be parallel
to the third side.
AD AE
Given : A ABC and a line  intersecting AB at D and AC at E such that = .
DB EC
To prove : DE || BC.
Proof : If possible let DE not be parallel to BC. Then there must be another line through D, which is
parallel to BC. Let DF || BC
Then, by Thales. theorem, we have
AD AF
= ....(i)
DB FC
For Unacademy Subscription Use “PJLIVE” Code | Join t.me/pjsir42 for Updates
For More Info: “75970 – 84242, 94590 – 43333 / 2222”
AD AE
But, = (given) ....(ii)
DB EC
From (i) and (ii) we get
AF AE
=
FC EC
AF AE
+1 = +1
FC EC
AF + FC AE + EC
=
FC EC
AC AC
=
FC EC
FC = EC.
This is possible only when E and F coincide. Hence, DE || BC.
Some important results and theorems
(i) The internal bisector of an angle of a triangle divides the opposite side internally in the ratio of the
sides containing the angle. (called as Angle Bisector Theorem)
(ii) In a triangle ABC, if D is a point on BC such that D divides BC in the ratio AB : AC, then AD is
the bisector of A.
(iii) The external bisector of an angle of a triangle divides the opposite sides externally in the ratio of
the sides containing the angle.
(iv) The line drawn from the mid-point of one side of a triangle parallel to another side bisects the third side.
(v) The line joining the mid-points of two sides of a triangle is parallel to the third side.
(vi) The diagonals of a trapezium divide each other proportionally.
(vii) If the diagonals of a quadrilateral divide each other proportionally, then it is a trapezium.
(viii) Any line parallel to the parallel sides of a trapezium divides the non-parallel sides proportionally.
(ix) If three or more parallel lines are intersected by two transversals, then the intercepts made by them
on the transversals are proportional.

Solved Examples

Example.1 Find each value.


LK KN
(i) (ii)
KJ LM
Solution Use the triangle proportionality theorem.
LK MN 6 2 KN JN 9 3
(i) = = = (ii) = = =
KJ NJ 9 3 LM JM 15 5
Example.2 In a ABC, D and E are points on the sides AB and AC respectively such that DE || BC. If AD
= 4x – 3, AE = 8x – 7, BD = 3x – 1 and CE = 5x – 3, find the value of x.
Solution. In ABC, we have DE || BC
AD AE
 = [By Basic Proportionality Theorem]
DB EC
4x − 3 8x − 7
 =
3x − 1 5x − 3
 20x2 – 15x – 12x + 9 = 24x2 – 21x – 8x + 7
 20x2 – 27x + 9 = 24x2 – 29x + 7
 4x2 – 2x – 2 = 0  2x2 – x – 1 = 0
1
 (2x + 1) (x – 1) = 0  x = 1 or x = –
2
1
So, the required value of x is 1. [x = – is neglected as length cannot be negative].
2
Example.3 D and E are respectively the points on the sides AB and AC of a ABC such that AB = 12 cm,
AD = 8 cm, AE = 12 cm and AC = 18 cm, show that DE || BC.

For Unacademy Subscription Use “PJLIVE” Code | Join t.me/pjsir42 for Updates
For More Info: “75970 – 84242, 94590 – 43333 / 2222”
Solution. We have,
AB = 12 cm, AC = 18 cm, AD = 8 cm and AE = 12 cm.
 BD = AB – AD = (12 – 8) cm = 4 cm
CE = AC – AE = (18 – 12) cm = 6 cm
AD 8 2
Now, = =
BD 4 1
AE 12 2 AD AE
And, = =  =
CE 6 1 BD CE
Thus, DE divides sides AB and AC of ABC in the same ratio. Therefore, by the converse of
basic proportionality theorem, we have DE || BC.
Area( ABD) AB
Example.4 In ABC, if AD is the bisector of A, prove that = .
Area(CD) AC
Solution. In ABC, AD is the bisector of A.
AB BD
 = ...(i) [By internal bisector theorem]
AC DC
From A draw AL ⊥ BC
1
BD.AL
Area ( ABD) 2 BD AB
 = = = [From (i)]
Area ( ACD) 1
DC.AL DC AC
2
Hence Proved.
Example.5 In the given figure, AB || CD. Find the value of x.
Solution. Since the diagonals of a trapezium divide each other proportionally.
AO BO
 =
OC OD
3x − 19 x − 4
 =
x −3 4
 12x – 76 = x2 – 4x – 3x + 12
 x2 – 19x + 88 = 0
 x2 – 11x – 8x + 88 = 0
 (x – 8) (x – 11) = 0
 x = 8 or x = 11.

Check Point - A

1. If three or more parallel lines are intersected by two transversals, prove that the intercepts made by
them on the transversal are proportional.
2. using converse of BPT prove that the line joining the midpoints of any two sides of a triangle is parallel
to the third side .
3. If D, E are points on the sides AB and AC of ABC such that AD = 6 cm, BD = 9 cm, AE = 8 cm,
EC = 12 cm. Prove that DE||BC.
AM AN
4. In the given figure, if LM||CB and LN||CD, prove that =
AB AD

5. ABCD is a trapezium in which AB is parallel to DC. If the diagonals intersect at O prove that
AO.DO = BO.CO.

For Unacademy Subscription Use “PJLIVE” Code | Join t.me/pjsir42 for Updates
For More Info: “75970 – 84242, 94590 – 43333 / 2222”
6. Find the value of each ratio or length.
GH
(a) (b) EF (c) DG
HD

Answer
1
6. (a) (b) 6 (c) 6
2

B. CRITERIA FOR SIMILARITY OF TWO TRIANGLES

(a) AAA Similarity Criteria


If two triangles are equiangular, then they are similar.
Given : A = D, B = E, C = F
To prove : ABC ~ DEF
Proof :
Case -1 : If AB = DE
In ABC and DEF
A = D
AB = DE
B = E
 ABC  DEF [By ASA congruence rule]
By CPCT,
BC = EF, AC = DF
AB BC AC
 = = =1
DE EF DF
Hence, ABC ~ DEF
Case -2 : If AB > DE
Construction : Mark points P and Q on AB and AC
respectively such that AP = DE and AQ = DF
Proof : APQ and DEF
AP = DE
A = D
AQ = DF
APQ  DEF [By SAS congruence rule]
By CPCT, P = E and Q = F
But B = E and C = F
 P = B and Q = C
 PQ || BC [ corresponding angles are equal]
AP AQ
By basic proportionality theorem in ABC, =
AB AC
DE DF
= ...(i) [By construction]
AB AC
DE EF
Similarly, we can prove that = ...(ii)
AB BC
DE DF EF
From (i) & (ii) = =
AB AC BC
 ABC ~ DEF

Case -3 : If AB < DE
Construction : Mark points P and Q on DE and DF respectively
such that DP = AB and DQ = AC
Proof : DPQ and ABC
DP = AB
D = A
For Unacademy Subscription Use “PJLIVE” Code | Join t.me/pjsir42 for Updates
For More Info: “75970 – 84242, 94590 – 43333 / 2222”
DQ = AC
DPQ  ABC [By SAS congruence rule]
By CPCT, P = B and Q = C
But E = B and F = C
 P = E and Q = F  PQ || EF [ corresponding angles are equal]
DP DQ
By basic proportionality theorem in DEF, =
DE DF
AB AC
= ...(i) [By construction]
DE DF
AB BC
Similarly, we can prove that = ...(ii)
DE EF
AB AC BC
From (i) & (ii) = =
DE DF EF
 ABC ~ DEF

(b) SSS Similarity Criteria


If the corresponding sides of two triangles are proportional, then they are similar.
AB BC AC
Given : = =
DE EF DF
To prove : ABC ~ DEF
Construction : Let AB > DE, now mark two points P and Q on
AB and AC respectively such that AP = DE and AQ = DF.
AB AC AB AC
Proof : Given =  =
DE DF AP AQ
 By converse of BPT in ABC, PQ II BC.
 P = B and Q = C [Corresponding angles]
APQ ~ ABC [By AA similarity]
AB BC AB BC
= and =
AP PQ DE EF
AB BC AB BC
 = and = [since AP = DE]
DE PQ DE EF
BC BC
 =  PQ = EF
PQ EF
Now, in APQ and DEF
AP = DE
PQ = EF
AQ = DF
APQ  DEF [By SSS congruence rule]
But APQ ~ ABC
 DEF ~ ABC.

(c) SAS Similarity Criteria


If in two triangles, one pair of corresponding sides are proportional and the included angles are equal
then the two triangles are similar.
AB AC
Given : = and A = D
DE DF
To prove : ABC ~ DEF
Construction : Let AB > DE, now mark two points and P and Q on AB and AC.
Proof : In APQ and DEF

For Unacademy Subscription Use “PJLIVE” Code | Join t.me/pjsir42 for Updates
For More Info: “75970 – 84242, 94590 – 43333 / 2222”
AP = DE
A = D
AQ = DF
 APQ  DEF [By SAS congruence rule]
AB AC
Given, =
DE DF
AB AC
 = [By construction]
AP AQ
 PQ || BC [By converse of Basic Proportionality Theorem]
 P = B, Q = C
 APQ ~ ABC [By AA similarity]
Hence, DEF ~ ABC

(d) Results based upon characteristic properties of Similar Triangles


(i) The ratio of the perimeters of two similar triangles is equal to the ratio of their corresponding sides.
(ii) If two triangles are equiangular, then the ratio of the corresponding sides is the same as the ratio of
the corresponding medians.
(iii) If two triangles are equiangular, then the ratio of the corresponding sides is same as the ratio of the
corresponding angle bisector segments.
(iv) If two triangles are equiangular then the ratio of the corresponding sides is same as the ratio of the
corresponding altitudes.
(v) If one angle of a triangle is equal to one angle of another triangle and the bisectors of these equal
angles divide the opposite side in the same ratio, then the triangles are similar.
(vi) If two sides and a median bisecting the third side of a triangle are respectively proportional to the
corresponding sides and the median of another triangle, then two triangles are similar.
(vii) If two sides and a median bisecting one of these sides of a triangle are respectively proportional to
the two sides and the corresponding median of another triangle, then the triangles are similar.

Solved Examples

Example.6 Is it possible to prove that the triangles in each pair are similar? Explain why or why not.

Solution. (i) Yes. AED  CEB because they are vertical angles. A  C because they are alternate
interior angles formed by two parallel lines and a transversal. So AED ~ ACEB by the
AA Similarity Postulate.
(ii) No. The congruent angles are not included between the given sides.

Example.7 Tell whether the triangles in each pair are similar. Explain your reasoning.

For Unacademy Subscription Use “PJLIVE” Code | Join t.me/pjsir42 for Updates
For More Info: “75970 – 84242, 94590 – 43333 / 2222”
Solution. (i) You know that LC ~ LF. Check whether the sides that include the angles are in proportion.
DF EF
=
AC BC
8.5 5.5
=
4 3
Check whether the
fractions are equal.
2.125  1.83
The sides are not in proportion, so the triangles are not similar.
(ii) Are the corresponding sides in proportion?
5.75 7.5 9.5
= =
11.5 15 19
1 1 1
= =
2 2 2
By the SSS Similarity Theorem, the triangles are similar.

Example.8 In a trapezium ABCD, AB || DC and DC = 2AB. EF drawn parallel to AB cuts AD in F and BC


BE 3
in E such that = . Diagonal DB intersects EF at G. Prove that 7FE = 10AB.
EC 4
Solution. In DFG and DAB,
1 = 2 [Corresponding  s AB || FG]
 FDG =  ADB [Common]
 DFG ~ DAB [By AA rule of similarity]
DF FG
 =  (i)
DA AB
Again in trapezium ABCD
EF || AB || DC
AF BE
 =
DF EC
AF 3  BE 3 
 =  EC = 4 (given) 
DF 4  
AF 3
 +1 = +1
DF 4
AF + DF 7
 =
DF 4
AD 7
 =
DF 4
DF 4
 = … (ii)
AD 7
From (i) and (ii), we get
FG 4 4
= i.e., FG = AB … (iii)
AB 7 7
In BEG and BCD, we have
BEG = BCD [Corresponding angle  EG || CD]

For Unacademy Subscription Use “PJLIVE” Code | Join t.me/pjsir42 for Updates
For More Info: “75970 – 84242, 94590 – 43333 / 2222”
GBE = DBC [Common]
 BEG ~ BCD [By AA rule of similarity]
BE EG
 =
BC CD
3 EG  BE 3 EC 4 EC + BE 4 + 3 BC 7 
 =  = i.e., =  =  =
7 CD  EG 7 BE 3 BE 3 BE 3 
3 3
 EG = CD = (2 AB) [CD = 2AB (given)]
7 7
6
 EG = AB ... (iv)
7
Adding (iii) and (iv), we get
4 6 10
FG + EG = AB + AB = AB
7 7 7
10
 EF = AB i.e., 7EF = 10AB. Hence proved.
7

Example.9 BAC = 90º, AD is its bisector. If DE ⊥ AC, prove that DE × (AB + AC) = AB × AC.
Solution. It is given that AD is the bisector of A of  ABC.
AB BD
 =
AC DC

AB BD
 +1 = +1 [Adding 1 on both sides]
AC DC
AB + AC BD + DC
 =
AC DC
AB + AC BD
 = ...(i)
AC DC
In 's CDE and CBA, we have
DCE = BCA [Common]
DEC = BAC [Each equal to 90º]
So, by AA-criterion of similarity
 CDE ~  CBA
CD DE
 =
CB BA
AB BC
 = ....(ii)
DE DC
From (i) and (ii), we have
AB + AC AB
 =
AC DE
 DE × (AB + AC) = AB × AC.

1 1 1
Example.10 In the given figure, PA, QB and RC are each perpendicular to AC. Prove that + = .
x z y
Solution. In PAC, we have BQ || AP
BQ CB
 = [ CBQ ~ CAP]
AP CA
y CB
 = .(i)
x CA
In ACR, we have BQ || CR
BQ AB
 =   ABQ ~ ACR
CR AC

For Unacademy Subscription Use “PJLIVE” Code | Join t.me/pjsir42 for Updates
For More Info: “75970 – 84242, 94590 – 43333 / 2222”
y AB
 = …(ii)
z AC
Adding (i) and (ii), we get
y y CB AB y y AB + BC
+ = +  + =
x z AC AC x z AC
y y AC y y
 + =  + =1
x z AC x z
y y 1
 + = . Hence Proved.
x z y

Check Point - B

1. Explain why the triangles in each pair are similar.

2. Is it possible to prove that the triangles in each pair are similar? Explain why or why not.

3. In the adjoining figure, check whether similar. If yes identify the similarity criterion two triangles FGH
and QPR are

4. ABC ~ PQR. If AB = 6 cm, BC = 4 cm, AC = 8 cm, PR = 6 cm then find PQ + QR =


5. In the figure, if DE|| BC, then find x.

6. ABC is a triangle and DE is drawn parallel to BC such that AD : DB = 2 : 3. If DE = 5 cm, find the
length of BC.
7. D is a point on the side QR of triangle PQR such that angles PDR and QPR are equal. Prove that
QR.DR = PR2.

Answers
3. Yes, SAS similarity 4. 7.5 5. 10 6. 12.5 cm

C. AREAS OF SIMILAR TRIANGLES

For Unacademy Subscription Use “PJLIVE” Code | Join t.me/pjsir42 for Updates
For More Info: “75970 – 84242, 94590 – 43333 / 2222”
Theorem : The ratio of the areas of two similar triangles is equal to the square of the ratio of their
corresponding sides.
Given : Two triangles ABC and PQR such that
 ABC ~  PQR [Shown in the figure]
2 2
ar(ABC)  AB   BC   CA 
2

To prove : =  =  =  .
ar(PQR)  PQ   QR   RP 
Construction : Draw altitudes AM and PN of the triangle ABC and PQR.
1 1
Proof : ar(ABC) = BC × AM and ar(PQR) = QR × PN
2 2
1
BC  AM
ar(ABC) 2 BC  AM
So, = = ... (i)
ar(PQR) 1 QR  PN QR  PN
2
Now, in ABM and PQN,
B = Q [As ABC  PQR]
M = N [ 90º each]
So, ABM  PQN [AA similarity criterion]
AM AB
Therefore, = ... (ii)
PN PQ
Also, ABC ~ PQR [Given]
AB BC CA
So, = = ... (iii)
PQ QR RP
ar(ABC) BC AB
Therefore, =  [From (i) and (ii)]
ar(PQR) QR PQ
AB AB
=  [From (iii)]
PQ PQ
2
 AB 
= 
 PQ 
Now using (iii), we get
2 2
ar( ABC)  AB   BC   CA 
2

=  =  = 
ar( PQR)  PQ   QR   RP 
Corollary : The areas of two similar triangles are proportional to the squares of their corresponding
altitude.
Given : ABC ~ DEF, AL ⊥ BC and DM ⊥ EF.
Ar e a of ABC AL2
To prove: =
Area of DEF DM 2
1
 BC  AL
Ar ea of ABC 2 1
Proof: = (Area of  = × Base × Height)
Area of DEF 1  EF  DM 2
2
Area of ABC BC AL
 =  ...(1)
Area of DEF EF DM
In ALB and DME, we have
(i) ALB = DME (Each equal to 90º)
(ii) ABL = DEM (ABC ~ DEF  B = E)
 ALB ~ DME (by AA-axiom)
AL AB
 = ... (2)
DM DE
(Corresponding sides of similar s are proportional.)
ABC ~ DEF (Given)
For Unacademy Subscription Use “PJLIVE” Code | Join t.me/pjsir42 for Updates
For More Info: “75970 – 84242, 94590 – 43333 / 2222”
AB BC AC
 = = ... (3)
DE EF DF
(Corresponding sides of similar s are proportional.)
From (2) and (3)
BC AL
=
EF DM
BC AL
Substituting = in (1),
EF DM
Area of ABC AL2
we get : = .
Area of DEF DM 2
Hence proved.
Corollary : The areas of two similar triangles are proportional to the squares of their corresponding
medians.
Given : ABC ~ DEF and AP, DQ are their medians.
Area of ABC AP 2
To prove: =
Area of DEF DQ2

Proof : ABC ~ DEF (Given)


Area of ABC AB2
 = ... (1)
Area of DEF DE 2
(Areas of two similar s are proportional to the squares of their corresponding sides.)
ABC~DEF
AB BC 2BP BP
 = = = ... (2)
DE EF 2EQ EQ
(Corresponding sides of similar s are proportional.)
AB BP
 = and B = E
DE EQ
(From (2) and the fact the ABC~ DEF)
 APB ~DQE (By SAS-similarity axiom)
BP AP
 = (3)
EQ DQ
From (2) and (3)
AB AP AB2 AP 2
 =  = ...(4)
DE DQ DE 2 DQ2
From (1) and (4)
Area of ABC AP 2
=
Area of DEF DQ2
Hence proved.
Corollary : The areas of two similar triangles are proportional to the squares of their corresponding
angle bisector segments.

Given : ABC ~ DEF and AX, DY are their bisectors of A and D respectively.
Area of ABC AX 2
To prove : =
Area of DEF DY 2
Area of ABC AB2
Proof : = ... (1)
Area of DEF DE 2
(Areas of two similar s are proportional to the squares of the
corresponding sides.)
ABC ~ DEF (Given)
1 1
 A = D  A = D
2 2

For Unacademy Subscription Use “PJLIVE” Code | Join t.me/pjsir42 for Updates
For More Info: “75970 – 84242, 94590 – 43333 / 2222”
 BAX = EDY …(2)
1 1
(BAX = A and EDY = D)
2 2
In ABX and DEY, we have (Given)
BAX = EDY (From (2))
B= E (ABC ~ DEF)
 ABX ~ DEY (By AA similarity axiom)
AB AX AB2 AX 2
 =  = ... (3)
DE DY DE 2 DY 2
From (1) and (3)
Area of ABC AX 2
=
Area of DEF DY 2
Hence Proved.

Solved Examples

Example.11 It is given that ABC ~ PQR, area (ABC) = 36 cm2 and area (PQR) = 25 cm2. If QR = 6
cm, find the length of BC.
Solution. We know that the areas of similar triangles are proportional
to the squares of their corresponding sides.
Area of ( ABC) BC2
 =
Area of ( PQR) QR 2
Let BC = x cm. Then,
36 x 2 36  36  6  6  36
= 2  x2 = x = = = 7.2.
25 6 25  5  5
Hence BC = 7.2 cm

Example.12 P and Q are points on the sides AB and AC respectively of ABC such that PQ || BC and
divides ABC into two parts, equal in area. Find PB : AB.
Solution. Area (APQ) = Area (trap. PBCQ) [Given)
 Area (APQ) = [Area (ABC) – Area (APQ)]
 2 Area (APQ) = Area (ABC)
Area of ( A PQ) 1
 = ...(i)
Area of (ABC) 2
Now, in APQ and ABC, we have
PAQ = BAC [Common A)
APQ = ABC [PQ || BC, corresponding s are equal]
 APQ ~ ABC. [By AA]
We known that the areas of similar s are proportional to the squares of their corresponding sides.
Area of ( A PQ) AP 2 AP 2 1
 =  = [Using (i)]
Area of (ABC) AB2 AB2 2
AP 1
 = i.e.,AB = 2.AP
AB 2
 AB = 2(AB− PB)
 2PB = ( 2 − 1) AB
PB ( 2 − 1)
 = .
AB 2
 PB   = ( 2 − ) 2

For Unacademy Subscription Use “PJLIVE” Code | Join t.me/pjsir42 for Updates
For More Info: “75970 – 84242, 94590 – 43333 / 2222”
Example.13 Prove that the area of the equilateral triangle described on the side of a square is half the area of
the equilateral triangle described on its diagonal.
Solution. Given : A square ABCD. Equilateral triangles BCE and ACF have been described on side
BC and diagonal AC respectively.
1
To Prove : Area (BCE) = .Area (ACF)
2
Proof : Since BCE and ACF are equilateral. Therefore, they are equiangular (each angle
being equal to 60º) and hence BCE ~ ACF.
 ABCDisa square 
Diagonal = 2(side) 
 AC 2BC 
 
Area ( BCE ) BC 2
 =
Area ( ACF ) AC 2
Area ( BCE ) BC2 1 Area ( BCE ) 1
 = =  = . Hence Proved.
Area ( ACF)
( ) Area ( ACF ) 2
2
2BC 2

Example.14 Area of triangle RST is


(A) 6 m2 (B) 72 m2
(C) 54 m2 (D) None of these
Solution. In RQP and RST
Q = S = 90º
PRQ = SRT (V.O.A.)
By AA
RQP ~ RST
RQ QP
=
RS ST
3 4
=
RS 12
 RS = 9
1
Area of RST = × 9 × 12 = 54 m2
2

Check Point - C

1. If ABC ~ DEF such that area of ABC is 9 cm2 and the area of DEF is 16 cm2 and BC = 2.1 cm
find the length of EF is
2. In the diagram, LM is parallel to BC and AL = 1 cm, LB = 3 cm, MC = 4.5 cm and BC = 8 cm. Find
the length of LM. If the area of triangle ALM is 18 sq cm, what is the area of triangle ABC?

3. D and E are points on AB and AC respectively of triangle ABC such that DE is parallel to BC. If
AD = 3 cm, DB = 2 cm, area of ABC is 10 sq cm, find the area of ADE.
4. In the given figure, ABC and DEF are similar BC = 3 cm, EF = 4 cm and area of  ABC = 54 cm2.
Find the area of DEF.

For Unacademy Subscription Use “PJLIVE” Code | Join t.me/pjsir42 for Updates
For More Info: “75970 – 84242, 94590 – 43333 / 2222”
ar( ADE)
5. In the adjacent figure, DE || BC and AD : DB = 5 : 4, then find the value of
ar(CED)

Answers
1. 2.8 cm 2. LM = 2 cm & Area = 288 sq. cm 3. 3.6 sq. cm
4. 96 sq. cm 5. 5 : 4

D. PYTHAGORAS THEOREM

Theorem : In a right triangle, the square of the hypotenuse is equal to the sum of the squares of the
other two sides.
Given : A right triangle ABC, right angled at B.
To prove : AC2 = AB2 + BC2
Construction : BD ⊥ AC
Proof : ADB & ABC
DAB = CAB [Common]
BDA = CBA [90º each]
So, ADB ~ ABC [By AA similarity]
AD AB
= [Sides are proportional]
AB AC
or, AD . AC = AB2 ... (i)
Similarly BDC ~ ABC
CD BC
So, =
BC AC
or CD . AC = BC2 ... (ii)
Adding (i) and (ii),
AD . AC + CD . AC = AB2 + BC2
or, AC (AD + CD) = AB2 + BC2
or, AC. AC = AB2 + BC2
or, AC2 = AB2 + BC2 Hence Proved.

(a) Converse of Pythagoras theorem


Theorem : In a triangle, if the square of one side is equal to the
sum of the squares of the other two sides, then the angle
opposite to the first side is a right angle.
Given : A triangle ABC such that AC2 = AB2 + BC2 .
Construction : Construct a triangle DEF such that DE = AB,
EF = BC and E = 90º.
Proof : In order to prove that B = 90º, it is sufficient to show ABC ~ DEF. For this we proceed as
follows.
Since DEF is a right-angled triangle with right angle at E. Therefore, by Pythagoras theorem, we have
DF2 = DE2 + EF2
 DF2 = AB2 + BC2 [DE = AB and EF = BC (By construction)]

For Unacademy Subscription Use “PJLIVE” Code | Join t.me/pjsir42 for Updates
For More Info: “75970 – 84242, 94590 – 43333 / 2222”
 DF2 = AC2 [ AB2 + BC2 = AC2 (Given)]
 DF = AC ....(i)
Thus, in  ABC and  DEF, we have
AB = DE, BC = EF [By construction]
And AC = DF [From equation (i)]
  ABC   DEF [By SSS criteria of congruency]
 B = E = 90º. Hence,  ABC is a right triangle, right angled at B.

(b) Internal angle bisector theorem


The internal bisector of an angle of a triangle divides the opposite side in the ratio of the sides
containing the angle.
Given : ABC in which AD is the internal bisector of A.
BD AB
To Prove : =
DC AC
Construction : Draw CE || DA, meeting BA produced at E.
Proof:
1 = 2 ... (1) [ AD is the bisector of A]
2 = 3 ... (2) [Alt. Ls are equal, as CE || DA and AC is the
transversal)
1 = 4 ... (3) [Corres. s are equal, as CE||DA and BE is the
transversal)
3 = 4 ... (4) [From (1), (2) and (3))
AE = AC ... (5) [Sides opposite to equal angles are equal)
In BCE, DA || CE
BD BA
 = [By Basic Proportionality Theorem]
DC AE
BD AB
 = [Using 5]
DC AC
Hence proved.

Converse of Angle Bisector Theorem


Theorem : If a straight line through one vertex of a triangle divides the opposite side internally in the
ratio of the other two sides, then the line bisects the angle internally at the vertex.
Proof :
Given : ABC is a triangle. AD divides BC in the ratio of the sides containing the angles A to meet
BC at D.
AB BD
That is = ... (1)
AC DC
To prove : AD bisects A i.e. 1 = 2

Construction : Draw CE || DA. Extend BA to meet at E.

Let EAD= 1 and DAC = 2 Assumption


BAD = AEC = l Since DA||CE and AC is transversal, corresponding
angles are equal
DAC = ACE= 2 Since DA||CE and A C is transversal, Alternate angles
are equal
BA BD
= …(2)
AE DC
In BCE by Thales theorem
AB BD
= From (1)
AC DC
AB BA
= From (1) and (2)
AC AE
For Unacademy Subscription Use “PJLIVE” Code | Join t.me/pjsir42 for Updates
For More Info: “75970 – 84242, 94590 – 43333 / 2222”
AC = AE …(3) Cancelling AB
1 = 2 ACE is isosceles by (3)
AD bisects A Since, l =BAD= 2 = DAC. Hence proved
(b) Some results deduced from Pythagoras theorem
(i) In the given figure ABC is an obtuse triangle, obtuse angled at B. If AD ⊥ CB,
then AC2 = AB2 + BC2 + 2BC . BD

(ii) In the given figure, if  B of ABC is an acute angle and AD ⊥ BC,


then AC2 = AB2 + BC2 – 2BC . BD

(iii) In any triangle, the sum of the squares of any two sides is equal to twice the square of half of the
third side together with twice the square of the median which bisects the third side (called as
Apollonius Theorem)
(iv) Three times the sum of the squares of the sides of a triangle is equal to four times the sum of the
squares of the medians of the triangle.

Solved Examples

3
Example.15 If ABC is an equilateral triangle of side a, prove that its altitude is a.
2
Solution. ABC an equilateral triangle.
We are given that AB = BC = CA = a. AD is the altitude, i.e., AD ⊥ BC.
Now, in right angled triangles ABO and ACD, we have
AB = AC [Given]
and AD=AD [Common side]
 ABD  ACD [By RHS congruence]
1 a
 BD = CD  BD = DC = BC =
2 2
From right triangle ABD,
AB2 = AD2 + BD2
2
a
 a2 = AD2 +  
2
a2 3 2 3
 AD2 = a2 – = a  AD = a
4 4 2

Example.16 In a ABC, obtuse angled at B, if AD is perpendicular to CB produced,


prove that : AC2 = AB2 + BC2 + 2BC × BD
Solution. In ADB, D = 90.
 AD2 + DB2 = AB2 ... (i) [By Pythagoras Theorem]
In ADC, D = 90 º
 AC2 = AD2 + DC2 [By Pythagoras Theorem]
For Unacademy Subscription Use “PJLIVE” Code | Join t.me/pjsir42 for Updates
For More Info: “75970 – 84242, 94590 – 43333 / 2222”
= AD2 + (DB + BC)2
= AD2 + DB2 + BC2 + 2DB × BC
= AB2 + BC2 + 2BC × BD [Using (i))
Hence, AC2 = AB2 + BC2 + 2BC × BD.

Example.17 In a ABC, AB = BC = CA = 2a and AD ⊥ BC. Prove that


(i) AD = a 3 (ii) area (ABC) = 3a 2
Solution. (i) Here, AD ⊥ BC.
Clearly, ABC is an equilateral triangle.
Thus, in ABD and ACD
AD = AD [Common]
ADB = ADC [90º each]
And AB = AC
 By RHS congruency condition
ABD ACD  BD = DC = a
Now, ABD is a right angled triangle
 AD = AB2 − BD2 [Using Pythagoras Theorem]
AD = 4a − a =
2 2
3a or a 3 .
1 1
(ii) Area (ABC) =  BC  AD =  2a  a 3 = a 2 3 .
2 2

Example.18 BL and CM are medians of ABC right angled at A. Prove that 4 (BL2 + CM2) = 5 BC2.
Solution. In BAL
BL2 = AL2 + AB2 [Using Pythagoras theorem] ... (i)
and, In CAM
CM2 = AM2 + AC2 [Using Pythagoras theorem] ... (ii)

Adding (i) and (ii) and then multiplying by 4, we get


4(BL2 + CM2) = 4(AL2 + AB2 + AM2 + AC2)
= 4{AL2 + AM2 + (AB2 + AC2)} [  ABC is a right triangle]
= 4(AL2 + AM2 + BC2)
= 4(ML2 + BC2) [ LAM is a right triangle]
= 4ML2 +4 BC2
[A line joining mid-points of two sides is parallel to third side and is equal to half of it, ML =BC/2]
= BC2 + 4BC2 = 5BC2. Hence proved.

Example.19 O is any point inside a rectangle ABCD. Prove that OB2 + OD2 = OA2 + OC2.
Solution. Through O, draw PQ || BC so that P lies on AB and Q lies on DC.
Now, PQ || BC A D
Therefore,
PQ ⊥ AB and PQ ⊥ DC [B = 90º and C = 90º]
So,  BPQ = 90º and  CQP = 90º
Therefore, BPQC and APQD are both rectangles.
Now, from  OPB,
OB2 = BP2 + OP2 ... (i)
Similarly, from  ODQ,
OD2 = OQ2 + DQ2 ... (ii)
From  OQC, we have

For Unacademy Subscription Use “PJLIVE” Code | Join t.me/pjsir42 for Updates
For More Info: “75970 – 84242, 94590 – 43333 / 2222”
OC2 = OQ2 + CQ2 ... (iii)
And from  OAP, we have
OA2 = AP2 + OP2 ... (iv)
Adding (i) and (ii)
OB2 + OD2 = BP2 + OP2 + OQ2 + DQ2
= CQ2 + OP2 + OQ2 + AP2 [As BP = CQ and DQ = AP]
= CQ2 + OQ2 + OP2 + AP2
= OC2 + OA2 [From (iii) and (iv)] Hence Proved.

Example.20 ABC is a right triangle, right-angled at C. Let BC = a, CA = b, AB = c and let p be the length of
perpendicular form C on AB, prove that
1 1 1
(i) cp = ab (ii) 2 = 2 = 2
p a b
Solution. (i) Let CD ⊥ AB. Then, CD = p
1 1 1
 Area of ABC = (Base × height) = (AB × CD) = cp
2 2 2
1 1
Also, Area of  ABC = (BC × AC) = ab
2 2
1 1
 cp = ab  cp = ab.
2 2

(ii) Since  ABC is a right triangle, right angled at C.


 AB2 = BC2 + AC2
 c2 = a 2 + b 2
2
 ab   ab 
   = a 2 + b2  cp = ab  c = p 
 p  
a 2 b2 1 1 1 1 1 1
 = a 2 + b2  2
= 2+ 2  2
= 2+ 2.
p2 p b a p a b

Example.21 In an equilateral triangle ABC, the side BC is trisected at D. Prove that 9 AD2 = 7AB2.
1
Solution. ABC be an equilateral triangle and D be point on BC such that BD = BC (Given)
3
Draw AE ⊥ BC, Join AD.

BE = EC (Altitude drown from any vertex of an equilateral


triangle bisects the opposite side)
BC
So, BE = EC =
2
In  ABC
AB2 = AE2 + EB2 ...(i)
AD2 = AE2 + ED2 ...(ii)
From (i) and (ii)
AB2 = AD2 – ED2 + EB2
BC 2 BC 2 BC BC BC BC
 AB2 = AD2 – + [ BD + DE =  + DE =  DE = ]
36 4 2 3 2 6
BC 2 BC 2 BC
 AB2 + − = AD2 [ EB = ]
36 4 2
AB2 BC 2
 AB2 + − = AD2 [ AB = BC]
36 4
36AB2 + AB2 − 9AB2
 = AD2
36
28AB 2
 = AD2
36
For Unacademy Subscription Use “PJLIVE” Code | Join t.me/pjsir42 for Updates
For More Info: “75970 – 84242, 94590 – 43333 / 2222”
 7AB2 = 9AD2.

Example.22 Prove that in any triangle, the sum of the squares of any two sides is equal to twice the square
of half of the third side together with twice the square of the median which bisects the third
side. (Appollonius Theorem)
Solution. Given : A ABC in which AD is a median.
2 A
1 
To prove : AB2 + AC2 = 2AD2 +  BC 
2 
or AB2 + AC2 = 2(AD2 + BD2)
Construction : Draw AE ⊥ BC.
Proof :
 AD is median. B D E C
 BD =DC
Now,
AB2 + AC2 = (AE2 + BE2) + (AE2 + CE2)
= 2AE2 + BE2 + CE2
= 2[AD2 – DE2] + BE2 + CE2
= 2AD2 – 2DE2 + (BD + DE)2 + (DC – DE)2
= 2AD2 – 2DE2 + (BD + DE)2 + (BD – DE)2
2
1 
= 2(AD2 + BD2) = 2AD2 + 2  BC 
2 
Hence Proved.

Example.23 Two poles of height 'a' metres and 'b' metres are 'p' metres apart. Prove that the height of the
point of intersection of the lines joining the top of each pole to the foot of the opposite po1e is
ab
given by metres.
a+b
Solution. Let AB and CD be two poles of height 'a' metres and 'b' metres respectively such that the poles
are 'p' metres apart. That is AC = p meters. Suppose the lines AD and BC meet at O, such that
OL= h metres

Let CL = x and LA = y.
Then, x + y = p
In ABC and LOG, we have
CAB = CLO [each equal to 90º]
C= C [C is common]
 CAB ~ CLO [By AA similarity]
CA AB p a
= gives =
CL LO x h
ph
So, x = ... (1)
a
In ALO and ACD, we have
ALO = ACD [each equal to 90º]
A= A [A is common]
ALO ~ ACD [by AA similarity]
For Unacademy Subscription Use “PJLIVE” Code | Join t.me/pjsir42 for Updates
For More Info: “75970 – 84242, 94590 – 43333 / 2222”
AL OL y h ph
= gives = we get, y = ….(2)
AC DC p b b
py ph
(1) + (2)gives x + y = +
a b
 1 1
p = ph =  +  (Since x + y = p)
 b
a+b
1= h 
 ab 
ab
Therefore, h =
a+b

Hence, the height of the intersection of the lines joining the top of each pole to the foot of the
ab
opposite pole is metres.
a+b

Check Point - D

1. A man of height 1.8 m, standing 5 m away from a lamp post observes that the length of his shadow is
1.5 m. What is the height of the lamp post ?
2. A rope from the top of a mast on a sailboat is attached to a point 2 metres from the base of the mast.
The rope is 8 metres long. How high is the mast ?
3. Identify the triangle as acute angled, obtuse angled, right angled whose sides are given below.
(i) a = 12, b = 15, c = 20 (ii) a = 15, b = 8, c = 17
(iii) a = 12, b = 5, c = 17 (iv) a = 8, b = 9, c = 12
4. In a triangle ABC, A = 90º. If AD ⊥ BC prove that AB2 – BD2 = AC2 – CD2.
5. A vertical stick 12 m long casts a shadow 8 m long on the ground. At the same time another tower casts
a shadow 40 m long on the ground. Find the height of the tower.
6. In ABC, AD is the bisector of A. If BC = 10 cm, BD = 6 cm and AC = 6 cm, find AB.

Answers
1. 7.8 m 2. 2 15 m
3. (i) obtuse (ii) Right (iii) obtuse (iv) acute
5. 60 m 6. 9

For Unacademy Subscription Use “PJLIVE” Code | Join t.me/pjsir42 for Updates
For More Info: “75970 – 84242, 94590 – 43333 / 2222”
BOARD LEVEL EXERCISE

TYPE (I) : VERY SHORT ANSWER TYPE QUESTIONS : [01 MARK EACH]
1. The lengths of the diagonals of a rhombus are 16 cm and 12 cm. Then, the length of the side of the
rhombus is
2. In Figure,  BAC = 90º and AD ⊥ BC. Then prove that BD . CD = AD2

3. If in two triangles DEF and PQR,  D =  Q and  R =  E, then show that ED . PR = FE. RQ
4. In triangles ABC and DEF,  B =  E,  F =  C and AB = 3 DE. Then, show that the two triangles
are similar but not congruent
BC 1 ar( PRQ)
5. If  ABC ~ PQR, with = , then is equal to
QR 3 ar(BCA)

TYPE (II) : SHORT ANSWER TYPE QUESTIONS : [02 MARKS EACH]


6. In Figure, two line segments AC and BD intersect each other at the point P such that PA = 6 cm, PB = 3
cm, PC = 2.5 cm, PD = 5 cm, APB = 50º and  CDP = 30º. Then, PBA is equal to

ar(ABC) 9
7. If  ABC~QRP, = , AB = 18 cm and BC = 15 cm, then PR is equal to
ar(PQR) 4

8. If S is a point on side PQ of a  PQR such that PS = QS = RS, then prove that PR2 + QR2 = PQ2
9. If ABC ~ DFE, A = 30º, C = 50º, AB = 5cm, AC = 8cm and DF = 7.5 cm. Then, DE and F.
10. Corresponding sides of two similar triangles are in the ratio of 2 : 3. If the area of the smaller triangle is
48 cm2, find the area of the larger triangle.
TYPE (III) : LONG ANSWER TYPE QUESTIONS: [03 MARK EACH]
11. Find the value of x for which DE || AB in Figure

12. In Figure, if  1 =  2 and  NSQ ~  MTR, then prove that  PTS ~  PRQ.

13. Find the altitude of an equilateral triangle of side 8 cm.


For Unacademy Subscription Use “PJLIVE” Code | Join t.me/pjsir42 for Updates
For More Info: “75970 – 84242, 94590 – 43333 / 2222”
14. In a PQR, PR2 – PQ2 = QR2 and M is a point on side PR such that QM ⊥ PR. Prove that QM2 = PM × MR.
15. ABCD is a trapezium in which AB || DC and P and Q are points on AD and BC, respectively such that
PQ ||DC. If PD = 18 cm, BQ = 35 cm and QC = 15 cm, find AD.
16. Hypotenuse of a right triangle is 25 cm and out of the remaining two sides, one is longer than the other
by 5 cm. Find the lengths of the other two sides.
17. A street light bulb is fixed on a pole 6 m above the level of the street. If a woman of height 1.5 m casts
a shadow of 3m, find how far she is away from the base of the pole.
TYPE (IV): VERY LONG ANSWER TYPE QUESTIONS [04 MARK EACH]
18. In Figure, if PQRS is a parallelogram and AB || PS, then prove that OC || SR.

19. In Figure, PA, QB, RC and SD are all perpendiculars to a line l, AB = 6 cm, BC = 9 cm, CD = 12 cm
and SP = 36 cm. Find PQ, QR and RS

20. In Figure, OB is the perpendicular bisector of the line segment DE, FA ⊥ OB and FE intersects OB at
1 1 2
the point C. Prove that + = .
OA OB OC

21. Legs (sides other than the hypotenuse) of a right triangle are of lengths 16cm and 8 cm. Find the length
of the side of the largest square that can be inscribed in the triangle.

PREVIOUS YEARS PROBLEMS

1. In figure, ABD is a right triangle, right-angled at A and AC ⊥ BD. Prove that AB2 = BC . BD
[2 MARKS/CBSE 10TH BOARD: 2013]

2. In The figure, ABC is a triangle with B = 90º, Medians AE and CD of respective lengths 40 cm
and 5 cm are drawn. Find the length of the hypotenuse AC. [3 MARKS/CBSE 10TH BOARD: 2013]

For Unacademy Subscription Use “PJLIVE” Code | Join t.me/pjsir42 for Updates
For More Info: “75970 – 84242, 94590 – 43333 / 2222”
3. If a line is drawn parallel to one side of a triangle to intersect the other two sides in distinct points,
prove that the other two sides are divided in the same ratio.
OR
Prove that in a triangle, if the square of one side is equal to the sum of the squares of the other two
sides, then the angle opposite to the first side is a right angle.
[4 MARKS/CBSE 10TH BOARD: 2013, 2014, 2015]
AD 2
4. In  ABC, D and E are points on the sides AB and AC respectively such that DE || BC. If = and
DB 3
AE = 18 cm, then find the AC. [CBSE 10TH BOARD: 2014]

5. In figure, two triangles ABC and DBC are on the same base BC in which A = D = 90º. If CA and
BD meet each other at E, show that AE × CE = BE × ED. [CBSE 10TH BOARD: 2014]

6. A triangle has sides 5 cm, 12 cm and 13 cm. Find the length to one decimal place, of The perpendicular
from the opposite vertex to the side whose length is 13 cm. [CBSE 10TH BOARD: 2014]

7. Calculate area (PQR) from figure [CBSE 10TH BOARD: 2014]

8. Prove that the ratio of the areas of two similar triangles is equal to the ratio of the squares of their
corresponding sides.
OR
Prove that, in a right triangle, the square of the hypotenuse is equal to the sum of squares of the other
two sides. [CBSE 10TH BOARD: 2014]

9.  ABC and  PQR are similar triangles such That  A = 32º and  R = 65º, Then  B is
[CBSE 10TH BOARD: 2015]
(A) 83º (B) 33º (C) 63º (D) 93º

10. In Figure, DE||BC and BD = CE. Prove That  ABC is an isosceles triangle.
[CBSE 10TH BOARD: 2015]

For Unacademy Subscription Use “PJLIVE” Code | Join t.me/pjsir42 for Updates
For More Info: “75970 – 84242, 94590 – 43333 / 2222”
11. In a  ABC, P and Q are points on sides AB and AC respectively, such that PQ || BC. If AP = 2.4 cm,
AQ = 2 cm, QC = 3 cm and BC = 6 cm, find AB and PQ. [CBSE 10TH BOARD: 2015]

12. Find The length of an altitude of an equilateral triangle of side 2 cm. [CBSE 10TH BOARD: 2015]

13. If ABC is an equilateral triangle with AD ⊥ BC, then prove AD2 = 3DC2. [CBSE 10TH BOARD: 2016]

14. The diagonals of a trapezium ABCD with AB||DC intersect each other at point O. If AB = 2CD, find
the ratio of the areas of triangles AOB and COD [CBSE 10TH BOARD: 2016]

15. If  ABC   RQP, A = 80º and B = 60º, the value of P is [CBSE 10TH BOARD: 2017]
(A) 80º (B) 30º (C) 40º (D) 50º

16. In figure, AB ⊥ BC, DE ⊥ AC and GF ⊥ BC, therefore ADE   GCF [CBSE 10TH BOARD: 2017]

17. In an isosceles triangle ABC, AB = AC = 25 cm, BC = 14 cm. Calculate the altitude from A on BC.
[CBSE 10TH BOARD: 2017]
PQ 7
18. If Figure, XY || QR, = and PR = 6.3 cm. Find YR. [CBSE 10TH BOARD: 2017]
XQ 3

For Unacademy Subscription Use “PJLIVE” Code | Join t.me/pjsir42 for Updates
For More Info: “75970 – 84242, 94590 – 43333 / 2222”
EXERCISE – 01

SUBJECTIVE QUESTIONS
Section (A) : Similar Triangles
A-1. Any point O, inside ABC, is joined to its vertices. From a point D on AO, DE is drawn so that DE ||
AB and EF || BC as shown in figure. Prove that DF || AC.

A-2. Kitchen garden of Ms. Sanjana is in the form of a triangle as shown. She wants to divide it in two parts;
one triangle and one trapezium.

She takes PE = 4m, QE = 4.5 m, PF = 8m and RF = 9m. Is EF || QR ? Justify your answer.


A-3. In given figure AB || DC. Find the value of x.

A-4. In figure, two triangles ABC and DBC lie on the same side of base BC. P is a point on BC such that PQ
|| BA and PR || BD. Prove that QR || AD.

Section (B) : Criteria for Similarity of triangles


B-1. In LMN, L = 50º and N = 60º. If LMN ~ PQR, then find Q.
B-2. In figure, DE || BC in ABC such that BC = 8 cm, AB = 6 cm and DA = 1.5 cm. Find DE.

For Unacademy Subscription Use “PJLIVE” Code | Join t.me/pjsir42 for Updates
For More Info: “75970 – 84242, 94590 – 43333 / 2222”
BE AC
B-3. In figure, DB ⊥ BC, DE ⊥ AB and AC ⊥ BC. Prove that = .
DE BC

B-4. In  ABC, D and E are points on AB and AC respectively such that DE || BC. If AD = 2.4 cm, AE =
3.2 cm, DE = 2 cm and BC = 5 cm, find BD and CE.

B-5. Given : GHE = DFE = 90º, DH = 8, DF = 12, DG = 3x – 1 and DE = 4x + 2.

Find the lengths of segments DG and DE.

B-6. In figure, QPS = RPT and PST = PQR. Prove that PST ~ PQR and hence find the ratio ST :
PT, if PR : QR = 4 : 5.

B-7. In figure, ABC and DBC are two right triangles with the common hypotenuse BC and with their sides
AC and DB intersecting at P. Prove that AP × PC = DP × PB.

B-8. In figure, ABC is an isosceles triangle in which AB = AC. E is a point on the side CB produced, such
that FE ⊥ AC. If AD ⊥ CB, prove that : AB × FE = AD × EC.

B-9. A girl of height 90 cm is walking away from the base of a lamp-post at a speed of 1.2 m/s. If the lamp is
3.6 m above the ground, find the length of her shadow after 4 seconds.

For Unacademy Subscription Use “PJLIVE” Code | Join t.me/pjsir42 for Updates
For More Info: “75970 – 84242, 94590 – 43333 / 2222”
B-10. In the figure, PQRS is a parallelogram with PQ = 16 cm and QR = 10 cm. L is a point on PR such that
RL : LP = 2 : 3. QL produced meets RS at M and PS produced at N.

Find the lengths of PN and RM.

B-11. In a triangle PQR, L and M are two points on the base QR, such that LPQ = QRP and RPM =
RQP. Prove that

(i) PQL ~ RPM (ii) QL × RM = PL × PM (iii) PQ2 = QR × QL

B-12. In figure, ABD is a right triangle, right-angled at A and AC ⊥ BD. Prove that AB2 = BC  BD.

B-13. In figure, M is mid-point of side CD of a parallelogram ABCD. The line BM is drawn intersecting AC
at L and AD produced at E. Prove that EL = 2BL.

Section (C) : Areas of Similar Triangles


C-1. If the areas of two similar triangles are in the ratio 25 : 64, write the ratio of their corresponding sides.

C-2. In the given figure, DE is parallel to the base BC of triangle ABC and AD : DB = 5 : 3. Find the ratio:-

AD Area of DEF
(i) (ii)
AB Area of CFB

ar(DEF)
C-3. D, E and F are the mid-points of the sides AB, BC and CA respectively of ABC. Find .
ar(ABC)

C-4. Prove that the ratio of the areas of two similar triangles is equal to the ratio of the squares on their
corresponding sides.
For Unacademy Subscription Use “PJLIVE” Code | Join t.me/pjsir42 for Updates
For More Info: “75970 – 84242, 94590 – 43333 / 2222”
Using the above, do the following :
The diagonals of a trapezium ABCD, with AB || DC, intersect each other at the point O. If AB = 2 CD,
find the ratio of the area of AOB to the area of COD.
C-5. AD is an altitude of an equilateral triangle ABC. On AD as base, another equilateral triangle ADE is
constructed.
Prove that area (ADE) : area (ABC) = 3 : 4.
Section (D) : Pythagoras Theorem
D-1. In figure, BAC = 90º, AD⊥ BC. Prove that AB2 = BD2 + AC2 – CD2.

D-2. In figure, ACB = 90º, CD ⊥ AB, prove that CD2 = BD.AD.

D-3. The perpendicular AD on the base BC of a ABC meets BC at D so that 2DB = 3CD. Prove that
5AB2 = 5AC2 + BC2.
D-4. D and E are points on the sides CA and CB respectively of ABC right-angled at C. Prove that
AE2 + BD2 = AB2 + DE2.
D-5. In a right triangle, prove that the square on the hypotenuse is equal to sum of the squares on the other
two sides. Using the above result, prove the following :
In figure PQR is a right triangle, right angled at Q. If QS = SR, show that PR2 = 4PS2 – 3PQ2.

D-6. In  ABC, ABC = 135º. Prove that AC2 = AB2 + BC2 + 4ar (ABC).
D-7. In a triangle, if the square of one side is equal to the sum of the square of the other two sides, prove that
the angle opposite to the first side is a right angle.
Use the above theorem to find the measure of PKR in figure

D-8. In a triangle, if the square of one side is equal to the sum of the squares of the other two sides, then
prove that the angle opposite the first side is a right angle.
Using the above, do the following :
In an isosceles triangle PQR, PQ = QR and PR2 = 2PQ2. Prove that Q is a right angle.

For Unacademy Subscription Use “PJLIVE” Code | Join t.me/pjsir42 for Updates
For More Info: “75970 – 84242, 94590 – 43333 / 2222”
OBJECTIVE QUESTIONS
Section (A) : Similar Triangles
A-1. In a ABC, AD is the bisector of A, meeting side BC at D. If AB = 10 cm, AC = 6 cm, BC = 12 cm,
find BD.

(A) 3.3 (B) 18 (C) 7.5 (D) 1.33

A-2. In figure, if PQ | | BC and PR | | CD, then

AR AR AR AQ AP AD
(A) = (B) = (C) = (D) None of these
AD AD AD AB PC AR

A-3. In figure, ABCD is a trapezium in which AB || EF || DC. The length of AE is

(A) 2 cm (B) 3 cm (C) 4 cm (D) 7 cm

A-4. If AD and AE are angle bisectors of BAE and DAC respectively, then

BD AB BD AD BD DE BD (AB)(AD)
(A) = (B) = (C) = (D) =
EC AC DE AC DE EC EC (AE)(AC)

Section (B) : Criteria for Similarity of triangles


B-1. The perimeters of two similar triangles are 25 cm and 15 cm respectively. If one side of first triangle is
9 cm, then the corresponding side of the other triangle is :
(A) 6.2 cm (B) 3.4 cm (C) 5.4 cm (D) 8.4 cm
QR
B-2. Two triangles ABC and PQR are similar, if BC : CA : AB = 2 : 3 : 4, then is :
PR
2 1 1 2
(A) (B) (C) (D)
5 2 2 3

B-3. ABC and PQR are similar triangles such that A = 32º and R = 65º, then B is :

For Unacademy Subscription Use “PJLIVE” Code | Join t.me/pjsir42 for Updates
For More Info: “75970 – 84242, 94590 – 43333 / 2222”
(A) 83º (B) 32º (C) 65º (D) 97º
B-4. The perimeters of two similar triangles ABC and LMN are 60 cm and 48 cm respectively. If LM = 8 cm,
length of AB is :
(A) 10 cm (B) 8 cm (C) 5 cm (D) 6 cm
B-5. In figure, ABC ~ PQR, then y + z is :

(A) 8 3 cm (B) 4 + 3 3 cm (C) 5 + 4 3 cm (D) 6 + 3 3 cm

AB BC
B-6. If in ABC and DEF, = , then they will be similar if :
DE FD
(A) B = E (B) A = D (C) B = D (D) A = F
B-7. A vertical stick 30 m long casts a shadow 15 m long on the ground. At the same time, a tower casts a
shadow 75 m long on the ground. The height of the tower is :
(A) 150 m (B) 100 m (C) 25 m (D) 200 m
B-8. If the ratio of the corresponding sides of two similar triangles is 2 : 3, then the ratio of their
corresponding altitude is :
(A) 3 : 2 (B) 16 : 81 (C) 4 : 9 (D) 2 : 3
B-9. In the given figure DA ⊥ AB, CB ⊥ AB and OM ⊥ AB. If AO = 5.4 cm, OC = 7.2 cm and BO = 6 cm,
then the length of DO is:

(A) 4.5 cm (B) 4 cm (C) 5 cm (D) 6.5 cm


Section (C) : Areas of Similar Triangles
C-1. In a triangle ABC, a straight line parallel to BC intersects AB and AC at point D and E respectively. If
the area of ADE is one-fifth of the area of ABC and BC = 10 cm, then DE equals :
(A) 2 cm (B) 2 5 cm (C) 4 cm (D) 4 5 cm

C-2. ABC ~ PQR. M is the mid point of BC and N is the mid point of QR. If the area of ABC = 100 sq.
cm. and the area of PQR = 144 sq. cm. If AM = 4 cm, then PN is :
(A) 4.8 cm (B) 12 cm (C) 4 cm (D) 5.6 cm
ar(APQ)
C-3. In the figure, PQ || BC and AP : PB = 1 : 2. Find .
ar(ABC)

(A) 1 : 4 (B) 4 : 1 (C)1 : 9 (D) 2 : 9

1
C-4. In the given figure, LM || NQ and LN || PQ. If MP = MN, find the ratio of the areas of LMN and QNP.
3
For Unacademy Subscription Use “PJLIVE” Code | Join t.me/pjsir42 for Updates
For More Info: “75970 – 84242, 94590 – 43333 / 2222”
(A) 9 : 4 (B) 1 : 9 (C) 1 : 3 (D) 3 : 1

ar ( DEF )
C-5. In the given figure, DE ||BC and AD : DB = 5 : 4. Find .
ar ( CFB )

(A) 25 : 16 (B) 25 : 81 (C)5 : 4 (D) 4:5

Section (D) : Pythagoras Theorem


D-1. In a triangle ABC, if angle B = 90º and D is the point in BC such that BD = 2 DC, then
(A) AC2 = AD2 + 3 CD2 (B) AC2 = AD2 + 5 CD2
2 2 2
(C) AC = AD + 7 CD (D) AC2 = AB2 + 5 BD2

D-2. In an isosceles ABC, if AC = BC and AB2 = 2 AC2, then C is equal to :


(A) 45º (B) 60º (C) 30º (D) 90º

D-3. In the following figure, AE ⊥ BC, D is the mid point of BC, then x is equal to :

1 2 a2  h+d c+d−h a 2 + b2 + d 2 − c2
(A)  b − d 2
−  (B) (C) (D)
a 4 3 2 4

D-4. P and Q are the mid points of the sides AB and BC respectively of the triangle ABC, right-angled at B,
then
4
(A) AQ2 + CP2 = AC2 (B) AQ2 + CP2 = AC2
5
5 3
(C) AQ2 + CP2 = AC2 (D) AQ2 + CP2 = AC2
4 5

For Unacademy Subscription Use “PJLIVE” Code | Join t.me/pjsir42 for Updates
For More Info: “75970 – 84242, 94590 – 43333 / 2222”
EXERCISE – 02

OBJECTIVE QUESTIONS
1. In the figure C is a right angle, DE ⊥ AB, A E = 6, EB = 7 and BC = 5. The area of the quadrilateral
EBCD is

(A) 27.5 (B) 25 (C) 22.5 (D) 20


2. The median AD of ABC meets BC at D. The internal bisectors of ADB and ADC meet AB and
AC at E and F respectively. Then EF :
(A) is perpendicular to AD (B) is parallel to BC
(C) divides AD in the ratio of AB : AC (D) none of these

3. Three squares have the dimensions indicated in the diagram. The area of the quadrilateral ABCD, is :

21 15 42
(A) (B) (C) (D) data not sufficient
4 4 4

4. ABCD is a parallelogram, M is the midpoint of DC. If AP = 65 and PM = 30 then the largest possible
integral value of AB is :

(A) 124 (B) 120 (C) 119 (D) 118

5. ABCD is a parallelogram, P is a point on AB such that AP : PB = 3 : 2. Q is a point on CD such that


CQ : QD = 7 : 3. If PQ meets AC at R, then AR : AC is :
(A) 5 : 11 (B) 6 : 13 (C) 4 : 7 (D) 2 : 5

6. If CD = 15, DB = 9, AD bisects A, ABC = 90º, then AB has length :

(A) 32 (B) 18 (C) 7 (D) 24

7. In a right triangle with sides a and b, and hypotenuse c, the altitude drawn on the hypotenuse is x. Then
which one of the following is correct ?
1 1 1 1 1 1
(A) ab = x2 (B) + = (C) a2 + b2 = 2x2 (D) 2 = 2 + 2
a b x x a b

For Unacademy Subscription Use “PJLIVE” Code | Join t.me/pjsir42 for Updates
For More Info: “75970 – 84242, 94590 – 43333 / 2222”
8. In the right triangle shown the sum of the distances BM and MA is equal to the distances BC and CA. If
MB = x, CB = h and CA = d, then x equals.

hd
(A) (B) d – h (C) h + d – 2d (D) h 2 + d2 − h
2h + d

9. A rhombus is inscribed in triangle ABC in such a way that one of its vertices is A and two its sides lie
along AB and AC and fourth vertex lies on BC, where AC = 6, AB = 12 and BC = 8, the side of the
rhombus, is :
(A) 2 (B) 3 (C) 4 (D) 5
12
10. ABCD (in order) is a rectangle with AB = CD = and BC = DA = 5. Point P is taken on AD such
5
that BPC = 90º. The value of (BP + PC) is equal to :
(A) 5 (B) 6 (C) 7 (D) 8
11. In the diagram, ABCD is a rectangle and point E lies on AB. Triangle DEC has DEC = 90º, DE = 3
and EC = 4. The length of AD is :

(A) 2.4 (B) 2.8 (C) 1.8 (D) 3.2


12. In the figure, DB is diagonal of rectangle ABCD and line l through A and line m through C divide DB in
three equal parts each of length 1 cm and are perpendicular to DB. Area (in cm2) of rectangle ABCD is :

(A) 2 2 (B) 2 3 (C) 3 2 (D) 3 3

13. In the quadrilateral ABCD :

(A) x = y, a = z (B) x = z, a = y (C) x = z, a =y (D) x = y, a = w

14. ‘O’ is any point inside the rectangle PQRS, then


(A) OP2 + OR2 = OQ2 + OS2 (B) OP2 + OQ2 = OR2 + OS2
(C) OP2 + OS2 = OQ2 + QR2 (D) None of the above
15. In a triangle ABC, if AB, BC and AC are the three sides of the triangle, then which of the statements is
necessarily true?
(A) AB + BC < AC (B) AB + BC > AC (C) AB + BC = AC (D) AB2 + BC2 = AC2

16. The sides of a triangle are 12 cm, 8 cm and 6 cm respectively, the triangle is
For Unacademy Subscription Use “PJLIVE” Code | Join t.me/pjsir42 for Updates
For More Info: “75970 – 84242, 94590 – 43333 / 2222”
(A) acute (B) obtuse (C) right (D) Can't be determined
17. If the sides of a triangle are produced then the sum the exterior angles i.e., DAB + EBC + FCA
equal to

(A) 180º (B) 270º (C) 360º E (D) 240º


18. In the given figure BC is produced to D arid BAC = 40º and ABC = 70º. Find the value ACD.

(A) 30º (B) 40º (C) 70º (D) 110º


19. In a ABC, BAC > 90º, then ABC and ACB must be

(A) acute (B) obtuse


(C) one acute and one obtuse (D) Can't be determined B · C
20. If the angles of a triangle are in the ratio 1 : 4 : 7 then the value of the largest angle is
(A) 135º (B) 84º (C) 105º (D) None of these
21. In the adjoining figure B = 70º and C = 30º. BO and CO are the angle bisectors of ABC and
ACB. Find the value of BOC.

(A) 30º (B) 40º (C) 120º (D) 130º


22. In the given diagram of ABC, B = 80º, C = 30º. BF and CF are the angle bisectors CBD and
BCE respectively. Find the value BFC.

(A) 110º (B) 50º (C) 125º (D) 55º


23. In an equilateral triangle, the incentre circumcentre, orthocentre and centroid are
(A) concylic (B) coincident (C) collinear (D) None of these
24. In the adjoining figure D is the midpoint of BC of ABC. DM and DN are the perpendiculars on AB
and AC respectively and DM = DN, then the ABC is.

(A) right angled (B) Isosceles (C) equilateral B (D) scalene

25. In the adjoining figure of ABC, AD is the perpendicular bisector of side BC. The triangle ABC is

For Unacademy Subscription Use “PJLIVE” Code | Join t.me/pjsir42 for Updates
For More Info: “75970 – 84242, 94590 – 43333 / 2222”
(A) right angled (B) isosceles (C) scalene (D) equilateral
26. Triangle ABC is such that AB = 9 cm, BC = 6 cm, AC = 7.5 cm. DEF is similar to ABC, If EF = 12 cm
then DE is
(A) 6 cm (B) 16 cm (C) 18 cm (D) 15 cm
27. In ABC, AB = 5 cm, AC = 7 cm. If AD is the angle bisector of A. Then BD : CD is
(A) 25: 49 (B) 49: 25 (C) 6: 1 (D) 5: 7
28. In a ABC, D is the mid-point of BC and E is mid-point of AD, BF passes through E. What is the ratio
of AF : FC?

(A) 1 : 1 (B) 1: 2 (C) 1 : 3 (D) 2 : 3


29. In a ABC, AB = AC and AD ⊥ BC, then
(A) AB < AD (B) AB > AD (C) AB = AD (D) AB  AD
30. The difference between altitude and base of a right angled triangle is 17 cm and its hypotenuse is 25
cm. What is the sum of the base and altitude of the triangle is?
(A) 24 cm (C) 34 cm (B) 31 cm (D) Can't be determined,
31. If AB, BC and AC be the three sides of a triangle ABC, which one of the following is true?
(A) AB – BC = AC (B) AB – BC > AC (C) AB – BC < AC (D) AB2 – BC2 = AC2
32. In the triangle ABC, side BC is produced to D. ACD = 100º, if BC = AC, then ABC is

(A) 40º (B) 50º (C) 80º (D) can’t be determined


33. In the adjoining figure, D, E and F are the mid-points of the sides BC, AC and AB respectively. DEF
congruent to triangle

(A) ABC (B) AEF (C) CDE, BFD (D) AFE, FBD and EDC
DE 1
34. In the given figure, If DE || BC, = and AE = 10 cm, find AC.
BC 2

(A) 16 cm (B) 12 cm (C) 20 cm (D) 18 cm

35. In the figure, DE || BC and AD = 12 cm, AB = 20 cm and AE = 10 cm. And EC.

For Unacademy Subscription Use “PJLIVE” Code | Join t.me/pjsir42 for Updates
For More Info: “75970 – 84242, 94590 – 43333 / 2222”
(A) 6.6 cm (B) 10 cm (C) 6.67 cm (D) 15 cm
36. In a right angled ABC, C = 90º and CD Is the perpendicular on the hypotenuse AB, AB = c, BC = a,
AC = b and CD = p Then

p p 1 1 1 1 1 1
(A) = (B) 2
+ 2 = 2 (C) p2 = b2 + c2 (D) 2
= 2+ 2
a b p b a p a b

37. If the medians of a triangle are equal, then the triangle is


(A) right angled (B) isosceles (C) equilateral (D) scalene

38. The circumcentre of a triangle is determined by the


(A) altitudes (B) median
(C) perpendicular bisectors (D) angle bisectors

39. The point of intersection of the angle bisectors of a triangle is


(A) orthocentre (B) centroid (C) lncentre : (D) clrcumcentre

40. A triangle PQR is formed by joining the mid-points of the sides of a triangle ABC. 'O' is the
cirumcentre of ABC, then for PQR, the point 'O' Is
(A) incentre (B) circumcentre (C) orthocenter (D) centroid

41. If m a ABC. ‘S’ is the circumventer then


(A) S is equidistant from all the vertices of a triangle
(B) S is equidistant from all the sides of a triangle
(C) AS, BS and CS are the angular bisectors
(D) AS, BS and CS produced are the altitudes on the opposite sides.

42. If AD, BE, CF are the altitudes of ABC whose orthocentre is H, then C is the orthocentre of
(A) ABH (B) BDH (C) ABD (D) BEA

43. In a right angled ABC, C = 90º and CD is the perpendicular on hypotenuse AB. If BC = 15 cm and
AC = 20 cm then CD is equal to

(A) 18 cm (B) 12 cm (C) 17.5 cm (D) Can't be determined


44. In an equilateral ABC, if a, b and c denote the lengths of perpendiculars from A, B and C respectively
on the opposite sides, then
(A) a > b > c (B) a > b < c (C) a = b = c (D) a = c  b

45. What is the ratio of side and height of an equilateral triangle?


(A) 2 : 1 (B) 1 : 1 (C) 2 : 3 (D) 3:2

46. PQR is the triangle formed by joining the mid-points of the sides AB, BC and CA of ABC and the
area of PQR is 6 cm2, then the area of ABC is
For Unacademy Subscription Use “PJLIVE” Code | Join t.me/pjsir42 for Updates
For More Info: “75970 – 84242, 94590 – 43333 / 2222”
(A) 36 cm2 (B) 12 cm2 (C) 18 cm2 · (D) 24 cm2

47. One side other than the hypotenuse of right angle isosceles triangle is 6 cm. The length of the
perpendicular on the hypotenuse from the opposite vertex is
(A) 6 cm (B) 6 2 cm (C) 4 cm (D) 3 2 cm

48. Any two of the four triangles formed by joining the midpoints of the sides of a given triangle are
(A) congruent and equal in area. (B) equal in area but not congruent.
(C) unequal in area and not congruent. (D) None of these

49. The internal bisectors of B and C of ABC meet at O. If A = 80º, then BOC is
(A) 50º (B) 160º (C) 100º (D) 130º

50. The point in the plane of a triangle which is at equal perpendicular distance from the sides of the triangle
(A) centroid (B) incentre (C) circumcentre (D) orthocentre

51. Incentre of a triangle lies in the interior of


(A) an isosceles triangle only (B) a right angled triangle only
(C) any equilateral triangle only (D) any triangle

52. In a triangle PQR, PQ = 20 cm and PR = 6 cm, the side QR is


(A) equal to 14 cm (B) less than 14 cm (C) greater than 14cm (D) None of these

53. If O is orthocentre of a triangle PQR, which is formed by joining the mid points of the sides of a
ABC, then for ABC, O is
(A) orthocenter (B) incentre (C) circumcentre (D) centroid

54. In a ABC, a line PQ parallel to BC cuts AB at P and AC at Q. If BQ bisects PQC, then which one of
the following relations is always true
(A) BC= CQ (B) BC = BQ (C) BC  CQ (D) BC  BQ

55. Which of the following is true, in the given figure, where AD is the altitude to the hypotenuse of a right
angled ABC?
(i) CAD and ABD are similar
(ii) CDA and ADB are congruent
(iii) MOB and CAB are similar
Select the correct answer using the codes given below
(A) (ii) and (ii) (B) (ii) and (iii) (C) (i) and (iii) (D) (i), (ii) and (iii)
AB BD
56. If D is such a point on the side, BC of ABC that = , then AD must be a/an
AC CD
(A) altitude of ABC (B) median of ABC
(C) angle bisector of ABC (D) perpendicular bisector of ABC

57. ln right angled ABC ABC = 90º, if P and Q are points on the sides AB and BC respectively, then

(A) AQ2 + CP2 = 2(AC2 + PQ2) (B) AQ2 + CP2 = AC2 + PQ2
1 1
(C) AQ2 + CP2 = (AC2 + PQ2) (D) AQ + CP = (AC + PQ)
2 2

58. If ABC is a right angled triangle at B and M, N are the mid-points of AB and BC, then 4(AN2 + CM2) is
equal to

For Unacademy Subscription Use “PJLIVE” Code | Join t.me/pjsir42 for Updates
For More Info: “75970 – 84242, 94590 – 43333 / 2222”
5
(A) 4AC2 (B) 6AC2 (C) 5AC2 (D) AC2
4
AB BC CA
59. If ABC and DEF are so related that = = , then which of the following is true?
FD DE EF
(A) A= F and B = D (B) C = F and A = D
(C) B = F and C = D (D) A = E and B = D
BD
60. ABC is a right angle triangle at A and AD is perpendicular to the hypotenuse. Then is equal to
CD
2 2
 AB   AB  AB AB
(A)   (B)   (C) (D)
 AC   AD  AC AD
61. Let ABC be an equilateral triangle. Let BE ⊥ CA meeting CA at E, then (AB2 + BC2 + CA2) is equal to
(A) 2BE2 (B) 3BE2 (C) 4BE2 (D) 6BE2
62. If D, E and F are respectively the mid-points of sides of BC, CA and AB of a ABC. If EF = 3 cm, FD
4 cm, and AB = 10 cm, then DE, BC and CA respectively will be equal to
AB
(A) 6, 8 and 20 cm (B) 4, 6 and 8 cm (C) 5, 6 and 8 cm (D) , 9 and 12 cm
AD
63. In the right angle triangle C = 90º. AE and BD are two medians of a triangle ABC meeting at F. The
ratio of the area of ABF and the quadrilateral FDCE is
(A) 1 : 1 (B) 1 : 2 (C) 2 : 1 (D) 2 : 3
64. ABC is a triangle and DE is drawn parallel to BC cutting the other sides at D and E. If AB = 3.6 cm,
AC = 2.4 cm and AD = 2.1 cm, then AE is equal to
(A) 1.4 cm (B) 1.8cm (C) 1.2 cm (D) 1.05 m
65. Consider the following statements:
(i) If three sides of a triangle are equal to three sides of another triangle, then the triangles are congruent.
(ii) If three angles of a triangle are equal to three angles of another triangle respectively, then the two
triangles are congruent. Of these statements
(A) (i) is correct and (ii) is false (B) both (i) and (ii) are false
(C) both (i) and (ii) are correct (D) (i) is false and (ii) is correct
66. In the figure ABE is an equilateral triangle in a square ABCD. Find the value of angle x.

(A) 60º (B) 45º (C) 75º (D) 90º


67. In the given diagram MN || PR and m LBN = 70º, AB = BC. Find m ABC.

(A) 40º (B) 30º (C) 35º (D) 55º


68. In the given diagram, equilateral triangle EDC surmounts square ABCD. Find the m BED represented
by x.

(A) 45º (B) 60º (C) 30º (D) None of these


69. In the given diagram XY || PQ. Find m xº and m yº.

For Unacademy Subscription Use “PJLIVE” Code | Join t.me/pjsir42 for Updates
For More Info: “75970 – 84242, 94590 – 43333 / 2222”
(A) 75º and 40º (B) 45º and 60º (C) 75º and 45º (D) 60º and 45º
70. In the adjoining figure m CAB == 62º, m CBA m ADE = 58º and DFG = 66º, find m FGE.

(A) 44º (B) 34º (C) 36º (D) None of these


71. In the given figure CE ⊥ AB, m ACE = 20º ABD = 50º. Find m BDA.

(A) 50º (B) 60º (C) 70º (D) 80º


72. In the ABC, BD bisects B, and is perpendicular to AC. If lengths of the sides of the triangle
expressed in terms of x and y as show, find the value of x and y.

(A) 6, 12 (B) 10, 12 (C) 16, 8 (D) 8, 15


73. In the following figure ADBC, BD =CD= ABC = 27º, m ACD = y. Find the value y.

(A) 27º (B) 54º (C) 72º (D) 58º


74. ABC is an isosceles triangle with AB = AC. Side BA is produced to D such that AB = AD. Find m BCD.
(A) 60º (B) 90º (C) 120º (D) Can't be determined
75. In ABC, AC = 5 cm. Calculate the length of AE where DE || BC, given that AD = 3 cm and BD = 7 cm.

(A) 2 cm (B) 1 cm (C) 1.5 cm (D) 2.5 cm

76. In PQR, AP = 2 2 cm, AQ = 3 2 cm and PR = 10 cm, AB || QR. find the length of BR.

(A) 6 2 cm (B) 6 cm (C) 5 2 cm (D) None of these


77. In the adjoining figure (not drawn to scale) AB, EF and. CD are parallel lines. Given that EG = 5 cm,
GC = 10 cm and DC = 18 cm. Calculate AC, If AB = 15 cm.

For Unacademy Subscription Use “PJLIVE” Code | Join t.me/pjsir42 for Updates
For More Info: “75970 – 84242, 94590 – 43333 / 2222”
(A) 21 cm (B) 25 cm (C) 18 cm (D) 28 cm
78. In the adjoining figure PA, QB and RC are each perpendicular to AC. Which one of the following is true?

1 1 1 1 1 1
(A) x + y = z (B) xy = 2z (C) + = (D) + + =0
x y z x y z

79. In the adjoining figure BAC and ADB are right angles. BA = 5 cm, AD = 3 cm and BD = 4 cm,
what is the length of DC?

(A) 2.5 (B) 3 (C) 2.25 (D) 2


80. The areas of two similar triangles are in the ratio of 25 : 36. What is the ratio of their respective heights?
(A) 5 : 6 (B) 6 : 5 (C) 1 : 11 (D) 2:3
81. In the given diagram, AB || CD, then which one of the following is true?

AB AO AB BO
(A) = (B) = (C) AOB ~ COD (D) All of these
CD OC CD OD
82. The diagonal BD of a quadrilateral ABCD bisects B and D, then

AB AD AB AD
(A) = (B) = (C) AB = AD × BC (D) None of these
CD BC BC CD
83. Two right triangles ABC and DBC are drawn on the same hypotenuse BC on the same side of BC. If
AC and DB intersects at P, then

AP BP
(A) = (B) AP × DP = PC × BP
PC DP
(C) AP × PC = BP × DP (D) AP × BP = PC × PD
84. A man goes 150m due east and then 200m due north. How far is he from the starting point?
(A) 200m (B) 350m (C) 250m (D) 175m

For Unacademy Subscription Use “PJLIVE” Code | Join t.me/pjsir42 for Updates
For More Info: “75970 – 84242, 94590 – 43333 / 2222”
85. In an equilateral triangle ABC, the side BC is trisected at D and E. Find the value of AD2.

9 7 3 4
(A) AB2 (B) AB2 (C) AB2 (D) AB2
7 9 4 5

86. ABC is a triangle in which A = 90º. AN ⊥ BC, AC = 12 cm and AB = 5 cm. Find the ratio of the
areas of ANC and ANB.
(A) 125 : 44 (B) 25 : 144 (C) 144 : 25 (D) 12 : 5

87. A vertical stick 15 cm long casts its shadow 10 cm long on the ground. At the same time a flag pole
casts a shadow 60 cm long. Find the height of the flag pole.
(A) 40 cm (B) 45 cm (C) 90 cm (D) None

88. Vertical angles of two isosceles triangles are equal and their corresponding altitudes are in the ratio 4 : 9.
Find the ratio of their areas.
(A) 16 : 49 (B) 16 : 81 (C) 16: 65 (D) None of these

89. In the figure ACB ~ APQ. If BC = 8 cm, PQ = 4 cm, AP = 2.8 cm, find CA.

(A) 8 cm (B) 6.5 cm (C) 5.6 cm (D) None of these

90. In the fig. BC || AD. Find the value of x.

(A) 9, 10 (B) 7, 8 (C) 10, 12 (D) 8, 9

91. In an equilateral triangle of side 2a, calculate the length of its altitude.
3
(A) 2a 3 (B) a 3 (C) a (D) None of these
2

92. In fig. AD is the bisector of BAC. If BD = 2 cm, CD = 3 cm and AB = 5 cm. Find AC.

(A) 6cm (B) 7.5 cm (C) 10 cm (D) 15 cm

93. In the fig. AB || QR. Find the length of PB.

(A) 2 cm (B) 3 cm (C) 2.5 cm (D) 4 cm Q

94. In the fig. QA and PB are perpendicular to AB. If AO = 10 cm, BO = 6 cm and PB = 9 cm. find AQ.

For Unacademy Subscription Use “PJLIVE” Code | Join t.me/pjsir42 for Updates
For More Info: “75970 – 84242, 94590 – 43333 / 2222”
P

A O
B

Q
(A) 8 cm (B) 9cm (C) 15 cm (D) 12 cm
95. In the given figure AB = 12 cm, AC = 15 cm and AD = 6 cm. BC || DE, find the length of AE.

(A) 6 cm (B) 7.5 cm (C) 9 cm (D) 10 cm


96. In figure, ABC is a right triangle, right angled at AD and CE are the two medians drawn from A and C
3 5
respectively. If AC = 5 cm and AD = cm, find the length of CE.
2

(A) 2 5 cm (B) 2.5 cm (C) 5 cm (D) 4 5 cm

97. In a ABC, AB = 10 cm, BC = 12 cm and AC = 14 cm. Find the length of median AD. If G is the
centroid, find length of GA.
4 5 10 8 8
(A) 7, 7 (B) 5 7,4 7 (C) , 7 (D) 4 7, 7
3 9 3 3 3

98. ABC Is a right angled triangle at A and AD is the altitude to BC. If AB = 7 cm and AC = 24 cm. Find
the ratio of AD Is to AM If M is the mid-point of BC.
336 625
(A) 25 : 41 (B) 32 : 41 (C) (D)
625 336
99. Area of ABC = 30 cm2. D and E are the mid-points of BC and AB respectively. Find ar (BDE).
(A) 10 cm (B) 7.5 cm (C) 15 cm (D) None of these
100. The three sides of' a triangles are given. Which one of the following is not a right triangle?
(A) 20, 21, 29 (B) 16, 63, 65 (C) 56, 90, 106 (D) 36, 35, 74
101. In ABC, AB2 + AC2 = 2500 cm2 and median AD = 25 cm, find BC.
(A) 25 cm (B) 40 cm (C) 50 cm (D) 48cm
102. In the given figure, AB = BC and BAC = 15º, AB = 10 cm. Find the area of ABC.

(A) 50 cm2 (B) 40 cm2 (C) 25 cm2 (D) 32 cm2


103. In ABC, G is the centroid, AB = 15 cm, BC = 18 cm and AC = 25 cm. Find GD, where D is the mid-
point of BC.
1 2 8
(A) 86 cm (B) 86 cm (C) 15 cm (D) None of these
3 3 3
104. In the given fig, BC = AC = AD, EAD = 81º. Find the value of x.

For Unacademy Subscription Use “PJLIVE” Code | Join t.me/pjsir42 for Updates
For More Info: “75970 – 84242, 94590 – 43333 / 2222”
(A) 45º (B) 54º (C) 63º (D) 36º

105. In an equilateral triangle the inradius r and circumradius R are connected by


R R
(A) r = (B) r = (C) r = 4R (D) None of these
3 2
106. The altitude of a triangle is equal to 6 cm and divides the vertex angle in the ratio 2 : 1 and the base of the
triangle into two segments the smaller of which is equal to 3cm. The perimeter of the triangle (in cm) is
(A) 21 + 3 5 (B) 21 + 5 (C) 3 5 + 31 (D) 3 5 + 26

107. Inscribed in a right angled triangle is a square having a common right angle with the triangle? The legs
of the triangle are a, b. The perimeter of the square is
ab 2ab 4ab a+b
(A) (B) (C) (D)
a+b a+b a+b 4ab
1
108. Two sides of a triangle are 10 cm and 5 cm length and the length of the median to the third side is 6
2
cm. The area of the triangle is 6 x cm2. The value of x is
A

10 1 5
6
2
B D C
(A) 12 (B) 13 (C) 14 (D) 15
109. In the adjoining figure, BAC is a 30º – 60º – 90º triangle with AB = 20. D is the midpoint of AC. The
perpendicular at D to AC meets the line parallel to AB through C at E. The line through E perpendicular to E
meets BA produced at F. If DF = 5 x the x =
E C
90º 90º 90º
D
60º 30º
F A B
(A) 6 (B) 14 (C) 7 (D) 10
110. In the following figure, PQR is right angled at R and S is the mid–point of hypotenuse PQ. If RS = 25
cm and PR = 48 cm, then find QR.

(A) 7 cm (B) 25 cm (C) 14 cm (D) Cannot be determined

111. In a ABC, AD is the bisector of the angle A. If AB = 3, AC = 6 and BC = 3 3 , find the length of AD
(A) 3 (B) 3 3 (C) 2 3 (D) None of these

112. In an obtuse angled ABC, obtuse angled at B if AD ⊥ CB produced then


(A) AC2 = AB2 + BC2 – 2BC × BD (B) AC2 = AB2 – BC2 + 2BC × BD

For Unacademy Subscription Use “PJLIVE” Code | Join t.me/pjsir42 for Updates
For More Info: “75970 – 84242, 94590 – 43333 / 2222”
(C) AC2 = AB2 + BC2 + 2BC × BD (D) None of these

113. In a ABC, B is an acute angle and AD ⊥ BC then


(A) AB2 = AB2 + BC2 – 2BC × BD (B) AB2 = AC2 + BC2 – 2BC × BD
(C) AC = AB – BC – 2BC × BD
2 2 2
(D) AC2 = AB2 + BC2 – 2BC × BD

114. In a triangle ABC with medians AD, BE and CF which of the following is true for relations between
sides and medians?
(A) 2(AB2 + BC2 + CA)2 = 3(AD2 + BE2 + CF)2 (B) 3(AB2 + BC2 + CA)2 = 4(AD2 + BE2 + CF)2
(C) 4(AB2 + BC2 + CA)2 = 3(AD2 + BE2 + CF)2 (D) None of these

EXERCISE – 03

NTSE PROBLEMS (PREVIOUS YEARS)


BD 3 CF
1. In the given figure = and AE = 6BE, then = __________ [Orissa NTSE Stage-1 2012]
CD 4 AF
A

B
D C

E
(A) 2/9 (B) 4/6 (C) 3/8 (D) 5/9
2. In a given figure in trapezium ABCD if AB || CD then value of x is : [Raj. NTSE Stage-1 2013]

29 8 1
(A) (B) (C) 20 (D)
8 29 20
area ABC
3. ABC ~ PQR and . If PQ = 18 cm and BC = 12 cm, then AB and QR are respectively:
area PQR
[Delhi NTSE Stage-1 2013]
(A) 9 cm, 24 cm (B) 24 cm, 9 cm (C) 32 cm, 675 cm (D) 135 cm, 16 cm
4. E and F are respectively, the mid points of the sides AB and AC of ABC and the area of the
quadrilateral BEFC is k times the area of ABC. The value of k is : [Delhi NTSE Stage-1 2013]
1 3
(A) (B) 3 (C) (D) 4
2 4
5. The ratio of the areas of two similar triangles is equal to : [M.P. NTSE Stage-1 2013]
(A) The ratio of corresponding medians
(B) The ratio of corresponding sides
(C) The ratio of the squares of corresponding sides
(D) None of these

6. In the figure, ABC is similar to EDC. If we have AB = 4cm, ED = 3 cm, CE = 4.2 cm and CD = 4.8
cm, then the values of CA and CB respectively are : [M.P. NTSE Stage-1 2013]

For Unacademy Subscription Use “PJLIVE” Code | Join t.me/pjsir42 for Updates
For More Info: “75970 – 84242, 94590 – 43333 / 2222”
(A) 6 cm, 6.6 cm (B) 4.8 cm, 6.6 cm (C) 6.4 cm, 5.6 cm (D) 5.6 cm, 6.4 cm
7. ABC and BDE are two equilateral triangles such that D is the mid-point of BC. Ratio of the areas of
triangles ABC and BDE is [Raj. NTSE Stage-1 2014]
(A) 2 : 1 (B) 1 : 2 (C) 4 : 1 (D) 1 : 4
8. In a  PRS, PRS = 120º.A point Q is taken on PR such that PQ = QS and QR = RS then QPS = .......
[Bihar NTSE Stage-1 2014]
(A) 15º (B) 30º (C) 45º (D) 12º
9. In below figure PQ || BC and AP : PB = 1 : 2. Then the ratio of area of APQ and ABC will be :
[Chhattisgarh NTSE Stage-1 2014]

(A) 1 : 2 (B) 1 : 4 (C) 1 : 9 (D) 4 : 1


10. In ABC, segment AD⊥BC, If BD = x units, then x is : [Delhi NTSE Stage-1 2014]

c2 + a 2 − b2 a 2 + b2 − c2 b2 + c2 − a 2 b2 + c2 − a 2
(A) x = (B) (C) (D)
2a 2c 2b 2abc

11. In ABC, AB = 6 3 cm, AC = 12 cm and BC = 6 cm. The angle B is [Raj. NTSE Stage-1 2014]
(A) 120º (B) 60º (C) 90º (D) 45º
12. In the figure given below, point D is on side BC of ABC such that ABC = DAC. If AB = 9, AD =
4, AC = 5, then A(ADC) : A(BAC) = [Maharashtra NTSE Stage-1 2014]

(A) 81 : 16 (B) 9 : 4 (C) 4 : 9 (D) 16 : 81


13. In trapezium PQRS, seg PQ || seg SR. Diagonal PR and diagonal QS intersects each other at point O.
If PO = x + 4; QO = x + 2; RO = x + 10 and SO = x + 7, then PQ : RS = ?
[Maharashtra NTSE Stage-1 2014]

(A) 3 : 2 (B) 2 : 3 (C) 4 : 9 (D) 9 : 4


14. In the figure given below, equilateral triangle EDC surmounts square ABCD. Find the angle DEB
represented by x [Delhi NTSE Stage-1 2015]

For Unacademy Subscription Use “PJLIVE” Code | Join t.me/pjsir42 for Updates
For More Info: “75970 – 84242, 94590 – 43333 / 2222”
(A) 60º (B) 15º (C) 30º (D) 45º

15. The length of sides of triangle are integers and its perimeter is 14. How many such distinct triangles are
possible? [Bihar NTSE Stage-1 2015]
(A) 6 (B) 5 (C) 4 (D) 3

16. In the figure given below, DE||AC, find the value of x. [Delhi NTSE Stage-1 2015]

(A) 2 (B) 3 (C) 1 (D) 4

17. In the given figure, ODC ~ OBA, BOC = 115º and CDO = 80º. Then OAB is equal to
[Raj. NTSE Stage-1 2015]

(A) 80º (B) 35º (C) 45º (D) 65º

18. In ABC, AD is median and E is the mid-point of AD. If BE is extended, it meets AC in F. AB = 8 cm,
BC = 21 cm and AC = 15 cm, then AF = ? [Jharkhand NTSE Stage-1 2015]

(A) 7cm. (B) 3 cm. (C) 12 cm. (D) 5 cm.

19. In the given figure, AD is the bisector of  BAC. If AB = 10 cm, AC = 6 and BC = 12 cm, find BD :
[Jharkhand NTSE Stage-1 2015]
A

B C
D
12 cm
(A) 4.5 cm. (B) 9 cm. (C) 7.5 cm. (D) 3 cm.

20. In the given figure, ACB = 90º and CD ⊥ AB, then [Jharkhand NTSE Stage-1 2015]

For Unacademy Subscription Use “PJLIVE” Code | Join t.me/pjsir42 for Updates
For More Info: “75970 – 84242, 94590 – 43333 / 2222”
A

C B
2 2
(A) CD = BD.AD (B) BC = AD.BD (C) AC2 = AD. BC (D) AD2 = CD.BD

21.  ABC is a right triangle in which  C = 90º and CD ⊥ AB. If BC = a, AC = b, AB= c and CD = p,
then [Jharkhand NTSE Stage-1 2015]
1 1 1 1 1 1
(A) p2 = a2 + b2 (B) 2 = 2 + 2 (C) 2 = 2 + 2 (D) None of these
p a b c a b

1
22. The ratio of areas of two similar triangles is 2 : 1. If the perimeter of large triangle is 36 cm, then
4
find the perimeter of smaller triangle. [Maharashtra NTSE Stage-1 2015]
(A) 16 cm (B) 20 cm (C) 12 cm (D) 24 cm

23. In the following figure, seg DE || side BC in  ABC. If 3A ( ADE) = A (DECB), then find the ratio
BC : DE? [Maharashtra NTSE Stage-1 2015]

(A) 1 : 2 (B) 16 : 1 (C) 1 : 16 (D) 2 : 1


24. In the above figure ABC, m B = 90, BD ⊥ AC, AD = 4.5, AB = 7.5, then find A(BDC) : A(ABC) .
[Maharashtra NTSE Stage-1 2016]

(A) 16 : 25 (B) 4 : 5 (C) 25 :16 (D) 5 : 4


25. In the given figure ABCD is a trapezium in which AB||DC and AB : DC = 3 : 2, The ratio of the areas
of AOB and COD is [Raj. NTSE Stage-1 2016]
(A) 3 : 2 (B) 2 : 3 (C) 4 : 9 (D) 9 : 4
26. In the figure D = 90º AB = 16 cm, BC = 12 cm and CA = 6 cm, then CD is:
[Delhi NTSE Stage-1 2016]

13 17 19 18
(A) cm (B) cm (C) cm (D) cm
6 6 6 5

For Unacademy Subscription Use “PJLIVE” Code | Join t.me/pjsir42 for Updates
For More Info: “75970 – 84242, 94590 – 43333 / 2222”
AD 4
27. In the below figure ABC, DE || BC, A(ADE) = 48 sq.cm., = . Find the area of BEC
DB 5
[Maharashtra NTSE Stage-1 2016]
A

D E

B C
(A) 60 sq. cm (B) 95 sq. cm (C) 108 sq. cm (D) 135 sq. cm

28. In  ABC, m B = 90º, AB = 4 5 . BD ⊥ AC, AD = 4, then ar (ABC) = ?


[Maharashtra NTSE Stage-1 2017]
(A) 96 sq. units (B) 80 sq. units (C) 120 sq. units (D) 160 sq. units

29. In the following figure, seg AB || seg CD. Diagonals AC and BD intersect at point O. If AO : OC = 1 : 3,
A(AOB)
then =? [Maharashtra NTSE Stage-1 2017]
A(ABD)
A B

O
D C
1 1
(A) (B) (C) 16 (D) 116
4 9

30. In ABC points P and Q trisect side AB, points T and U trisect side AC and points R and S trisect side
BC. Then perimeter of hexagon PQRSTU is how many times of the perimeter of ABC ?
[Maharashtra NTSE Stage-1 2017]
1 2 1 1
(A) times (B) times (C) times (D) times
3 3 6 2

31. In the given figure, AD is the bisector of ÐBAC. If AB =10cm., AC = 6 cm, and BC =12 cm., find BD
[Jharkhand NTSE Stage–I/18]

(A) 4.5 cm (B) 9 cm (C) 7.5 cm (D) 3 cm

32. If ABC ~ DEF such that DE = 3cm, EF = 2cm, DF = 2.5 cm and BC = 4 cm.
Then the perimeter of ABC will be... [Chhattisgarh NTSE Stage-I/18]
(A) 18 cm (B) 20 cm (C) 12 cm (D) 15 cm

33. If the correspondence ABC  EFD is a similarity in ABC and DEF, then following is not true.
[Gujarat NTSE Stage-I/18]
BC AC AB BC AB AC BC AB
(A) = (B) = (C) = (D) =
DF DE DE DF EF DE DF EF

34. The areas of two similar triangles ΔABC and ΔDEF are 48 cm2 and 12 cm2 respectively. If EF = 3 cm
then BC is– [Uttar Pradesh NTSE Stage–I/18]
(A) 6 cm (B) 4 cm (C) 2 cm (D) 12 cm

For Unacademy Subscription Use “PJLIVE” Code | Join t.me/pjsir42 for Updates
For More Info: “75970 – 84242, 94590 – 43333 / 2222”
35. The areas of two similar triangles are 36 cm2 and 81 cm2 respectively. If the median of smaller triangle
is 12 cm then the corresponding median of the larger triangle is [Raj. NTSE Stage-1 2019]
(A) 12 cm (B) 18 cm (C) 24 cm (D) 10 cm

VALUE BASED / PRACTICAL BASED QUESTIONS

1. An old man was trying to place a ladder 13m long in such a way that it reaches a window of a building
12m above the ground. Anoop decided to help the old man. He placed the foot of the ladder at such a
distance from the wall that the top of ladder reached the window.
(a) Find the distance of the foot of ladder from the wall.
(b) What value is depicted by Anoop?

For Unacademy Subscription Use “PJLIVE” Code | Join t.me/pjsir42 for Updates
For More Info: “75970 – 84242, 94590 – 43333 / 2222”
Answer Key

BOARD LEVEL EXERCISE


TYPE (I) : VERY SHORT ANSWER TYPE QUESTIONS :
1. 10 cm 5. 9

TYPE (II) : SHORT ANSWER TYPE QUESTIONS :


6. 100º 7. 10 cm 9. DE = 12 cm, F =100º 10. 108 cm2

TYPE (III) : LONG ANSWER TYPE QUESTIONS:


11. 2 13. 4 3 cm 15. 60 cm 16. 15 cm and 20 cm
17. 9 m

TYPE (IV): VERY LONG ANSWER TYPE QUESTIONS


16
19. PQ = 8 cm, QR = 12 cm, RS = 16 cm 21. cm
3

PREVIOUS YEAR PROBLEMS

2. 52 4. 45 6. 4.6 cm 7. 120 cm2


9. (A) 11. AB = 6cm, PQ = 2.4cm 12. 3
14. 4 : 1 15. (C) 17. AD = 24 cm 18. YR = 2.7 cm

EXERCISE – 01

SUBJECTIVE QUESTIONS
Section (A) : Similar Triangles
A-2. Yes A-3. x = 7

Section (B) : Criteria for Similarity of triangles


B-1. Q is 70º. B-2. 2 cm. B-4. BC = 3.6 cm, CE = 4.8 cm
5
B-5. DG = 20 units, DE = 30 units B-6. B-9. 1.6 m
4
B-10. PN = 15 cm and RM = 10.67 cm

Section (C) : Areas of Similar Triangles


5 25 1
C-1. 5 : 8. C-2. (i) (ii) C-3.
8 64 4
4
C-4. .
1

Section (D) : Pythagoras Theorem


D-7. 90º.

OBJECTIVE QUESTIONS
Section (A) : Similar Triangles
A-1. (C) A-2. (B) A-3. (B) A-4. (D)

Section (B) : Criteria for Similarity of triangles


B-1. (C) B-2. (D) B-3. (A) B-4. (A) B-5. (B)
For Unacademy Subscription Use “PJLIVE” Code | Join t.me/pjsir42 for Updates
For More Info: “75970 – 84242, 94590 – 43333 / 2222”
B-6. (C) B-7. (A) B-8. (D) B-9. (A)

Section (C) : Areas of Similar Triangles


C-1. (B) C-2. (A) C-3. (C) C-4. (A) C-5. (B)

Section (D) : Pythagoras Theorem


D-1. (B) D-2. (D) D-3. (A) D-4. (C) D-5. (C)
D-6. (C)

EXERCISE – 02

OBJECTIVE QUESTIONS
Ques. 1 2 3 4 5 6 7 8 9 10 11 12 13 14 15 16 17 18 19 20
Ans. C B A A B B D A C C A C A A B B C D A C
Ques. 21 22 23 24 25 26 27 28 29 30 31 32 33 34 35 36 37 38 39 40
Ans. D D B B B C D B B B C B D C C D C C C C
Ques. 41 42 43 44 45 46 47 48 49 50 51 52 53 54 55 56 57 58 59 60
Ans. A A B C C D D A D B D C C A C C B C A A
Ques. 61 62 63 64 65 66 67 68 69 70 71 72 73 74 75 76 77 78 79 80
Ans. C C A A A C A A A B B C C B C B B C C A
Ques. 81 82 83 84 85 86 87 88 89 90 91 92 93 94 95 96 97 98 99 100
Ans. D B C C B C C B C D B B A C B A D C B D
Ques. 101 102 103 104 105 106 107 108 109 110 111 112 113 114
Ans. C C B B B A C C C C C C D B

EXERCISE – 03

Ques. 1 2 3 4 5 6 7 8 9 10 11 12 13 14 15 16 17 18 19 20
Ans. A C B C C D C A C A C D B D C C B D C A
Ques. 21 22 23 24 25 26 27 28 29 30 31 32 33 34 35
Ans. B D D A D C D B A B C D B A B

VALUE BASED / PRACTICAL BASED QUESTIONS

1. 5m, Helpful nature

For Unacademy Subscription Use “PJLIVE” Code | Join t.me/pjsir42 for Updates
For More Info: “75970 – 84242, 94590 – 43333 / 2222”
9 CIRCLES
Introduction

A circle is the locus of a point which moves in a plane in such a way that its distance from a
fixed point remains constant. The fixed point is called the centre and the constant distance is
called the radius of these circle.
The perimeter of a circle is called its circumference. Circumference = 2r
Line segment joining any two points on circumference a circle is called chord of the circle.

Diameter : A chord of the circle passing through the centre of a circle is called its diameter. In the figure, AOB
is a diameter of a circle with centre O.
Diameter = 2 × Radius
(i) Diameter is the Largest chord of a circle.
(ii) All diameters of a circle are equal in length.

A. CIRCLES

(a) Secant and Tangent


Let us consider a circle and a line AB. There can be three different situations as shown in figure.

In figure (i) the line AB and the circle have no common point.
In figure (ii) the line AB intersects the circle at point P and Q.
Therefore, the line AB and the circle have two common points P and Q. The line AB is called secant of
the circle.
In figure (iii) the line touches the circle at point P. Therefore, the line AB and the circle have only one
common point P. The line AB is called tangent of the circle.
Secant : A line which intersects a circle at two distinct points is called the secant of the circle.
Tangent : A line which meets a circle at only one point is called the tangent to the circle. The point a
which the line meets the circle is called the point of contact.
There is only one tangent passing through a point lying on the circle.

Tangent as a limiting case of a secant : In the figure, the secant  cuts the circle
at A and B. If this secant  is turned around the point A, keeping A fixed then B
moves on the circumference closer to A In the limiting position, B coincides with
A The secant  becomes the tangent at A. Tangent to a circle is a secant when the

For Unacademy Subscription Use “PJLIVE” Code | Join t.me/pjsir42 for Updates
For More Info: “75970 – 84242, 94590 – 43333 / 2222”
two end points of its corresponding chord coincide.

In the figure, is a secant which cuts the circle at A and B. If the secant is
moved parallel to itself away from the centre, then the points A and B come
closer and closer to each other. In the limiting position, they coincide into a
single point A at the secant l becomes the tangent at A Thus a tangent line
is limiting case of a secant when the two points of intersection of the secant
and a circle coincide with the point A The point A is called the point of
contact of the tangent. The line  touches the circle at the point A i.e., the
common point of the tangent and the circle is called the point of contact.

Remark : The line containing the radius through the point of contact is called normal to the circle at the point.

Theorem – 1 :
Statement : A tangent to a circle is perpendicular to the radius through the point of contact.

Given : A circle C (O, r) and a tangent AB at a point P.


To prove : OP ⊥ AB.
Construction : Take any point Q, other than P on the tangent AB. Join OQ. Suppose OQ meets the
circle at R.
Proof : Among all line segments joining the point O to a point on AB, the shortest one is perpendicular
to AB. So, to prove that OP ⊥ AB, it is sufficient to prove that OP is shorter than any other segment
joining O to any point of AB.
257 Clearly, OP = OR (Radius)
Now, OQ = OR + RQ  OQ > OR  OQ > OP ( OP = OR)
Thus, OP is shorter than any other segment joining O to any point of AB. Hence, OP ⊥ AB.

Theorem - 2 : (Converse of Theorem 1)


Statement: A line drawn through the end of a radius and perpendicular to it is a tangent to the
circle.
Given : OP is a radius of a circle with center O. AB is a line through P and OP ⊥ AB.
To prove : AB is tangent to the circle at point P.
Construction : Take a point R other than P on AB. Join OR which intersect the circle at Q.
Proof : OP ⊥ AB (given)
 OP is the shortest line segment drawn from point O to AB.
 OP < OR
 OR > OP
 R lies outside the circle.
Thus every point on AB, other then P, lies outside the circle.
This shows that AB meets the circle only at point P.
Hence, AB is the tangent to the circle at point P.

For Unacademy Subscription Use “PJLIVE” Code | Join t.me/pjsir42 for Updates
For More Info: “75970 – 84242, 94590 – 43333 / 2222”
(b) Number of tangents from a point on a circle

(a) If a point is inside the circle then it is not possible to draw any tangent to the circle through this
point as shown in figure (i).
(b) If a point is on the circle then only one tangent to the circle through this point can be drawn as
shown in figure (ii).
(c) If a point is outside the circle then exactly two tangents can be drawn to the circle through this point
as shown in figure (iii). PA and PB are the lengths of tangents drawn from P to the circle.
Theorem-3 : The tangent at any point of a circle and the radius through the point are perpendicular to
each other.
Given : A circle with centre O. AB is a tangent to the circle at a point P and OP is the radius through P.
To prove: OP ⊥ AB.
Construct: Take a point Q, other than P, on tangent AB. Join OQ.
Proof:
Q is a point on tangent AB, other than the point P,
so Q will lie outside the circle
 OQ will intersect the circle at some point R
 OR< OQ (Whole is greater than the part.)
 OP < OQ (OR = OP = radius.)
Thus, OP is shorter than any other line segment joining O to any point of AB of all line segments drawn
from O to line AB, the perpendicular is the shortest
 OP ⊥ AB
Remark : 1. A pair of tangents drawn at two points of a circle are either parallel or they intersect each other
at a point outside the circle.
2. If two tangents drawn to a circle are parallel to each other, then the line-segment joining their
points of contact is a diameter of the circle.
3. The distance between two parallel tangents to a circle is equal to the diameter of the circle, i.e.,
twice the radius.
4. A pair of tangents drawn to a circle at the end points of a diameter of a circle are parallel to
each other.
5. A pair of tangents drawn to a circle at the end points of a chord of the circle, other than a
diameter, intersect each other at a point outside the circle.
Corollary-1 : A line drawn through the end point of a radius and perpendicular to it is a tangent to the circle.
Given : O is the centre and r be the radius of the circle. OP is a radius of the circle. line l is drawn
through P so that OP ⊥ .
To prove : Line  is tangent to the circle at P.
Construction : Suppose that the line e is not the tangent to the circle at P. Let us draw another straight
line m which is tangent to the circle at P. Take two points A and B (other than P) on the line C and two
points C and D on m.
Proof : As OP ⊥  (Given)
  OPB = 90º
 OP ⊥ m (By theorem)
 LOPD = 90º
 OPD = OPB (Each = 90º)
But a part cannot, be equal to whole. This gives contradiction.
Hence, our supposition is wrong.
Therefore, the line  is tangent to the circle at P.

For Unacademy Subscription Use “PJLIVE” Code | Join t.me/pjsir42 for Updates
For More Info: “75970 – 84242, 94590 – 43333 / 2222”
Corollary-2 : If O be the centre of a circle and tangents drawn to the circle at the points A and B of the
circle intersect each other at P, then
AOB + APB = 180º.
Proof: As OA ⊥ PA & OB ⊥ PB
 OAP = OBP = 90º (By theorem)
AOB + OBP + OAP + APB = 360º
 AOB + 90º + 90º + APB = 360º
 AOB + APB + 180º = 360º
 AOB + APB = 360º – 180º
 AOB + APB = 180º

Theorem-4 : If two tangents are drawn to a circle from an exterior point, then
(i) the tangents are equal in length
(ii) the tangents subtend equal angles at the centre
(iii) the tangents are equally inclined to the line joining the exterior point and the centre of the circle.
Given : PA and PB are two tangents drawn to a circle with centre O, from an exterior point P.
To prove : (i) PA = PB (ii) AOP = BOP, (iii) APO = BPO.
Proof :
In AOP and BOP
OA = OB (Radii of the same circle.)
OAP = OBP = 90º (Radius through point of contact is perpendicular to the tangent)
OP = OP (Common)
 AOP = BOP (By RHS congruence)
Hence, we have
(i) PA=PB (c.p.c.t)
(ii) AOP = BOP (c.p.c.t)
(iii) APO = BPO (c.p.c.t)
Corollary 3 : If PA and PB are two tangents from a point to a circle with centre O touching it at A and
B. Prove that OP is perpendicular bisector of AB.
Proof:
In ACP and BCP
(i) PA = PB (Lengths of two tangents from P are equal)
(ii) PC = PC (Common)
(iii) APC = PC (PO Bisector APB)
ACP  BCP (SAS congruency)
AC = BC (c.p.c.t)
ACP = BCP = 90º
Hence OP is perpendicular bisector of AB.

(c) Common tangents to two circles


Definition : A line which touches the two given circles is called common tangent to the two circles.
Let C(O1, r1), C(O2, r2) be two given circles. Let the distance between centres O1 and O2 be d i.e.,
O1O2 = d.
Case-1 : Common tangent to two circle intersecting circles

In figure d < r1 + r2. In this case two circles intersect in two distinct points and there are only two
common tangents.

For Unacademy Subscription Use “PJLIVE” Code | Join t.me/pjsir42 for Updates
For More Info: “75970 – 84242, 94590 – 43333 / 2222”
Case-2 : Common tangent of two circle which touch each other
(i) externally

In figure, d = r1 + r2. In this case, two circles touch externally and there are three common tangents.
(ii) Internally

In figure, d = r1 – r2 (r1 > r2), in this case, two circles touch internally and there is only one
common tangent.

Case-3 : Common tangent of two non-intersecting and non-touching circles


(i) externally

In figure d > r1 + r2 i.e. two circles do not intersect.


In this case, four common tangents are possible.
The tangent lines l and m are called direct common tangents and the tangent lines p and q are
called indirect (transverse) common tangents.
(ii) Internally

In figure, the circle C(O2, r2) lies wholly in the circle C(O1, r1) and there is no common tangent.

Remark : Length of Direct common tangent


= (distance between centre O1 and O 2 ) 2 − (r1 − r2 ) 2  d 2 − (r1 − r2 ) 2
Length of transverse common tangent
= (distance between centre O1 and O 2 ) 2 − (r1 + r2 ) 2  d 2 − (r1 + r2 ) 2

For Unacademy Subscription Use “PJLIVE” Code | Join t.me/pjsir42 for Updates
For More Info: “75970 – 84242, 94590 – 43333 / 2222”
Solved Examples

Example.1 A point M is 26 cm away from the centre of a circle and the length of tangent drawn from M to
the circle is 24 cm. Find the radius of the circle.
Solution. Given OM = 26 cm, MN = 24 cm
We know that the tangent at any point to a circle is perpendicular to the radius through point of
contact.
 ONM is a right angled 
OM2 = ON2 + MN2 (By Pythagoras theorem)
 (26)2 = ON2 + (24) 2
 ON2 = (26)2 – (24)2
 ON2 = 676 – 576 = 100
 ON = 10 cm.
Hence, radius of circle is 10 cm.

Example.2 If all the sides of a parallelogram touch a circle, show that the parallelogram is a rhombus.
Solution. Let ABCD be a parallelogram whose sides AB, BC, CD and DA touch a circle at the points P,
Q, R and S respectively.
Since the lengths of tangents drawn from an external point to a circle are equal, we have
AP = AS, BP = BQ, CR = CQ and DR = DS.
 AB + CD = AP + BP + CR + DR
= AS + BQ + CQ + DS
= (AS + DS) + (BQ + CQ)
= AD + BC
Now, AB + CD = AD + BC
 2AB = 2BC ( Opposite sides of a parallelogram are equal)
 AB = BC
 AB = BC = CD = AD.
Hence, ABCD is a rhombus.

Example.3 In the given figure, the incircle of ABC touches the sides AB, BC and CA at the points P, Q,
R respectively.
1
Show that AP + BQ + CR = BP + CQ + AR = (Perimeter of ABC)
2

Solution. Since the lengths of two tangents drawn from an external point to a circle are equal, we have
AP = AR, BQ = BP
and CR = CQ
 AP + BQ + CR = AR + BP + CQ … (i)
Perimeter of ABC = AB + BC + CA = AP + BP + BQ + CQ + AR + CR
= (AP + BQ + CR) + (BP + CQ + AR)
= 2(AP + BQ + CR) [Using (i)]
 AP + BQ + CR = BP + CQ + AR
1
= (Perimeter of ABC).
2

For Unacademy Subscription Use “PJLIVE” Code | Join t.me/pjsir42 for Updates
For More Info: “75970 – 84242, 94590 – 43333 / 2222”
Example.4 In figure AM and BM are the tangents to a circle with centre O. Show that the point O, A, M, B
are concyclic.
Solution. Given : AM and BM are the tangents to a circle with centre O.
To prove : O, A, M and B are concyclic, i.e., AOBM is a cyclic quadrilateral.
Proof : OA ⊥ AM and OB ⊥ BM (A tangent is perpendicular
to radius at the point of contact).
i.e., OAM = 90º and OBM = 90º
In quadrilateral AOBM
OAM + OBM = 90º + 90º = 180º
Thus, the sum of opposite angles of quadrilateral AOBM is 180º.
 AOBM is a cyclic quadrilateral.

Example.5 Two tangents PA and PB are drawn to a circle with centre O from an external point P. Prove
that APB = 2OAB.
Solution. Given : PA and PB are two tangents drawn from external point P to a circle with centre O.
To prove : APB = 2OAB.
Proof : OA ⊥ PA ( A tangent to a circle is perpendicular to radius at the point of contact).
 OAP = 90º
In PAB
PA = PB ( Tangents drawn from an external point to a circle are equal)
 PAB = PBA ( In a , angles opposite to equal sides are equal)
PAB + PBA + APB = 180º (Angle sum property)
PAB + PAB + APB = 180º ( PAB = PBA)
2PAB = 180º – APB.
1
 PAB = 90º − APB ...(i)
2
Now OAP = 90º
 OAB + PAB = 90º
1
 OAB + 90º − APB = 90º (using (i))
2
 OAB = 7 APB  APB = 2OAB.

Example.6 If the radii of the two concentric circles are 15 cm and 17 cm, show that the length of the chord
of one circle which is tangent to other circle is 16 cm.
Solution. Given : OA = 17 cm, OC = 15 cm (given)
AB is a chord of larger circle which is the tangent for smaller circle at point C.
 OC ⊥ AB ( A tangent is perpendicular to radius at the point of contact).
In AOC
OA2 = OC2 + AC2
 172 = 152 + AC2
 AC2 = 172 – 152
 AC2 = 289 – 225 = 64
 AC = 8 cm.
We know that perpendicular drawn from centre of a chord bisects the chord.
So, AB = 2AC = 2 × 8 = 16 cm. Ans.

Example.7 If all the sides of a parallelogram touches a circle, show that the parallelogram is a rhombus.
Solution. Given : Sides AB, BC, CD and DA of a ||gm ABCD touch a circle at P, Q, R and S respectively.
To prove : ||gm ABCD is a rhombus.
Proof : AP = AS.......(i)
BP = BQ .......(ii)
CR = CQ .......(iii)
DR = DS .......(iv)

For Unacademy Subscription Use “PJLIVE” Code | Join t.me/pjsir42 for Updates
For More Info: “75970 – 84242, 94590 – 43333 / 2222”
[Tangents drawn from an external point to a circle are equal]
Adding (i), (ii), (iii) and (iv), we get
 AP + BP + CR + DR = AS + BQ + CQ + DS
 (AP + BP) + (CR + DR) = (AS + DS) + (BQ + CQ)
 AB + CD = AD + BC
 AB + AB = AD + AD
[In a ||gm ABCD, opposite sides are equal]
 2AB = 2AD or AB = AD
But AB = CD and AD = BC [Opposite sides of a ||gm]
 AB = BC = CD = DA
Hence, ||gm ABCD is a rhombus.

Example.8 A circle touches the side BC of a  ABC at P and touches AB and AC when produced at Q and
1
R respectively as shown in figure, Show that AQ = (Perimeter of ABC).
2
Solution. Given : A circle is touching side BC of ABC at P and touching AB and AC when produced at
Q and R respectively.
1
To prove : AQ = (perimeter of ABC)
2
Proof : AQ = AR....(i)
BQ = BP ....(ii)
CP = CR ....(iii)
[Tangents drawn from an external point to a circle are equal]
Now, perimeter of ABC = AB + BC + CA
= AB + BP + PC + CA
= (AB + BQ) + (CR + CA) [From (ii) and (iii)]
= AQ + AR = AQ + AQ [From (i)]
1
AQ = (perimeter of ABC).
2

Example.9 Prove that the tangents at the extremities of any chord make equal angles with the chord.
Solution. Let AB be a chord of a circle with centre O, and let AP and BP be the tangents at A and B
respectively. Suppose, the tangents meet at point P.

Join OP. Suppose OP meets AB at C.


We have to prove that
PAC = PBC
In triangles PCA and PCB
PA = PB [ Tangent from an external point are equal]
APC = BPC [ PA and PB are equally inclined to OP]
And PC= PC [Common]
So, by SAS criteria of congruence
PAC  PBC  PAC = PBC [By CPCT]

Example.10 Prove that the segment joining the points of contact of two parallel tangents passes through the
centre.
Solution. Let PAQ and RBS be two parallel tangents to a circle with centre O. Join OA and OB. Draw
OC||PQ

For Unacademy Subscription Use “PJLIVE” Code | Join t.me/pjsir42 for Updates
For More Info: “75970 – 84242, 94590 – 43333 / 2222”
Now, PA || CO
 PAO + COA = 180º [Sum of co-interior angle is 180º]
 90º + COA = 180º [ PAO = 90]
 COA = 90º
Similarly, COB = 90º
 COA + COB = 90º + 90º = 180º
Hence, AOB is a straight line passing through O.
Example.11 Two circles touch externally at P and a common tangent touch them at A and B. Prove that
(i) The common tangent at P bisects AB.
(ii) AB subtends a right angle at P.
Solution. Let PT be the common tangent at any point P. Since the tangent to a circle from an external
point are equal,
 TA = TP, TB = TP
 TA = TB
i.e. PT bisects AB at T
TA = TP gives TAP = TPA (from PAT)
TB = TP gives TBP = TPB [from PBT]
 TAP + TBP = TPA + TPB = APB
 TAP + TBP + APB = 2 APB
 2 APB = 180º [sum of s of a  = 180º]
 APB = 90º. Hence proved

Example.12 In a right triangle ABC, the perpendicular BD on the hypotenuse AC is drawn. Prove that
(i) AC × AD = AB2 (ii) AC × CD = BC2
Solution. We draw a circle with BC as diameter. Since BDC = 90º.
 The circle on BC as diameter will pass through D. Again
 BC is a diameter and AB ⊥ BC.
 AB is a tangent to the circle at B.
Since AB is a tangent and ADC is a secant to the circle.
 AC × AD = AB2 This proves (i)
Again AC × CD = AC × (AC – AD) = AC2 – AC × AD
= AC22 – AB2 [Using (i)]
= BC2 [ABC is a right triangle]
Hence, AC × CD = BC2. This proves (ii).
Example.13 Two circles of radii R and r touch each other externally and PQ is the direct common tangent.
Then show that PQ2 = 4rR
Solution. Draw O'S|| PQ,
 O'SPQ is rectangle
O'S = PQ, PS = QO' = r
OO' = R + r
OS = OP – PS = R – r
In  O'OS
(O'S)2 = (OO')2 – (OS)2
PQ2 = (R + r)2 – (R – r)2
PQ2 = R2 + r2 + 2Rr – (R2 + r2 – 2Rr)
PQ2 = R2 + r2 + 2Rr – R2 – r2 + 2Rr
PQ2 = 4rR

For Unacademy Subscription Use “PJLIVE” Code | Join t.me/pjsir42 for Updates
For More Info: “75970 – 84242, 94590 – 43333 / 2222”
Example.14 A triangle ABC is drawn to circumscribe a circle of radius 4 cm such that the segments BD and
DC into which BC is divided by the point of contact D are of lengths 8 cm and 6 cm
respectively (see figure). Find the sides AB and AC.
Solution. In figure BD = 8 cm and DC = 6 cm
Then we have BE = 8 cm ( BE = BD)
and CF = 6 cm ( CF = CD)
Suppose AE = AF = x cm
In ABC, a = BC = 6 cm + 8 cm = 14 cm
b = CA = (x + 6) cm, c = AB = (x + 8) cm
a + b + c 14 + (x + 6) + (x + 8)
s= = cm
2 2
2x + 28
= cm = (x + 14) cm
2
Area of ABC
= s(s− a)(s− b)(s− c) = (x + 14)  x 8  6
= 48x  (x + 14) cm2 … (i)
Also, area of ABC = area of OBC + area of OCA + area of OAB
1 1 1
= ×4×a+ ×4×b+ ×4×c
2 2 2
= 2 (a + b + c) = 2 × 2s = 4s
= 4 (x + 14) cm2 … (ii)
From (i) and (ii), 48x  (x + 14) = 4 × (x + 14)
 48x × (x + 14) = 16 × (x + 14)2
 3x = x + 14  x = 7 cm
Then AB = c = (x + 8) cm = (7 + 8) cm = 15 cm
and AC = b = (x + 6) cm = (7 + 6) cm = 13 cm

Check Point - A

1. A quadrilateral PQRS is drawn to circumscribe a circle. If PQ = 12 cm, QR = 15 cm and RS = 14 cm


then find length of PS.

2. O is the centre of the circle. PA and PB are tangent. If PAB = 70º, then find APB.

3. Find the maximum number of common tangents that can be drawn to two circles that do not intersect
each other.

4. A tangent is drawn to a circle from a point 17 cm away from its centre. If the length of the tangent is 15
cm then find the radius of the circle.

5. Two concentric circles are of radii 5 cm and 3 cm. Find the length of the chord to the larger circle
which is a tangent to the smaller circle.

Answers
1. 11 cm 2. 40º 3. 4 4. 8 cm 5. 8 cm

For Unacademy Subscription Use “PJLIVE” Code | Join t.me/pjsir42 for Updates
For More Info: “75970 – 84242, 94590 – 43333 / 2222”
BOARD LEVEL EXERCISE

TYPE (I) : VERY SHORT ANSWER TYPE QUESTIONS : [01 MARK EACH]
1. In the figure, AB is a chord of the circle and AOC is its diameter such that ACB = 50º. If AT is the
tangent to the circle at the point A, then BAT is equal to

2. In Figure, if AOB = 125º, then COD is equal to

3. If radii of two concentric circles are 4 cm and 5 cm, then the length of each chord of one circle which is
tangent to the other circle is

4. In the figure, if O is the centre of a circle, PQ is a chord and the tangent PR at P makes an angle of 50º
with PQ, then POQ is equal to
P
50º R

5. If two tangents inclined at an angle 60º are drawn to a circle of radius 3 cm, then length of each tangent
is equal to

6. In the figure, AT is a tangent to the circle with centre O such that OT = 4 cm and OTA = 30º. Then
AT is equal to

30º
A T

For Unacademy Subscription Use “PJLIVE” Code | Join t.me/pjsir42 for Updates
For More Info: “75970 – 84242, 94590 – 43333 / 2222”
7. In the figure, if PQR is the tangent to a circle at Q whose centre is O, AB is a chord parallel to PR and
BQR = 70º, then AQB is equal to
A B

70º

P Q R

TYPE (II) : SHORT ANSWER TYPE QUESTIONS : [02 MARKS EACH]


8. Prove that the centre of a circle touching two intersecting lines lies on the angle bisector of the lines.

9. Two tangents PQ and PR are drawn from an external point to a circle with centre O. Prove that QORP
is a cyclic quadrilateral.

10. If from an external point B of a circle with centre O, two tangents BC and BD are drawn such that
DBC = 120º, prove that BC + BD = BO, i.e., BO = 2BC.

11. In a right triangle ABC in which B = 90º, a circle is drawn with AB as diameter intersecting the
hypotenuse AC and P. Prove that the tangent to the circle at P bisects BC.

12. In the figure, tangents PQ and PR are drawn to a circle such that RPQ = 30º. A chord RS is drawn
parallel to the tangent PQ. Find the RQS.
S R

30º
Q P

13. In figure, common tangents AB and CD to two circles intersect at E. Prove that AB = CD.
A
D

E
B
C

14. Prove that the tangents drawn at the ends of a chord of a circle make equal angles with the chord.
15. Prove that a diameter AB of a circle bisects all those chords which are parallel to the tangent at the
point A.

TYPE (III) : LONG ANSWER TYPE QUESTIONS: [03 MARK EACH]


16. Two circles with centres O and O' of radii 3 cm and 4 cm, respectively intersect at two points P and Q
such that OP and O'P are tangents to the two circles. Find the length of the common chord PQ.

17. If a hexagon ABCDEF circumscribe a circle, prove that AB + CD + EF = BC + DE + FA.

For Unacademy Subscription Use “PJLIVE” Code | Join t.me/pjsir42 for Updates
For More Info: “75970 – 84242, 94590 – 43333 / 2222”
18. If AB is a chord of a circle with centre O, AOC is a diameter and AT is the tangent at A as shown in
figure. Prove that BAT = ACB

19. In the figure, the common tangent, AB and CD to two circles with centres O and O' intersect at E.
Prove that the points O, E, O' are collinear.
A
D

O O’
E
B
C

TYPE (IV): VERY LONG ANSWER TYPE QUESTIONS [04 MARK EACH]
20. A is a point at a distance 13 cm from the centre O of a circle of radius 5 cm. AP and AQ are the
tangents to the circle at P and Q. If a tangent BC is drawn at a point R lying on the minor arc PQ to
intersect AP at B and AQ at C, find the perimeter of the ABC

21. If an isosceles triangle ABC, in which AB = AC = 6 cm, is inscribed in a circle of radius 9 cm, find the
area of the triangle.

22. In the figure, from an external point P, a tangent PT and a line segment PAB is drawn to a circle with
centre O. ON is perpendicular on the chord AB. Prove that :
(i) PA. PB = PN2 – AN2 (ii) PN2 – AN2 = OP2 – OT2
2
(iii) PA.PB = PT
T

P
O
A
N
B C

For Unacademy Subscription Use “PJLIVE” Code | Join t.me/pjsir42 for Updates
For More Info: “75970 – 84242, 94590 – 43333 / 2222”
PREVIOUS YEARS PROBLEMS

1. Prove that the parallelogram circumscribing a circle is a rhombus.


[2 MARKS/CBSE 10TH BOARD: 2013]

2. In Figure, a circle inscribed in triangle ABC touches its sides AB, BC and AC at points D, E and F
respectively. If AB = 12 cm, BC = 8 cm and AC = 10 cm, then find the lengths of AD, BE and CF.
[2 MARKS/CBSE 10TH BOARD: 2013]

3. In Figure, PA and PB are two tangents drawn from an external point P to a circle with centre C and
radius 4 cm. If PA ⊥ PB, then the length of each tangent is : [1 MARK/CBSE 10TH BOARD:
2013]

(A) 3 cm (B) 4 cm (C) 5 cm (D) 6 cm

4. In Figure, a circle with centre O is inscribed in a quadrilateral ABCD such that, it touches the sides BC, AB,
AD and CD at points P, Q, R and S respectively, If AB = 29 cm, AD = 23 cm, B = 90º and DS = 5 cm,
then the radius of the circle (in cm.) is : [1 MARK/CBSE 10TH BOARD: 2013]
(A) 11 (B) 18 (C) 6 (D) 15

5. In Figure, l and m are two parallel tangents to a circle with centre O, touching the circle at A and B
respectively. Another tangent at C intersects the line l at D and m at E. Prove that DOE = 90º
[4 MARKS/CBSE 10TH BOARD: 2013, 2016]

6. Prove that the tangent at any point of a circle is perpendicular to the radius through the point of contact.
[4 MARKS/CBSE 10TH BOARD: 2013]

7. Two concentric circles are of radii 5 cm and 3 cm. Length of the chord of the larger circle (in cm),
which touches the smaller circle is. [1 MARK /CBSE 10TH BOARD: 2013]
(A) 4 (B) 5 (C) 8 (D) 10

For Unacademy Subscription Use “PJLIVE” Code | Join t.me/pjsir42 for Updates
For More Info: “75970 – 84242, 94590 – 43333 / 2222”
8. In Figure, a quadrilateral ABCD is drawn to circumscribe a circle such that its sides AB, BC, CD and
AD touch the circle at P, Q, R and S respectively. If AB = x cm, BC = 7 cm, CR = 3 cm and AS = 5
cm, find x. [1 MARK /CBSE 10TH BOARD: 2013]
D R
C

S
Q

A P B
(A) 10 (B) 9 (C) 8 (D) 7

9. A quadrilateral is drawn to circumscribe a circle. Prove that the sums of opposite sides are equal.
[4 MARKS/CBSE 10TH BOARD: 2013, 2015, 2017]
10. In Figure, XP and XQ are two tangents to the circle with centre O, drawn from an external point X.
ARB is another tangent, touching the circle at R. Prove that XA + AR = XB + BR.
[2 MARKS/CBSE 10TH BOARD: 2014]
P
A

O R
X

B
Q

11. In Figure, PQ is a chord of a circle with centre O and PT is a tangent. If QPT = 60º, find PRQ.
[1 MARKS/CBSE 10TH BOARD: 2014]

P T

12. In Figure, two tangents RQ and RP are drawn from an external point R to the circle with centre O. If
PRQ = 120º, then prove that OR = PR + RQ. [2 MARKS/CBSE 10TH BOARD: 2014]

O R

For Unacademy Subscription Use “PJLIVE” Code | Join t.me/pjsir42 for Updates
For More Info: “75970 – 84242, 94590 – 43333 / 2222”
13. Prove that the tangent drawn at the mid-point of an arc of a circle is parallel to the chord joining the end
points of the arc. [2 MARKS/ CBSE 10TH BOARD: 2015]

14. In Figure, from an external point P, two tangents PT and PS are drawn to a circle with centre O and
radius r. If OP = 2r, show that OTS = OST = 30º. [2 MARKS/ CBSE 10TH BOARD: 2015]
T

Q
O P

15. In the figure, two equal circles, with centres O and O', touch each other at X.OO' produced meets the
circle with centre O' at A. AC is tangent to the circle with centre O, at the point C. O'D is perpendicular
DO '
to AC. Find the value of . [4 MARKS/ CBSE 10TH BOARD: 2015]
CO
C

D
A
O’ X O

16. Prove that the lengths of the tangents drawn from an external point to a circle are equal.
[4 MARKS/CBSE 10TH BOARD: 2015, 2016, 2017]

17. If the angle between two tangents drawn from an external point P to a circle of radius a and centre O, is
60º, then find the length of OP. [1 MARK/CBSE 10TH BOARD: 2017]

18. Prove that the tangents drawn at the end points of a chord of a circle make equal angles with the chord.
[2 MARKS/CBSE 10TH BOARD: 2017]

EXERCISE – 01

SUBJECTIVE QUESTIONS
Section (A) : Circles
A-1. In figure, a circle touches all the four sides of a quadrilateral ABCD with AB = 6 cm, BC = 7 cm and
CD = 4 cm. Find AD.

For Unacademy Subscription Use “PJLIVE” Code | Join t.me/pjsir42 for Updates
For More Info: “75970 – 84242, 94590 – 43333 / 2222”
A-2. In figure, if ATO = 40º, find AOB.

A-3. Find the length of tangent, drawn from a point 8 cm away from the centre of a circle of radius 6 cm.

A-4. Two circles touch each other externally, then find the number of common tangents to the circles.

A-5. ABCD is a quadrilateral such that D = 90º. A circle C (O, r) touches the sides AB, BC, CD and DA at
P, Q, R and S respectively. If BC = 38 cm, CD = 25 cm and BP = 27 cm, find r.

A-6. PQR is a right-angled triangle with PQ = 12 cm and QR = 5 cm. A circle with centre O and radius x is
inscribed in  PQR. Find the value of x.

A-7. From an external point P, two tangents PA and PB are drawn to the circle with centre O. Prove that OP
is the perpendicular bisector of AB.

A-8. Two tangents TP and TQ are drawn to a circle with centre O from an external point T. Prove that
PTQ = 2OPQ.

A-9. A circle touches the sides of a quadrilateral ABCD at P, Q, R, S respectively. Show that the angles
subtended at the centre by a pair of opposite sides are supplementary.

A-10. In figure OP is equal to diameter of the circle. Prove that ABP is an equilateral triangle.

A-11. The radius of the incircle of a triangle is 4 cm and the segments into which one side is divided by the
point of contact are 6 cm and 8 cm. Determine the other two sides of the triangle.

A-12. Prove that the tangent at any point of a circle is perpendicular to the radius through the point of contact.
For Unacademy Subscription Use “PJLIVE” Code | Join t.me/pjsir42 for Updates
For More Info: “75970 – 84242, 94590 – 43333 / 2222”
Using the above, do the following :
In figure, O is the centre of the two concentric circles. AB is a chord of the larger circle touching the
smaller circle at C. Prove that AC = BC.

A-13. In figure, CP and CQ are tangent from an external point C to a circle with centre O. AB is another
tangent which touches the circle at R. If CP = 11 cm and BR = 4 cm, find the length of BC.

A-14. PQ is a chord of length 8 cm of a circle of radius 5 cm. The tangents at P and Q intersect at a point T
(see figure). Find the length TP.
P

5 cm

T O
R

A-15. In the given figure, PQ is a transverse common tangent to two circles with centers A and B and of radii
5 cm and 3 cm respectively. If PQ intersects AB at C such that CP = 12 cm, calculate AB.

OBJECTIVE QUESTIONS
Section (A) : Circles
A-1. The length of the tangent drawn from a point 8 cm away from the centre of a circle of radius 6 cm is :
(A) 7 cm (B) 2 7 cm (C) 10 cm (D) 5 cm

A-2. A tangent PQ at a point P of a circle of radius 5 cm meets a line through the centre O at a point Q, so
that OQ = 12 cm. Length of PQ is :
(A) 12 cm (B) 13 cm (C) 8.5 cm (D) 119 cm

A-3. If tangents PA and PB from a point P to a circle with centre O are inclined to each other at an angle of
80º, then POA is equal to :
(A) 50º (B) 60º (C) 70º (D) 80º

A-4. PQ is a tangent to a circle with centre O at the point P. If OPQ is an isosceles triangle, then OQP is equal to :
(A) 30º (B) 45º (C) 60º (D) 90º
For Unacademy Subscription Use “PJLIVE” Code | Join t.me/pjsir42 for Updates
For More Info: “75970 – 84242, 94590 – 43333 / 2222”
A-5. From a point P which is at a distance of 13 cm from the centre O of a circle of radius 5 cm, the pair of
tangents PQ and PR to the circle are drawn. Then the area of the quadrilateral PQOR is :
(A) 60 cm2 (B) 65 cm2 (C) 30 cm2 (D) 32.5 cm2

A-6. In figure, PA and PB are tangents from a point P to a circle with centre O. Then the quadrilateral OAPB
must be a :

(A) Square (B) rhombus (C) cyclic quadrilateral (D) parallelogram

A-7. In figure, ABC is circumscribing a circle. Then the length of AB is :

(A) 6 cm (B) 8 cm (C) 12 cm (D) 14 cm

A-8. In figure, AB is a chord of a circle with centre O and AP is the tangent at A such that BAP = 75º.
Then ACB is equal to :

(A) 135º (B) 120º (C) 105º (D) 90º

A-9. In figure, if PA and PB are tangents to circle with centre O such that APB = 80º, then OAB is equal to

(A) 25º (B) 30º (C) 40º (D) 50º

For Unacademy Subscription Use “PJLIVE” Code | Join t.me/pjsir42 for Updates
For More Info: “75970 – 84242, 94590 – 43333 / 2222”
EXERCISE – 02

OBJECTIVE QUESTIONS
1. The three circles in the figure centered at A, B and C are tangent to one another and have radii 7, 21 and
6 respectively. The area of the triangle ABC, is

(A) 54 (B) 64 (C) 74 (D) 84

2. Triangle PAB is formed by three tangents to circle O and APB = 40º, then angle AOB

(A) 45º (B) 50º (C) 60º (D) 70º

3. On a plane are two points A and B at a distance of 5 unit apart. The number of straight lines in this
plane which are at distance of 2 units from A and 3 units from B, is:
(A) 1 (B) 2 (C) 3 (D) 4

4. Let C be a circle centre O. Let T be a point on the circle, and P a point outside the circle such that PT is
tangent to C. Assume that the segment OP intersects C in a point Q. If PT = 12 and PQ = 8, the radius
of C, is :
(A) r = 40 (B) r = 5 (C) r = 4 5 (D) r = 4 13

5. Three circles are mutually tangent externally. Their centres form a triangle whose sides are of lengths 3,
4 and 5. The total area of the three circles (in square units), is :
(A) 9 (B) 16 (C) 21 (D) 14

6. Two circle touch each other externally at C and AB is a common tangent to the circles. Then, ACB =
(A) 60º (B) 45º (C) 30º (D) 90º

7. AB and CD are two common tangents to circles which touch each other at C. If D lies on AB such that
CD = 4 cm, then AB is equal to :
(A) 4 cm (B) 6 cm (C) 8 cm (D) 12 cm

8. In figure, ABC is circumscribing a circle. Find the length of BC.

(A) 8 cm (B) 10 cm (C) 12 cm (D) 14 cm

For Unacademy Subscription Use “PJLIVE” Code | Join t.me/pjsir42 for Updates
For More Info: “75970 – 84242, 94590 – 43333 / 2222”
9. In figure, there are two concentric circles with centre O and of radii 5 cm and 3 cm. From an external
point P, tangents PA and PB are drawn to these circles. If AP = 12 cm, find the length of BP.

(A) 4 10 cm. (B) 2 10 cm. (C) 10 cm (D) 3 10 cm

10. Which of the following shapes of equal perimeter, the one having the largest area is :
(A) circle (B) equilateral triangle (C) square (D) regular pentagon

11. A triangle with side lengths in the ratio 3 : 4 : 5 is inscribed in a circle of radius 3. The area of the
triangle is equal to :
(A) 8.64 (B) 12 (C) 6 (D) 10.28

12. At one end A of a diameter AB of a circle of radius 5 cm, tangent XAY is drawn to the circle. The
length of the chord CD parallel to XY at a distance 8 cm from A is :
(A) 4 cm (B) 5 cm (C) 6 cm (D) 8 cm

13. In the given figure, O is the centre of the circle. Radius of the circle is 17 cm. If OC = 8 cm, then the
length of the chord AB is

(A) 35 cm (B) 30 cm (C) 15 cm (D) 18 cm

14. In the given figure OM ⊥ AB, radius of the circle 5 cm and length of the chord AB = 8 cm. Find the
measure of OM.

(A) 3 cm (B) 2.5 cm (C) 2 cm (D) 6 cm

15. In the given figure, two circles with their respective centres intersect each other at A and B and AB
intersects OO' at M, then m OMA is

(A) 60º (B) 80º (C) 90º (D) Can't be determined

16. In the above question (no. 3) what is the ratio of AM: BM?
(A) 5 : 6 (B) 3: 2 (C) 1 : 1 (D) Can't be determined

For Unacademy Subscription Use “PJLIVE” Code | Join t.me/pjsir42 for Updates
For More Info: “75970 – 84242, 94590 – 43333 / 2222”
17. In the given figure the two chords AC and BC are equal. The radius OC intersect AB at M, then AM : BM is

(A) 1: 1 (B) 2 :3 (C) 3 : 2 (D) None of the above

18. In the adjoining figure, O is the centre of circle and diameter AC = 26 cm. If chord AB = 10 cm, then
the distance between chord AB and centre O of the circle is

(A) 24 cm (B) 16 cm (C) 12 cm (D) None of the above

19. In the adjoining circle C(O, r) the degree measure of minor arc AB = 130º. Find the degree measure of
major arc

(A) 230º (B) 260º (C) 310º (D) None of the above

20. If the diagonals of a cyclic quadrilateral are equal, then the quadrilateral is
(A) rhombus (B) square (C) rectangle (D) None of these

21. In the given figure, AB is diameter of the circle. C and D lie on the semicircle. ABC = 65º and CAD = 45º.
Find m DCA.

(A) 45º (B) 25º (C) 20º (D) None of these

22. In the given figure, chords AB and CD are equal. If OBA = 55º, then m COD is

(A) 65º (B) 55º (C) 70º (D) 50º

23. The quadrilateral formed by angle bisectors of a cyclic quadrilateral is a


(A) rectangle (B) square (C) parallelogram (D) cyclic quadrilateral

For Unacademy Subscription Use “PJLIVE” Code | Join t.me/pjsir42 for Updates
For More Info: “75970 – 84242, 94590 – 43333 / 2222”
24. ABC and DBC have a common base and drawn towards same side of it. BAC = BDC = 60º. If
AC and DB intersect of P, then

(A) AP × PC = BP × PD (B) AP × BP = PC × PD
(C) AP × PO = PC × BP (D) None of these

25. In the given figure, BAC and BDC are the angles of same segments. DBC = 30º and BCD = 110º.
Find m BAC.

(A) 35º (B) 40º (C) 55º (D) 60º

26. In the given figure, O is the centre of the circle. ABO = 60º. Find the value of ACB.

(A) 40º (B) 60º (C) 50º (D) 30º

27. In the given figure, AOC = 120º. Find m CBE, where O is the centre.

(A) 60º (B) 100º (C) 120º (D) 150º

28. In the adjoining figure, O is the centre of the circle and OBO = 50º. Find them BAD.

(A) 60º (B) 40º (C) 80º (D) 150º

29. In the given figure, ABC is an equilateral triangle. Find m BEC.

(A) 120º (B) 60º (C) 80º (D) None of the above

For Unacademy Subscription Use “PJLIVE” Code | Join t.me/pjsir42 for Updates
For More Info: “75970 – 84242, 94590 – 43333 / 2222”
30. In the given figure, AB is the diameter of the circle. Find the value of ACD.

(A) 30º (B) 60º (C) 45º (D) 25º

31. In the given figure, ABCD is a cyclic quadrilateral and diagonals bisect each other at P. If DBC = 60º
and BAC = 30º, then BCD is

(A) 90º (B) 60º (C) 80º (D) None of the above

32. In the given figure, ABCD is a cyclic quadrilateral and AB is the diameter. ADC = 140º, then find m BAC.

(A) 45º (B) 40º (C) 50º (D) None of the above

33. In the given figure, COB = 40º, AB is the diameter of the circle. Find m CAB.

(A) 40º (B) 20º (C) 30º (D) None of these

34. In the given figure, O is the centre of circle. AOB = 80º and AOC = 120º. Find m BAC.

(A) 120º (B) 80º (C) 100º (D) None of the above

35. In the given figure, O is the centre of the circle and AOC = 100º. Find the ratio of m ADC : m ABC.

(A) 5: 6 (B) 1: 2 (C) 5: 13 (D) None of the above

For Unacademy Subscription Use “PJLIVE” Code | Join t.me/pjsir42 for Updates
For More Info: “75970 – 84242, 94590 – 43333 / 2222”
36. In the given figure, O is the centre of the circle and, AOB = 100º. Find m BCD.

(A) 80º (B) 60º (C) 50º (D) 40º

37. In the given figure, AB is the diameter of the circle. ADC = 120º, Find m CAB.

(A) 20º (B) 30º (C) 40 (D) Can’t be determined

38. In the given figure, O is the centre of the circle. AOB = 70º, find m OCD.

(A) 70º (B) 55º (C) 65º (D) 110º

39. In the given figure, CAB = 40º and AKB = 105º. Find KCD.

(A) 65º (B) 35º (C) 40º (D) 72º

40. In the given figure, ABC is an isosceles triangle in which AB = AC and m ABC = 50º, m BDC is

(A) 80º (B) 60º (C) 65º (D) 100º

41. In the given figure, AB is the diameter, m BAD = 70º and m DBC = 30º. Find m BDC

(A) 25º (B) 30º (C) 40º (D) 60º

For Unacademy Subscription Use “PJLIVE” Code | Join t.me/pjsir42 for Updates
For More Info: “75970 – 84242, 94590 – 43333 / 2222”
42. Find the value of DCE.

(A) 100º (B) 80º (C) 90º (D) 75º

43. 'O' is the centre of the circle, line segment BOD is the angle bisector of AOC, m COD = 50º. Find
m ABC.

(A) 25º (B) 50º (C) 100º (D) 120º

44. In the given figure, O is the centre of the circle. AOB = goo. Find m APB. P

(A) 130º (B) 150º (C) 135º (D) Can't be determined

45. In the given figure, PQ is the tangent of the circle. Line segment PR intersects the circle at N and R.
PQ = 15 cm, PR = 25 cm, find PN.

(A) 15 cm (B) 10 cm (C) 9 cm (D) 6 cm

46. In the given figure, there are two circles with the centres O and O touching each other internally at P.
Tan gents TQ and TP are drawn to the larger circle and tangents TP and TR are drawn to the smaller
circle. Find TQ : TR.

(A) 8 : 7 (B) 7: 8 (C) 5:4 (D) 1: 1

For Unacademy Subscription Use “PJLIVE” Code | Join t.me/pjsir42 for Updates
For More Info: “75970 – 84242, 94590 – 43333 / 2222”
47. In the given figure, PAQ is the tangent. BC is the diameter of the circle. m BAQ = 60º, find m ABC.

(A) 25º (B) 30º (C) 45º (D) 60º

48. ABCD is a cyclic quadrilateral PQ is a tangent at B. If DBQ = 65º, then BCD is

(A) 35º (B) 85º (C) 115º (D) 90º

49. In the given figure, AP = 2 cm, BP = 6 cm and CP = 3 cm. Find DP.

(A) 6 cm (B) 4 cm (C) 2 cm (D) 3 cm

50. In the given figure, AP = 3 cm, BA = 5 cm and CP = 2 cm. Find CD.

(A) 12 cm (B) 10 cm (C) 9 cm (D) 6 cm

51. In the given figure, tangent PT = 5 cm, PA = 4 cm, find AB.

7 11 9
(A) cm (B) cm (C) cm (D) Can't be determined
4 4 4

52. Two circles of radii 13 cm and 5 cm touch internally each other. Find the distance between their centres.
(A) 18 cm (B) 12 cm (C) 9 cm (D) 8 cm

53. Three circles touch each other externally. The distance between their centre is 5 cm, 6 cm and 7 cm.
Find the radii of the circles.
(A) 2 cm, 3 cm, 4 cm (B) 3 cm, 4 cm, 1 cm (C) 1 cm, 2.5 cm, 3.5 cm (D) 1 cm, 2 cm, 4 cm

For Unacademy Subscription Use “PJLIVE” Code | Join t.me/pjsir42 for Updates
For More Info: “75970 – 84242, 94590 – 43333 / 2222”
54. A circle touches a quadrilateral ABCD. Find the true statement.

(A) AB + BC =CD + AD (B) AB + CD = BC + AD


(C) BD = AC (D) None of the above

55. O and O' are the centres of two circles which touch each other externally at P. AB is a common tangent
touching the circles at A and B. Find APB.
(A) 90º (B) 120º (C) 60º (D) Data insufficient

56. If AB is a chord of a circle, P and Q are two points on the circle different from A and B, then
(A) the angle subtended by AB at P and Q are either equal or supplementary.
(B) the sum of the angles subtended by AB at P and Q is always equal two right angles.
(C) the angles subtended at P and Q by AB are always equal.
(D) the sum of the angles subtended at P and Q is equal to four right angles.

57. In the given figure, AB and CD are two common tangents to the two touching circles. If CD = 6 cm,
then AB is equal to

(A) 9 cm (B) 15 cm (C) 12 cm (D) None of the above

58. In the given figure, CD is a direct common tangent to two circles intersecting each other at A and B,
then CAD + CBD =?

(A) 120º (B) 90º (C) 360º (D) 180º

59. O and O' are the centres of circle of radii 20 cm and 37 cm. AB = 24 cm. What is the distance OO'?

(A) 51 cm (B) 45 cm (C) 35 cm (D) 48 cm

60. In a circle of radius 5 cm, AB and AC are the two chords such that AB = AC = 6 cm. Find the length of
the chord BC.
(A) 4.8 cm (B) 10.8 cm (C) 9.6 cm (D) None of these

61. In a circle of radius 17 cm, two parallel chords are drawn on opposite sides of a diameter. The distance
between the chords is 23 cm. If the length of one chord is 16 cm, then the length of the other is
(A) 23 cm (C) 15 cm (B) 30 cm (D) None of these

For Unacademy Subscription Use “PJLIVE” Code | Join t.me/pjsir42 for Updates
For More Info: “75970 – 84242, 94590 – 43333 / 2222”
62. A circle has two parallel chords of lengths 6 cm and 8 cm. If the chords are 1 cm apart and the centre is
on the same side of the chords, then a diameter of the circle is of length
(A) 5 cm (B) 6 cm (C) 8 cm (D) 10 cm

63. Three equal circles of unit radius touch each other. Then, the area of the circle circumscribing the three
circles is
 
(A) 6p (2 + 3 )2 (B) (2 + 3 )2 (C) (2 + 3 )2 (D) 3(2 + 3 )2
6 3

64. Through any given set of four points P, Q, R, S it is possible to draw


(A) at most one circle (B) exactly one circle (C) exactly two circles (D) exactly three circles

65. The number of common tangents that can be drawn to two circles is at the most
(A) 1 (B) 2 (C) 3 (D) 4

66. ABC is a right angled triangle AB = 3 cm, BC = 5 cm and AC = 4 cm, then the in radius of the circle is

(A) 1 cm (B) 1.25 cm (C) 1.5 cm (D) none of these

67. In the diagram, PQ and QR are tangents to the circle with centre O, at P and R respectively. Find the
value of x.

(A) 25 (B) 35 (C) 45 (D) 55

Passage (Q.68 & Q.69) : If a chord is drawn through the contact point of a tangent to a circle, then the angles
which this chord makes with the given tangent are respectively equal to the angles formed in the corresponding
alternate segment.

68. In the given fig. PQ is a tangent to the circle at A, BAQ = 50° and BAC = 35°, then ABC is

(1) 50º (B) 35° (C) 85° (D) 95º

For Unacademy Subscription Use “PJLIVE” Code | Join t.me/pjsir42 for Updates
For More Info: “75970 – 84242, 94590 – 43333 / 2222”
69. In the fig., PQ is a tangent to the circle at P. QRS is a straight line. Find the value of x.

(A) 25º (B) 30º (C) 35º (D) 40º

70. AB and CD are two paratlel chords of a circle such that AB= 10 cm and CD = 24 cm. The chords are on
opposite sides of the centre and the distance between them is 17 cm. Find the radius of the circle
(A) 11 cm (B) 12 cm (C) 13 cm (D) 14 cm

71. In the fig, RTP and STQ are common tangents to the two circles with centres A and B. The radii of the two
circles are 3 cm and 5 cm respectively. If ST : TQ = 3 : 5 and RT = 4 cm. Find the length of QT and AB.

(A) 11 cm, 12 cm (B) 20/3 cm, 40/ 3 cm (C) 18 cm, 12 cm (D) 12 cm, 15 cm

72. The radii of two concentric circles are 13 cm and 8 cm. AB is a diameter of bigger circle. BD is a
tangent to the smaller circle touching it at D. Find the length of AD.
(A) 15 cm (B) 16 cm (C) 18 cm (D) 19 cm

73. AB is the chord of a circle with centre O. AB is produced to C, such that BC = OB. CO is joined and
produced to meet the circle in D. If ACD = yº and AOD = xº, then

(A) 3xº = yº (B) xº = 3yº (C) xº = yº (D) xº = 4yº

74. In the adjoining figure, PQRS is a cyclic quadrilateral and the sides PS and QR are produced to meet at
B. Then out of the following the true statement is

(A) PR = QS (B) PB = QB (C) PA = QA (D) All of these

75. An equilateral triangle has side 2 3 cm. The radius of its circumcircle will be
(A) 2 cm (B) 3 cm (C) 3 cm (D) 4 cm

For Unacademy Subscription Use “PJLIVE” Code | Join t.me/pjsir42 for Updates
For More Info: “75970 – 84242, 94590 – 43333 / 2222”
76. In the given figure, APB is tangent, LQPS = 80º, PSQ=65o and SQR = 25º, then

(A) SPB = 35º (B) APQ = 65º (C) QSR = 55º (D) All of these

77. In a circle of radius 5 cm, AB and AC are two chords such that AB = AC = 6 cm. The distance of the
chord BC from centre is

(A) 3.6 cm (B) 1.4 cm (C) 1 cm (D) 4 cm

78. The chord ED is parallel to the diameter AC, as shown in the figure. The value of CED is equal to

(A) 30º (B) 40º (C) 50º (D) 60º

79. The line AB is 6 m in length and is tangent to the inner one of the two concentric circles at point C. It is
known that the radii of the two circles are integers. The radius of the outer circle is

(A) 5m (B) 4m (C) 6m (D) 3m

80. Find the length of a perpendicular from the centre of a circle to a chord if the diameter of the circle is
30 cm and its chord is 24 cm.
(A) 6 cm (B) 7 cm (C) 6 m (D) 3 m

81. Two circles touch internally. The sum of their areas is 116  cm2 and the distance between their centres
is 6 cm. The radii of the circles are
(A) 4 cm and 9 cm (B) 4 cm and 10 cm (C) 5 cm and 10 cm (D) 4 cm and 20 cm

82. In the figure, AB is a diameter of the circle. TD is tangent. If AHD = 36º, then CDT is

(A) 120º (B) 116º (C) 106º (D) 126º

For Unacademy Subscription Use “PJLIVE” Code | Join t.me/pjsir42 for Updates
For More Info: “75970 – 84242, 94590 – 43333 / 2222”
EXERCISE – 03

NTSE PROBLEMS (PREVIOUS YEARS)


1. In the following figure. O is the centre of the circle. The value of x is [Raj. NTSE Stage-1 2007]

(A) 45º (B) 65º (C) 85º (D) 95º


2. Two circles of equal radius touch each other externally at point C. AB is their common tangent. Value
of CAB is : [NTSE Stage-I/Rajasthan/2009]

(A) 30º (B) 40º (C) 45º (D) 60º


3. The chord of maximum length in a circle is called : [Raj. NTSE Stage-1 2013]
(A) Radius (B) Arc (C) Diameter (D) Point
4. One of the side of a triangle is divided into line segment of lengths 6 cm and 8 cm by the point of
tangency of the incircle of the triangle. If the radius of the incircle is 4 cm, then the length (in cm) of
the longer of the two remaining sides of the triangle is : [Haryana NTSE Stage-1 2013]
(A) 12 (B) 13 (C) 15 (D) 16
5. The circumference of the circumcircle of the triangle formed by x -axis, y-axis and graph of 3x + 4y = 12
is: [Haryana NTSE Stage-1 2013]
(A) 3 units (B) 4 units (C) 5 units (D) 6.25 units

6. AB and CD are two parallel chords of a circle such that AB = 10 cm and CD = 24 cm. If the chords are
on the opposite sides of the centre and the distance between them is 17 cm, the radius of the circle is :
[Delhi NTSE Stage-1 2013]
(A) 14 cm (B) 10 cm (C) 13 cm (D) 15 cm

7. In the figure given below, point O is orthocentre of ABC and points D, E and F are foot of the
perpendiculars, then how many sets make the 4 cyclic points from the point O ?
[Maharashtra NTSE Stage-1 2013]

(A) 4 (B) 3 (C) 2 (D) 6

8. If two circles are such that one is not contained in the other and are non-intersecting, then number of
common tangents are : [UP NTSE Stage-1 2013]
(A) One (B) Two (C) Three (D) Four

For Unacademy Subscription Use “PJLIVE” Code | Join t.me/pjsir42 for Updates
For More Info: “75970 – 84242, 94590 – 43333 / 2222”
9. AB and AC are equal chord of a circle with centre O. Then by which angle OA bisects BC.
[MP NTSE Stage-1 2014]
(A) 30º (B) 60º (C) 90º (D) 120º

10. In the following figure O is the centre of circle and BAC = nº, OCB = mº then
[UP NTSE Stage-1 2014]
A


O


B C
(A) mº + nº = 90º (B) mº + nº = 180º (C) mº + nº = 120º (D) mº + nº = 150º

11. In the below figure AB is a diameter of circle and AT is tangent line then value of x will be :
[Chhattisgarh NTSE Stage-1 2014]

(A) 65º (B) 50º (C) 45º (D) 90º

12. The hypotenuse of a right triangle is 10 cm and radius of the inscribed circle is 2 cm. The perimeter of
the triangle is : [Delhi NTSE Stage-1 2014]
(A) 15 cm (B) 22 cm (C) 24 cm (D) 18 cm

13. In figure, for ABC, chord AB = chord BC, ABC = 72º and the angle bisector of ABC intersects
the circle in Point D, then what is the measure of angle BEA ? [Maharashtra NTSE Stage-1 2014]

(A) 100º (B) 36º (C) 18º (D) 54º

14. In figure, A, B, C and D are four point on a circle. AC and BD intersect at a point E such that BEC =
125º and ECD = 30º. Then BAC = [Raj. NTSE Stage-1 2014]

(A) 95º (B) 110º (C) 85º (D) 105º

For Unacademy Subscription Use “PJLIVE” Code | Join t.me/pjsir42 for Updates
For More Info: “75970 – 84242, 94590 – 43333 / 2222”
15. In the given figure find PQR (where O is centre of the circle) [UP NTSE Stage-1 2014]

(A) 60º (B) 80º (C) 100º (D) 120º

16. If two equal circles of radius r passes through centre of the other then the length of their common chord
is [UP NTSE Stage-1 2014]
r 3
(A) (B) r 3 (C) r (D) r 2
3 4

17. In the given figure, DBC = 22º and DCB = 78º then BAC is equal to [Raj. NTSE Stage-1 2015]

(A) 90º (B) 80º (C) 78º (D) 22º

18. The radii of two concentric circle with centre O are 5 cm and 13 cm respectively. The line drawn from
the point A of outer circle touches the inner circle at point M and line AE intersects the inner circle at
the points C and D. If AE = 25 cm, then find AD. (A.C.D.E) [Maharashtra NTSE Stage-1 2015]
(A) 16 cm (B) 10 cm (C) 12 cm (D) 8 cm

19. If two chords of a circle are equidistance from the centre of the circle, then they are............
[MP NTSE Stage-1 2015]
(A) Equal to each other (B) Not equal to each other.
(C) Intersect each other. (D) None of these

20. In the given figure O is the centre of a circle, XY, PQ, AB are tangents of the circle. If XY || PQ, then
the value of AOB is [Raj. NTSE Stage-1 2016]
(A) 80º (B) 90º (C) 70º (D) 100º

21. In ABC, m B = 140, ‘P’ is the centre of the circumcircle of ABC. Find m PBC.
[Maharashtra NTSE Stage-1 2016]
(A) 40º (B) 50º (C) 80º (D) 100º

22. The incircle of ABC touches the sides AB, BC and AC in the point P, Q and R respectively. If AP = 7
cm, BC = 13 cm, find the perimeter of ABC [Maharashtra NTSE Stage-1 2016]
(A) 27 cm (B) 30 cm (C) 40 cm (D) 41 cm

For Unacademy Subscription Use “PJLIVE” Code | Join t.me/pjsir42 for Updates
For More Info: “75970 – 84242, 94590 – 43333 / 2222”
23. In the following figure secants QS and TR intersect each other at point P, which is outside the circle. O
is the point of intersection of chords SR and TQ. If OS = 5 cm, OT = 10 cm, TR = 12 cm, PR = 8 cm,
then find (PQ) . [Maharashtra NTSE Stage-1 2017]

S
Q
O P
R
T

(A) 6 cm (B) 10 cm (C) 12 cm (D) 16 cm

24. Radius of a circle with centre 'O' is 4 5 cm. AB is the diameter of the circle AE || BC and BC = 8 cm.
Line EC is tangent to the circle at point D. Find the length of DE. [Maharashtra NTSE Stage-1 2017]
A
E

O
D

B C

(A) 4 5 cm (B) 6 5 cm (C) 8 cm (D) 10 cm

25. In the adjoining figure, O is the centre of a circle; PQL and PRM are the tangents at the points Q and R
respectively and S is a point on the circle such that SQL = 50º and SRM = 60º then the value of
QSR is : [Bihar NTSE Stage-I/18]
L

Q
50º

S O
S
60º P
R
M
(A) 40º (B) 50º (C) 60º (D) 70º

26. In the given figure, a circle is centered at O. APB is a tangent at a point P, if QPB = 50º, then the
measurement of POQ will be... [Chhattisgarh NTSE Stage-I/18]

Q
O

50º
A B
P
(A) 100º (B) 120º (C) 140º (D) 150º

For Unacademy Subscription Use “PJLIVE” Code | Join t.me/pjsir42 for Updates
For More Info: “75970 – 84242, 94590 – 43333 / 2222”
27. In a right-angled triangle ABC, AB = 3 cm, BC = 4 cm and B = 90º. A circumcircle is constructed.
Radius of circumcircle will be.... [Chhattisgarh NTSE Stage-I/18]
(A) 3 cm (B) 4 cm (C) 5 cm (D) 2.5 cm

28. In the given figure, three circles with centres A, B, C respectively touch each other externally. If AB = 5 cm,
BC = 7 cm and CA = 6 cm, then the radius of the circle with A is : [Jharkhand NTSE Stage–I/18]

(A) 1.5 cm (B) 2 cm (C) 2.5 cm (D) 3 cm

VALUE BASED / PRACTICAL BASED QUESTIONS


1. Two roads starting from P are touching a circular path at A and B. Sarita runs from P to A, 20 km and
A to O, 15 km and Rita runs from P to O directly.

(i) Find the distance covered by Rita


(ii) Who will win the race?
(iii) Which value is depicted by Rita?

2. As a part of a campaign, a huge balloon with message of 'AWARENESS OF CANCER' was displayed
from the terrace of a tall building. It was held by strings of length 8 m each and inclined at an angle of
60º at the point, where it was tied as shown in the figure.

(i) What is the length of AB ?


(ii) if the perpendicular distance from the centre of the circle to the chord AB is 3m, then find the
radius of the circle.
(iii) Which method should be apply to find the radius of circle ?
(iv) What do you think of such campaign ?

For Unacademy Subscription Use “PJLIVE” Code | Join t.me/pjsir42 for Updates
For More Info: “75970 – 84242, 94590 – 43333 / 2222”
Answer Key

BOARD LEVEL EXERCISE


TYPE (I) : VERY SHORT ANSWER TYPE QUESTIONS : [01 MARK EACH]
1. 50º 2. 55º 3. 6 cm 4. 100º 5. 3 3 cm
6. 2 3 cm 7. 40º

TYPE (II): SHORT ANSWER TYPE QUESTIONS : [02 MARK EACH]


12. 30º

TYPE (III): LONG ANSWER TYPE QUESTIONS: [03 MARK EACH]


16. 4.8 cm

TYPE (IV): VERY LONG ANSWER TYPE QUESTIONS [04 MARK EACH]
20. 24 cm 21. 8 2 cm2

PREVIOUS YEAR PROBLEMS


2. AD = 7 cm, BE = 5 cm, CF = 3 cm 3. (B) 4. (A) 7. (C)
8. (B) 11. 120º 15. 1/3 17. OP = 2a

EXERCISE – 01

SUBJECTIVE QUESTIONS
Section (A) : Circles
A-1. 3 cm A-2. 100º A-3. 28 cm A-4. 3 A-5. 14 cm.
A-6. x = 2 cm. A-11. 13 cm, 15 cm A-13. 7 cm

OBJECTIVE QUESTIONS
Section (A) : Circles
A-1. (B) A-2. (D) A-3. (A) A-4. (B) A-5. (A)
A-6. (C) A-7. (D) A-8. (C) A-9. (C)

For Unacademy Subscription Use “PJLIVE” Code | Join t.me/pjsir42 for Updates
For More Info: “75970 – 84242, 94590 – 43333 / 2222”
EXERCISE – 02

OBJECTIVE QUESTIONS
Ques. 1 2 3 4 5 6 7 8 9 10 11 12 1 2 3 4 5 6 7 8
Ans. D D C B D D C B A A A D B A C C A C A D
Ques. 9 10 11 12 13 14 15 16 17 18 19 20 21 22 23 24 25 26 27 28
Ans. C C D A B D C B A C A C B B C C B B B A
Ques. 29 30 31 32 33 34 35 36 37 38 39 40 41 42 43 44 45 46 47 48
Ans. C B B C C D B C B B C D A B A A C D A C
Ques. 49 50 51 52 53 54 55 56 57 58 59 60 61 62 63 64 65 66 67 68
Ans. B D C A D A C D D C B D B D A D B B A C
Ques. 81 82
Ans. B D

EXERCISE – 03

Ques. 1 2 3 4 5 6 7 8 9 10 11 12 13 14 15 16 17 18 19 20
Ans. C C C C C C B D C A A C C A B B B A A B
Ques. 21 22 23 24 25 26 27 28
Ans. B C B D D A D B

VALUE BASED / PRACTICAL BASED QUESTIONS


1. (i) 25 km
(ii) Rita
(iii) Rita choose shortest path to reach at O. So, it shows her intelligence.

2. (i) 8 m
(ii) 5 m
(iii) Pythagoras theorem
(iv) Creating awareness and taking initiative

For Unacademy Subscription Use “PJLIVE” Code | Join t.me/pjsir42 for Updates
For More Info: “75970 – 84242, 94590 – 43333 / 2222”
10 AREA RELATED TO CIRCLES

Introduction

In our daily life we come across many objects which are circular in shape. For example, cycle wheels, bangles,
circular paths etc. That is why the problem of finding perimeters and areas related to circular figures is of great
importance.
Circle : The collection of all points in a plane, which are at a fixed distance from a fixed point in the plane, is
called a circle.
The fixed point is called the centre of the circle and the fixed distance is called the radius of the circle. The
diameter of a circle is twice its radius.
In figure, O is the centre and the length OP is the radius of the circle.

A. PERIMETER AND AREA OF A CIRCLE

(a) Circumference
The length of the boundary of a circle is called its circumference or its perimeter. We know that the
ratio of the circumference of a circle to its diameter is always a constant. This constant ratio is denoted
by the Greek letter .
Circumference

Diameter
Circumference =  × 2r = 2r [Diameter = 2r]
The exact value of  is not known, because  is an irrational number. For all practical purposes, the
22
value of  is approximately taken as or 3.14
7
If r is the radius of a circle, then
(i) Circumference = 2r or d, where d = 2r is the diameter of the circle.
(ii) Area = r2.
r 2
(iii) Area of semi-circle = .
2
(iv) Perimeter of the semi-circle = r + 2r.
(v) Area enclosed by two concentric circles
= R2 – r2 =  (R2 – r2) = (R + r) (R – r)
Where R and r and radii of two concentric circles.

Remarks :(i) If two circles touch each other externally, then the distance between their centres is equal to
sum of their radii.
(ii) If two circles touch each other internally, then the distance between their centres is equal to
difference of their radii.
(iii) The distance moved by a rotating wheel in one revolution is equal to the circumference of the
wheel.

For Unacademy Subscription Use “PJLIVE” Code | Join t.me/pjsir42 for Updates
For More Info: “75970 – 84242, 94590 – 43333 / 2222”
Solved Examples

Example.1 Find the area of a circle whose circumference is 22 cm.


Solution. Let r be the radius of the circle.
Then, Circumference = 22 cm
22 7
 2r = 22  2  r = 22  r = cm
7 2
22 7 7 2
 Area of the circle = r 2 =   cm = 38.5cm2 .
7 2 2

Example.2 If the perimeter of a semi-circular protractor is 66 cm, find the diameter of the protractor (Take
22
= ).
7
Solution. Let the radius of the protractor be r cm. Then, Perimeter = 66 cm
 22 
 r + 2r =66  r  + 2 − 66
7 
 36  66  7 77
 r   − 66  r−  r− cm
 7  36 6
77
 Diameter = 2r = 2r = cm .
3

Example.3 Two circles touch externally. The sum of their areas is 130  sq. cm. and the distance between
their centres is 14 cm. Find the radii of the circles.
Solution.

If two circles touch externally, then the distance between their centres is equal to the sum of
their radii.
Let the radii of the two circles be r1 cm and r2 cm respectively.
Let C1 and C2 be the centres of the given circles. Then,
C1C2 = r1 + r2  ClC2 = 14 cm (given)
 14 = r1 + r2
 r1 + r2 = 14 ...(i)
It is given that the sum of the areas of two circles is equal to 130  cm2.
 r12 + r22 = 130 
 r12 +r22 =130 ...(ii)
Now, (r1 + r2) = r1 = r1 + r + 2r1r2
2 2 2

 142 = 130 + 2r1r2 [Using (i) and (ii)]


 196 – 130 = 2r1r2
 r1r2 = 33 ...(iii)
Now, (r1 – r2)2 = r12 + r12 + – 2r1r2
 (r1 – r2)2 = 130 – 2 × 33 [Using (ii) and (iii)]
 (r1 – r2) = 64
2

 r1 – r2 = 8 ...(iv)
Solving (i) and (iv), we get r1 = 11 cm and r2 = 3 cm.
Hence, the radii of the two circles are 11 cm and 3 cm.

For Unacademy Subscription Use “PJLIVE” Code | Join t.me/pjsir42 for Updates
For More Info: “75970 – 84242, 94590 – 43333 / 2222”
Example.4 A race track is in the form of a ring whose inner circumference is 352 m, and the outer
circumference is 396 m. Find the width of the track.
Solution. Let the outer and inner radii of the ring be R metres and r metres respectively.
Then, 2R = 396 and 2r= 352
22 22
 2  E = 396and 2   r = 352
7 7
7 1 7 1
 R = 396   and r = 352  
22 2 22 2
 R = 63 m and r = 56 m
Hence, width of the track = (R – r) m = (63 – 56) m = 7 m.

Example.5 The diameter of a cycle wheel is 28 cm. How many revolutions will it make in moving 13.2 km?
Solution. Distance travelled by the wheel in one revolution
22 28
= 2r = 2   = 88cm
7 2
and the total distance covered by the wheel
= 13.2 × 1000 × 100 cm
= 1320000 cm
 Number of revolutions made by the wheel
1320000
= = 15000.
88

Example.6 A wire is looped in the form of a circle of radius 28 cm. It is re-bent into a square form.
Determine the length of the side of the square.
Solution. We have,
Length of the wire = Circumference of the circle
 22 
Length of the wire = 2   28 cm
 7 
Length of the wire = 176 cm ....(i)
Let the side of the square be x cm.
Then,
Perimeter of the square = Length of the wire
 4x = 176
 x = 44 cm
Hence, the length of the side of the square is 44 cm.

Check Point - A

1. The difference between the circumference and the radius of a circle is 37 cm. Find the area of the circle.
2. A wheel makes 1000 revolutions in covering a distance of 88 km. Find the radius of the wheel.

3. A circular road runs round a circle. If the difference between the circumference of the outer circle and
inner circle is 66 m, find the width of the road.

4. If the diameter of a circle is increased by 100%, find the percentage increase in its area.

5. The short and long hands of a clock are 4 cm and 6 cm long respectively. Find the sum of the distances
travelled by their tips in 2 days.

Answers
1. 154 cm2 2. 14 m 3. 21/2 m 4. 300º 5. 1910.8 cm

For Unacademy Subscription Use “PJLIVE” Code | Join t.me/pjsir42 for Updates
For More Info: “75970 – 84242, 94590 – 43333 / 2222”
B. SECTOR AND SEGMENT OF A CIRCLE

(a) Sector of a Circle and its Area


The region bounded by an arc of a circle and its two bounding radii is
called a sector of the circle.
If the arc is a minor arc then the corresponding sector is called the minor
sector and the remaining part bounded by the major arc is called the
major sector.
In figure shaded region OAMB is the minor sector and the remaining
portion OANB is the major sector.
(i) Length of Arc and Area of sector
Let r be the radius of the circle with centre O and AOB be a sector of the circle such that AOB = .
0 <  < 180º, then the arc AB is a minor arc of the circle.

Now, if  increases, the length of arc AB is also increases and if  becomes 180º, then arc AB
becomes the circumference of a semi-circle.
When an arc subtends an angle 180º at the centre, then the length of the arc of the semi-circle = r.
r r
When an arc subtends angle  at the centre, then length of the arc  = .
180º 180º
r
If  be the length of the arc AB, then =
180º
Again, when an arc subtends angle 180º at the centre, the corresponding sector is a semi-circular
1 2
region of area r .
2
 When an arc subtends an angle 180º at the centre, then the area of the corresponding sector is
1 2
r .
2
r 2 1 r 2
 When an arc subtends an  at the centre, then area of the sector =    = .
2 180º 360º
r 2 r
If A be the area of the sector, then A = and length of an arc =
360º 180º

Remarks (i) Angle described by minute hand in 60 minutes = 360º.


 360º  º
 Angle described by minute hand in one minute =   = 6º .
 60 
Thus, minute hand rotates through an angle of 6º in one minute.
(ii) Angle described by hour-hand in -12 hours. = 360º.
 360º  º
 Angle described by hour hand in one hour =   = 30º .
 12 
 30º  º  1  º
 Angle described by hour hand in one minute =   = 
 60   2 
 1 º
Thus, hour hand rotates through an angle of   in one minute.
2

For Unacademy Subscription Use “PJLIVE” Code | Join t.me/pjsir42 for Updates
For More Info: “75970 – 84242, 94590 – 43333 / 2222”
(b) Segment of a Circle and its Area
The region enclosed by an arc and a chord is called the segment of
the circle.
The segment containing the minor arc is called a minor segment
and the remaining segment containing the major arc is called the
major segment.
In the figure, the shaded region is the minor segment and the
remaining part of the circle is major segment.
(i) Area of a Segment of a Circle
Let r be the radius of a circle with centre O and let AB be an arc subtending an angle  at the centre
O. We shall find the area of the shaded segment AMB.
Let AP ⊥ O
Now, area of the segment AMB
= Area of the sector OAMB – area of OAB
r 2  1
= −  OB  AP
360º 2
r 2 1
= − r  OA  sin 
360º 2
AP
[ From AOP, sin  =  AP = OA sin ]
OA
r 2   2
= − r [ OA = OB = r]
360º 2
r 2  1 2
Hence, area of the segment = − r sin 
360º 2

Solved Examples

Example.7 A sector is cut from a circle of radius 21 cm. The angle of the sector is 150º. Find the length of
its arc and area.
Solution. The length or arc  and area A of a sector of angle  in a circle of radius r are given by
 
=  2r and A =  r 2 respectively.
360º 360º
Here, r = 21 cm and  = 150
150 22   150 22 2
 l=  2   21 cm = 55cm and A =    ( 21)  cm 2
 360 7   360 7 

Example.8 In figure, there are shown sector of two concentric circles of radii 7 cm and 3.5 cm. Find the
22
area of the shaded region. (Use  = ).
7
Solution. Let A1 and A2 be the areas of sectors OAB and OCD respectively.
Then,
A1 = Area of a sector of angle 30º in a circle of radius 7 cm
 30 22 2  
 A1 =    7  [Using : A = × r2]
 360 7  360
77 2
  = cm
6
A2 = Area of a sector of angle 30º in a circle of radius 3.5 cm.
 30 22 2  1 22 7 7  77 2
 A2 =    ( 3.5 )   A2 =      = cm
 360 7  12 7 2 2  21
For Unacademy Subscription Use “PJLIVE” Code | Join t.me/pjsir42 for Updates
For More Info: “75970 – 84242, 94590 – 43333 / 2222”
 77 77  2
 Area of the shaded region = A1 – A2 =  −  cm
 6 24 
77 77 2
= × (4 – 1) cm2 = cm
24 8
= 9.625 cm2.

Example.9 The minute hand of a clock is 10 cm long. Find the area of the face of the clock described by
the minute hand between 9 A.M. and 9.35 A.M.
Solution. We have,
Angle described by the minute hand in one minute = 6º.
 Angle described by the minute hand in 35 minutes = (6 × 35)º = 210º
 Area swept by the minute hand in 35 minutes.
= Area of a sector of angle 210º in a circle of radius 10 cm
 210 22 
=    (10) 2  cm2 = 183.3 cm2.
 360 7 

Example.10 A chord of circle 14 cm makes an angle of 60º at the center of the circle. Find
(i) area of minor sector (ii) area of the minor segment
(iii) area of the major sector (iv) area of the major segment
Solution. Given, r = 14 cm,  = 60º

(i) Area of minor sector OAPB = r2
360º
60º
= 60º × 3.14 × 14 × 14 = 102.57 cm2
360º
r 2  r 2
(ii) Area of minor segment APB = − sin
360º 2
14 14
= 102.57 − sin 60º
2
3
= 102.57 – 98 × = 17.80 cm2.
2
(iii) Area of major sector = Area of circle – Area of minor sector OAPB =  (14)2 – 102.57
= 615.44 – 102.57 = 512.87 cm2.
(iv) Area of major segment AQB
= Area of circle. Area of minor segment APB
= 615.44 – 17.80 = 597.64 cm2.

Example.11 Two circles touch externally. The sum of their areas is 130 sq. cm and the distance between
their centres is 14 cm. Determine the radii of the circles.
Solution. Let the radii of the given circles be R cm and r cm respectively (Fig.5). As the circles touch
externally, distance between their centres = (R + r) cm.
 R + r = 14 ... (i)
Sum of their areas= (R + r ) cm = (R2 + r2) cm2.
2 2 2

 (R2 + r2) = 130


 R2 + r2 = 130 ... (ii)
We have the identity, (R + r) + (R – r) = 2(R2 + r2)
2 2

 (14)2 + (R – r)2 = 2 × 130 [From (i) and (ii)].


 (R – r) = 64
2

 R–r=8 ... (iii)


On solving (i) and (iii), we get R = 11 cm and r = 3 cm.
Hence, the radii of the given circles are 11 cm and 3 cm.

For Unacademy Subscription Use “PJLIVE” Code | Join t.me/pjsir42 for Updates
For More Info: “75970 – 84242, 94590 – 43333 / 2222”
(pi) occupies the most significant place in measurement of surface area as well as volume of
various solid and plane figures. The value of  is not exactly known. According to S.
355
Ramanujan, the value of  = ,
113
10 10
According to Archimedes ; the value of  is given below :  = 3 << 3
71 70
17
Ptolemy,  = 3 .
120
2
 16 
The Egyptians.  =   = 3.160
9

Example.12 A chord of a circle of radius 14 cm makes a right angle at the centre. Calculate : (i) The area of
the minor segment of the circle, (ii) the area of the major segment of the circle.
Solution. Let AB be the chord of a circle with centre O and radius 14 cm such that AOB = 90º. Thus,
r = 14 cm and  = 90º. [Fig.6)
r 2   22 90  2
(i) Area of sector OACB = =  14 14  2
 cm = 154 cm .
360  7 360 

1 2 1 
Area of DOAB = r sin  =   14  14  sin 90  cm2 = 98 cm2.
2 2 
 Area of minor segment ACBA = (Area of sector OACB) – (Area of sector OACB) –
Area of OAB) = (154 – 98) cm2 = 56 cm2.
(ii) Area of major segment BDAB = (Area of the circle) – (Area of minor segment ACBA)
 22  
=   14  14  − 56 cm2 = (616 – 56) cm2 = 560 cm2.
 7  

Example.13 The minute hand of a clock is 10.5 cm long. Find the area swept by it in 15 minutes.
Solution Angle described by minute hand in 60 minutes = 360º.
 360 º
Angle described by minute hand in 15 minutes =   15  = 90º.
 60 
Thus, the required area is the area of a sector of a circle with central angle,  = 90º and
radius, r = 10.5 cm.
  2    22 90  2
 Required area =   =  10.5 10.5 
2
 cm = 86.63 cm
 360   7 360 

Check Point - B

1. Find the area of a quadrant circle of circumference 22 cm.


2. The perimeter of a sector of a circle of radius 5.6 cm is 27.2 cm. Find the area of the sector.
3. A pendulum swings through an angle of 30º and describes an arc 8.8 cm in length. Find the length of
the pendulum.
4. The perimeter of a sector of a circle of radius 7 cm is 44 cm. Find area of the sector.
5. An arc of a circle is of length 5 cm and the sector it bounds has an area of 25 cm2. Find the radius of
the circle.
Answers
1. 77/8 cm2 2. 44.8 cm2 3. 16.8 cm 4. 105 cm2 5. 10 cm

For Unacademy Subscription Use “PJLIVE” Code | Join t.me/pjsir42 for Updates
For More Info: “75970 – 84242, 94590 – 43333 / 2222”
C. APPLICATIONS OF AREA RELATED TO CIRCLES
In our daily life we come across various plane figures, which are combinations of two or more plane figures.
For example, window designs, flower beds, circular paths etc. In this section, we shall discuss problems
of combinations of plane figures.
IMPORTANT FORMULA
(i) Heron’s formula : Area of a triangle = s(s − a )(s − b)(s − c)
Where s = Semi-perimeter and a, b, c are the sides of the triangle.
1
(ii) Area of a right-angled triangle = × base × altitude
2
3 2
(iii) Area of an equilateral triangle = a.
4
(iv) Area of a rectangle = Length × breadth
(v) Perimeter of a rectangle of sides ‘a’ and ‘b’ = 2(a + b).
(vi) Area of a square of side ‘a’ = a2.
(vii) Length of diagonal of a square of a side ‘a’ = 2 a.
(viii) Perimeter of a square of side ‘a’ = 4a.
(ix) Area of a parallelogram = Base × Height
1
(x) Area of a rhombus = d1d2.
2
Where d1 and d2 are the lengths of its diagonals.
1
(xi) Area of a trapezium = (a + b)h.
2
Where a and b are lengths of two parallel sides and h is the distance A
between them.
(xii) In an equilateral triangle of side a with (AD)
3
(a) Height = a unit
2 O
9
(b) In radius (OD) = unit
2 3
9 B D C
(c) Circum radius (OB) = unit
3

Solved Examples

Example.14 In the given figure ABCP is a quadrant of a circle of radius 14 cm. With AC as diameter, a
semicircle is drawn. Find the area of the shaded portion.
Solution. In right angled triangle ABC,
we have
AC2 = AB2 + BC2
AC2 = 142 + 142
AC = 2  14 = 14 2 cm
2

Now required Area


= Area APCQA
= Area ACQA – Area ACPA
= Area ACQA – (Area ABCPA – Area of ABC)
2
1  14 2   1 1 
=      −   (14) +  14  14 
2

2  2  4 2 

For Unacademy Subscription Use “PJLIVE” Code | Join t.me/pjsir42 for Updates
For More Info: “75970 – 84242, 94590 – 43333 / 2222”
1 22 1 22
=   7 2  7 2 −  × 14 × 14 + 7 × 14
2 7 4 7
= 154 – 154 + 98 = 98 cm2.
Example.15 Find the area of the shaded region in figure, where radii of the two concentric circles with
centre O are 7 cm and 14 cm respectively and AOC = 40º.
Solution. We have,
Area of ring =  (R2 – r2)
=  × (142 – 72)
= 462 cm2
Area of the region ABDC
= Area of sector AOC. Area of sector BOD
 40 22 40 22 
=   14  14 −   7  7  cm 2
 360 7 360 7 
 1 1 
=   22 14  2 −  22  7 1 cm2
9 9 
Hence, Required shaded area
 154  2 1232 2
=  462 −  cm = cm = 410.67 cm2
 3  3
Example.16 In an equilateral triangle of side 24 cm, a circle is inscribed touching its sides. Find the area of
the remaining portion of the triangle [Take 3 = 1.732].
Solution. Let ABC be an equilateral triangle of side 24 cm, and let AD be perpendicular from A on BC.
Since the triangle is equilateral, so AD bisects BC.
 BD = CD = 12 cm
The centre of the inscribed circle will coincide with the centroid of ABC.
AD
 OD =
3
In  ABD, we have
AB2 = AD2 + BD2 [Using Pythagoras Theorem]
 242 = AD2 + 122
 AD = 242 − 122 = (24 − 12)(24 + 12) = 36 12 = 12 3 .
1 1 
 OD = AD=  12 3  cm = 4 3 cm
3 3 
 22
( )
2  22 
Area of the incircle = (OD)2=   4 3  cm2 =   48 cm2 =150.85 cm2
7  7 
3 3
Area of the triangle ABC = (Side)2 (24)4 = (24)4 = 249.4 cm2
4 4
 Area of the remaining portion of the triangle = (249.4 – 150.85) cm2 = 98.55 cm2.
Example.17 A horse is placed for grazing inside a rectangular field 70 m by 52 m and is tethered to one
corner by a rope 21 m long. On how much area can it graze ?
Solution. Shaded portion indicates the area which the horse can
graze. Clearly, shaded area is the area of a quadrant of a
circle of radius r = 21 m.
1 2
 Required area = r
4
 1 22 2
 Required area =    (21)  cm
2

4 7 

For Unacademy Subscription Use “PJLIVE” Code | Join t.me/pjsir42 for Updates
For More Info: “75970 – 84242, 94590 – 43333 / 2222”
693 2
 Required area = cm 346.5 cm2
2

Example.18 In figure, AOBCA represents a quadrant of a circle of radius 3.5 cm with centre O. Calculate
22
the area of the shaded portion (Take  = )
7
Solution. We have,

1 2 1 22
Area of quadrant AOBCA = r =   () = 9.625 cm2
4 4 7
1 1
Area of AOD = × Base × Height = (3.5)(2) = 3.5 cm2
2 2
Hence, Area of the shaded portion = Area of quadrant – Area of AOD = (9.625 – 3.5)cm2
= 6.125 cm2

Example.19 The figure shows a running track surrounding a grass enclosure PQRSTU. The enclosure consists of
a rectangle PQST with a semi-circular region at each end. Given, PQ = 200m and PT = 70 m.

(i) Calculate the area of the grassed enclosure in m2.


(ii) Given that the track is of constant Width 7 m, calculate the outer perimeter ABCDEF of the track.

Solution. (i) Diameter of each semi-circular region of grassed enclosure = PT = 70 m,


 Radius of each one of them= 35m.
Area of grassed enclosure
 (Area of rect. PQST) + 2 (Area of semi-circular region with radius 35 m)

1 2  22  2
 (PQ × PT) + 2 × r = (200  70) + 7  35  35 m = 17850 m .
2
2  
3
In an equilateral triangle of side, a unit, height = a unit
2
a a
inradius = units Circumradius = units
2 3 3
(ii) Diameter of each outer semi-circle of the track
For Unacademy Subscription Use “PJLIVE” Code | Join t.me/pjsir42 for Updates
For More Info: “75970 – 84242, 94590 – 43333 / 2222”
 AE = (PT + 7 + 7) m = 84 m.
 Radius of each one of them = 42 m.
Outer perimeter ABCDEF = (AB + DE + semi-circle BCD + semi-circle EFA)
 (2PQ + 2 × circumference of semi-circle with radius 42 m)
 22 
 (2 × 200 + 2 ×  × 42) m =  2  200 + 2   42 m = 664 m.
 7 

Example.20 Two circles touch internally. The sum of there are is 116 sq.cm and the distance between
centres is 6 cm. Find the radii of the given circle.
Solution. Let the radii of the given circles be R cm and r respectively. As the circles touch internally,
distance between their centres = (R – r) cm.
 R–r=6 ... (i)
Sum of their areas
= (R2 + r2) cm2 = (R2 + r2) cm2
 (R2 + r2) =116   R2 + r2 =116 .... (ii)
We have the identity, (R + r)2 + (R – r)2 = 2(R2
 (R + r)2 + 62 = 2 × 116 [Using (i) and (ii)]
 (R + r) = 196
2

 R + r = 196 = 14 cm … (iii)
On solving (i) and (iii), we get R = 10 cm and r = 4 cm.
Hence, the radii of the given circles are 10 cm and 4cm.

Check Point - C

1. Find the area of largest possible square inscribed in a circle of unit radius?

2. Four circular cardboard pieces each of radius 7 cm are placed in such a way that each piece touches two
other pieces. Find the area of the space enclosed by the four pieces.

3. Four horses are tethered at four corners of a square plot of side 63 metres so that they just cannot reach
one another. Find the area left un-grazed inside the plot.

4. A building with base in the form of an equilateral triangle of side 14 m is built in a huge grass field. In
one corner of the building a cow is tethered with a rope of length 21 m. Find the area grazed by the
cow.

5. A square grass field has side 14 metres. Two goats are tethered with a rope of 14 metres long each at
opposite corners. Find the common area grazed by both the goats.

Answers
1. 2 2. 42 cm2 3. 850.5 m2 4. 1257.66 m2 5. 112 m2

For Unacademy Subscription Use “PJLIVE” Code | Join t.me/pjsir42 for Updates
For More Info: “75970 – 84242, 94590 – 43333 / 2222”
BOARD LEVEL EXERCISE

TYPE (I) : VERY SHORT ANSWER TYPE QUESTIONS : [01 MARK EACH]
1. If the circumference of a circle and the perimeter of a square are equal, then show that area of the circle
is greater than area of the square.

2. If the perimeter of a circle is equal to that of a square, then find the ratio of their areas.

3. Find the area of the circle that can be inscribed in a square of side 6 cm.

4. Find the diameter of a circle whose area is equal to the sum of the areas of the two circles of radii 24 cm
and 7 cm.

5. Find the area of a sector of circle of radius 21 cm and central angle 120º.

6. The areas of two sectors of two different circles with equal corresponding arc lengths are equal. Is this
statement true? If no, give reasons in support of your answer.

TYPE (II) : SHORT ANSWER TYPE QUESTIONS : [02 MARKS EACH]


7. In Figure, a square is inscribed in a circle of diameter d and another square is circumscribing the circle.
Find the ratio of area of the outer square to the area of the inner square.

8. In figure, AB is a diameter of the circle, AC = 6 cm and BC = 8 cm. Find the area of the shaded region
(Use  = 3.14).
C

A B

9. Find the area of the shaded field shown in figure.


8m

4m
6m

10. Find the area of the minor segment of a circle of radius 14 cm, when the angle of the corresponding
sector is 60º.

For Unacademy Subscription Use “PJLIVE” Code | Join t.me/pjsir42 for Updates
For More Info: “75970 – 84242, 94590 – 43333 / 2222”
11. Find the area of the shaded region in figure, where arcs drawn with centres A, B, C and D intersect in
pairs at mid-points P, Q, R and S of the sides AB, BC,CD and DA, respectively of a square ABCD (Use
 = 3.14).
P
A B

S Q 12 cm

D C
R
12. A circular park is surrounded by a road 21 m wide. If the radius of the park is 105 m, find the area of
the road.

13. A circular pond is 17.5 m is of diameter. It is surrounded by a 2 m wide path. Find the cost of
constructing the path at the rate of Rs 25 per m2.

14. Find the difference of the areas of a sector of angle 120º and its corresponding major sector of a circle
of radius 21 cm.

TYPE (III) : LONG ANSWER TYPE QUESTIONS: [03 MARK EACH]


15. Find the area of the shaded region in figure.

4m

3m 3 m 12 m

4m

26 m

16. In figure, arcs are drawn by taking vertices A, B and C of an equilateral triangle of side 10 cm. to
intersect the sides BC, CA and AB at their respective mid-points D, E and F. Find the area of the
shaded region (Use  = 3.14).
A

F E

B D C

For Unacademy Subscription Use “PJLIVE” Code | Join t.me/pjsir42 for Updates
For More Info: “75970 – 84242, 94590 – 43333 / 2222”
17. The diameters of front and rear wheels of a tractor are 80 cm and 2 m respectively. Find the number of
revolutions that rear wheel will make in covering a distance in which the front wheel makes 1400
revolutions.

A B

D C

307
18. In figure, ABCD is a trapezium with AB || DC, AB = 18 cm, DC = 32 cm and distance between AB and
DC = 14 cm. If arcs of equal radii 7 cm with centres A, B, C and D have been drawn, then find the area
of the shaded region of the figure.
19. Three circles each of radius 3.5 cm are drawn in such a way that each of them touches the other two.
Find the area enclosed between these circles.
20. On a square cardboard sheet of area 784 cm2, four congruent circular plates of maximum size are
placed such that each circular plate touches the other two plates and each side of the square sheet is
tangent to two circular plates. Find the area of the square sheet not covered by the circular plates.
21. Floor of a room is of dimensions 5 m × 4 m and it is covered with circular tiles of diameters 50 cm each
as shown in figure. Find the area of floor that remains uncovered with tiles. (Use  = 3.14)
5m

22. An archery target has three regions formed by three concentric circles as shown in figure. If the
diameters of the concentric circles are in the ratio 1: 2:3, then find the ratio of the areas of three regions.

23. Area of a sector of central angle 200º of a circle is 770 cm2. Find the length of the corresponding arc of
this sector.
TYPE (IV): VERY LONG ANSWER TYPE QUESTIONS [04 MARK EACH]
24. Sides of a triangular field are 15 m, 16 m and 17 m. With the three corners of the field a cow, a buffalo
and a horse are tied separately with ropes of length 7 m each to graze in the field. Find the area of the
field which cannot be grazed by the three animals.

25. All the vertices of a rhombus lie on a circle. Find the area of the rhombus, if area of the circle is 1256
cm2. (Use  = 3.14).

For Unacademy Subscription Use “PJLIVE” Code | Join t.me/pjsir42 for Updates
For More Info: “75970 – 84242, 94590 – 43333 / 2222”
PREVIOUS YEARS PROBLEMS

1. Two circular pieces of equal radii and maximum area, touching each other are cut out from a
rectangular card board of dimensions 14 cm × 7 cm. Find the area of the remaining card board.
22
[use  = ]. [2 MARKS/CBSE 10TH BOARD: 2012]
7

22
2. If the difference between the circumference and the radius of a circle is 37cm, then using  = , the
7
circumference (in cm) of the circle is : [1 MARK /CBSE 10TH BOARD: 2012]
(A) 154 (B) 44 (C) 14 (D) 7

3. In Figure, AB and CD are two diameters of a circle with centre O, which are perpendicular to each
other. OB is the diameter of the smaller circle. If OA = 7cm, find the area of the shaded region.
22
[using  = ] [3 MARKS /CBSE 10TH BOARD: 2012]
7
C

B O A

4. In a circle of radius 21 cm, an arc subtends an angle of 60º at the centre.


22
Find : (i) the length of the arc (ii) area of the sector formed by the arc [ use  = ]
7
[3 MARKS/ CBSE 10TH BOARD: 2013]

5. In Figure, OABC is a quadrant of a circle of radius 7 cm. If OD = 4 cm, find the area of the shaded
22
region. [Use  = ]. [2 MARKS/ CBSE 10TH BOARD: 2013]
7
A
B
D

O C

6. In Figure, from a rectangular region ABCD with AB = 20 cm, a right triangle AED with AE = 9 cm and
DE = 12 cm, is cut off. On the other end, taking BC as diameter, a semicircle is added on outside the
region. Find the area of the shaded region. [Use  = 3.14 ]. [3 MARKS/ CBSE 10TH BOARD: 2013]
A B
E

D C

For Unacademy Subscription Use “PJLIVE” Code | Join t.me/pjsir42 for Updates
For More Info: “75970 – 84242, 94590 – 43333 / 2222”
7. In Figure, ABCD is a quadrant of a circle of radius 28 cm and a semi-circle BEC is drawn with BC as
22
diameter. Find the area of the shaded region. [Use  = ]. [3 MARKS/ CBSE 10TH BOARD: 2014]
7
B E
B
D

A C

8. Find the area of the minor segment of a circle of radius 14 cm, when its central angle is 60º. Also find
22
the area of the corresponding major segment. [Use  = ] [3 MARKS/ CBSE 10TH BOARD: 2014]
7

9. In Figure PQRS is a square lawn with side PQ = 42 metres. Two circular flower beds are there on the
sides PS and QR with centre at O, the intersection of its diagonals. Find the total area of the two flower
beds (shaded parts). [4 MARKS/ CBSE 10TH BOARD: 2014]
S R

P Q

10. In figure, O is the centre of a circle such that diameter AB = 13 cm and AC = 12 cm. BC is joined. Find
the area of the shaded region. (Take  = 3.14) [3 MARKS/CBSE 10TH BOARD: 2014]
A

B C

11. In figure, find the area of the shaded region, enclosed between two concentric circles of radii 7 cm and
22
14 cm, where AOC = 40º. (Use  = ). [3 MARKS/CBSE 10TH BOARD: 2014]
7

O
40º
B D
A C

For Unacademy Subscription Use “PJLIVE” Code | Join t.me/pjsir42 for Updates
For More Info: “75970 – 84242, 94590 – 43333 / 2222”
12. In Figure, is shown a sector OAP of a circle with centre O, containing  . AB is perpendicular the
radius OA and meets OP produced at B. Prove that the perimeter of shaded region is
  
tan sec. r  tan  + sec  + −1 . [4 MARKS/ CBSE 10TH BOARD: 2014]
 180 

B


A
O r

13. In figure, PQ is a tangent at a point C to a circle with centre O. If AB is a diameter and CAB = 30º,
find PCA. [1 MARK/CBSE 10TH BOARD: 2015]
P

Q
A B
O

14. In the given figure, ABCD is rectangle of dimensions 21 cm × 14 cm. A semicircle is drawn with BC as
diameter. Find the area and the perimeter of the shaded region in the figure.
[4 MARKS/CBSE 10TH BOARD: 2015]

15. Three semicircles each of diameter 3 cm, a circle of diameter 4.5 cm and a semicircle of radius 4.5 cm
are drawn in the given figure. Find the area of the shaded region.
[3 MARKS/CBSE 10TH BOARD: 2015]

3 cm 3 cm 3 cm
311

For Unacademy Subscription Use “PJLIVE” Code | Join t.me/pjsir42 for Updates
For More Info: “75970 – 84242, 94590 – 43333 / 2222”
16. In the given figure, two concentric circles with centre O have radii 21 cm and 42 cm. If AOB = 60º,
22
find the area of the shaded region. [Use  = ] [3 MARKS / CBSE 10TH BOARD: 2017]
7

O
60º
C D

A B

EXERCISE – 01

SUBJECTIVE QUESTIONS
Section (A) : Perimeter and Area of a Circle
A-1. If the diameter of a semi-circular protractor is 14 cm, then find its perimeter.

A-2. Diameter of a tyre is 1.26 m. Find the distance covered by it in 500 revolutions.

A-3. Two circles touch internally. The sum of their areas is 116  cm2 and distance between their centres is 6
cm. Find the radii of the circles.

A-4. The inner circumference of a circular track is 220 m. The track is 7 m wide everywhere. Calculate the
22
cost of putting up a fence along the outer circle at the rate of Rs 2 per metre. [Use  = ]
7

A-5. In figure there are three semicircles, A, B and C having diameter 6 cm each, and another semicircle E
having a circle D with diameter 9 cm are shown. Calculate :
(i) The area of the shaded region
(ii) The cost of painting the shaded region at the rate of 25 paise per cm2, to the nearest rupee.

9 cm
E F
0D

A C

6 cm B 6 cm

6 cm

Section (B) : Sector and Segment of a Circle


B-1. A chord of a circle of radius 12 cm subtends an angle of 60º at the centre. Find the area of the
corresponding segment of the circle. (Use  = 3.14 and 3 = 1.73).

B-2. The length of minute hand of a clock is 14 cm. Find the area swept by the minute hand in one minute.
22
[Use  = ]
7

B-3. In a circle with centre O and radius 5 cm, AB is a chord of length 5 3 cm. Find the area of sector AOB.

For Unacademy Subscription Use “PJLIVE” Code | Join t.me/pjsir42 for Updates
For More Info: “75970 – 84242, 94590 – 43333 / 2222”
B-4. A chord AB of a circle of radius 10 cm makes a right angle at the centre of the circle. Find the area of
the major and minor segments (Take  = 3.14)

22
B-5. ABCD is a flower bed. If OA = 21 cm and OC = 14 m, find the area of the bed. [Use  = ]
7

Section (C) : Applications of area related to Circles


C-1. Find the perimeter of figure, where is a semi-circle and ABCD is a rectangle.

C-2. A round table cover has six equal designs as shown in figure. If the radius of the cover is 28 cm, find
the cost of making the designs at the rate of Rs. 3.50 per cm2. (Use 3 = 1.7).

C-3. In a circular table cover of radius 32 cm, a design is formed leaving an equilateral triangle ABC in the
middle as shown in figure. Find the area of the design (shaded region).

C-4. Calculate the area of the designed region in figure common between the two quadrants of circles of
radius 8 cm each.
For Unacademy Subscription Use “PJLIVE” Code | Join t.me/pjsir42 for Updates
For More Info: “75970 – 84242, 94590 – 43333 / 2222”
C-5. Figure, shows a sector of a circle, centre O, containing an angle º. Prove that :

  
(i) Perimeter of the shaded region is r  tan  + sec  + − 1
 180 
r2   
(ii) Area of the shaded region is  tan  − 
2 180 

OBJECTIVE QUESTIONS
Section (A) : Perimeter and Area of a Circle
A-1. The area of the shaded portion in the given figure is :

(A) 7.5  sq.units (B) 6.5  sq.units (C) 5.5  sq.units (D) 4.5  sq.units

A-2. The radius of a circle is increased by 1 cm, then the ratio of the new circumference to the new diameter is :
1
(A)  + 2 (B)  + 1 (C)  (D)  −
2

A-3. If the sum of the circumferences of two circles with radii R 1 and R2 is equal to the circumference of a
circle of radius R, then :
(A) R1 + R2 = R
(B) R1 + R2 > R
(C) R1 + R2 < R
(D) Nothing definite can be said about the relation among R1, R2 and R

A-4. It is proposed to build a single circular park equal in area to the sum of areas of two circular parks of
diameters 16 m and 12 m in a locality. The radius of the new park could be :
(A) 10 m (B) 15 cm (C) 20 m (D) 24 cm

A-5. The circumference of a circle exceeds its diameter by 15 cm then, the circumference of the circle is :
(A) 22.00 cm (B) 22.5 cm (C) 21.01 cm (D) None

For Unacademy Subscription Use “PJLIVE” Code | Join t.me/pjsir42 for Updates
For More Info: “75970 – 84242, 94590 – 43333 / 2222”
A-6. If the circumference of a circle increases from 4 to 8 then its area is :
(A) halved (B) doubled (C) tripled (D) quadrupled

A-7. The ratio of the outer and inner circumferences of a circular path is 23: 22. If the path is 5 m wide, the
radius of the inner circle is :
(A) 55 m (B) 110 m (D) 220 m (D) 230 m
A-8. If a wire is bent into the shape of a square, then the area of the square is 81 cm2. When the same wire is
bent into a semi-circular shape, then the area of the semi circle will be :
(A) 22 cm2 (B) 44 cm2 (C) 77 cm2 (D) 154 cm2

A-9. A circular park has a path of uniform width around it. The difference between the outer and inner
circumferences of the circular park is 132 m. Its width is :
(A) 20 m (B) 21 m (C) 22 m (D) 24 m

A-10. A circular ground whose diameter is 140 meters is to be fenced by wire three times around its
22
circumference. Find the length of wire needed. [Use  = ]
7
(A) 440 m (B) 1320 m (C) 660 m (D) None of these

Section (B) : Sector and Segment of a Circle


B-1. The perimeter of the following shaded portion of the figure is :

(A) 40 m (B) 40. 07 m (C) 40. 28 m (D) 35 m

B-2. The area of the shaded region in the given figure is :

  
(A) sq. units (B) sq. units (C) sq. units (D) 2 sq. units
3 2 4

B-3. In figure, ABCD is square of side 42 cm. HEA, EFB, FGC and GHD are four quadrants of circles, then
shaded area is :

(A) 374 cm2 (B) 372 cm2 (C) 376 cm2 (D) 378 cm2

B-4. A lawn is the form of a square of side 30 m. A cow is tied with a rope of 10 m to a pole standing at one
of its corners. The maximum area of the lawn grazed by this cow is :
(A) 300 m2 (B) 150 m2 (C) 78.5 m2 (D) 450 m2
For Unacademy Subscription Use “PJLIVE” Code | Join t.me/pjsir42 for Updates
For More Info: “75970 – 84242, 94590 – 43333 / 2222”
B-5. Find the area of the shaded region in figure. where ABCD is a square of side 10 cm. (Use  = 3.14)

(A) 57 cm2 (B) 55 cm2 (C) 60 cm2 (D) 63 cm2

Section (C) : Applications of area related to Circles


C-1. If AC passes through the centre of the circle, then the area of the shaded region in the given figure is

a2   a2   
(A) (3 − )
2
(B) a  − 1 (C) 2a2 ( –1) (D)  − 1
2 2  2 2 

C-2. If a rectangle of sides 5 cm and 15 cm is be divided into three squares of equal area, then the sides of
the squares will be :
(A) 4 cm (B) 6 cm (C) 7 cm (D) None

C-3. The area of the square that can be inscribed in a circle of radius 12 cm is :
(A) 72 cm2 (B) 144 cm2 (C) 288 cm2 (D) 576 cm2

C-4. From a square metal sheet of side 28 cm, a circular sheet of largest possible radius is cut off the area of
the remaining sheet is :
(A) 784 cm2 (B) 78.4 cm2 (C) 168 cm2 (D) 84 cm2

C-5. A square with side length 1 is inscribed in a semicircle such that one side of the square is on the
diameter of the semicircle. The perimeter of the semicircle is :
 5   5
(A)  5 (B) (C) 5  + 1 (D) (  + 1)
2 2  2

For Unacademy Subscription Use “PJLIVE” Code | Join t.me/pjsir42 for Updates
For More Info: “75970 – 84242, 94590 – 43333 / 2222”
EXERCISE – 02

OBJECTIVE QUESTIONS
1. In the given figure, the diameter of the biggest semi-circle is 56 cm and the radius of the smallest circle
is 7 cms. then find the area of the shaded portion

(A) 462 cm2 (B) 231 cm2 (C) 924 cm2 (D) None of these

2. In the figure given, find the radius of the inner circle, if other circles are of radii 1 m.

(A) ( 2 + 1) m (B) 2m (C) ( 2 − 1) m (D) None of these

3. In this figure, AOB is a quarter circle of radius 10 and PQRO is a rectangle of perimeter 26. Find the
perimeter of the shaded region.

(A) 17 – 5 (B) 17 + 5 (C) 13 + 5 (D) 13 – 5

4. ABC is an isosceles right triangle with area P. The radius of the circle that passes through the point A,
B and C is
P P
(A) P (B) (C) (D) 2P
2 2

5. A circle passes through the three vertices of an isosceles triangle that has sides of length 3 and a base of
length 2. The area of the circle is
9 81 27 5
(A) (B) (C) (D)
4 32 16 2

6. Two circles, each with radius 6 , intersect in the two points A and B. For each of the circles
diameters from point A are drawn and the opposite ends C and D connected to point B. If the area of
figure ACBD is 2 11 , then the length of AB may have :
(A) two rational values (B) only one rational value
(C) one irrational and one rational value (D) two irrational values

7. Two circles I and II are externally tangent. A tangent to the circle I passes through the centre of the
circle II. The distance from the point of tangency to the centre of the circle II is three times the radius of
the circle II. The ratio of the circumference of the circle I to the circumference of the circle II.
(A) 2 (B) 3 (C) 4 (D) 16

For Unacademy Subscription Use “PJLIVE” Code | Join t.me/pjsir42 for Updates
For More Info: “75970 – 84242, 94590 – 43333 / 2222”
8. In the given figure, OPQR is a rhombus, three of whose vertices lie on a circle with centre O. If the area
of rhombus is 323 cm2. The radius of circle is :

(A) 8 (B) 9 (C) 10 (D) 11

9. A wire in the shape of an equilateral triangle encloses an area of S sq. cm. If the same wire is bent to
form of a circle. The area of the circle will be :
S2 3S2 3S 3 3S
(A) (B) (C) (D)
9   

10. Each of the congruent circles shown is externally tangent to other circles and/or to the side(s) of the
rectangle as shown. If each circle has circumference 16, then the length of a diagonal of the rectangle, is

(A) 80 (B) 40 (C) 20 (D) 15

11. In figure, arcs have been drawn with radii 14 cm each and with centres P, Q and R. Find the area of the
shaded region.
P

Q R

(A) 204 cm2 (B) 308 cm2 (C) 320 cm2 (D) None of these

12. A piece of wire 20 cm long is bent into the form of an arc of a circle subtending an angle of 60º at its
centre. Find the radius of the circle.
60 30 40 20
(A) cm (B) cm (C) cm (D) cm
   

13. The area of an equilateral triangle is 49 3 cm2. Taking each angular point as centre, a circle is
described with radius equal to half the length of the side of the triangle as shown in figure. Find the area
of the triangle not included in the circle. [Take 3 = 1.73]

(A) 7.77 cm2 (B) 7.24 cm2 (C) 7 cm2 (D) None of these

For Unacademy Subscription Use “PJLIVE” Code | Join t.me/pjsir42 for Updates
For More Info: “75970 – 84242, 94590 – 43333 / 2222”
14. In figure ABC is a right-angled triangle right-angled at A. Semicircles are drawn on AB, AC and BC as
diameters. Find the area of the shaded region.
(A) 4 cm2 (B) 3 cm2 (C) 12 cm2 (D) 6 cm2

15. In figure, AB and CD are two perpendicular diameters of a circle with centre O. If OA = 7 cm, find the
22
area of the shaded region. [Use  = ]
7

(A) the area of the shaded region is 12.5 cm2 (B) the area of the shaded region is 66.5 cm2
(C) the area of the shaded region is 10.5 cm2 (D) None of these

16. ABCD is a square, four equal circles are just touching each other whose centres are the vertices A, B,
C, D of the square. What is the ratio of the shaded to the unshaded area within square?

8 3 5 6
(A) (B) (C) (D)
11 11 11 11

17. If the radius of a circle is increased by 100%, then the area of the circle increases by
(A) 100% (B) 200% (C) 300% (D) 400%

18. If the perimeter of an isosceles right triangle is (6 + 3 2 )m then the area of the triangle is
(A) 4.5 m2 (B) 5.4 m2 (C) 9 m2 (D) 81 m2

19. In the adjoining figure ACB is a quadrant with radius 'a'. A semicircle is drawn outside the quadrant
taking AB as a diameter. Find the area of shaded region. ·

1 1 a2
(A) ( – 2a2) (B)   (a2 – a2) (C) (D) Can't be determined
4 4 2

For Unacademy Subscription Use “PJLIVE” Code | Join t.me/pjsir42 for Updates
For More Info: “75970 – 84242, 94590 – 43333 / 2222”
20. ABCD is a square, inside which 4 circles with radius 1cm each are touching each other. What is the
area of the shaded region?

(A) (2 – 3) cm2 (B) (4 – ) cm2 (C) (16 – 4) cm2 (D) None of these
21. The opposite pairs of sides of a square are increased by 40% and 30% respectively. The area of the
resulting rectangle exceeds the area of the square by
(A) 42% (B) 62% (C) 82% (D) 72%
22. If the length and width of a rectangular garden plot were each increased by 20 percent, then what would
be the percent increase in the area of the plot?
(A) 20% (B) 24% (C) 36% (D) 44%
1 1 1
23. The sides of a triangle are in the ratio of : : If the perimeter is 52 cm, then the length of the
2 3 4
smallest side is
(A) 9 cm (B) 10 cm (C) 11 cm (D) 12 cm
24. The length of a rope by which cow must be tethered at the center of circle in order that it may be able to
graze an area of 9856 sq. meters is
(A) 56 m (B) 64 m (C) 88 m (D) 168 m
25. A horse is tethered to one comer by a rope 21 m long for grazing inside a rectangular field of 70 m by
52 m. On how much area can it graze?
(A) 386.5 m2 (B) 325.5 m2 (C) 346.5 m2 (D) 246.5 m2
26. In the figure below, the rectangle at the comer measures 10 cm × 20 cm. The corner A of the rectangle
is also a point on the circumference of the circle. What is the radius of the circle in cm?

(A) 10 cm (B) 40 cm (C) 50 cm (D) None of these

27. There are two circles intersecting each other. Another smaller circle with centre O, is lying between the
common region of two larger circles. Centres of the circles (i.e., A, O and B) are lying on a straight
line. AB = 16cm and the radii of the larger circles are 10cm each. What is the area of the smaller circle?

4 
(A) 4 cm2 (B) 2 cm2 (C) cm2 (D) cm2
 4

28. Three circles of equal radii touch each other as shown in figure. The radius of each circle is 1cm. What
is the area of shaded region?

2 3 − 2 3 2 − 2 2 3
(A)   cm (B)   cm (C) cm2 (D) None of these
 2   3  

For Unacademy Subscription Use “PJLIVE” Code | Join t.me/pjsir42 for Updates
For More Info: “75970 – 84242, 94590 – 43333 / 2222”
29. A triangle and a parallelogram are constructed on the same base such that their areas are equal. If the
altitude of the parallelogram is 100 m, then the altitude of the triangle is
(A) 100 m (B) 200 m (C) 100 2 m (D) 10 2 m

30. There are two concentric circles whose areas are in the ratio of 9 : 16 and the difference between their
diameters is 4cm. What is the area of the area of the outer circle?
(A) 32 cm2 (B) 64 cm2 (C) 36 cm2 (D) 48 cm2

3x
31. The length of each side of a square is +1. What is the perimeter of the square?
4
9 2 3
(A) x + 1 (B) 3x + 1 (C) 3x + 4 (D) x + x+1
16 2
32. The area of a square increases by................. if its side increases by 30%.
(A) 71% (B) 60% (C) 69% (D) 30%

33. A track is in the form of a ring whose inner circumference, is 352 m and the outer circumference is 396
m. The width of the track is
(A) 44 m (B) 14 m (C) 22 m (D) 7 m

34. A person wishes to make a 100 sq. m rectangular garden. Since he has only 30 m barbed wire for fencing, he
fences only three sides letting the house wall act as the fourth side. The width of the garden is
(A) 20 m (B) 5 m (C) 50 m (D) 100 m

35. In the figure, ABCD is a square with side 10. BFD is an arc of circle with center C. BGD is an arc of a
circle with center A. What is the area of the shaded region?

(A) 100 – 50 (B) 100 – 257 (C)  – 100 (D) 25 – 100

36. A steel wire bent in the form of a square of area 121 cm 2. If the same wire is bent in the form of a
circle, then the area of the circle is:
(A) 130 cm2 (B) 136 cm2 (C) 154 cm2 (D) None of these

37. A pond 100 m in diameter is surrounded by a circular grass walk 2 m wide. How many square meters of
grass is there on the walk?
(A) 98  (B) 100  (C) 204  (D) 202 

38. The length of a rectangle is increased by 60%. By what percent would the width be decreased so as to
maintain the same area?
1
(A) 37 % (B) 60% (C) 75% (D) 120%
2

39. A circular grassy plot of Land, 42 min diameter, has a path 3.5 m wide running round it on the outside.
Find the cost of gravelling the path at 4 per square meter. ;
(A) 2002 (B) 2003 (C) 2004 (D) 2000

40. If the circumference and the area of a circle are numerically equal, then what is the numerical value of
the diameter?
(A) 1 (B) 2 (C) 4 (D) 

For Unacademy Subscription Use “PJLIVE” Code | Join t.me/pjsir42 for Updates
For More Info: “75970 – 84242, 94590 – 43333 / 2222”
41. A rhombus OABC is drawn inside a circle whose center is at O in such a way that the vertices A, B and C of
the rhombus are on the circle. If the area of the rhombus is 32 3 m2, then the radius of the circle is
(A) 64 m (B) 8 m (C) 32 m (D) 46 m

42. Four equal circles are described about the four corners of a square so that each touch two of the others.
If each side of the square is 14 cm, then the area enclosed between the circumferences of the circles is
(A) 24 sq cm (B) 42 sq cm (C) 154 sq cm (D) 196 sq cm

43. A wire is in the form of a circle of radius 35 cm. If it is bent into the shape of a rhombus, what is the
side of the rhombus? ·
(A) 32 cm (B) 70 cm (C) 55 cm (D) 17 cm

44. To make marriage tent, poles are planted along the perimeter of a square field at a distance of 5 meters from
each other and the total number of poles used is 20. What is the area (in sq. meters) of the square field?
(A) 500 (B) 400 (C) 900 (D) None of these

45. A rectangular carpet has an area of 60 m2. Its diagonal and longer side together equal 5 times the
shorter side. The length of the carpet is
(A) 5 m (B) 12 m (C) 13 m (D) 14.5 m

46. How many squares are there in a 5 inch by 5 inch square grid, if the grid is made up of one inch by one
inch squares?
(A) 50 (B) 150 (C) 55 (D) 25

47. The area of a rhombus is 2016 sq cm and its side is 65cm. The lengths of the diagonals (in cm) are
(A) 125, 35 (B) 126, 32 (C) 132, 26 (D) 135, 25

48. The number of revolutions made by a wheel of diameter 56 cm in covering a distance of 1.1 km is
 22 
 use  = .
 7 
(A) 31.25 (B) 56.25 (C) 625 (D) 62.5

49. Semi-circular lawns are attached to the edges of a rectangular field measuring 42 m × 35 m. The area of
the total field is
(A) 3818.5 m2 (B) 8318 m2 (C) 5813 m2 (D) 1358 m2

50. A wire is looped in the form of a circle of radius 28 cm. It is re-bent into a square form. Determine the
length of the side of the square.
(A) 44 cm (B) 45 cm (C) 46 cm (D) 48 cm

51. The circumference of a circular ground is 88 meters. A strip of land, 3 meters wide, inside and along
circumference of the ground is to be levelled. What is the budgeted expenditure if the levelling costs
Rs. 7 per square meter?
(A) Rs. 1050 (B) Rs. 1125 (C) Rs. 1325 (D) Rs. 1650

52. In the figure, when the outer circles all have radii 'r', then the radius of the inner circle will be

1 2
(A) 2r (B) ( 2 – 1)r (C) (D)
2r ( 2r + 1)r

For Unacademy Subscription Use “PJLIVE” Code | Join t.me/pjsir42 for Updates
For More Info: “75970 – 84242, 94590 – 43333 / 2222”
53. A square and an equilateral triangle have the same perimeter. If the diagonal of the square is 12 2 cm,
then the area of the triangle is
(A) 24 3 cm2 (B) 24 2 cm2 (C) 64 3 cm2 (D) 32 3 cm2

54. A horse is tethered to one corner of a rectangular grassy field 40 m by 24 m with a rope 14 m long.
Over how much area of the field can it graze?
(A) 154 m2 (B) 308 m2 (C) 150 m2 (D) None of these

55. Four horses are tied on the four corners of a square field of 14 m length so that each horse can touch the
other two horses. They were able to graze in the area accessible to them for 11 days. For how many
days is the ungrazed area sufficient for them?
(A) 3 days (B) 4 days (C) 5 days (D) 2 days

56. If the diagonal of a square is decreased by 15%, then the area of the square is decreased by
(A) 22.5% (B) 72.25% (C) 27.75% (D) None of these

57. The ratio of the circumference of two circles is 2 : 3. What is the ratio of their areas?
(A) 4 : 9 (B) 2 : 3 (C) 9 : 4 (D) None of these

58. The ratio of the area of a square inscribed in a semicircle to that of the area of a square inscribed in the
circle of the same radius is
(A) 2 : 1 (B) 2 : 3 (C) 2 : 5 (D) None of these ·

59. In the figure AB = CD = 2BC = 2BP = 2 CQ. In the middle, a circle with radius 1cm is drawn. In the
rest figure all are the semi-circular arcs. What is the perimeter of the whole figure?

(A) 4  (B) 8 (C) 10  (D) None of these

60. If BC passes through centre of the circle, then the area of the shaded region in the given figure is

a2  a2   
(A) (3 – ) (B) a2 ( – 1) (C) 2a2 ( – 1) (D)  − 1
2 2 2 2 

61. Two circles of unit radii, are so drawn that the centre of each lies on the circumference of the other. The
area of, the region common to both the circles, is
(4 − 3 3) (4 − 6 3) (4 − 3 3) (4 − 6 3)
(A) (B) (C) (D)
12 12 6 6

62. The area of the largest possible square inscribed in a circle of unit radius (in square unit) is
(A) 3 (B) 4 (C) 2 3  (D) 2

63. The area of the largest triangle that can be inscribed in a semicircle of radius r is
2
r
(A) r2 cm2 (B)   cm2 (C) r 2 cm2 (D) 3 3r cm2
 
3

64. If a regular hexagon is inscribed in a circle of radius r, then its perimeter is


(A) 6 3 r (B) 6 r (C) 3 r (D) 12 r

65. If a regular hexagon circumscribes a circle of radius r, then its perimeter is


For Unacademy Subscription Use “PJLIVE” Code | Join t.me/pjsir42 for Updates
For More Info: “75970 – 84242, 94590 – 43333 / 2222”
(A) 4 3 r (B) 6 3 r (C) 6r (D) 12 3 r

Directions (Q.66 to Q.68) : In the adjoining figure ABCD is a square. A circle ABCD is passing
through all the four vertices of the square. There are two more circles on the sides AD and BC touching
each other inside the square, AD and BC are the respective diameters of the two smaller circles. Area of
the square is 16 cm2.

66. What is the area of region 1?


 
(A) 2.4 cm2 (B)  2 −  cm2 (C) 8 cm2 (D) (4 – 2) cm2
 4

67. What is the area of region 2?


(A) 3( – 2) cm2 (B) ( – 3) cm2 (C) ( – 3) cm2 (D) 4( – 2) cm2

68. What is the area of region 3?


(A) (4 – ) cm2 (B) 4(4 – ) cm2 (C) ( – 2) cm2 (D) (3 + 2) cm2

69. A circular paper is folded along its diameter, then again it is folded to form a quadrant. Then it is cut as
shown in the figure, after it the paper was reopened in the original circular shape·. Find the ratio of area
of the original paper to that of the remaining paper?
(The shaded portion is cut off from the quadrant. The radius of quadrant OAB is 5 cm and radius of
each semicircle is 1 cm)

(A) 25 : 16 (B) 25 : 9 (C) 20 : 9 (D) None of these

70. In the figure below, ABCDEF is a regular hexagon and AOF = 90º. FO is parallel to ED. What is the
ratio of the area of the triangle AOF to that of the hexagon ABCDEF?

1 1 1 1
(A) (B) (C) (D)
12 6 24 18

71. The wheel of a cycle covers 660 metres by making 500 revolutions. What is the diameter of the wheel
(in cm)?
(A) 42 (B) 21 (C) 30 (D) 60

72. Twenty-nine times the area of a square is one square metre less than six times the area of the second
square and nine times the side of it exceeds the perimeter of another square by one metre. The
difference in s1des of these squares is
(A) 5 m (B) 54/11 m (C) 11 m (D) 6 m
73. The radius of one circular field is 5 m and that of the other is 13 m. Find the radius of the circular field
whose area is the difference of the areas of first and second fields.

For Unacademy Subscription Use “PJLIVE” Code | Join t.me/pjsir42 for Updates
For More Info: “75970 – 84242, 94590 – 43333 / 2222”
(A) 15 m (B) 13 m (C) 17 m (D) 12 m

74. The ratio of the areas of the incircle and the circumcircle of a square is
(A) 1 : 2 (B) 1: 3 (C) 1 : 4 (D) 1 : 2

75. The area of largest circle that can be drawn inside a rectangle with side 18 cm by 14 cm is
(A) 49 cm2 (B) 154 cm2 (C) 378 cm2 (D) 1078 cm2

EXERCISE – 03

NTSE PROBLEMS (PREVIOUS YEARS)


1. The circumference of a circle and perimeter of a square are equal. The ratio of their areas is-
[Raj. NTSE Stage-1 2005]
(A)  : 4 (B) 2 :  (C)  : 2 (D) 4 : 

2. In the given figure, AB = 4 cm, BC = 3 cm, the area of shaded portion is : [Raj. NTSE Stage-1 2005]

(A) 6.25 cm2 (B) 7.64 cm2 (C) 12 cm2 (D) 19.64 cm2

3. In the following figure, if O is the centre of the circle and radius OA = 14 cm, then the area of the
shaded portion is : [Raj. NTSE Stage-1 2006]

(A) 7 cm2 (B) 49 cm2 (C) 98 cm2 (D) 196 cm2

4. In the following figure the perimeter is [Raj. NTSE Stage-1 2007]

(A) 56 cm (B) 88 cm (C) 196 cm (D) None

5. A thin wire is bent into the form of a circle of radius 7 cm. If a square is made out of this wire, the side
of the square would be : (NTSE Stage -I/Bihar/2009)
(A) 7 cm (B) 14 cm (C) 11 cm (D) 22 cm

6. As shown in the figure diameter of outer circle is 12 cm. Ratio of shaded area of upper semi-circle to
that of the shaded area in the lower semicircle. [Rajasthan NTSE Stage -I 2009]

For Unacademy Subscription Use “PJLIVE” Code | Join t.me/pjsir42 for Updates
For More Info: “75970 – 84242, 94590 – 43333 / 2222”
(A) 1 : 1 (B) 11 : 3 (C) 4 : 3 (D) 8 : 27

7. The area of a circle inscribed in an equilateral triangle is 48 square units. The perimeter of the triangle
is ________units. [UP NTSE Stage-1 2012]
(A) 24 (B) 32 (C) 36 (D) 72

8. The perimeter of square and circumference of Circle are equal, the area of square is 121 m2, then the
area of circle is : [Raj. NTSE Stage-1 2013]
(A) 7  m 2
(B) 14  m 2
(C) 21  m 2
(D) 49  m2

9. A circle is inscribed in an equilateral triangle of side 'a' cm. The area (in cm2) of a square inscribed in
the circle is : [Haryana NTSE Stage-1 2013]
(A) a2/6 (B) a2/3 (C) 3a2/4 (D) a2/12

10. Four circular cardboard pieces, each of radius 7 cm. are placed in such a way that each piece touches
the two other pieces. The area of the space enclosed by the four pieces is: [Delhi NTSE Stage-1 2013]
(A) 21 cm2 (B) 42 cm2 (C) 84 cm2 (D) 168 cm2

11. In figure, ABC is a quadrant of a circle of radius 14 cm and a semicircle is drawn with BC as diameter.
The area of the shaded region is [Raj. NTSE Stage-1 2014]

(A) 98 cm2 (B) 154 cm2 (C) 56 cm2 (D) None of these

12. In the given figure, ABC is an equilateral triangle whose side is 2 3 cm. A circle is drawn which
passes through the midpoints D, E and F of its sides. The area of the shaded region is
[Raj. NTSE Stage-1 2014]

(A)
1
4
( )
4 − 3 3 cm2 (B)
1
4
(
2 − 3 cm2 ) (C)
1
4
( )
 − 3 3 cm2 (D)
1
4
( )
3 − 3 cm2

13. In the given figure, AB = 4 cm and BD = 4 3 cm. Then the relation between  [area of triangle ADC]
and S (shaded area bounded by three semicircles) is : [Haryana NTSE Stage-1 2014]

(A)  > S (B)  < S (C)  = S (D) None of the above

14. A chord of a circle of radius 7 cm. subtends an angle of 90º at its centre. The ratio of areas of smaller
and larger segment is : [Bihar NTSE Stage-1 2014]

For Unacademy Subscription Use “PJLIVE” Code | Join t.me/pjsir42 for Updates
For More Info: “75970 – 84242, 94590 – 43333 / 2222”
(A) 2 : 7 (B) 1 : 10 (C) 1 : 11 (D) None of these

15. If the radius of circle is , then its area will be- [Chhattisgarh NTSE Stage-1 2014]
(A)  (B) 2 (C) 3 (D) 3 

16. In below figure ABC is a right angle triangle in which B = 90 and BC = 6 cm and AB = 8 cm. Then
radius of incircle will be : [Chhattisgarh NTSE Stage-1 2014]

(A) 5 cm (B) 4 cm (C) 3 cm (D) 2 cm

17. The area of triangle formed by joining the centres of three congruent externally touching circles, is
49 3 sq-cm, then find the radius of circle. [Maharashtra NTSE Stage-1 2014]
(A) 7 cm (B) 7 cm (C) 7 3 cm (D) 4 3 cm

18. The radii of two circles are 9 cm and 12 cm. The circumference of a circle whose area is equal to sum
of the areas of the two circles is : [Delhi NTSE Stage-1 2014]
(A) 15 cm (B) 15 cm (C) 30 cm (D) 225 cm

5
19. The area of a rectangle is same as that of a circle of radius cm. If the length of the rectangle
11
exceeds its breadth by 3 cm., then the length of the rectangle is : [Jharkhand NTSE Stage-1 2014]
(A) 2 cm (B) 3 cm (C) 4 cm (D) 5 cm

20. The sum of the areas of two circles which touch each other externally is 153  sq. units. If the sum of
their radii is 15 units, then the ratio of large radius to the smaller radius is equal to :
[Jharkhand NTSE Stage-1 2014]
(A) 4 (B) 2 (C) 3 (D) none of these

21. In the given figure, three circles with centres A, B, C respectively touch each other externally. If AB = 5 cm,
BC = 7cm. and CA = 6 cm., then the radius of the circle with centre A is
[Jharkhand NTSE Stage-1 2015]

C
Q

R P
A B

(A) 1.5 cm (B) 2 cm (C) 2.5 cm (D) 3 cm

22. In the given figure ABCD is a square of side 14 cm. Find the area of the shaded region.
[Jharkhand NTSE Stage-1 2015]

For Unacademy Subscription Use “PJLIVE” Code | Join t.me/pjsir42 for Updates
For More Info: “75970 – 84242, 94590 – 43333 / 2222”
C
Q

R P
A B

(A) 56 cm2 (B) 48 cm2 (C) 42 cm2 (D) 44 cm2

23. Find the area of circle in sq. cms, in which the chord of length 18 cm is at the distance of half of the
radius of circle from the centre of circle : [MP NTSE Stage-1 2015]
(A) 108  (B) 54  (C) 27  (D) 81 

1
24. th region of the circle with the radius 3 7 cm is shaded. Then find the area of unshaded region.
9
[MP NTSE Stage-1 2015]
(A) 154 sq. cm (B) 176 sq. cm (C) 44 sq. cm (D) 22 sq. cm

25. A circle with area A cm2 is contained in the interior of a larger circle with area (A + B) cm 2 and the
radius of the larger circle is 4 cm. If A, B, A + B are in arithmetic progression, then the diameter (in
cm) of the smaller circle is : (NTSE Stage-2 /2015)
3 4 3 8 3
(A) (B) (C) (D) 2 3
2 3 3
26. ABC is a right-angled triangle with A = 90º, AB = b cm, AC = a cm, and BC = c cm A circle is
inscribed in this triangle. The radius of the circle, in cm, is : [Haryana NTSE Stage-1 2016]
1 1
(A) a + b – c (B) (a + b – c) (C) (a – b + c) (D) a 2 + b2 + c2
2 2
27. The wheel of a motor car makes 1000 revolutions in moving 440 m. The diameter of the wheel is
[Bihar NTSE Stage-1 2016]
(A) 0.44 m (B) 0.14 m (C) 0.24 m (D) 0.34 m

28. The area of the largest circle that can be drawn inside a square side 28 cm is
[Bihar NTSE Stage-1 2016]
(A) 17248 (B) 784 (C) 8624 (D) 616

29. The radius of a wheel is 0.25m the number of revolution to travel a distance of 11 km will be-
[UP NTSE Stage-1 2017]
(A) 1000 (B) 4000 (C) 8000 (D) 7000

30. In the adjoining figure ABCPA is a quadrant of a circle of radius 14 cm. With AC as diameter, a
semicircle is drawn. The area of the shaded region is : [Bihar NTSE Stage-I/18]

(A) 35 cm2 (B) 64 cm2 (C) 98 cm2 (D) 132 cm2

31. In the given figure, chord AB subtends an angle 90º at centre O of the circle having radius 4 cm. Area
of the shaded region will be [Rajasthan NTSE Stage-I/18]

For Unacademy Subscription Use “PJLIVE” Code | Join t.me/pjsir42 for Updates
For More Info: “75970 – 84242, 94590 – 43333 / 2222”
(A) (4 – 2)cm2 (B) 4( – 2)cm2 (C) ( – 8)cm2 (D) ( – 2)cm2

32. In the given figure O is the centre of circle. If AC = 8cm, BC = 6cm. Then the area of the shaded part
will be.... [Chhattisgarh NTSE Stage-I/18]

(A) 24 cm2 (B) 78 – 50 cm3 (C) 39 – 25 cm2 (D) 15.25 cm2

33. Length of Minute hand of a clock is 14cm. Area formed by this hand in 5 minutes is:
[Madhya Pradesh NTSE Stage-I/18]
154 215 205
(A) (B) 154 (C) (D)
3 3 3

2
34. How much time the minute hand of a clock will take to describe an angle of radians?
3
[Rajasthan NTSE Stage-I/19]
(A) 15 minutes (B) 20 minutes (C) 10 minutes (D) 25 minutes

35. Find the area of shaded portion in the figure given below, where ABCD id s square of the 28 cm:
[Rajasthan NTSE Stage-I/19]

(A) 784 cm2 (B) 616 cm2 (C) 668 cm2 (D) 168 cm2

For Unacademy Subscription Use “PJLIVE” Code | Join t.me/pjsir42 for Updates
For More Info: “75970 – 84242, 94590 – 43333 / 2222”
VALUE BASED / PRACTICAL BASED QUESTIONS
1. For the inauguration of Eco club to the school, badges were given to teachers. Sangeeta made these
5
badges in the shape of an equilateral triangle of side 5 cm with a circle of radius cm inscribed in it
2 3
as shown in the figure (take  = 3.14)

(i) Find the area of the shaded portion.


(ii) Which value is depicted by Sangeeta.

For Unacademy Subscription Use “PJLIVE” Code | Join t.me/pjsir42 for Updates
For More Info: “75970 – 84242, 94590 – 43333 / 2222”
Answer Key

BOARD LEVEL EXERCISE


TYPE (I) : VERY SHORT ANSWER TYPE QUESTIONS :
2. 14 : 11 3. 9  cm2 4. 50 cm 5. 462 cm2 6. No

TYPE (II) : SHORT ANSWER TYPE QUESTIONS :


7. 2 : 1 8. 54.5 cm2 9. (32 + 2)m2 10. 17.8 cm2 11. 30.96 cm2
2
12. 15246 cm 13. Rs 3061.50 14. 462 cm2

TYPE (III) : LONG ANSWER TYPE QUESTIONS:


15. 248 – 4 16. 39.25 cm2 17. 560 18. 196 cm2 19. 1.967 cm2
1
20. 168 cm2 21. 4.3 m2 22. 1 : 3 : 5 23. 73 cm
3

TYPE (IV): VERY LONG ANSWER TYPE QUESTIONS


24. 24 21 − 77 25. 800 cm2

PREVIOUS YEAR PROBLEMS

1. 21 cm2 2. (B) 3. 66.5 cm2 4. (i) 22 cm (ii) 231 cm2


5. 24.5 cm2 6. 334.39 cm2 7. 392 cm2 8. 17.79 cm , 598.21 cm2
2

9. 1386 m2 10. 36.39 cm2 11. 410.66cm2 13. 60º


14. 217 cm2, 78 cm 15. 99/8 cm2 16. 3465 cm2

EXERCISE – 01

SUBJECTIVE QUESTIONS
Section (A) : Perimeter and Area of a Circle
A-1. 36 cm A-2. 1980 m
A-3. radii of the given circles are 10 cm and 4 cm respectively.
A-4. Rs 528 A-5. (i) 49.5 cm2 (ii) Rs. 12.375 (Approximately)

Section (B) : Sector and Segment of a Circle


25 2
B-1. 13.08 cm2 B-2. 10.26 B-3. cm B-4. 28.5 cm2, 285.5 cm2
3
B-5. 192.5 m2

Section (C) : Applications of area related to Circles


C-1. 76 cm. C-2. Cost of making the design = Rs. 1626.8.
 22528  256 2
C-3.  − 768 3  cm 2 . C-4. cm .
 7  7

OBJECTIVE QUESTIONS
Section (A) : Perimeter and Area of a Circle
A-1. (D) A-2. (C) A-3. (A) A-4. (A) A-5. (A)
A-6. (D) A-7. (B) A-8. (C) A-9. (B) A-10. (B)

For Unacademy Subscription Use “PJLIVE” Code | Join t.me/pjsir42 for Updates
For More Info: “75970 – 84242, 94590 – 43333 / 2222”
Section (B) : Sector and Segment of a Circle
B-1. (C) B-2. (A) B-3. (D) B-4. (C) B-5. (A)

Section (C) : Applications of area related to Circles


C-1. (D) C-2. (D) C-3. (C) C-4. (C) C-5. (C)

EXERCISE – 02

Ques. 1 2 3 4 5 6 7 8 9 10 11 12 13 14 15 16 17 18 19 20
Ans. A C B A B D C A D A B A A D B B C A C B
Ques. 21 22 23 24 25 26 27 28 29 30 31 32 33 34 35 36 37 38 39 40
Ans. C D D A C C C A B B C C D B C C C A A C
Ques. 41 42 43 44 45 46 47 48 49 50 51 52 53 54 55 56 57 58 59 60
Ans. B B C D B D B C A A D B C A A C A C C D
Ques. 61 62 63 64 65 66 67 68 69 70 71 72 73 74 75
Ans. C D A B A D B B A A A D D D B

EXERCISE – 03

Ques. 1 2 3 4 5 6 7 8 9 10 11 12 13 14 15 16 17 18 19 20
Ans. D B C B C C D D A B A A A B C D B C D A
Ques. 21 22 23 24 25 26 27 28 29 30 31 32 33 34 35
Ans. B C A B C B B D D C B D A B D

VALUE BASED / PRACTICAL BASED QUESTIONS

1. (i) 4.27 cm2 (ii) Social responsibility

For Unacademy Subscription Use “PJLIVE” Code | Join t.me/pjsir42 for Updates
For More Info: “75970 – 84242, 94590 – 43333 / 2222”

You might also like